(PDF) The Complete Guide To The TOEFL Test | Cinthia Calvillo Tovar - Academia.edu
книга выложена группой vk.com/create_your_english JIo.-\1 "' ~ ~ e sentials TOEfl® IS a r gist red trademark of Educational esting ervic s (ETS) . T lis publication IS not e I ed or approved by ETS. книга выложена группой vk.com/create_your_english The Complete Guide to the TOEFL® Test --- -----~- - ---- ------ ---- --- _._- ---- - -- -------- The Complete Guide to the TOEFL@ Test PBT Edition Bruce Rogers ~... J# HEINLE _- (ENGAGE Learning - - - - - - - - - - Australia· Brazil· Japan. Korea· Mexico· Singapore. Spain. United Kingdom. United States - ~...~ HEINLE ,.., (ENGAGE Learning· The Complete Guide to the TOEFL ® Test: PBT Edition Bruce Rogers Publisher: Sherrise Roehr Acquisitions Editor: Tom Jefferies Director of Global Marketing: Ian Martin Director of Content and Media Production: Michael Burggren Content Project Manager: Daisy Sosa Print Buyer: Susan Spencer Cover Designer: Lisa Mezikofsky Cover Image: iStockphoto Compositor: MPS Limited, A Macmillan Company © 2011 Heinle, Cengage Learning ALL RIGHTS RESERVED. No part of this work covered by the copyright herein may be reproduced, transmitted, stored or used in any form or by any means graphic, electronic, or mechanical, including but not limited to photocopying, recording, scanning, digitizing, taping, Web distribution, information networks, or information storage and retrieval systems, except as permitted under Section 107 or 108 of the 1976 United States Copyright Act, without the prior written permission of the publisher. For product information and technology assistance, contact us at Cengage learning Customer & Sales Support, 1-800-354-9706 For permission to use material from this text or product, submit all requests online at cengage_com/permissions Further permissions questions can be emailed to permissionrequest@cengage.com ISBN-13: 978-1-111-22059-4 ISBN-lO: 1-111-22059-X Heinle Cengage Learning 20 Channel Center Street Boston, Massachussetts 02210 USA Cengage Learning is a leading provider of customized learning solutions with office locations around the globe, including Singapore, the United Kingdom, Australia, Mexico, Brazil, and Japan. Locate your local office at: international.cengage.com/region Cengage Learning products are represented in Canada by Nelson Education, Ltd. Visit Heinle online at elt.heinle.com Visit our corporate website at www.cengage.com This edition is an updated printing of the Complete Guide to the TOEFL®, Second Edition. Printed in the United States of America 12345671413121110 книга выложена группой vk.com/create_your_english f/ Contents Preface Acknowledgements ix xiii GETTING STARTED xv Questions and Answers about TOEFL Twelve Keys to High Scores on TOEFL xvi xx SECTION 1: GUIDE TO LISTENING COMPREHENSION 1 About Listening Comprehension Sample Listening Comprehension Test 2 Part A: Dialogs About Dialogs Lesson 1. Anticipating Questions about Dialogs Lesson 2. Dialogs with Sound Confusion Lesson 3. Dialogs with Homonyms and Words with Multiple Meanings Lesson 4. Dialogs with Idiomatic Expressions Lesson 5. Answering Inference Questions about Dialogs Lesson 6. Dialogs Involving Agreement and Disagreement Lesson 7. Dialogs Involving Suggestions, Invitations, Offers, and Requests Lesson 8. Dialogs Involving Contradictions, Assumptions, and Questions Lesson 9. Answering Questions about Activities, Plans, Topics, and Problems Lesson 10. Dialogs with Special Verbs Mini-Test 1: Dialogs Part B: About Extended Conversations About Extended Conversations Lesson 11. Anticipating Questions about the Extended Conversations Lesson 12. Answering Overview Questions about Extended Conversations Lesson 13. Answering Detail Questions about Extended Conversations Min i-Test 2: Extended Conversations Part C: Mini-Talks About Mini-Talks Lesson 14. Anticipating Questions about Mini-Talks Lesson 15. Answering Overview Questions about Mini-Talks Lesson 16. Answering Detail Questions about Mini-Talks 3 11 11 16 18 21 24 29 32 35 42 46 49 52 55 55 59 62 64 67 68 68 72 74 76 Mini-Test 3: Mini-Talks 79 Mini-Lessons for Section 1: Idiomatic Expressions 81 vi Contents SECTION 2: GUIDE TO STRUCTURE AND WRITTEN EXPRESSION 101 About Structure and Written Expression Sample Structure and Written Expression Test 102 104 Part A: About Structure Introduction Lesson 17. Incomplete Independent Clauses Lesson 18. Incomplete Adjective Clauses Lesson 19. Incomplete or Missing Participial Phrases Lesson 20. Incomplete or Missing Appositives Lesson 21. Incomplete Adverb Clauses Lesson 22. Incomplete Noun Clauses Lesson 23. Missing or Incomplete Prepositional Phrases 111 111 114 121 126 129 132 138 141 Lesson Lesson Lesson Lesson Lesson Lesson Lesson Lesson Mini-Test 4: Structure 144 24. 25. 26. 27. 28. 29. 30. 31. 147 152 155 159 162 165 168 171 Word Order Items Items Involving Verb Problems Incomplete or Missing Infinitive and Gerund Phrases Items Involving Parallel Structures Items Involving Misplaced Modifiers Incomplete or Missing Comparisons Missing Conjunctions Missing Negative Words Mini-Test 5: Structure Part B: Written Expression About Written Expression Lesson 32. Errors with Word Forms Lesson 33. Errors in Word Choice Lesson 34. Errors with Verbs Lesson 35. Errors with Parallel Structures Lesson 36. Errors with Pronouns Lesson 37. Errors with Singular and Plural Nouns Lesson Lesson Lesson Lesson Lesson Lesson 174 176 176 179 189 202 209 212 219 Mini-Test 6: Written Expression 225 38. 39. 40. 41. 42. 43. 228 234 240 247 250 254 Errors with Verbals Errors with Prepositions Errors with Articles Errors with Comparatives and Superlatives Errors in Word Order Errors with Conjunctions Mini-Test 7: Written Expression 257 Mini-Lessons for Section 2: Preposition Use 260 Contents vii SECTION 3: GUIDE TO READING COMPREHENSION About Reading Comprehension Sample Reading Comprehension Test Lesson 44. Overview Questions Lesson 45. Factual Questions, Negative Questions, and Scanning Questions Lesson 46. Inference Questions and Purpose Questions Lesson 47. Vocabulary-in-Context Questions Lesson 48. Reference Questions 269 270 276 290 303 313 324 334 Mini-Test 8: Reading Comprehension 343 Mini-Lessons for Section 3: Vocabulary Building 352 Guide to the Test of Written English (TWE) About the Test of Written English Ten Keys to Writing theTWE Essay Three Practice TWE Tests 387 388 Three Complete Practice Tests About Taking the Practice Tests Scoring the Practice Tests Practice Test I Practice Test 2 Practice Test 3 409 About the Author 497 Answer Sheets 499 395 399 410 411 413 441 469 Preface About This Book If you are preparing for TOEFL, you are not alone. Over a million people all over the world take either the Internet- or Paper-based version of the test every year. A high score on this test is an essential step in being admitted to graduate or undergraduate programs at almost all colleges and universities in North America. A strong TOEFL score may also be needed to complete an English program at a language school or may be a requirement for graduation from a university. But preparing for this test can be a difficult, frustrating experience. Perhaps you haven't taken many standardized, multiple choice tests such as TOEFL. Perhaps you are not familiar with the format of TOEFL. Maybe you've taken TOEFL once but were not satisfied with your score, or maybe you've taken the test several times but can't seem to improve your score beyond a certain point. In any of these cases, you need a guide. That's why this book was written-to help students preparing for this important exam to maximize their scores. This is the most complete and accurate TOEFL preparation book for the Paper-based test now available. It is based on many years of classroom experience teaching TOEFL preparation classes in the United States and abroad, and on several years of research on the test. The Complete Guide to the TOEFL Test is simply written and clearly organized and is suitable for any intermediate or advanced student of English as a second or foreign language. The Complete Guide to the TOEFL Test offers a step-by-step program that teaches you critical test taking techniques, helps you polish the language skills specifically needed for the Paper-based test, and generally makes you a smarter test-taker. And the Guide is an efficient way to prepare for TOEFL; by concentrating only on the points that are actually tested on the TOEFL, it lets you make the most of your preparation period and never wastes your time. Good luck on TOEFL! x Preface About This Edition In 200S, Educational Testing Service (ETS) began administering the Internet-based TOEFL (iBT). It was introduced at first in the United States and a few other countries. Over the next few years, ETS began to administer the iBT in more and more countries. However, the original Paper-based TOEFL (PBT) is still given in many locations around the world. Whether you take the iBT or the PBT depends on the location where you take the test. It may also depend on when you take the test, because some centers offer the PBT on certain dates and the iBT on others. To find out if the iBT or the PBT is given in your area, you should check the TOEFL website. Go to www.ets.org.click on TOEFL and then click on Register for the Test. In addition, the Institutional TOEFL (called the TOEFL Institutional Testing Program (ITP) by ETS) is a paper-based test which is administered by language centers, universities, and other institutions all over the world. Over 400,000 people take the Institutional test every year. For students or classes that must prepare for the Internet-based test, The Complete Guide to the TOEFL: iET Edition is also available from this publisher. This edition of The Complete Guide to the TOEFL Test meets the needs of people who want to take the TOEFL PBT rather than the iBT because the design of the two versions of the test is quite different. I believe The Complete Guide to the TOEFL Test is the most complete and accurate and up-to-date guide to preparing for the Paper-based TOEFL available. If you have comments about this edition of The Complete Guide to the TOEFL Test or about any aspect of the TOEFL test, I welcome your ideas, questions, and suggestions. please feel free to contact me at the e-mail address below. And, as always, good luck on TOEFL! Bruce Rogers Bruce_Rogers_CGT@mail.com Preface xi Organization of this Book Getting Started, th~ first section of the book, serves as an introduction to the Paper-based test. The opening<portion;of this section, Questions and Answers about TOEFL, provides you with information about the format of the test, guides you through the process of registering for the exam, and helps you understand your scores. The next portion of this section, Keys to High Scores, presents the "secrets" of being a good test-taker: arranging your preparation time, using the process of elimination, coping with test anxiety, pacing yourself during the test, and other important techniques. The main body of the book is divided into three parts, reflecting the three main sections of the Paper-based test: Listening Comprehension, Structure and Expression, and Reading Comprehension. Each of these sections consists of the following components: • An introduction containing basic strategies for that section of the test. • A full-length sample test to give you a feel for each part of the test and to provide a basis for understanding the lessons. • An introduction to each subsection of the test, with specific tactics for approaching the items in each portion of the test. • Lessons that break down the knowledge and skills needed for each part of the test into comprehensible "bites" of information. Each of the forty-eight lessons in the book contains sample items that illustrate exactly how the point brought up in that lesson is tested in TOEFL. Furthermore, each lesson contains one or more exercises for practicing the relevant points. Some of these items follow TOEFL format, others follow formats appropriate for the particular point being taught, but all of them have the same "feel" as actual TOEFL items. There are around 125 of these exercises in the GUide-totaling over 2,500 practice items! • Mini-tests that review the points brought up in the previous lessons. These tests put together the points practiced in isolation in the lessons and allow you to chart your progress. All the items on the mini-tests are identical in form to items on actual tests. • Mini-lessons covering important testing points which require more time to master than points brought up in the lessons. You should begin studying and working the exercises in the minilessons as soon as you he gin each section of the guide. • Section 1: Listening Comprehension This section is subdivided into three parts, each one designed to familiarize you with the problems commonly encountered on the three parts of the revised Section 1: Dialogues, Extended Conversations, and Mini-Talks. The exercises and tests in this part of the book are intended to be used with the audios that accompany the Guide. The mini-lessons for this section teach common idioms and figurative expressions that are often tested in Part A. • Section 2: Structure and Written Expression The first part of this section categorizes common grammatical points tested in Structure problems and suggests ways to solve these problems. The second part lists the usual errors that must be identified in Written Expression problems and offers ways to identify these mistakes. The mini-lessons for this section cover preposition usage, one of the most difficult of grammar skills. • Section 3: Reading Comprehension This section of the book prepares you for the third section of TOEFL. The Reading Comprehension portion of the book suggests reading attack skills, lists the various types of questions asked about the passages, and offers suggestions for answering each type of question. There are in-depth reading exercises to practice these techniques. The mini-lessons for this section systematically build your vocabulary, especially concentrating on the type of words that are asked about in the vocabulary items in the Reading section. • The Test of Written English This part of the book introduces the TWE and presents the best methods for planning, writing, and checking the essay you must write if you take this portion of the exam. There are two models of strong TWE essays and there are two practice TWE exams. xii Preface • Three Complete Practice Tests Practice tests provide one of the best ways to get ready for TOEFL because they draw together all the points you have studied. These tests duplicate the format, content, and level of difficulty of the actual exam. You may want to take one of these tests as a diagnostic when you begin your studies. To get the most out of these exams, follow the recommendations in the section titled ~bout Taking the Practice Tests. Scoring information is also included here. Suggestions for Using This Book The Complete Guide to the TOEFL Test: PBT Edition is designed to be used either as a textbook for TOEFL preparation classes or as a tool for individuals preparing for the exam by themselves. If you are working alone, you will need the audio program to accompany the textbook as well as the answer keys and audio scripts (available online). Whether working alone or in a group, you should begin your preparation for TOEFL by reading the introductory chapters titled Getting Started. You can then work through the book in the order in which it is written or begin with the section in which you are weakest (or in which the majority of the students in a class are weakest.) Generally, you can make the fastest progress by working in your weakest area. You can determine which area is your weakest by looking at the scores from a previous test or by using one of the practice tests as a diagnostic test. The amounts of time required to cover each segment of the Guide are given below. Keep in mind that these times are very approximate and do not include review sessions. Getting Started Guide to Listening Comprehension Guide to Structure and Written Expression Guide to Vocabulary and Reading Comprehension Guide to the Test of Written English Three Complete Practice Tests 1-3 hours 15-20 hours 20-30 hours 15-25 hours 4-8 hours 8-10 hours Acknowledgements I would like to thank all who worked on the original second edition of the book and to Tom Jefferies and Daisy Sosa for their help in preparing this updated printing. Getting Started xv xvi Getting Started ------ --- -- - --- -- - ------ Questions and Answers about TOEFL Q: What is TOEFL? A: TOEFL stands for Test of English as a Foreign Language. It is a test designed to measure the English-language ability of people who do not speak English as their first language and who plan to study at colleges and universities in North America either as undergraduate or graduate students. Educational Testing Service (ETS) of Princeton, New Jersey prepares and administers TOEFL. This organization produces many other standardized tests, such as the Test of English for International Communication (TOEIC), the Scholastic Aptitude Test (SAT), and the Graduate Record Exam (GRE). Although there are other standardized tests of English, TOEFL is by far the most important in North America. ETS has offered this exam since 1965. Each year, over a million people take the TOEFL test at testing centers all over the world. Around 7,000 colleges and universities in the United States and Canada accept TOEFL scores from students from non-English speaking countries as part of their application process. Q: What format does the Paper-based TOEFL (PBT) follow? How long does it take to complete? A: All the questions on the PBT (except for the TWE Prompt) are multiple choice questions with four answer choices. The test is divided into three sections, each with its own time limit. These sections are always given in the same order. Listening Comprehension 50 items 30 minutes (approximately) Structure and Written Expression 40 items 25 minutes Reading Comprehension 50 items 55 minutes Totals 140 items 1 hour 50 minutes Because of the time it takes to check identification, show people to their seats, give directions, and pass out and collect exams, you will actually be in the testing room for about two and a half hours. Q: How is the Internet-Based TOEFL (iBT) different from the PBT? A: The two versions of the test are quite different. Some of the most important differences are as follows: • All the items on the PBT are multiple choice but there are some special computer-unique item types on the iBT. • There is no Structure section on the iBT • There is no Speaking section on the PBT but there are six speaking tasks on the iBT • There is one Writing task on the PBT but two on the iBT • There are several "integrated" tasks on the iBT which test several skills at once, such as Listening Reading, and Speaking, but the PBT tests all the skills independently. • Note taking is allowed on the iBT but not on the PBT ____, '---; __________________________G_e_tt_i_n_9_S_t_a_rt_ed ----------------------------- xvii Q: What is the Institutional TOEFL? A. Institutional TOEFL (called the TOEFL Institutional Testing Program (ITP) by ETS) tests are given by English-language centers, universities, and other institutions. Sometimes Institutional TOEFL scores are used for placement in a school's English program or for testing a student's progress. Sometimes s£ores or a certain level are required for graduation. Institutional tests are made up of items that previously appeared 6nPaper-hased tests adminIstered by ETS. The answer sheets are sent to ETS or a TOEFL representative for scoring. Because ETS does not supervise these tests, most universities won't accept the results for admission purposes. The questions types, number of items, and time limits for the Institutional TOEFL are the same as the Paper-based TOEFL. Q: What is the Test of Written English (TWE)? A. The TWE tests your ability to communicate written English by requiring you to write a short essay on a specified topic. You have twenty-five minutes to complete your essay. There is no additional fee for taking TWE. There is no TWE on the Institutional TOEFL. Q: How do I register for TOEFL? A: You can register for the Paper-based test administered by ETS on their website. For the Institu- tional TOEFL, each institution has its own system for registering for the test. Q: What should I bring with me to the exam site? A: You should bring the following: • Your passport or other appropriate identification document • Your admission ticket • A watch • Several HB or No.2 pencils • Your photo file record, with a recent photograph attached Don't bring any reference books, such as dictionaries. You are not permitted to smoke, eat, or drink in the test center. Remember, you should arrive 30 minutes before the test begins. Q: When will I receive the results of the test? A: ETS sends scores for Paper-based tests about five weeks after you have taken the test. ETS will not send your scores early or give out scores over the telephone. Results for the Institutional TOEFL tests are usually available sooner, often in about four weeks. Q: How does ETS calculate my TOEFL PBT score? A: There is a total score and three sub-scores, one for each section of the test. Each section counts equally toward the total score. To obtain these scores, ETS's computers count the number of correct answers in each section. The results are called raw scores. The raw scores are then converted into scaled scores. The scaled scores from each section are added together, multiplied by 10, and divided by 3 to arrive at a total score, as shown: Scaled scores Part 1 49 (162 X 10) Part 2 + 58 1620 + Part 3 55 162 3 Total Score 540 Total scores range from a high of 677 to a low of 310, although scores of below 420 are rare. You must answer at least 25% of the questions in all three sections to receive a test score. xviii Getting Started Q: How is the PBT scoring system different from the iBT scoring system? A: The scores for the PBT range from 677 (the highest) to 310 (the lowest). This is also the scoring system used on the Institutional TOEFL test. The scores on the iBT range from 120 (the highest) to 0 (the lowest). Use the chart below to compare scores on the iBT and the PBT. 677 120 650 115 637 110 620 105 600 100 587 95 570 90 563 85 550 80 537 75 523 70 513 65 497 60 480 55 463 50 450 45 433 40 417 35 397 30 377 25 350 20 327 15 317 10 310 0 Q: What is a passing score on the Paper-based TOEFL? A: There isn't any. Each university has its own standards for admission, so you should check the catalogues of universities you are interested in, or contact their admissions offices. Most undergraduate programs require scores between 500 and 550, and most graduate programs ask for scores between 525 and 600. In recent years, there has been a tendency for universities to raise their minimum TOEFL requirements. Of course, the higher your score, the better your chance of admission. On the Institutional TOEFL, acceptable scores are set by each institution. Getting Started --------------------' Q: How are universities informed of my ----- xix ------- --- sc~!es? A: ETS reports your score to three institutions for free. For a charge, ETS will send your scores to additional institutions. Some universities will also accept photocopies of the test results that were mailed directly to you. Institutes that administer the institutional test are not allowed to send scores to universities. Q: How can I get more information about the Paper-based TOEFL? A: There is a lot more information about the Paper-based test on the ETS website at www.ets.org. xx Getting Started Twelve Keys to High Scores on TOEFL Key #1: Increase your general knowledge of English. There are two types of knowledge that will lead to high TOEFL scores: • A knowledge of the tactics used by good test takers and of the "tricks" of the test (which you will learn by using this Guide) • A general command of English (which must be built up over a long period) Following a step-by-step TOEFL preparation program, such as that presented in The Complete Guide to the TOEFL Test, will familiarize you with the tactics you need to raise your scores. The practice tests in this book will help you polish these techniques. The best way to increase your general knowledge of English is simply to use English as much as possible. Classes in English will be useful, and so will opportunities to speak, read, write, or listen to English. Some people who are preparing for TOEFL think that conversation classes and practice are a waste of time because speaking skills are not tested on the exam. In fact, one of the best ways to get ready for the exam is to converse in English whenever you can. Not only will you improve your ability to listen to everyday English, but you'll also learn to think in English. If you are living in an English-speaking country, don't spend all your time with people from your own country. If you are living in your home country, try to arrange opportunities for conversations in English. You can improve your listening comprehension skills by going to English-language lectures and movies. Listening to news and informational broadcasts on the radio is especially useful. Reading books, magazines, and newspapers in English can help you prepare for the Reading Comprehension part of the test. One of your most important jobs is to systematically improve your vocabulary. Vocabulary building will help you, not just in the Reading test, but throughout the exam. You may want to keep a personal vocabulary list. When you come across an unfamiliar word, look it up in a dictionary and write the word and its definition in your personal vocabulary list. Keep this list with you and study it when riding buses, eating lunch, taking coffee breaks, or whenever else you have a free moment. Key #2: Make the most of your preparation time. You need to train for TOEFL just as you would train for any important competitive event. Obviously, the sooner you can start training, the better, but no matter when you begin, you need to get the most out of your preparation time. Make a time-management chart. Draw up an hour-by-hour schedule of your week's activities. Block out those hours when you are busy with classes, work, social activities, and other responsibilities. Then pencil in times for TOEFL preparation. You will remember more of what you study if you schedule a few hours every day or several times weekly instead of scheduling all your study time in large blocks on weekends. After following this schedule for a week, make whatever adjustments are necessary. After that, try to keep to this schedule until the week before the testing date. During that last week, reduce your study time and begin to relax. If possible, reserve a special place where you do nothing but work on TOEFL preparation, separate from where you do your regular homework or other work. This place should be as free of distractions as possible. A good method of studying for TOEFL is the "30-5-5" method: • Study for thirty minutes. • Take a five-minute break-leave your desk and do something completely different. Getting Started - - - - - - xxi ------- • When you return, take five minutes to review what you studied before the break and preview what you are going to study next. Incidentally, it's an excellent idea to meet regularly with a small group of people who are also preparing for TOEFL. Research has shown that this study-group approach to test preparation is very effective. Key #3: Be in good physical condition when you take the exam. When you make out your time-management schedule, don't forget to leave time for physical activities-sports, aerobics, jogging, bicycling, or whatever else you prefer. The most important physical concern is that you not become exhausted during your preparation time. If you aren't getting enough sleep, you'll need to reduce your study time or another activity. This is especially important in the last few days before the exam. Key #4: Choose your test date carefully. If you require PBT scores for admission to a university, you need to plan ahead. Most students at North American universities enter their programs in the fall term, with late classes beginning in August or September. The deadline for applying for the fall term is usually December of the previous year or January of that year. Therefore, you should take the test sometime in the spring or summer of the previous year. If your score on the test is not satisfactory, you will then have time to take the test again. Of course, if you are taking a TOEFL preparation program, you should wait until you have completed or almost completed that course before you take the test so that the tactics and skills you have learned will be fresh in your mind. l_ Key #5: Be familiar with the format and directions. l You should have a clear map of the TOEFL test in your mind. Then, as you're taking the exam, you'll know exactly where you are and what's coming next. You can familiarize yourself with the basic TOEFL format by looking over the chart on page xvi. The directions for each part of the TOEFL test are always the same; even the same examples are used. If you are familiar with the directions, you won't have to waste time reading them during the test. You can become familiar with these directions by studying the directions for the practice tests in this book. xxii Getting Started Key #6: Organize your pre-exam time. You shouldn't try to "cram" (study intensively) during the last few days before the exam. Last-minute studying can leave you exhausted, and you need to be alert for the test. The night before the exam, don't study at all. Gtt together the materials you'll need in the morning, then go to a movie, take a long walk, or do something else to take your mind off the test. Go to bed when you usually do. If the exam is in the morning, have breakfast before you leave. Wear comfortable clothes because you'll be sitting in the same position for a long time. Give yourself plenty of time to get to the test site, keeping in mind traffic, weather, and parking problems. If you have to rush, that will only add to your stress. Key #7: Use time wisely during the test. TOEFL would be a far easier test if you could spend an unlimited amount of time working on it. However, there are strict time limits. Doing well on TOEFL means that you must find a balance between speed and accuracy. You don't want to rush through any section, but you do want to finish each section before time is called. The ideal is to finish Sections 2 and 3 with a few minutes remaining so that you can go back to questions that you found difficult. (The timing on Section 1 is controlled by the audio and you can't go back and check your answers after completing this section.) The questions on TOEFL are not equally difficult. Items can be classified as easy, medium, and difficult. The approximate distribution of items on a typical TOEFL are as follows: easy-30%; medium-40%; difficult-30%. Easy items are usually found at the beginning of each part of the test, medium items are usually found in the middle of each part, and difficult items are usually found at the end of each part. You may be tempted to rush through the easy items to save time for the difficult ones at the end of each part. This is not a good strategy. Your goal is to get as many right answers as possible. Therefore, you want to concentrate on the items that give you the best chance of a correct answer-in other words, the easiest ones. Hint: Remember, you don't get any extra points for answering difficult questions. Work steadily. Never spend too much time on anyone problem. If you are unable to decide on an answer, guess and go on. Answer each question as you come to it, even if you are not sure of the answer. You can mark difficult items on your answer sheet with check marks (as shown in Key #8). Then, if you have time at the end of the section, you can return to these problems. Sometimes when you come back to an item, you will find it easier. (Be sure to erase all of these check marks before you hand in your answer sheet.) The most important tool for timing yourself is a watch, preferably one with a "count down" feature that you can set at the beginning of Sections 2 and 3. (Watches with alarms are not permitted.) Key #8: Know how to mark your answer sheet. One of the worst surprises you can have during a test is to suddenly discover that the number of the item that you are working on doesn't correspond to the number of the answer you are marking for that item. You have to go back to find where you first got off track, then change all the answers after that number. You can avoid this problem by using the test book itself as a marker. Cover all the unanswered items in each column on your answer sheet. Then uncover one item at a time as you advance. Every five items or so, quickly glance at the number of the question that you are working on and the number of the answer to make sure they are the same. Getting Started xxiii ---------------------------------------------------- Mark answers by filling in the oval so that the letter cannot be seen. Don't mark answers any other way. Correct Incorrect 1 1 CDCD~CD 1 CDCD~CD Bring several #2 black lead pencils. Make sure each has a functioning eraser. Do not use a pen, liquid lead pencil, or any other kind of marker. By the way, you may see either of two types of answer sheet. On one the answer choices are displayed horizontally, while on the other they are displayed vertically. Horizontal 1 CD CD CD'- 2 CD'-CDCD 3 CDCD'-CD ~ CDCD'-CD 5 '-CDCDCD ~ CD CD CD'- Vertical V 4 V 6 , 8 , 0 , 0 , 00 0 , , 1 2 3 5 8 8 8 8 @ @ @ @ @ @ @@ Always be sure you have filled in a circle completely and have filled in only one answer per item. If you have to erase an answer, erase it completely. Notice the check marks by numbers 4 and 6. The test taker found these items difficult. He or she guessed at the answers, and then used the marks as a reminder to come back to these items if the time allowed. These marks should be erased before the end of the test. Incidentally, if you mark the same answer four times in a row, you'll know one of those four answers is wrong. The same correct answer will occur at most three times in a row on TOEFL. Hint: Don't sharpen your pencils too much before the exam. You can fill in circles more quickly if your pencil is not too sharp. Key #9: Improve your concentration. -- The ability to focus your attention on each item is an important factor in scoring high. Two and a half hours or more after all is a long time to spend in deep concentration. However, if your concentration is broken, it could cost you points. When an outside concern comes into your mind, just say to yourself, "I'll think about this after the test." Like any skill, the ability to concentrate can be improved with practice. Work on it while you are taking the practice tests in this book. xxiv Getting Started /-_._---......._-, Key #10: Use the process of/~ii~to make the best guess. Unlike some standardized exams, TOEFL has no penalty for guessing. In other words, incorrect answers aren't subtracted from your total score. Even if you are not sure which answer is correct, you should always, always, always guess. But you want to make an educated guess, not a blind guess. To do so, use the process of elimination, To understand the process of elimination, it may be helpful to look at the basic structure of a multiple-choice item. On TOEFL, multiple choice items consist of a stem and four answer choices, (The stem in the Listening section is spoken; in the other two sections, it is written.) One answer choice, called the key, is correct. The three incorrect choices are called distractors because their function is to distract (take away) your attention from the right answer. STEM .............. . (A) distractor (B) distractor (C) key (D) distractor The three distractors, however, are usually not equally attractive. One is usually "almost correct." This choice is called the main distractor. Most people who answer an item incorrectly will chose this answer. STEM ......... . (A) distractor (B) key (C) distractor (D) main distractor To see how this works in practice, look at this simple Structure item: Winter wheat _ _ _ _ planted in the fall. (A) because (B) is (C) which (D) has If you are sure of the answer, you should mark your choice immediately and go on. If not, you should use the process of elimination. In this item, choices (A) and (C) are fairly easy to eliminate. Because this sentence consists of a single clause, connecting words such as because and which are not needed. It may be a little more difficult to choose between choices (B) and (D) because both form correct verb phrases. Even if you are unable to decide between these two choices, you have a 50% chance of guessing correctly. That's twice as good as the 25% chance you would have if you had guessed blindly. (Choice (B) is the key, of course; a passive verb, not a present perfect verb, is required to complete the sentence correctly.) Choice (D) is the main distractor. What if you eliminate one or two answers but can't decide which of the remaining choices is correct? If you have a "hunch" (an intuitive feeling) that one choice is better that the others, choose it. If not, just pick any remaining answer and go on. If you have no idea which of the four answers is correct, it's better to use a standard "guess letter," such as (C), than to guess at random. You should NEVER leave any items unanswered. Even if you don't have time to read an item, you have a 25% chance of guessing the key. If you are unable to finish a section, fill in all the unanswered ovals on your answer sheet with your guess letter in the last few seconds before time is called. Remember: Use the same guess letter all the time so that you can fill in the ovals quickly. Getting Started xxv Key #11: Learn to control test anxiety. A little nervousness before an important test is normal. After all, these tests can have an important effect on your plans for your education and career. If you were going to participate in a big athletic contest or give an important business presentation, you would feel the same way. There is an expression in English that describes this feeling quite well: "butterflies in the stomach." These "butterflies" will mostly disappear once the test starts. And a little nervousness can actually help by making you more alert and focused. However, too much nervousness can slow you down and cause you to make mistakes. You may become anxious during the test because it seems very hard and it seems that you are making many mistakes. Try not to panic. The test seems hard because it is hard. You can miss quite a few items and still get a high score. One way to avoid stress on the day of the test is to give yourself plenty to time to get to the test center. If you have to rush, you'll be even more nervous during the exam. If you begin to feel extremely anxious during the test, try taking a short break-a "ten-second vacation." Close your eyes and put down your pencil. Take a few deep breaths, shake out your hands, roll your head on your neck, relax-then go back to work. Of course, you can't take a break during the Listening Comprehension section when the items are being read. However, if you're familiar with the directions, you can relax during the times when directions are being read. A positive, confident attitude toward the exam can help you overcome anxiety. Think of TOEFL not as a test of your knowledge or of you as a person but as an intellectual challenge, a series of puzzles to be solved. Key #12: Learn from taking practice test and official TOEFL exams. One of the most important steps in preparing for TOEFL is taking realistic, complete practice tests. There are now three tests in this book. In addition, you may take the official TOEFL test several times. Each time you take a test, either a practice test or a real one, you should learn from it. Immediately after the exam, write down your reactions: Which section seemed difficult? Did you have problems finishing any sections? When you look at your results, is the score for one section Significantly lower of higher than the scores for the other two sections? You can use this information to focus your studies for the next time you take the test. Hint: Whenever you take a practice test, pretend that you are taking an actual TOEFL exam. Whenever you take an actual exam, pretend you are taking a practice test. T SEC T ION книга выложена группой 13' vk.com/create_your_english Guide to Listening Comprehension 2 Section 1 • Guide to Listening Comprehension About Listening Comprehension The Listening Comprehension section of TOEFL is always given first. The purpose of this section is to test your understanding of spoken English. The directions for this section are given on the audio as well as printed in your test book. There are four speakers, two men and two women. The speakers read the items at a normal speed. All four have standard North American accents. The tone of the items is conversational, much less formal than the items in the two other test sections. Section 1 is divided into three parts: Part A: Dialogs 30 items Part B: Extended Conversations 2 conversations 7-8 items Part C: Mini-Talks 3 talks 12-13 items Totals: 50 items +30 minutes Listening Comprehension actually tests both your listening ability and your reading skills since you must understand both the material on the audio and the answer choices written in your test book. Many test takers find the Listening Comprehension section the most difficult. Because it is given first, you may be more nervous during this part of the test. Furthermore, it is difficult to understand voices on audio (just as it is on the telephone or radio) because you can't see the speakers' gestures, facial expressions, or lip movements as you can during "live" listening. Finally, the test writers at ETS employ a number of"tricks"that make choosing the correct answer more difficult. The exercises and tests in the Listening Comprehension section of this text are designed to help you overcome these difficulties. You will become more comfortable listening to audio materials in general and to TOEFL items in particular. You'll also become alert to many of the test writers' tricks. • Familiarize yourself with the directions for each part before the exam. But remember, you are not permitted to turn the page to look over answer choices while the directions are being read. (No answer choices appear on the same page as the directions for Section 1.) • If you have any difficulties hearing the audio, inform one of the proctors during the introductory section. Once the test has begun, the proctors cannot stop the audio. • Never skip any items. If you're not sure of an answer, guess. • Answer each item as quickly as you can; then preview the answer choices for the next item. Try to guess what the next item will be by the form of the answer choices. • Concentration is very important in this part of the test. Once you choose an answer, don't think about the last item-start thinking about the next one. Don't daydream. Focus your attention on the audio and on the choices in your test book. Begin your preparation for Section 1 by taking the Sample Listening Comprehension Test on the following pages. This will familiarize you with the first section of the exam. !!'j Section 1 • Guide to Listening Comprehension [!J Sample Listening Comprehension Test This section tests your ability to comprehend spoken English. It is divided into three parts, each with its own directions. You are not permitted to turn the page during the reading of the directions or to take notes at any time. Part A Directions: Each item in this part consists of a brief conversation involving two speakers. Following each conversation, a third voice will ask a question. You will hear the conversations and questions only once, and they will not be written out. When you have heard each conversation and question, read the four answer choices and select the one-(A), (B), (C), or (D)-that best answers the question based on what is directly stated or on what can be inferred. Then fill in the space on your answer sheet that matches the letter of the answer that you have selected. Here is an example. You will hear: Sample Answer You will read: (A) Open the window. (B) Move the chair. (C) Leave the room. (D) Take a seat. From the conversation you fmd out that the woman thinks the man should put the chair over by the window. The best answer to the question "What does the woman think the man should do?" is (B), "Move the chair." You should fill in (B) on your answer sheet. 8 3 4 Section 1 • Guide to Listening Comprehension It's brand new. She just repaired it. Someone painted it. It's just been sold. 2. (A) Give the woman cash. (B) Go to his car. (C) Return some merchandise. (D) Use his credit card. 3. (A) He shouldn't have thrown away the list. (B) He doesn't have to read alI the books. (C) All of the books on the list are required. (D) Some of the books are available now. 1. (A) (B) (C) (D) 4. (A) She enjoyed it very much. (B) She thOUght it was too long. (C) She thought it was boring. (D) She only liked the ending. 5. (A) Either a pen or pencil can be used. (B) It's not necessary to fill out the form. (C) She doesn't have either a pen or a pencil. (D) A pen is better than a pencil. 6. (A) The software isn't convenient to use. (B) He's not familiar with the software. (C) Using the software is simple. (D) He wishes he'd bought that software. 7. (A) The man ordered it, but it hasn't arrived yet. (B) It isn't working. (C) Someone else is using it. (D) The man doesn't know how to operate it. 8. (A) What time his brother called. (B) Where to meet his brother. (C) Why his brother called. (D) When to meet his brother. 9. (A) He left on a long trip yesterday. (B) His letter arrived unexpectedly. (C) He seemed to be sad yesterday. (D) The letter he sent was very funny. 10. (A) It's on the wrong floor. (B) There are too many bedrooms. (C) It's too small. (D) The rent is too high. Cij.i.';'i."UjUijiW!t-. Section 1 • Guide to Listening Comprehension [IJ[IJ[IJ[IJ[IJ[IJ[IJ[IJ[IJ[IJ[IJ[IJ 11. (A) He'll probably give the man another grade. (B) He doesn't teach chemistry anymore. (C) He rarely changes his grades. (D) He'll probably retire soon. 12. (A) She (B) She (C) She CD) She mailed the grades to her students. left the students' tests in her office. can't get into her office. put a list of grades on the door. 13. (A) He should get something for his friends to eat. (B) There isn't time for him to go out now. (C) The game won't be played today. (D) He should have invited his friends to the game. 14. (A) His class has been canceled. (B) He shouldn't drop the class. (C) An earlier class would be better for him. CD) He doesn't need to study political science. 15. (A) Sitting in other seats. (B) Going home. (C) Turning up the music. (D) Asking the usher for a refund. 16. (A) He has a good excuse for being late. (B) He's been feeling very weak recently. (C) He's stiII waiting to be contacted. (D) He doesn't take responsibility for errors. 17. (A) She doesn't have her camera. (B) The sun hasn't set yet. (C) There isn't any film in the camera. (D) Her camera is broken. 18. (A) He got on the wrong bus. (B) He's afraid he'll be late for his flight. (C) He's sorry he took a bus instead of flying. (D) He had to wait for the bus. 19. (A) The meeting will have to be rescheduled. (B) She doesn't care whom the board picks as dean. (C) She's not sure where the meeting will be. (D) The board will not choose a dean this month. 20. (A) (B) (C) (D) He's upset about the card game. He's getting ready for the game. He knocked over the card table. He sat down to have dinner. @.i.';'(.',UjU§!ii@V. 5 6 Section 1 • Guide to Listening Comprehension [I] [I] [I] [I] [I] [I] [I] [I] [I] [I] [I] [I] 21. (A) They wish they hadn't paid attention to Harvey. (B) They asked for some information about Harvey. (C) Harvey told them not to ignore him. (0) Only Harvey could give them any assistance. 22. (A) Most of the audience joined in the performance. (B) Some people don't enjoy performing. (C) Not many people were in the audience. (D) A few people didn't like the performance. 23. (A) A hotel room. (B) The man's family. (C) A reasonable offer. (0) The man's schedule. 24. (A) He must change his syllabus. (B) The woman cannot take his class. (C) He has extra copies of the syllabus. (D) Some students are not on his list. 25. (A) It's inconvenient for him to go to Mount Pleasant Street. (B) Those antique stores aren't very nice. (C) There are many inexpensive shops on Mount Pleasant Street. (0) The antiques in those stores are a little expensive. 26. (A) He's gone to San Diego many times. (B) He's attended a lot of conferences. (C) He has already gotten enough information. (D) He's living in San Diego now. 27. (A) He once drove in a race. (B) He's going to the races soon. (C) He drives quite fast. (D) He's thinking about a new car. 28. (A) The bowls are stacked on the shelves. (B) This soup is no worse than the other brands. (C) The new bowls are very attractive. (D) He plans to stock up on this soup. 29. (A) Peter wouldn't be favored in the match. (B) The match had already been played. (C) The match wouldn't be played. (0) Peter would win the match. 30. (A) He hasn't fmished working on the bookshelves. (B) The tools have been misplaced. (C) He's not very good with tools. (0) The tools have already been returned. @,i.",t,',Ujii43'.S t•t- . Section 1 • Guide to Listening Comprehension [I] [I] [I] [I] [I] [I] [I] [I] [I] [I] [I] [I] Part B Directions: This part of the test consists of extended conversations between two speakers. After each of these conversations, there are a number of questions. You will hear each conversation and question only once, and the questions are not written out. When you have heard the questions, read the four answer choices and select the one-(A), (B), (C), or (D)-that best answers the question based on what is directly stated or on what can be inferred. Then fill in the space on your answer sheet that matches the letter of the answer that you have selected. Don't forget: during actual exams, taking notes or writing in your test book is not permitted. 8 7 8 Section 1 • Guide to listening Comprehension ITJITJITJITJITJITJITJITJITJITJITJITJ 31. (A) Student and advisor. (B) Museum curator and visitor. (C) Manager and job applicant. (D) Professor and teaching assistant. 32. (A) In a few weeks. (B) Next year. (C) In three years. (D) In four years. 35. (A) Amounts of money. (B) Names of riders. (C) Types of cars. (D) Regions of the United States. 33. (A) Change her major. (B) Make a quick decision. (C) Take elective courses in art history. (D) Work full time at a museum. 37. (A) In the campus cinema. (B) Next door to the Student Union building. (C) In a travel agent's office. (D) On the second floor of the Student Union building. 34. (A) She couldn't get airline reservations. (B) She can't find an important book. (C) She's been studying too much. (D) She doesn't have a car. 36. (A) Information about places to visit. (B) Help with expenses and driving. (C) Plane reservations. (0) A used car. w.,.",t.i';Ui U43.!Wt--' Section 1 • Guide to Listening Comprehension [I] [I] [I] [I] [I] [I] [I] [I] [I] [I] [I] [I] Part C Directions: This part of the test consists of several talks, each given by a single speaker. After each of these talks, there are a number of questions. You will hear each talk and question only once, and the questions are not written out. When you have heard each question, read the four answer choices and select the one-(A), (B), (C), or (D)-that best answers the question based on what is directly stated or on what can be inferred. Then fill in the space on your answer sheet that matches the letter of the answer that you have selected. Here is an example. You will hear: Now here is a sample question. You will hear: Sample Answer CDCD . . CB You will read: (A) (B) (C) (D) Philosophy. Meteorology. Astronomy. Photography. The lecture concerns a lunar eclipse, a topic that would typically be discussed in an astronomy class. The choice that best answers the question "In what course is this lecture probably being given?" is (C), "Astronomy." You should fill in (C) on your answer sheet. Here is another sample question. You will hear: Sample Answer . . CDCDCB You will read: (A) The Earth's shadow moves across the Moon. (B) Clouds block the view of the Moon. (C) The Moon moves between the Earth and the Sun. (D) The Sun can be observed without special equipment. From the lecture you learn that a lunar eclipse occurs when the Earth moves between the Sun and the Moon and the shadow of the Earth passes across the Moon. The choice that best answers the question "According to the speaker, which of the following occurs during a lunar eclipse?" is (A), "The Earth's shadow moves across the Moon." Don't forget: during actual exams, taking notes or writing in your test book is not permitted. 8 9 10 Section 1 • Guide to Listening Comprehension ITJITJITJITJITJITJITJITJITJITJITJITJ 38. (A) An airplane. (B) A satellite. (C) A fireworks display. (D) A flying saucer. 39. (A) To change tires. (B) To avoid the danger. (C) To get a hotel room. (D) To change drivers. 40. (A) From the news on the radio. (B) From a newspaper. (C) From his mother. (D) From the news on television. 41. (A) It burned up in the upper atmosphere. (B) It injured a woman as she was sleeping. (C) It caused damage to a parked car. (D) It broke into pieces before striking the ground. 42. (A) Frightened. (B) Upset. (C) Fortunate. (D) Relieved. 43. (A) On board a bus. (B) At the top of the Washington Monument. (C) On an elevator. (D) At the Lincoln Memorial. 44. (A) Four years. (B) Thirty-six years. (C) Forty years. (D) Forty-eight years. 45. (A) Walk up 898 steps. (B) Take the elevator to the top. (C) Come down on the elevator. (D) Walk down the stairs. 46. (A) They jumped over it. (B) They took pictures of it. (C) They wrote their names on it. (D) They touched it. 47. (A) Music appreciation. (B) American history. (C) Dance. (D) Geography. 48. (A) They were an important part of the daily lives of the people of the frontier. (B) They were all extremely old. (C) They were all written as theme songs for political campaigns. (D) They were primarily written as dance music. 49. (A) They weren't as enduring. (B) They were harder to sing and play. (C) They were livelier. (D) They weren't concerned with politiCS. 50. (A) Sing songs. (B) Look at some sheet music. (C) Go to a dance. (D) Listen to a recording. THIS IS THE END OF THE SAMPLE LISTENING COMPREHENSION TEST. STOP WORK ON THIS TEST. PAR T A Dialogs About Dialogs The first part of the Listening Comprehension section consists of spoken dialogs (conversations) between two speakers. A third speaker asks a question about what was said or implied in the conversation. You must decide which of the four answer choices printed in your test book is the best answer for the question you hear and then mark that choice on your answer sheet. Between each of the dialogs is a twelve-second pause. There are thirty dialogs. Sample Item You will hear: Ml: Do you think I should leave this chair against the wall or put it somewhere else? F1 : Over by the window, I'd say. M2: What does the woman think the man should do? You will read: (A) (8) (C) (D) Open the window. Move the chair. Leave the room. Take a seat. The woman indicates that she thinks the man should put the chair over by the window rather than leave it where it is. In other words, he should move it. The best 11 12 Section 1 • Guide to Listening Comprehension answer is therefore (B). Note: M1 = first male voice M2 = second male voice F1 = first female voice F2 = second male voice The Dialogs Most of the dialogs in Part A involve a man and a woman. A few involve two men or two women. Each speaker usually speaks one or two sentences. Many dialogs (about 25%) are about facets of life at American universities: attending classes, talking to professors, writing research papers, taking tests. Other dialogs are about more general activities: shopping in grocery stores, looking for hoUSing, taking vacations, going to meetings and parties. The tone of the dialogs is informal. Idioms, first names, contractions (I'm, doesn't, can't) are often heard. Some of the items test your ability to understand various language functions. For example, you must be able to determine if a speaker is agreeing or disagreeing with the other speaker, or if one speaker is accepting or rejecting the other speaker's offer. The Questions Most of the questions about the dialogs focus on what the second speaker says. However, it is usually necessary to understand the entire dialog in order to choose the correct answer. For example, in the Sample Item, it is not clear what the woman means when she says "Over by the window" unless you understand what the man says first. One or two questions in each test may focus instead on what the first speaker says. Co 1. Meaning questions These are the most common questions (about 50%). They ask for a restatement of what the second speaker or both speakers say. They may be general questions or ask what the speakers say ahout some specific topic. They often follow dialogs that contain idioms. "What does the man/woman mean?" "What do the speakers say about ---?" 2. Inference questions These are the second most common Part A questions (about 20%). The answers for these questions are not directly stated in the dialog, but they can he inferred (concluded) from what the speakers say. "What does the man/woman imply?" "What can be inferred from the conversation about - - - ? " "What can be concluded from the conversation about---?" 3. Questions about suggestions Generally, the first speaker talks about a prohlem or asks for advice. The second speaker makes a suggestion for solving the problem. "What does the woman suggest the man do?" "What does the man suggest they do?" "What does the woman suggest?" "What does the woman think the man should do?" 4. Questions about future actions These ask what one or hoth of the speakers will do next or in the near future, or what one or hoth are planning to do. "What will the man do?" "What will they probably do next?" "What are the speakers planning to do?" Section 1 • Guide to Listening Comprehension 13 5. Topic questions These ask about the subject of the dialog. "What are they talking about?" "What are they discussing?" 6. Questions about opinions These ask how one or both of the speakers feel about some topic. "How does the man/woman feel about ---?" "What is their opinion of ---?" 7. Questions about assumptions These ask what the second speaker thought (assumed) before he or she spoke to the first speaker. "What had the man assumed about ---?" "What had the woman previously assumed?" 8. Questions about questions The first speaker makes a statement; the second speaker asks a question to get more information. "What does the man want to know?" 9. Questions about time These ask when a conversation is taking place or when an event the speakers mention in the conversation will take place. "When is this conversation taking place?" "When will the - - - take place?" 10. Questions about reasons These ask why one or both of the speakers did something. "Why did the man/woman ---?" 11. Questions about problems These ask about some trouble one or both of the speakers are having. "What problem is the man having?" "What is the problem?" 12. Questions about activities These ask what one or both of the speakers are doing. "What are the speakers probably doing?" Note: Two types of questions that were commonly asked about dialogs in the past are seldom or never asked about in the new-format test. These are location questions ("Where does this conversation probably take place?") and occupation questions ("What is the man's occupation?" or "Who is the man?"). The Answer Choices All four of the answer choices are logical answers for the question, but only one-the key-is correct according to the dialog. However, as in all parts of the TOEFL, not all of the answer choices are equally attractive. You can often eliminate one or two choices easily even if you are not sure which answer is correct and so make a better guess. Correct answers are seldom stated word for word by either of the speakers. Correct answers often contain synonyms (words with the same meaning) for words in the dialogs and use different sentence structures. Grammatically, there are three types of answer choices: 1. Complete sentences (about 75%) 2. Incomplete sentences, usually beginning with verb forms-most often the simple form of the verb (about 20%) 3. Short noun or prepositional phrases The form of the answer choice can sometimes help you guess what the question will be, and you can therefore focus your listening. 14 Section 1 • Guide to Listening Comprehension Meaning questions: "What does the man mean?" Complete sentences: (A) He prefers coffee to tea. (B) He'd like some lemon in his tea. Questions about inferences: "What does the woman imply about the article?" Complete sentences: (A) She will probably read it today. (B) She wasn't able to find it in the library. Questions about suggestions: "What does the woman suggest John do?" Incomplete sentences beginning with simple forms of verbs or -ing forms: (A) Call his cousin. (B) Take his cousin home. "What does the man suggest?" (A) Taking a bus to campus. (B) Walking to class. Questions about future actions: "What will the speakers probably do next?" Incomplete sentences beginning with simple forms of verbs: (A) Park their car. (B) Get some gasoline. Topic questions: "What are the speakers discussing?" Noun phrases: (A) The man's new schedule. (B) A homework assignment. Questions about opinions: "What was their opinion of the play?" Complete sentences or adjective phrases: (A) They didn't enjoy it very much. (B) They liked it more than they thought they would. "How does the man feel about the announcement he heard?" (A) Angry. (B) Enthusiastic. Questions about assumptions: "What had the man assumed about Kathy?" Complete sentences often containing the auxiliary verb would or had: (A) She had already finished the paper. (B) She wouldn't finish the research on time. Questions about questions: "What does the woman ask about Professor Tolbert?" Incomplete sentences beginning with the word if or one of the wh- words or noun phrases: (A) If she is still in her office. (B) Where her office is. "What does the man ask about the department store?" (A) (B) Its location. Its hours of operation. Questions about time: "When will the man play the piano?" Prepositional phrases of time: (A) At noon. (B) Before the ceremony. Questions about reasons: "Why did Jerry miss the party?" Complete sentences or incomplete sentences beginning with infinitives (to + simple form): (A) He didn't receive an invitation. (B) He had other plans for the evening. "Why did Linda talk to Professor Delgado?" (A) (B) To ask him about a grade. To explain why she missed class. Section 1 • Guide to Listening Comprehension 15 QU..ti()n Questions about problems: "What problem did the man have?" Complete sentences: (A) He didn't bring enough money for the tickets. (B) There were no tickets available. Questions about activities: "What are they probably doing?" Incomplete sentences beginning with -ing verbs: (A) Buying groceries. (B) Cooking breakfast. The test writers sometimes make it more difficult to pick the correct answer by using sound-alike words, homonyms, words with multiple meaning, and other techniques. You'll practice avoiding these traps in this part of the Guide. • Be familiar with the directions for Part A. • Remember that the answer for the question is generally contained in what the second speaker says. • If you are not sure of the answer, eliminate as many answer choices as you can. • After you have chosen an answer, use the remaining time to preview the choices for the next item. If the answer choices are long, just skim over them quickly. Try to anticipate what the question will be by the form of the answer choices. • If you don't understand all or part of a conversation, guess and go on. 16 Section 1 • Guide to Listening Comprehension LESSON 1 ANTICIPATING QUESTIONS ABOUT DIALOGS Between each dialog in PartA, there is a twelve-second pause. During the pause, here's what you should do: • Answer the question you have just heard as quickly as you can. • Preview the choices for the next item. A look at the answer choices may tell you the topic of the upcoming dialog and what question will be asked about it. Consider the answer choices below: (A) Before she leaves her dormitory. (B) During chemistry class. (C) After the lab period. (D) While she's eating lunch. Even a quick glance will tell you that the dialog must be about a student's schedule and that the question will begin, "When. . . :' If you have an idea of the topic of the dialog and if you know what the question about the dialog will be, you will know what to listen for, and your listening task will be easier. Exercise 1 Focus: Guessing which type of question will be asked about dialogs by looking at the four answer choices. Directions: QUickly look over the five Part A items in each set. Try to guess the topic ofthe dialog and the type of question that would be asked about it. Then look at the list of questions following each set of items. Put the letter of the appropriate question in the blank provided. One question in each set will not be used. The first one is done as an example. Note: There is no audio material for this exercise. SetA 1. (A) Pleased. (B) Cold. (C) Disappointed. (D) Hungry. Question: b 2. (A) Go to her office. (B) Call a taxi. (C) Show the man where to find a taxi. (D) Get directions. Question: 3. (A) Cloudy but much warmer. (B) Rainy. (C) Clear but cold. (D) The same as today's. Question: 4. (A) He left it in the lock. (B) It's still in his dorm room. (C) He put it in his pocket. (D) It doesn't work in this lock. Question: 5. (A) Playing a game. (B) Attending a play. (C) Learning some lines. (D) Trying to find tickets. Question: a) What will tomorrow's weather probably be like? b) How does the man feel? c) What are these people doing? d) Where will the man go tomorrow? e) What does John say about the key? f) What will the woman do next? Section 1 • Guide to Listening Comprehension 17 Set B Set C 6. (A) He had given his textbooks to a friend. (B) He would receive more money. (e) He wouldn't get to the bookstore on time. (D) He hadn't sold his textbooks. Question: 7. (A) She enjoyed it very much. (B) She thought it was too long. (e) She liked it more than the movie reviewer did. (D) She found it confusing. Question: 8. (A) A tuition increase. (B) A policy change. (e) A new class. (D) A recent proposal. 11. (A) His brother helped him move the piano. Question: 9. (A) Buy some new software. (B) Get her computer ftxed. (e) Use the computers at the library. (D) Borrow his computer. Question: 10. (A) He doesn't have the right notebook. (B) He forgot to bring a pen. (e) He went to the wrong lecture hall. (D) He was late for the lecture. Question: a) What did the woman think about the movie? b) What is the man's problem? c) What does the man suggest Ann do? d) What had the woman assumed about the man? e) What information does the man want? 1) What are they discussing? (B) He moved the piano to his brother's house. (e) His brother taught him to play the piano. (D) He and his brother hired professional movers. Question: 12. (A) To ask her a question. (B) To get her advice. (C) To give her a suggestion. (D) To disagree with her idea. Question: 13. (A) Who Katie is. (B) What was said. (e) When Katie called. (D) What the problem was. Question: 14. (A) She's probably an expert on modern art. (B) She didn't paint the picture herself. (e) She's just begun to study painting. (D) She probably doesn't like modern art. Question: 15. (A) At the beginning of the spring semester. (B) During spring break. (C) During ftnal exams. (D) Right after fmal exams. Question: a) How does the man feel about the woman's remark? b) Why did the man call Professor Wilkey? c) What does the woman want to know? d) When does this conversation take place? e) What does the man mean? 1) What can be inferred about the woman? 18 Section 1 • Guide to Listening Comprehension LESSON 2 DIALOGS WITII SOUND CONFUSION Some of the items in Part A involve a confusion between words that have similar sounds. Here's how they work: one of the speakers uses a word or phrase that sounds like a word or phrase in one or more of the answer choices. If you don't hear the word clearly, you might incorrectly choose an option with a sound-alike word or phrase. Sample Item You will hear: Ml: I've never had this type of fruit before. I don't even know what to do with it. Fl: You just have to peel it and eat it. M2: What does the woman mean? You will read: (A) (B) (C) (D) She doesn't feel like eating fruit. The man should take the pill before eating. The fruit shouldn't be eaten until it's been peeled. She isn't familiar with this type of fruit either. The word feel in choice (A) sounds like the word peel in the dialog. In a different way, the word pill in choice (B) also sounds like the word peel. Notice that choice (C)-the correct answer-and choice (D) do not contain sound-alike words. Many sound-alike expressions in Part A are minimal pairs. Minimal pairs are two words that are pronounced alike except for one vowel sound (peel and pill, lack and lake, point and paint) or one consonant sound (peel andfeel, vine and wine, mop and mob). Another sound problem involves two words that sound like one word, such as mark it and market, sent her and center, in tents and intense. A third type of sound problem involves one word that sounds like part of a longer word, such as nation and imagination, mind and remind, give andforgive. Hint: If an answer choice contains a word that sounds like a word in the spoken sentence, that choice is probably wrong. For example, if you hear the word spell and you read the word spill in an answer choice, you can eliminate that choice. When you're taking Part A during an actual exam, you can use the context of the dialogs to help you solve problems with sound confusion. If you hear and understand all of the dialog, you won't have much trouble eliminating choices involving sound-alike words. However, if you only understand part of a dialog or if you "mis-hear" one or two words, you may easily choose an incorrect answer. Exercise 2.1 Focus: Discriminating between sound-alike words in dialogs and answer choices. Directions: Listen to the dialogs. Decide which of the two choices, (A) or CB), best answers the question, and mark the appropriate blank. The first one is done as an example. -4J)) Now start the audio. Section 1 • Guide to Listening Comprehension 1. (A) _ _ (B)~ Get in a different lane. Stand in another line. Go down the slide. (B) _ _ Play on the sled. 2. (A) _ _ Its taste has improved. (B) ____ It tastes slightly bitter. 8. (A)_._ How much the ticket cost. (B) ___ What Ellen might win. 9. (A) ____ 3. (A) _ _ 10. (A) _ _ _ 4. (A) _ _ _ 11. (A) _ _ (B) _ _ He tripped in the aisle. He slipped in some oil. 12. (A) _ _ (B) _ _ For its fast horses. For its natural resources. Put them in a file. (B) _ _ Throw them in a pile. He can't shut his suitcase. (B) _ _ His suitcase doesn't fit in the closet. 5. (A) ____ She made bread from whole wheat. (B) __ --- She baked some white bread. 6. (A) _____ It's being audio. (B) __ Brenda is typing it. 7. (A) _ _ _ (B) _ _ Emily bought new clothes. Emily recently moved. 19 It's been chipped. <..B) _ _ There's a ship inside it. Thinking about the decision. (B) _ _ Arguing about the issue. 13. (A) _ _ 14. (A) _ _ (B) _ _ The color is too bright. It doesn't fit around the neck. 15. (A) _ _ (B) _ _ Wrote his name on the paper. Told his students to write a paper. Exercise 2.2 Focus: Identifying sound-alike expressions in answer choices and choosing correct answers. l1li))) Directions: Listen to the dialogs. Each dialog contains a word or phrase that sounds like a word or phrase in two of the answer choices. Underline these words. Underline only those words with similar sounds, not words that are exactly the same. Then mark the answer choice that has the same meaning as the spoken sentence. (The correct answer will not contain any sound-alike words.) The first one is done as an example. Now start the audio. If necessary, repeat this exercise to make sure that you have underlined all the sound-alike words. 1. (A) ___ She went to the center with her friend. (B) ~ She wrote her friend a letter. (C) ___ She told her friend to call her later. 2. (A) ___ He has an appointment with the president. (B) _ _ _ He was just appointed vice-president. (C) _ _ _ He's unhappy because he lost the election. 3. (A) ___ It is a study of the life of plants. (B) _ _ _ It concerns the breeding of cattle. (C) ___ It deals with life on Earth. 4. (A) ___ They can't leave until the rain is over. (B) _ _ _ Their drain has stopped up. (C) ___ He shouldn't board the train until it completely stops. 5. (A) ___ He offered his help to Darlene. (B) ___ He made an offer to Darlene's sister. (C) ___ When Darlene was gone, he missed her. 20 Section 1 • Guide to Listening Comprehension 6. (A) ____ _ (R) L (C) He didn't hear what the woman said. He can lend the man a pen. He had a pain behind his ear. 7. (A) (B) '--.-- (C i 8. (A; (ll) _._ (C 9. (AI , > ' (B! (C) Get a copy made. Buy some cough drops. Eat in the coffee shop. , The food in this town isn't very good. She needed boots when she left home. The flooding in her neighborhood was severe. She's been weakened by the sickness. She was awakened by the coughing. She missed class because of her cough. 10. (A) (B) (C) __ Evaluate the texts. Correct the exams. Collect the tests. II. (A) __ His apartment is more comfortable now. He recently bought a new van. He's been feeling fine lately. (B) __ (C) 12. (A) _ _ Her name is not on the list. (B) ___ The lease is difficult to read. (C) ___ The lawyer told her to call the police. Section 1 • Guide to Listening Comprehension 21 LESSON 3 DIALOGS WITH HOMONYMS AND WORDS WITH MULTIPLE MEANINGS Two words are homonyfilS if they have the same pronunciation but are spelled differently and have different meanings. The words flour and flower, bare and bear are homonyms. In some items in Part A, one or more incorrect answer choices refer to a homonym of a word that is used on the audio, as in the example below. Sample Item You will hear: Ml: Eugene missed a lot of classes last week. Fl: That's because he was sick. I think he had the flu. M2: What is learned about Eugene? You will read: (A) He has been feeling weak for a long time. \j:B) Because of sickness, Eugene was absent. (C) Eugene's eyesight isn't very strong, so he needs glasses. CD) Eugene flew to another city this week. The dialog contains the word week, meaning a seven-day period. Choices (A) and CC) refer to a homonym of that word, weak, which means "not strong." The dialog also contains the word flu, an illness similar to a bad cold. Choice (D) refers to a homonym of that word, flew (took a trip by plane). --------------------------------------------- The dialogs may also contain words with multiple meanings. In these items, one or two of the answer choices refer to another defmition of a word as it is used in the dialog. Sample Item You will hear: Fl: Are you sure this is how Lois spells her last name? Ml: It doesn't look right, does it? In fact, I'm not even sure it starts with that letter. M2: What does the man mean? You will read: (A) CB) (C) CD) The letter to Lois was incorrectly addressed. Lois's last name may be incorrectly spelled. Lois's name appeared on the rigl1t side of the page. Lois hasn't begun writing the letter yet. The dialog contains the words right, meaning "correct," and the word letter, meaning a character in the alphabet. Choices CA) and (D) also contain the word letter, but in those choices the word has another defmition-a message sent through the mail. Choice (C) also contains the word right, but in that choice, it refers to a direction-the opposite of left. You won't be confused by these items if you uns;:lerstand the entire dialog. Again, the context of the dialog can help you choose the correct answer. But if you focus only on single words, like week and flU or letter and right in the two samples, you can easily make mistakes. 22 Section 1 • Guide to listening Comprehension Exercise 3.1 Focus: Using the context of dialogs to identify homonyms. Directions: Listen to the dialogs. Decide which of the pair of homonyms appears in the dialogs and mark the appropriate answer, CA) or (B). The first one is done as an example. ~))) Now start the audio. 1. _ _ CA) presence ~ (B) presents ,,/7. _ _ CA) board \ _ _ (B) bored 2. _ _ CA) overdue - - ( B ) overdo 3. _ _ CA) pain _ _ (B) pane 8. _ _ CA) brakes ~(B) breaks ."'-., ' 9. _ _ CA) sail *-- (B) sale f " 4. _ _ CA) where - ' - ( B ) wear <./ 10. _ _ CA) site _ _ (B) sight 5. _ _ CA) fmed _ _ CB) fmd lt~ ." 11. _ _ CA) rose - - ( B ) rows 6. _ _ CA) right - - ( B ) write 12. _ _ CA) aloud _ _ (B) allowed ,. Exercise 3.2 Focus: Using the context of dialogs to identify the defmitions of words with multiple meanings. Directions: Listen to the dialogs. One word from the dialog is given, along with two possible definitions of the word. Choose the definition of the word as it is used in the dialog and mark the appropriate answer, CA) or (B). The first one is done as an example. IIIIIIII))) Now start the audio. 1. cold ~CA) minor illness - - ( B ) chilly weather 2. kind _ _ CA) type '-.." - - ( B ) considerate 3. light _ _ CA) not heavy " - - ( B ) not dark , ~. ,-/ , \~)< 6. coat _ _ CA) layer _ _ (B) warm clothing 7. field ~CA) outside the classroom -_(B) area of study 8. playing _ _ CA) taking part in a game X;,...-/ _ _ CB) appearing 4. wing _ _ CA) part of an airplane - - - ( B ) part of a building V_ CA) 5. tables _ _ CA) charts _ _ CB) furniture 10. period _ _ CA) punctuation mark _ _ (B) class time V/ 9:. party celebration group --(B) Section 1 • Guide to Listening Comprehension Exercise 3.3 Focus: Using the context of dialogs to answer questions involving both homonyms and words with multiple defInitions. Directions: Listen to the statements. Decide which of the two choices best answers the question and mark the appropriate answer, (A) or (B). The fIrst one is done as an example. -4))) Now start the audio. 1. ~ (A) Look for mistakes. _ _ _ (B) Write a check. 2. _ _ (A) Events in the past. _ _ _ (B) The man's performance in class. , 3. _ _ (A) He'd never heard buffaloes before. ~ ___ (B) This was the fIrst herd he'd ever seen. 4. ______ (A) Follow the directions on the sign. _____ (B) Sign up for another class. , 5. ____ (A) Buy a second suit. ___ (B) Consider it for a little while. .~' 6. ____ (A) He can't carry the luggage by himself. _ . _ (B) The handle on the suitcase is broken. 7. _. __ (A) He was surprised by the rain. ____ (B) He just got out of the shower. 8. ___ (A) The class has a better opinion of him. ____ (B) He was standing in front of the class. 9. ___ (A) She works in an offIce by herself. _"_ (B) She's in charge of making loans. V,10. ____ (A) She's sorry the seminar is over. _ _ _ (B) She was often absent from the seminar. Y 11. ___ (A) They can park their car at the zoo. __._ (B) The park is located near the zoo. e/ 12. ___ (A) If she has some money for a phone call. _ _ _ (B) If her phone number has changed recently. 23 24 Section 1 • Guide to Listening Comprehension LESSON 4 DIALOGS WITH IDIOMATIC EXPRESSIONS On many TOEFL exams, up to half the dialogs in PartA contain idiomatic expressions. Many of the idiomatic expressions are two- or three-word verbs, such as call off and look out for. Sample Item You will hear: Fl: I wonder where Mike is. Ml: He'll show up as soon as the work is done, I bet. M2: What does the man say about Mike? You will read: (A) He probably won't arrive until the work is fInished. (B) He went to a show instead of going to work. (C) He can show them how to do the work. (D) He'll probably work late today. The idiom show up means "arrive." Choices (B) and (C) contain the word show, but it is not used in the idiomatic sense. In most dialogs, the second speaker uses the idiomatic expression. Most questions about this type of dialog are questions about meaning ("What does the man mean?" for example), but some are inference questions or other types of questions. The correct answer often contains a synonym for the idiom (arrive for show up in choice (A) of the Sample Item). Incorrect choices often contain references to the literal meaning of idioms, as in choices (B) and (C). The Mini-Lessons for Section 1, at the end of the Listening Comprehension section (pages 81100), are intended to familiarize you with a large number of idioms. You should work on these lessons and study these expressions as often as possible. However, memorizing these phrases does not guarantee that you will recognize all the idiomatic expressions that you will hear in the Listening Comprehension section. There are, after all, thousands of these expressions in English. You must develop "a good ear" for guessing the meaning of idioms. The context of the sentence will help you understand the expression, even if you're unfamiliar with it. Exercise 4.1 Focus: Recognizing synonyms for idiomatic expressions. Directions: Listen to the spoken statements. Each contains an idiomatic or fIgurative expression which is written out. First decide which of the two choices best answers the question and mark the appropriate answer, (A) or (B). Then underline the phrase in the correct answer that has the same meaning as the idiom. If necessary, rewind the audio and listen to the exercise again. The fIrst one has been done as an example. -4))) Now start the audio. 1. get into hot water --1L- (A) She was in trouble. _ _ _ (B) She took a warm bath. Section 1 • Guide to Listening Comprehension J r~,n 25 into (A) He met Caroline unexpectedly at the coffee shop. __ (B) Caroline and I jogged to the coffee shop. 3. hit ,; ,)it" . . . (A) ~_v._ (B) He and Chuck argued as soon as they met. He and Chuck quickly became friends. 4. a piece of cake _ . _ (A) The exam was simple. \ _ . . (B) She had a snack after the test. 5. at thl..' drop of a hat __ .~ (A) He can't leave until he finds his hat. ___ (B) He's ready to leave immediately. 6. on edge _ . __ (A) _ _ (B) He walks back and forth when he's nervous. He likes to walk along the edge. 7. under the weather _ _ (A) She didn't want to practice because of the bad weather. _ ......_ (B) She ,vasn't there because she felt a little sick. 8. take after _\.__ (A) He looks like his grandfather. , _~\_'_ (B) He takes care of his grandfather. ; 9. for good ___ (A) He doesn't want the professor to quit teaching permanently. ___ (B) He hopes Professor Holmes has a good reason for quitting. ~. 10. give a hand with ___ (A) Hand her the box . . ___ (B) Help her carry the box. 11. a stone's throw from \. ___ (A) He likes to throw rocks in the park. ___ (8) He lives close to the park. 12. not think much of '. ___ (A) She didn't consider it. ___ (B) She didn't like it. Exercise 4.2 Focus: Understanding dialogs involving idiomatic and figurative expressions. Directions: Look over the idiomatic expressions listed before each set of items. If you are unfamiliar with any of the idioms, you may want to look them up in the Mini-Lessons for Section 1 that follow the Listening Comprehension section of this book (pages 81-100). The dialogs each contain one of the listed expressions. Listen to the dialogs and mark the one answer choice, (A) or (B), that best answers the question. The first one has been done as an example. I11III))) Now start the audio. 26 Section 1 • Guide to Listening Comprehension Set A get off the ground run of the mill clear llP push one's luck ~. ~ \/ hours on end short for over one's head turn in (A) He's not sure Max's business will succeed. He doesn't know where Max has gone. _ _ (B) 2 .. _"_ _ (A) Gary is lucky to have such a good car. It's time for Gary to get some new tires. ,\-/ _ _ (B) 3./_·_ _ (A) Go to bed. ,~ _ _ (B) Watch a different program. \A. _ _ (A) She didn't understand all the jokes. She left before the performance was over. _ _ (B) 0. - - (A) If the weather gets better. _ _ (B) If she doesn't have any other plans. \ft.. - - (A) Elizabeth is taller than Liz. _ _ (B) People call Elizabeth "Liz." \). _ _ (A) She's stopped listening to it. _ _ _ (B) She listens to it constantly. 8. _ _ (A) The service is very fast there. _ _ _ (B) It's just an average restaurant. ~. Set B believe one's eyes lend a hand a breeze look who's talking chip in music to one's ears \.,.9:'/_- (A) Ice water sounds perfect. _ _ (B) The doctor told her to drink a lot of water. 00. --- (A) Skiing can be a dangerous sport. _ _ _ (B) It's easy to get into the habit of skiing. J}:/_- (A) She enjoys the sound of nature. _ _. (B) She wishes she'd brought a radio. \J2. _ _ (A) She wants to talk to Norman. _ _ (B) Norman doesn't study much himself. \.,13": ___ (A) There's not enough wind to go sailing today. ___ (B) It won't be too hard to learn to sail. \...,.l~ _ _ (A) Lend him some money. _ _ (B) rS'. __ (A) \/ Give him some help. They'll all pay for the gasoline. _ _ (B) Gasoline is very inexpensive. , 1f./ _ _ (A) She doesn't think the man is telling the truth. She was surprised to see the snow. V . --- (B) get in one's blood what the doctor ordered Section 1 • Guide to Listening Comprehension Set C by heart ring a bell call it a day slowly but surely come around take a lot of nerve go without saying take into account 17. ___ (A) She seems too nervous. -"--___ (B) She took a bold approach. 18. ~_ _ ./ ___ (B) (A) He doesn't want to do any more painting today. He'll phone the woman later today. 19. ____ (A) She spoke the lines in an emotional way. / _ ( B ) She's memorized all the lines. v:20. ~_(A) At some point, they'll agree to let her go. ~/_/_~ (B) They'll come with her to Alaska. 2 . •/ Y. __ (A) Rob Martin hasn't called him yet . ___ (B) He doesn't think Rob Martin was on the team . 22. _ _ (A) He didn't count his money carefully. ___ (B) He forgot about the tax. 23. ___ (A) She's making steady progress. ___ (B) She thinks the work is going too slowly. 24. ___ (A) Of course she was sorry that Molly left. "---/ ___ (B) Molly left without saying goodbye. Exercise 4.3 Focus: Using the context of dialogs to understand the meaning of idioms. Directions: Listen to the following dialog. Decide which of the choices-(A), (B), or (C)-best answers the question about the dialog and mark the appropriate answer. The first one is done as an example. -4))) Now start the audio. 1. ___ (A) Go to work with Jim. ___ (B) Go out for coffee. ~ (C) Get some exercise. 2. ___ (A) If the woman will go to the party with him. ___ (B) If the red tie looks good with his shirt. ___ (C) If he should wear a tie to the party. 3. _ _ (A) She missed Friday's class, too. ___ (B) They both missed class because they went sailing. _ _ (C) He should take better notes during Professor Morrison's class. . '-> // 4. _ _ (A) He cut himself while he was preparing food. _ _ (B) He doesn't want to work in a restaurant. , ,- - - - - (C) He's planning to open up his own restaurant. 5. _ _ (A) He wants to know if the woman is joking. - - ( B ) He wants the woman to leave him alone. _ _ (C) He'd like to know what the quiz will be about. 6. _ _ (A) The program was canceled. _ _ (B) The shuttle was launched yesterday. _ _ (C) The launch was delayed. 27 28 Section 1 • Guide to Listening Comprehension 7. _ _ (A) She stood up and left the lecture. _ _ (B) l I.- She was waiting outside the lecture hall. _ _ (C) Her sweater made her easy to spot. 8. _ _ (A) He deserved to get a ticket. _ _ (B) He was going to a good restaurant. _ _ (C) He probably wasn't speeding. 9. _ _ (A) He'll be glad to help. - - ( B ) If he helps, it will save the man some money. L- _ _ (C) He won't be very cooperative. 10. _ _ (A) It's about buying large real estate properties. _ _ (B) There are too many students in his class. _ _ (C) In general, he likes his real estate class. 11. _ _ (A) The man didn't get Jill a watch. The weather will be coolon graduation day. _ _ (C) Jill won't be graduating. --(B) 12. _ _ (A) She ordinarily works in a florist shop. L In the end, she won't have a problem. _ _ (C) She wears too much perfume to work. --(B) 13. _ _ (A) She doesn't want any fruit. She doesn't want to celebrate her birthday. _ _ (C) She doesn't like candy. --(B) 14. _ _ (A) He doesn't have any questions for her. He won't be able to take a trip. _ _ (C) He can't study during spring break. --(B) 15. _ _ (A) Mick's father told him to go to medical school. '1-__ • _ _ (B) Mick's father studied medicine. _ _ (C) Mick and his father walked to the school. 16. ___ (A) Fred would be upset if he'd lost money. _ _ (B) Fred shouldn't be paid for singing. _ _ (C) Fred is generally very sympathetic. 17. _ _ (A) If Wally has been injured. '- _ _ (B) IfWally has been informed. _ _ (C) If Wally's trip has been canceled. 18. _ _ (A) He can't fmd some of his pictures. _/ He didn't go to the Grand Canyon. _ _ (C) Not all of his photos were good. --(B) 19. _ _ (A) She thinks they're certain to do well. _ _ (B) She thinks they're talented but lack experience. _ _ (C) She doesn't like their style of photography. 20. _ _ (A) The lake is not very scenic. Her parents won't let them use the cabin. _ _ (C) The cabin is not luxurious. --(B) \,....-- Section 1 • Guide to Listening Comprehension 29 LESSON 5 ANSWERING INFERENCE QUESTIONS ABOUT DIALOGS Sometimes the answer to a question about a dialog is not directly stated in the dialog. How can you answer this type of question? You must be able to make an inference about the dialog. In other words, information in the dialog will indirectly provide you with the answer to the question. This type of question can be phrased in two ways: • What does the man/woman imply? • What can be inferred from the conversation? Some inference questions involve overstatement, or exaggeration. F: Are you interested in selling your car? M: Sure-if someone has a million dollars! Because of the exaggeration, we can infer that the man doesn't want to sell his car at all. Sample Item You will hear: M 1: Can I take this bus to the art museum? Fl: No, this bus goes north to Bank Street. You want a bus that goes the opposite way. M2: What can be inferred from this conversation? You will read: (A) The man needs to take a southbound bus. (B) There is no bus to the museum. (C) It takes a long time to get to the museum by bus. (D) The art museum is on Bank Street. From the information in the dialog it can be inferred that (A) is correct because the first bus is going north, but the man must take a bus going in the opposite direction to get to the art museum. Choice (B) is incorrect; it is possible to get to the museum by bus. There is no information about (C). Choice (0) can't be true because Bank Street is where the first bus is going. Exercise 5 Focus: Listening to dialogs that are followed by inference questions and identifying the best answers. Directions: Listen to the following dialogs. Decide which of the three choices-(A), (B), or (C)best answers the question and mark the appropriate answer. The first one has been done as an example . .-))) Now start the audio. 1. ___ (A) He's not related to Larry. ___ (B) He doesn't believe Larry won the contest. ~ (C) He's not a very good dancer. 30 Section 1 • Guide to Listening Comprehension 2. ___ (A) The suit costs a lot of money. ___ (B) The man dresses as if he were very wealthy. _ _ _ (C) The man already has an expensive suit. 3. ___ (A) There is just enough food. ___ (B) Many uninvited guests will come. _ _ _ (C) The woman has prepared too much food. 4. ___ (A) Dave is a painter. ___ (B) Dave's apartment has been recently painted. _ _ ~ (C) Dave's brother doesn't like the smell of paint. 5. ___ (A) He's changed his major often. ___ (B) He hasn't really changed his major. _ _ _ (C) He won't do well in his new major. 6. ___ (A) His lectures put his students to sleep. ~ (B) He's a middle-aged man. _ _ _ (C) He lectures about history. 7. _____ (A) He hasn't been to the dentist for years. _ _ (B) He wasn't able to see the dentist yesterday. ____ (C) Before he saw the dentist, he had a long wait. 8. ____ (A) They have agreed on it. ____ (B) They have different opinions about it. ____ (C) It depends on their co-operation. 9. ___ (A) Louis's new boss shouldn't have been promoted. ___ (B) Louis and his old boss argued. _ _ (C) Louis should get a better job. 10. ___ (A) There's not enough snow to cause a cancellation yet. _ _ (B) It will probably snow all night. _ _ (C) The university has already decided to cancel classes. 11. ___ (A) He's been interested in folk dancing for a long time. ___ (B) He's interested in making new friends. _ _ _ (C) He wants to form a new folk-dancing club. 12. _ _ (A) She didn't enjoy the music. _ _ (B) She couldn't see the concert very well. _ _ _ (C) She had a good seat near the stage. 13. ___ (A) Last summer was even hotter. ___ (B) This is the hottest summer he can remember. _ _ _ (C) He didn't live here last year. 14. ___ (A) Students must pay to swim in the pool. _ _ (B) The public cannot use the pool on campus. _ . _ (C) The pool can be used by students for free. 15. _ ~_ (A) They can't see the stars clearly. ___ (B) They're not in the city tonight. _ _ _ (C) They are looking at the lights of the City. 16. ___ (A) He doesn't know many people at work. ___ (B) He wasn't expecting a phone call. _ _ _ (C) He's not allowed to get phone calls at work. Section 1 • Guide to Listening Comprehension 17._ 18. _ __ (A) Those aren't Shelly's photographs. __ (B) Shelly has begun to take color photographs. ___ (C) Shelly took the photographs hanging in the hall. (A) That scarf looks great on Fran. _ (B) Fran wears that scarf too often. (C) In this weather, Fran needs a scarf. 19. __.__ (A) She doesn't have an accent. _______ (B) Her parents have very strong accents. ___ (C) Her accent is stronger than her parents'. 20. ___ (A) Robert usually has trouble skiing. _ _ (B) That's not a difficult slope. ___ (C) Robert is an excellent skier. 31 32 Section 1 • Guide to Listening Comprehension lESSON 6 DIALOGS INVOLVING AGREEMENT AND DISAGREEMENT To answer questions about some of the dialogs in PartA, it is necessary to understand if the second speaker agrees or disagrees with the first speaker's ideas or proposals. There are many ways to express agreement and disagreement: So do I. I'll say! Me too. You can say that again. Neither do 1.* Is!Has!Was it ever! I don't either. * You bet! Who wouldn't? I couldn't agree with you more. Isn't he/she/it though! (Didn't he/wasn't she!hasn't it though!) I feel the same way you do about it. I'll second that. *Tbese two expressions show agreement with a negative statement: I don't really like my schedule this term. I don't either. OR Neither do I. I don't think so. I'm afraid I don't agree. That's not what I think. Probably not. That's not the way I see it. Not necessarily. Not really. I'm afraid not. I can't say I agree. I'm not so sure. I couldn't agree with you less. There are, of course, other expressions that show agreement and disagreement. Some are practiced in the exercises. Sample Items You will hear: Ml: Howard certainly is a talented journalist. Fl: Isn't he though! M2: What does the woman mean? You will read: (A) She doesn't know if Howard is a journalist. (B) She agrees that Howard is talented. (C) She read Howard's journal. (D) She doesn't think Howard is talented. Section 1 • Guide to Listening Comprehension 33 Although the woman's reply seems negative in form, it actually signals agreement. Therefore, the best answer is (B). You will hear: FI: I thought Cheryl's photographs were the best at the exhibit. MI: I didn't really see it that way. M2: What does the man mean? You will hear: CA) He thought Cheryl's photos were the best. (B) He didn't look at Cheryl's photos. CC) He thought other photos were better than Cheryl's. CD) He didn't go to the exhibit. The man's response, "I didn't really see it that way;' means that he disagreed with the woman's opinion that Cheryl's photographs were the best. The best answer is therefore CC). Exercise 6.1 Focus: Determining if one speaker agrees or disagrees with the other speaker. Directions: listen to the following dialogues. Decide if the second speaker agrees or disagrees with the first speaker, and mark the appropriate blank. The first one is done as an example. III))) Now start the audio. Agrees Disagrees 7. ___ CA) Agrees ___ (B)- Disagrees 2. _____ CA) Agrees - - ( B ) Disagrees 8. ___ CA) Agrees _ _ CB) Disagrees 3. ______ CA) Agrees ____ CB) Disagrees 9. ___ CA) Agrees _ _ CB) Disagrees 1. ~CA) --(B) 4. _____ CA) Agrees ----- (B) Disagrees 5. ____ CA) Agrees ______ CB) Disagrees 6. _____ CA) Agrees --(B) Disagrees 10. _ _ CA) Agrees - - ( B ) Disagrees 11. _ _ CA) Agrees - - ( B ) Disagrees 12. _ _ CA) Agrees - - - (B) Disagrees Exercise 6.2 Focus: listening to dialogs that involve agreement and disagreement, and answering questions about them. Directions: listen to the following dialogs. Decide which choice-CA), (B), or CC)-best answers the question and mark the appropriate answer. The first one is done as an example. III))) Now start the audio. 34 1. Section 1 • Guide to Listening Comprehension ~ (A) He prefers taking a final exam. He's thinks an exam takes too much time. ___ (C) He'd rather write a research paper. _ _ _ (B) 2. ___ (A) It was difficult, but she understood it. _ _ _ (B) It wasn't very useful. ___ (C) It's probably easier than the other chapters. 3. ___ (A) He completely disagrees with it. _ _ _ (B) He doesn't believe the university will accept it. _ _ (C) He thinks it's a good one. 4. _ _ (A) She doesn't think that Jack wrote it. _ _ (B) She thinks it had too many details. _ _ (C) She found it well-written. 5. _ _ (A) He thinks it's a good day for bike riding, too. _ _ _ (B) He doesn't agree with the woman's opinion of the weather. ___ (C) He didn't hear what the woman said. 6. ___ (A) Arthur wasn't doing well in the class. _ _ _ (B) She's not sure why Arthur dropped the class. _ _ (C) She believes Arthur dropped the class for no reason. 7. _ _ (A) It might work. _ _ (B) It's very impractical. _ _ (C) It's unnecessary. 8. _ _ (A) He didn't understand it. _ _.. (B) It made him angry. _ . _ (C) He agreed with it. 9. ___ (A) She's never been there during final exam week. _ _ _ (B) It's crowded because students will be taking exams soon. ___ (C) It's not crowded now, but it soon will be. 10. _ _ (A) He likes the costumes Madelyn made. _ _ _ (B) He wouldn't recommend the play. _ _ (C) He doesn't think the costumes are attractive. 11. _ _ (A) It was very happy. _ _ (B) It was exciting. _ _ (C) It was unhappy. 12. _ _ (A) She thinks Pamela is right. _ _ (B) She thinks the regulations are fair. _ _ (C) She disagrees with the man's opinion. Section 1 • Guide to Listening Comprehension 33 Although the woman's reply seems negative in form, it actually signals agreement. Therefore, the best answer is (B). You will hear: F1: 1 thought Cheryl's photographs were the best at the exhibit. M1: I didn't really see it that way. M2: What does the man mean? You will hear: (A) He thought Cheryl's photos were the best. (B) He didn't look at Cheryl's photos. (C) He thought other photos were better than Cheryl's. (D) He didn't go to the exhibit. The man's response, "I didn't really see it that way," means that he disagreed with the woman's opinion that Cheryl's photographs were the best. The best answer is therefore (C). Exercise 6.1 Focus: Determining if one speaker agrees or disagrees with the other speaker. Directions: Listen to the following dialogues. Decide if the second speaker agrees or disagrees with the first speaker, and mark the appropriate blank. The first one is done as an example. ~»)) Now start the audio. 1. ~(A) _ _ (B) Agrees Disagrees 2. ____ (A) Agrees 7. ___ (A) Agrees ___ (B)- Disagrees Disagrees 8. ___ (A) Agrees - - ( B ) Disagrees 3. ______ (A) Agrees _____ (B) Disagrees 9. ___ (A) Agrees _ _ (B) Disagrees 4. ____ (A) Agrees _____ (B) Disagrees 10. _ _ (A) Agrees - - ( B ) Disagrees 5. _______ (A) Agrees _____ (B) Disagrees 11. _ _ (A) Agrees _ _ (B) Disagrees 6. ___ (A) Agrees -----(B) Disagrees 12. ___ (A) Agrees ___ (B) - - ( B ) Disagrees Exercise 6.2 Focus: Listening to dialogs that involve agreement and disagreement, and answering questions about them. Directions: Listen to the following dialogs. Decide which choice-(A), (B), or (C)-best answers the question and mark the appropriate answer. The first one is done as an example. ~»)) Now start the audio. Section 1 • Guide to Listening Comprehension LESSON 7 DIALOGS INVOLVING SUGGESTIONS, INVITATIONS, OFFERS, AND REQUESTS A number of dialogs in Part A involve a speaker making and/or responding to suggestions, invitations, offers, and requests. There are many ways to express these language functions. Some are listed in the charts in this lesson, while others are practiced in the exercises. A) Suggestions These are pieces of advice that one speaker gives another. In most dialogs, the first speaker poses a problem and the second speaker suggests a possible solution to that problem. In some dialogs, the first speaker makes a suggestion, and the second speaker responds to that suggestion positively or negatively. Why don't you/we ... If I were you ... Why not ... If I were in your shoes ... Have you ever thought of ... You/We should .. . You/We might want to .. . Shouldn't you/we .. . YoujWe could always .. . What about ... Maybe you/we could .. . What if you/we ... Try ... How about '" Why not! I don't think so. Good idea! I don't believe so. That's an idea. I already thought of that. Sounds good to me. I don't think that will work. By all means! Don't look at me! Why didn't I think of that? Can I take a rain check?* That's worth a try. Thanks, I'll give that a try. *This means, "Could we do this some other time?" 35 36 Section 1 • Guide to Listening Comprehension • Sample Item You will hear: M 1: I'm doing so poorly in math class, I think I'm going to have to drop it. F1: You know, Frank, you should talk to Professor de Marco before you do anything. He's given special help to lots of students who were having trouble. M2: What does the woman suggest Frank do? You will read: (A) Study with a group of students. (B) Drop his mathematics course. (C) Discuss the problem with the professor. (0) Take no action at this time. The woman suggests that the man talk to Professor de Marco because the professor has helped many students in the past. A. B) Invitations These are requests for someone to come somewhere or to take part in some activity. The tlrst speaker may invite the second speaker to do something and the second speaker responds, or the second speaker may invite the tlrst speaker to do something. Shall we ... Let's ... Would you like to .. . Do you want to ... Would you care to .. . Could you .. . Would you be able to ... Can you .. . Want to ... Yes, let's. I'm sorry, but .. . Sure, thanks. I'd like to, but .. . Sounds good. I'd love to, but .. . All right, I'd love to. Thanks a lot, but .. . I'd like that. That sounds nice, but ... What a great idea! I'll pass. Sure. Thanks for inviting me. Thanks for the invitation, but ... If you want me to. I don't think I'll be able to make it this time. Don't mind if I do. Section 1 • Guide to Listening Comprehension 'Y 37 Sample Item You will hear: Ml: Would you like to join us on Sunday? We're going to go on a picnic at the lake. Fl: I'd love to, but I have a test Monday, and I have to get ready for it. M2: What will the woman probably do on Sunday? You will read: (A) Study for a test. (B) Go on a picnic. (C) Take an exam. (D) Join a club. The man invites the woman to come to a picnic. The woman says that she'd love to go, but that she must study for a test she is taking Monday. (If the woman had accepted the man's invitation, Choice (B) would have been correct.) A C) Offers These are proposals to help someone or allow someone to do something. Either speaker in the dialog may make an offer. Let me ... Can I ... Shall I ... May I ... Would you like me to .. . Should I ... Do you want me to .. . I could ... That would be nice. I don't think so. Yes, please. I'm afraid not. Please do. That won't be necessary. Sure, thanks. Thanks anyway. Please don't. - - - - - - - 38 Section 1 • Guide to Listening Comprehension ~ Sample Item You will hear: Fl: Should I make reservations for dinner Friday night? M 1: Thanks anyway, but I've already made them. M2: What does the man mean? You will read: CA) CB) CC) CD) He can't go to dinner Friday night. Reservations won't be required. He made reservations earlier. He'd like the woman to make reservations. The woman offers to make reservations, but the man replies that he's already made them .... D) Requests To make a request is to ask someone to do something, or to ask for help or information. Would you .. . Will you ... Could you/I .. . May I ... Do you mind if ... Can you/l ... Would you mind if ... I'd be glad to. Sorry, but ... I'd be delighted. I'm afraid not. Sure thing. I'd like to, but ... Certainly. I wish I could, but ... Why not? *Actually, I do/would. If you want to. *I'm afraid I do/would. If you'd like. ,.As a matter of fact, I do/would. You bet. *Not at all. *Of course not. *Responses for "Do you mind if ..." or "Would you mind if ...") Section 1 • Guide to Listening Comprehension ... 39 Sample Item You will hear: Ml: I have to make one more phone call before I go. F2: Take your time. Would you just lock the office door when you finish? M2: What does the woman want the man to do? You will read: (A) Lock the office. (B) Finish his phone call quickly. (C) Tell her what time it is. (D) Look up a phone number. The woman requests that the man lock up the office. .A. Exercise 7.1 Focus: Identifying suggestions, invitations, offers, and requests and responses to them. Directions: Listen to the following dialogs. Decide which ofthe two choices best completes the sentence, and mark the appropriate space. The first one is done as an example. l1li))) Now start the audio. 1. The man is ... ~CA) declining an offer _ _ CB) making a suggestion 2. The woman is ... _ _ CA) accepting an invitation _ _ CB) making an offer 3. The woman is ... _ _ CA) declining an offer _ _ CB) making a suggestion 4. Mark is ... _ _ CA) rejecting a request _ _ CB) agreeing to a request 5. The woman is ... ___ CA) giving an invitation _ _ CB) making a suggestion 6. The man is ... _ _ CA) agreeing to a request - - ( B ) turning down an offer 7. Ed is probably going to ... _ _ CA) receive a suggestion _ _ CB) make an offer 8. The woman is ... _ _ CA) suggesting a solution - - ( B ) offering help 9. Cynthia is ... _ _ CA) giving an invitation - - ( B ) accepting an offer 10. The woman is ... ___ CA) declining an offer ___ CB) making a request 11. The man will probably ... _ _ _ (A) do what the woman suggests ___ CB) turn down the woman's invitation 12. Bob is ... ___ CA) agreeing to an offer ___ CB) refusing a request 13. The man is ... ___ CA) making a suggestion ___ CB) accepting an invitation 14. The man is ... ___ CA) requesting that the man do something ___ CB) giving the woman a suggestion 15. Paul is ... ___ CA) rejecting a suggestion ___ CB) agreeing with a suggestion 16. James tells the woman that ... ___ CA) he can't accept her invitation ___ CB) he'd enjoy another sandwich 40 Section 1 • Guide to Listening Comprehension Exercise 7.2 Focus: Listening to dialogs involving suggestions, invitations, offers, and requests, and answering questions about them. Directions: Listen to the t()lIowing dialogs. Decide which choice-(A), (B), or (C)-best answers the question about the dialogs, and mark the appropriate answer. The first one is done as an example. __ i)) Now start the audio. 1. _ _ (A) He would like a cigarette. ___ (B) The woman can smoke if she likes. ~ (C) He doesn't want the woman to smoke. 2. ____ (A) The man wears it quite often. _ _ (B) It needs to be cleaned. __ (C) The man should wear it. 3 . _ (A) Make more popcorn. _ (B) Go to another theater. __ (C) Buy some popcorn. 4. _ (A) She could plan the trip. _ (B) She may not feel well. _ (C) She can go on the class trip. S. .__ (A) He doesn't want more coffee. _ (B) He doesn't want to use his credit card. _ (C) He'd like to make coffee. 6 . _ (A) She doesn't think it's warm. ____ (B) She'll open the window herself. __ (C) She wants the window closed. 7. _ . __ (A) The soup is more expensive than sandwiches. ___ (B) She doesn't know what kind of soup there is. ___ (C) The man might enjoy some soup. 8. ___ (A) The kitchen also needs cleaning. ___ (B) The living room doesn't have to be cleaned. ___ (C) The man shouldn't do the cleaning. 9. _____ (A) Go with her to the registrar's office. _____ (B) Help her find her way to the registrar's office. ____ (C) Tell her where to get her own map. 10. ._ (A) Work on their chemistry homework. ___ (B) Have breakfast. ____ (C) Stop studying for a little while. 11. ____ (A) Buy a new toaster. ___ (B) Replace her old shoes. ___ (C) Have repairs done. 12. ___ (A) He'd like her to go away. ___ (B) She can read his magazine. ___ (C) He hasn't finished reading. Section 1 • Guide to Listening Comprehension 13. _____ (A) Get an antique desk. _____ (B) Buy a new computer. _____ (C) Sit down and get to work. 14. ____ (A) He's already passed the test . .___ (B) He doesn't like to study at the library. ___ (C) He doesn't plan to study tonight. 15. ___ (A) He hasn't seen the letters. _ _ _ (B) He doesn't know the right answers. ___ (C) He doesn't want to respond to the letters. 16. ___ (A) She wants to go even though it's raining. ___ (B) She can't come to lunch today. _ _ (C) She'll pay for their lunch. 41 42 Section 1 • Guide to Listening Comprehension LESSON 8 DIALOGS INVOLVING CONTRADICTIONS, ASSUMPTIONS, AND QUESTIONS A) Contradictions These involve the second speaker correcting what the first speaker says, as in the samples below: T Sample Items You will hear: Fl: Amy didn't work overtime last week. M 1: As a matter of fact, she did. M2: What does the man say about Amy? You will read: (A) She is always late for work. (B) She never works overtime. (C) She worked extra hours last week. (D) She hasn't had her job very long. The man's emphatic use of the auxiliary verb did shows that he is contradicting what the woman said. You will hear: MI: Martin always talks about how he loves to dance. F1: Yes, but you don't see him out on the dance floor very often, do you? M2: What does the woman say about Martin? You will read: (A) He is an excellent dancer. (B) He doesn't like dancing very much. (C) He doesn't talk about dancing very often. (D) He goes dancing four times a week. The woman's use of the word but and the tag question (" ... do you?") suggests that she doesn't believe that Martin really loves to dance. You will hear: FI: All of the students voted for the proposal to expand the Student Council. MI: Well, most of them did, anyway. M2: What does the man mean? You will read: (A) All of the students voted. (B) Some of the students opposed the proposal. (C) The proposal was defeated. (D) The Student Council voted. Section 1 • Guide to Listening Comprehension 43 The man says that most of the students voted for the proposal, contradicting the idea that all of them did. Therefore, some of the students must have opposed the proposal. In some dialogs, such as the third Sample Item, the second speaker does not completely contradict what the first speaker says but rather limits the first speaker's idea.... B) Assumptions These are the beliefs that one speaker has until he or she receives information from a second speaker. You will generally hear dialogs involving assumptions near the end of PartA. These questions are considered difficult, but once you understand how they work and practice answering them, you should find them no more difficult than any other type of question. In this type of dialog, the first speaker makes a statement. The second speaker is surprised because the first statement contradicts what he or she believes to be true. The second speaker's response often begins with the word "Oh" and ends with the phrase" ... after all." The answer to assumption questions is the reverse of what the second speaker thinks, and so what is "true" according to the first speaker is not the correct choice. ... Sample Item You will hear: Fl: No, Judy's not here right now. She's at her economics class. Ml: Oh, so she decided to take that course after all. M2: What had the man assumed about Judy? You will read: (A) She wouldn't take the course. (B) She had already completed that course. (C) She was busy studying economics. (D) She wouldn't find economics difficult. The man is surprised that Judy is in economics class because he thought that she had decided not to take the course. Therefore, he had obviously assumed that Judy was not going to take the course before he spoke to the woman. ... C) Questions The second speaker in a dialog sometimes asks about what the first speaker says. The third speaker then asks what the second speaker wants to know. ... Sample Item You will hear: Professor Petrakis said that MarkTwain was his favorite writer. M 1: When did he say that? M2: What does the man want to know? Fl: 44 Section 1 • Guide to Listening Comprehension You will read: (A) When Mark Twain lived. (B) What the professor said about Mark Twain. (C) When the professor made his remark. (D) What books Mark Twain wrote. The man asks when Professor Petrakis called Mark Twain his favorite author. ... Two question phrases that may give you trouble are What . .. for? and How come . .. ? Both mean Why ... ? Exercise 8 Focus: Answering questions about dialogs involving contradictions, assumptions, and questions. Directions: Listen to the following dialogs. Decide which one of the answer choices-(A), (B), or (C)-is correct, and mark the appropriate answer. The ftrst one is done as an example. -4))) Now start the audio. 1. ___ (A) Ginny is deftnitely coming to dinner. ___ (B) Ginny likes ftsh better than chicken. ~ (C) Ginny likes chicken. 2. _ _ (A) She had already moved. _ _ _ (B) She hadn't found a new apartment yet. ___ (C) She'd already made an appointment. 3. ___ (A) What the man's name is. _ _ _ (B) Who told the man to see the dean. _ _ (C) Who the dean is. 4. ___ (A) He wants to take part in the election. ___ (B) He's not interested in running for offtce. ___ (C) He wants to get more facts from the president. 5. _ _ (A) She couldn't type very fast. _ _ _ (B) She had already finished the ftnal draft. ___ (C) She hadn't completed the research. 6. ___ (A) He doesn't really like horseback riding. _ _ (B) He rides horses whenever possible. ___ (C) He doesn't talk about riding very much. 7. ___ (A) When his insurance agent called. ___ (B) What his insurance agent wanted. ___ (C) What time he should return the call. 8. _ _ (A) He was working full time. ___ (B) He was eating in the cafeteria. ___ (C) He didn't want a job. 9. _ _ (A) When they returned. (B) How long their hike was. _ _ (C) Where they hiked. _ i_ Section 1 • Guide to Listening Comprehension 10. _ _ (A) He thinks the clothes are expensive. ____ (B) He doesn't think the clothes are very nice. ______ (C) He thinks the woman is being unreasonable. 11. ____ (A) Where the meeting will be held. (B) When the meeting will start . ._ (C) Where the recreation center will be built. 12 .... _._ (A) Joy did not want to study abroad. . ___ (B) The overseas program had been canceled. _ _ (C) Joy would study overseas sooner than next year. 13 ! 1.. 1';. 16. ___ (A) If the party was at Ben's house. ____ (B) What time the party ended. ____ (C) If the man enjoyed the party. (A) All of Ted's answers were incorrect. _ (B) Most of the problems were done correctly. _ (C) Ted doesn't have to solve the problems. _ (A) How she got to the grocery store. (B) Why she went to the grocery store. (C) How much she paid for groceries. _ (A) Robin's brother didn't help her get a job. __ (B) Robin didn't get a job. __ (C) Robin was able to help her brother. 17. __ (A) The flashlight had needed batteries. __.__ (B) There had been some other problem with the flashlight . ._. __._ (C) The woman hadn't changed the batteries. 1R. ____ (A) Professor Brennon surely won't lead the seminar. ___ .. (B) Professor Brennon's permission is not required. _ (C) The woman doesn't need to take the seminar. 19. _ . _ (A) Why Steve is in the Pacific Northwest. _ ..... _ (B) How long Steve has been traveliri'g. _._ .._ (C) When Steve will return from his trip. 20. __ . _ (A) It hadn't been released yet. _ (B) It wouldn't be very good. __ (C) It can no longer be considered new. 45 46 Section 1 • Guide to Listening Comprehension LESSON 9 ANSWERING QUESTIONS ABour ACTIVITIES, PLANS, TOPICS, AND PROBLEMS A) Questions About Activities These questions follow dialogs that involve people talking about what they are doing. They are a kind of inference question because the activity itself is not mentioned in the dialog. Instead, you must determine the activity from the special vocabulary used by the speakers. T Sample Item You will hear: Fl: Is there room for that box up there? Ml: I can fit it in the trunk. And this suitcase should fit in the back seat. M2: What are the speakers probably doing? You will read: (A) Boarding an airplane. (B) Unpacking a box. (C) Loading a car. (D) Buying a suitcase. The words box, trunk, suitcase, and back seat all indicate that the speakers are putting things into a car. .... B) Questions About Plans These questions follow dialogs in which two speakers discuss what one or both of them are going to do in the future. T Sample Item You will hear: F 1: Are you going to go to Boston with Michael this summer? M1: Wish I could, but if I want to graduate next year, I've got to stay here and take a couple classes. M2: What does the man plan to do this summer? You will read: (A) Graduate. (B) Attend classes. (C) Visit Michael. (D) Go to Boston. The man indicates that he must stay where he is and take classes in order to graduate next year. .... Section 1 • Guide to Listening Comprehension 47 C) Questions About Topics The third speaker asks what the other two speakers are talking about. The topic is not usually mentioned directly in the dialog; it must be inferred from a general understanding of the dialog. The topic can be a person, a thing, or an activity. T Sample Item You will hear: Have you seen this letter from the bursar's office? Oh, no, not another increase! If you ask me, we're already spending too much to go to school here. M2: What are these speakers talking about? Fl: F2: You will read: (A) Higher tuition costs. (B) A poor grade. (C) Higher postage rates. (D) A letter from a relative. From the fact that the letter comes from the bursar's office (the financial office of a university) and that the second woman is upset about an increase and feels they are spending too much to go to school, it is clear that they are talking about an increase in tuition. .6. D) Questions About Problems These questions follow dialogs in which the speakers are discussing some trouble one or both of them are having. The third speaker asks what the problem is. T Sample Item You will hear: M2: Gordon, what happened to your window? Ml: When I was painting the house last week, I hit it with the ladder. M2: What problem does Gordon probably have? You will read: (A) (B) (C) (D) His house needs painting. He broke his ladder. He spilled some paint. His window is broken. Gordon, the second speaker, says that he hit the window with the ladder when he was painting the house. The logical result-a broken window. A. 48 V Section 1 • Guide to Listening Comprehension Exercise 9 Focus: Answering questions about activities, plans, topics, and problems. Directions: Listen to the dialogs and the questions about them. Decide which of the answer choices-(A), (B), or (C)-best answers the question, and mark the appropriate blank. I11III ))\ \Iow start the audio. 1. (A) Road conditions. A weather report. _ . _ (C) Motel reservations. ~ _ _ _ (B) 2. ___ (A) Go to the concert. ___ (B) Listen to jazz on the radio. _ _ _ (C) Buy more tickets. 3. ____ (A) The man's car is not running. ____ (B) The man isn't going to the party. ___ (C) The car isn't big enough for four people. 4. ___ (A) Stay inside. _ _ (B) Find his umbrella, _ _ (C) Look outside. ';. _ _ (A) A store. _ _ (B) A bridge. _ _ CC) A street. 6. _ CA) Shop for groceries. _ _ (B) Leave for a camping trip. _ _ (C) Go to a circus. 7. ___ (A) Ask for medicine for his headaches. ___ (B) Buy some new frames for his eyeglasses. ___ (C) Get different lenses for his glasses. 8. _ _ (A) Clothing. ___ (B) Hair styling. ____ (C) Painting. 9. ___ (A) Take a trip. _ _ (B) Watch television. ___ (C) Examine some documents. 10. ___ (A) He lent it to someone else. _ (B) It was ruined in the rain. _ (C) He forgot where he left it. 1 1 . _ (A) A car. ____ (B) A magazine. __ . _ (C) A computer. 12. ____ (A) Go directly to business school. __ ._ (B) Look for a joh with a big company. ___ (C) Start her own business. 13. ___ (A) Playing cards. _ _ (B) Making dinner. _ _ (C) Repairing a boat. 14. _ _ (A) He doesn't have Phyllis's address. _ _ (B) He's upset with Phyllis. ___ (C) He doesn't have a stamp. 15. ___ (A) Orcler a salad. ___ (B) Go to another restaurant. ___ (C) Put some salt in her soup. Section 1 • Guide to Listening Comprehension 49 LESSON 10 DIALOGS WITH SPECIAL VERBS A) Causative Verbs These verbs indicate that someone causes someone else to do something. When a dialog contains a causative verb, you must understand who performs the action. The verbs have, get, make, and let are the most common causative verbs. They are used in the following patterns: Have have someone do something have something done Dave had the mechanic flx his car. Dave had his car flxed. The causative verb have indicates that one person asks or pays another to do something. The subject of this sentence, Dave, does not perform the action. In the flrst sentence, the mechanic does. In the second sentence, an unnamed person does. get someone to do something get something done Jerry got his cousin to cut his hair. Jerry got his hair cut. The causative verb get usually means to persuade someone to do something. Again note that the subject, Jerry, does not perform the action. In the flrst sentence, Jerry's cousin does. In the second sentence, an unnamed person does. make someone do something Cathy made her son do his homework. The causative verb make means to force someone or compel someone to do something. Cathy's son is compelled to do his homework. let someone do something The boss let us go home. The verb let means permit or allow. The boss gives permission; we go home. ... Sample Item You will hear: M1 : Did you speak to the head of the department? Fl: No, she had her assistant meet with me. M2: What does the woman mean? You will read: (A) (B) /.(C) (D) She spoke to the head of the department. The head of the department had a meeting with her assistant. She met with the assistant to the head of the department. The assistant will soon become head of the department. According to the dialog, the head of the department directed her assistant to meet with the woman. A 50 Section 1 • Guide to Listening Comprehension B) Used to The expression used to has two forms, each with different meanings: used to + simple form I used to live in New York means be/get + used to I once lived in New York (but now I don't). <- + gerund (-ing verb) + noun phrase I'm not used to driving on the left side of road. means I'm not accustomed to driving on the left side. I've tlnally gotten used to my new job. means I've tlnaUy become accustomed to my new job. The dialogs in Part A sometimes take advantage of these two functions of used to. T Sample Items You will hear: Fl: What does Hank's father do for a living? Ml: He's a salesman now, but he used to be a truck driver. M2: What does the man say about Hank's father? You will read: (A) He once drove trucks. (B) He sells used trucks. (C) His truck is still useful. (D) He's accustomed to his job. The man says that Hank's father used to be a truck driver. In other words, Hank's father once drove trucks, but he no longer does so. You will hear: Fl: Nancy is working late again today? Ml: Yeah, she must be getting used to it by now. M2: What does the man say about Nancy? You will read: (A) She probably has a more difficult job now. (B) She once worked later than she does now. (C) She seldom comes to work late. (D) She is becoming accustomed to late hours at work The second speaker indicates that Nancy has probably adjusted to working late. Section 1 • Guide to Listening Comprehension Exercise 10 Focus: Listening to dialogs that contain causative verbs or expressions with used to. Directions: Listen to the dialogs and the questions about them. Then decide which of the two answer choices-(A) or (B)-best answers the question, and mark the appropriate blank. The fIrst one is done as an example. liliiii))) Now start the audio. 1. ___ (A) Doug is happy to be Rose's friend. ~ (B) Doug and Rose are no longer good friends. 2. ___ (A) He can do the job as well as a professional. ___ (B) He should hire an electrician to do the job. 3. ___ (A) This station now plays classical music. ___ (B) The station doesn't broadcast anything but news. 4. ___ (A) Changing the oil was easy for her. ___ (B) The oil didn't need to be changed. 5. ___ (A) He's not accustomed to early classes yet. ___ (B) His classes are diffIcult, too. 6. ___ (A) She's fInally accustomed to roller-skating. ___ (B) She doesn't go skating as often as she once did. 7. ___ (A) He's going to clean his tie. ___ (B) He's going to take his tie to the laundry. 8. ~~ ~ __ 9. 10. ~~ _~ (A) He moved the poster. (B) He no longer likes the poster. (A) She may not be able to take a vacation in August. (B) She's not sure when the busiest time will be. _ (A) She asked Greg to explain the point. (B) She explained the point to Greg. _~ __ ~ 11. _ _ ~ (A) He isn't accustomed to his glasses. _ _ ~ (B) He looks quite different without glasses. 12. ___ (A) This type of weather is not new to him. ___ (B) He once lived in a very different climate. 13. ___ (A) She's going to take a picture of the members of her club. _ _ (B) Someone is going to photograph her club. 14. ___ (A) She's never cooked with it. ___ (B) She doesn't feel comfortable using it. 15. ___ (A) If the deer will come near them. ___ (B) If they can approach the deer. 51 52 Section 1 • Guide to Listening Comprehension 23. 24. CDCD·.·<:£)c[) CDCD€0c[) CDCD€0c[) CE>c[) MINI-TEST 1: DIALOGS Directions: Listen to the conversations and the questions about them. Decide which of the four answer choices-(A), (B), (C), or (D)-is the best answer to the question. Then mark the appropriate answer on the answer sheet . .-))) Now start the audio. 1. ___ (A) She met her during the summer. ___ (B) She's never liked her very much. ___ (C) She warned her of a problem. ___ (D) Her impression of her has changed. 2. ___ ___ ___ ___ (A) (B) (C) (D) Review the last point. Go on to the next chapter. Leave the classroom. Point out the teacher's mistake. 3. ___ (A) The weather will not be as nice tomorrow. _ _ _ (B) She no longer reads the weather report. ___ (C) She went to the store for a newspaper. ___ (D) The weather reports will change soon. 4. ___ __ ___ ___ (A) (B) (C) (D) 5. ___ ___ ___ ___ 6. ___ (A) (B) (C) (D) (A) Writing an advertisement. Playing a game. Looking at a newspaper. Discussing a book. Grace may want to live with the man's sister. The woman thinks Grace already has a roommate. The woman doesn't know where Grace has moved. Grace doesn't know the man's sister. Go to a meeting. _ _ _ (B) Keep a budget. ___ (C) Reduce his expenses. ___ (D) Get some exercise. Section 1 • Guide to Listening Comprehension 7. (A) The man should clean out his closet. (8) The lamp will look better in a small space. (C) She doesn't like the lamp very much. (D) The living room is the best place for the lamp. 8. (A) (B) (C) (D) What ,\flark is writing. Where :Vlark is living now. Why .'vlark doesn't want to go. Why Mark is in a hurry. 9. (A) He certainly likes Ernie's red car. (B) The man in the red car resembles Ernie. (C) Ernie has a car just like that red one. (D) He can't see the man in the red car. 10. (A) The man doesn't need his hat. (H) It's not very cold today. (C) She likes the way the hat looks. (D) The man ought to wear his hat. 11. (A) She's never seen it. (B) It was made a long time ago. (e) She likes it a lot. (D) It's a very unusual movie. 12. (A) He doesn't believe what the woman told him. (B) He thinks the team was unprepared too. (e) He disagrees with the woman's idea. (D) He isn't ready to go to the game either. 15. (A) The man would like to use that computer. (B) Becky will need the computer for a long time. (e) There are no longer any computers in the library. (D) Becky would like the man to go to the library. 14. (A) He hurt his hand when he was scuba diving. (H) He hasn't gone scuba diving for a long time. (C) He's not too old to go scuba diving. (D) He's an experienced scuba diver. 15. (A) His shoes hurt his feet. (B) He was injured in a skiing accident. (e) His shoes are old and in bad shape. (D) He walked so far that his legs hurt. 16. (A) She doesn't have time to listen now. (H) She doesn't know what song she wants to hear. (C) She wants to hear his song right away. (D) She prefers old songs to new ones. A television commercial. A history class. (e) The woman's field of study. (D) Some famous artists. ):' (A) (B) 18. (A) She was about to suggest the same thing. (H) She doesn't feci like giving a party. (e) She's completely surprised by the man's remark. (D) She isn't hungry right now. 53 54 Section 1 • Guide to Listening Comprehension 19. (A) (B) (C) (D) Sophie won't be at her apartment. The man can inform Sophie at the meeting. The man shouldn't bother Sophie. Sophie will think this is bad news. 20. (A) (B) (C) (D) He lie He He 21. (A) (B) (C) (D) The ring is quite attractive. Laura got a bargain on the ring. The ring was probably expensive. Laura had to sell her ring. doesn't go out as often as he once did. doesn't always tell the truth. isn't as friendly as he once was. hasn't always been so sociable. 22. (A) She didn't realize Bill had to work. (B) Bill has not finished his work. (C) The break has not lasted long enough. (D) The work didn't take long to complete. 23. (A) (B) (C) (D) The woman has just begun to collect rocks. The man is unwilling to help. The box is very heavy. There's nothing in the box. 24. (A) (B) (C) (D) She She She She doesn't like the length of her hair. thinks the haircut took too long. doesn't know where to get a haircut. thinks haircuts are too expensive. 25. (A) Professor Clayburn is going to speak some other night. en) He's never heard of Professor Clayburn. (C) He didn't realize Professor Clayburn was speaking tonight. (D) Professor Clayburn is giving his speech in this room. 26. (A) The man may see her drawing. (B) She'd like the man to visit. (C) The man should draw a second picture. (D) She's going to take a guess. 27. (A) The weather has been very warm. (B) Her car doesn't need a tune-up. (C) She's starting to feel ill. (D) She's already too warm. 28. (A) Joe has been making too much noise. (B) Dogs are not allowed in the dorm. (C) No one understands the parking regulations. (D) Joe is not allowed to leave his room. 29. (A) Where the Medical Center is located. (B) Which office Dr. Norton is in. (C) What Dr. Norton told the man. (D) Why the man went to see Doctor Norton. 30. (A) She liked chemistry. (B) She would graduate in May. (C) She didn't have to repeat a course. (D) She hadn't completed the required courses. PAR T B About Extended Conversations About Extended Conversations The second part of the Listening Comprehension section consists of longer conversations between a man and a woman or (sometimes) between two men or two women. Each conversation lasts from thirty to ninety seconds. It is preceded by brief introductory comments. After each of the conversations, there are from three to five questions. The questions are separated by a twelve-second pause. You must decide which one of the four answer choices in your test book is the best answer for the question, then mark that answer on your answer sheet. You're not permitted to take notes. There are two longer conversations. The Introductory Comments These comments tell you which questions the conversation refers to and provide some brief information about the conversation: "Questions "Questions "Questions "Questions "Questions 31 to 31 to 35 to 36 to 35 to 34: 35: 38: 39: 39: Listen to Listen to Listen to Listen to Listen to two students talk about their psychology class." a conversation about plans for a class trip." two friends discussing a performance that they attended." a conversation in a student housing office." a conversation between two teaching assistants." Not only do these introductory comments tell you to get ready to listen; they also tell you how many questions in your test book to preview at one time. Moreover, they give you a general idea of what to expect as you listen. 55 56 Section 1 • Guide to Listening Comprehension ..... Sample Item You will hear: Listen to a conversation between two students. Bill, you're a physics major, aren't YOll? That's right. I need some advice. I want to take an introductory physics class, and I have to choose between two teachers, Professor Hunter and Professor McVey. Do you know much about them? MI: I've taken classes from both of them. To tell you the truth, I don't really like Hunter's style of teaching. He doesn't seem to care if his students understand or not, and his lectures are pretty dry. FI: Well, then, what about McVey? I've heard his course is difficult. M 1: It's not easy, but you'll learn a lot, and he always encourages his students to ask questions and join in discussions. M2: FI: MI: Fl: You will then hear: M2: What does the woman ask Bill to do? You will read: Give her information about two teachers. (B) Help her with a physics assignment. (C) Speak to a professor for her. (D) Lead a discussion. (A) The woman asks Bill for some advice about the two professors who are teaching basic physics courses. Therefore, the best answer is (A). You will then hear: M2: What does Bill imply that the woman should do? You will read: (A) Change her major to physics. (B) Discuss her problem with Professor Hunter. (C) Sign up for an easy class. (D) Take Professor McVey's class. CD CD CD . . Bill speaks critically of Professor Hunter's teaching methods, but favorably of Professor McVey's, so he would probably advise her to take McVey's course. The best answer is (D). • The Conversations The extended conversations are similar to the Part A dialogs in style, but are longer. They frequently take place in a campus setting between two students or between a professor and a student. Section 1 • Guide to Listening Comprehension 57 The Questions The questions may be overview questions or detail questions. The first question after the conversation is often an overview question. Overview questions require a broad understanding of the entire conversation. To answer them correctly, you must understand what both speakers say. There are several types of overview questions: ~p.;ofi.•,erview Question Topic question "What are the speakers discussing?" Questions about setting (time and location) "Where did this conversation take place?" "When did this conversation take place?" Questions about the speakers "Who are the speakers?" "What is the probable relationship between the speakers?" It's important to listen carefully to the first few lines of an extended conversation to answer overview questions because this part of the talk often sets the scene. It often establishes the time and location of the conversation. the identity of the speakers, and the main idea of the rest of the conversation. Detail questions ask about specific points in the conversation. The answer will usually be contained in what one speaker says. Detail questions follow the order of information in the conversation. In other words, the first of these questions refers to a point made early in the conversation, and the last asks about a point made near the end of the conversation. Most detail questions are factual questions; the answers are directly stated in the conversation. Many factual questions begin with these phrases: According to the conversation, ... According to the man, ... According to the woman, ... A few of the detail questions are inference questions. In other words, the information is not directly given by the speakers; it can be concluded from the information that is stated, however. What can be inferred from the man's comment about ... ? What does the man imply about ... ? What will the speakers probably try to do' The Answer Choices The four choices are all plausible answers for the question. Usually the answer choices are mentioned in some way in the conversation, but only one, the key, answers that particular question correctly. Some people prefer to close their eyes or look away while listening to the extended conversations in order to concentrate on the voices on the audio. However, it is better if you read over the answer choices in the test book while listening. This technique is difficult- but it has several advantages: • It allows you to get an overall sense of what the topic of the conversation will be. • It enables you to anticipate what the questions will be, then concentrate on listening for those points. • It permits you to confirm some of the details that you hear by comparing them with the answer choices in the test book. 58 Section 1 • Guide to Listening Comprehension In the next section of this Guide, you will practice previewing answer choices, listening to Part B conversations, and answering both overview and detail questions about the conversations. TACTICS FOR EXTENDED CONVERSATIONS • Be familiar with the directions, but remember that you cannot turn the page to look over answer choices while the directions are being read. • Pay attention to the introductory comment for each conversation. These comments will tell you which items each conversation refers to and may give you an idea of what the conversation will be about. • Preview the answer choices during the conversations and the pauses between questions. Try to guess what the questions will be. • Listen for overall concepts: • Who is taking part in the conversation? • Where and when does the conversation take place? • What is the main topic of the conversation? The answers to these questions are often suggested in the first few lines of the conversations. • You are not permitted to take written notes, but try to take "mental notes" on specific details: facts, figures, dates, places, and so on. You can sometimes check the information you think you hear against information you read in the answer choices while you are previewing. • Answer items right away. • Never leave any blanks on your answer sheet. Always guess. Section 1 • Guide to Listening Comprehension 59 LESSON 11 ANTICIPATING QUESTIONS ABOUT THE EXTENDED CONVERSATIONS When you are previewing the items in Part C, you should try to anticipate what the questions will be by the form of the answer choices. You usually won't be able to guess exactly what the question will be, but you can guess what type of question will be asked. For example, if the four answer choices are the names of places, a "Where ..." question will be asked; if the answer choices are times of day or dates, you will hear a "When ..." question. If you have some idea of what the question will be, you can focus your listening during the talks. You can also get a good idea of the situation in which the conversation is taking place by previewing the items. Remember: The introductory comments at the beginning of the conversation tell you which items to preview. Exercise 11 Focus: Guessing the questions that will be asked about an extended conversation and the situation in which the conversation takes place by previewing answer choices. Directions: Look over the answer choices below. First try to guess the general type of question that will be asked about each item. Then look at the list of questions after each group of items and match the letter of the appropriate question with the item. Then answer the question about the overall topic of the conversation. The first one is done as an example. There is no audio material for this exercise. Conversation 1 1. (A) A grade the student received. (B) A story about a dance recital. (C) The need for correct spelling. (D) The role of a reporter. Question: b 2. (A) Business. (B) Architecture. (C) Journalism. (D) Dance. Question: 3. (A) (B) (C) (D) She submitted it too late. It was too long. Some important details were omitted. Almost every word was misspelled. Question: 4. (A) Rewrite the story. (B) Buy a better dictionary. (C) Go to more dance recitals. (D) Get a job as a reporter. Question: Questions for Conversation 1 a) For what class did the man do the assignment? b) What is the main topic of this conversation? c) What does the man advise the woman to do? d) What problem does the man mention in connection with the story? Situation Question: Conversation 1 Which of the following best describes the situation in which the first conversation probably takes place? ___ ___ ___ ___ (A) (B) (C) (D) One student is telling another a story about a dance. A professor is criticiZing a student's story about a dance performance. An instructor is teaching a student a new dance. One student is suggesting ways in which the other student can improve her spelling. 60 Section 1 • Guide to Listening Comprehension Conversation 2 5. (A) He doesn't get enough exercise. (B) He's nervous about an important test. (C) He's spending too much time at the Recreation Center. (D) He doesn't know how to swim. Question: 6. (A) Across campus from the Student Center building. (B) South of the stadium. (C) On the north side of campus. (D) Between the Student Center and the stadium. Question: 7. (A) (B) (C) (D) Sign up for some classes at the Recreation Center. Spend more time studying for exams. Take a break from his studies. Take a bus to the Recreation Center. Question: 8. (A) Just before the beginning of the semester. (B) During mid-term exams. (C) Near the end of the semester. (D) Just after the end of the semester. Question: Questions for Conversation 2 a) Where is the Recreation Center? b) What does the woman suggest the man do? c) What problem does the man complain about? d) At what point in the semester does this conversation take place? Situation Question: Conversation 2 Which of the following best describes the situation in which the conversation probably takes place? ___ (A) One student recommends that another get some exercise in order to relax during exams. _ _ _ (B) Two students discuss their plans for a vacation. ___ (C) A physical education instructor suggests that a student register for classes in her department. ___ (D) A student explains to a visitor to campus how to get to the stadium. Conversation 3 9. (A) (B) (C) (D) At an art gallery. At an art museum. In an artist's studio. In a special room in the library. 12. (A) (B) (C) (D) An art historian and a student. Two students. Two visitors to a museum. A tour guide and a tourist. Question: 10. (A) Paintings. (B) Sculptures. (C) Book covers. (D) Photographs. Question: Question: Question: 11. (A) (B) (C) (D) A story in a newspaper. An article in an art magazine. A class she attended. A show she saw on television. Question: 13. (A) Primitive. (B) Life-sized. (C) Realistic. (D) Stylized. Section 1 • Guide to Listening Comprehension 61 -----------------------------------------------------------------------------------Questi()1ls for Conversation 3 a) Who is taking part in this conversation? b) How would the woman probably describe the works of art that she saw? c) What was the source of the woman's information? d) Where did the exhibit take place? e) What kind of art are the speakers discussing? Situation Question: Conversation 3 Which of the following best describes the situation in which the conversation probably takes place' ____~ ____~ ___~ _ _~ (A) (B) (C) (D) One speaker describes to the other her techniques for painting pictures. The woman tells the man about a recent book she read. An art expert gives some advice to the other speaker about becoming an artist. One speaker tells the other about some art she saw at an exhibit. 62 Section 1 • Guide to Listening Comprehension LESSON 12 ANSWERING OVERVIEW QUESnONS ABOUT EXTENDED CONVERSATIONS After each extended conversation in Part B, there are four to five questions. Usually the first and sometimes the last question are overview questions. To answer these questions, you need an understanding of the whole conversation rather than of any specific point. Overview Questions for the Extended Conversations • • • • • • • What is the main topic of this conversation? What are these people primarily discussing? Where does this conversation take place? When does this conversation take place? What is the relationship between the speakers? What is the man's/woman's occupation? What is one speaker's attitude toward the other speaker? Main topic questions must correctly summarize the conversation. Incorrect answers for these questions are too general, too specific, or incorrect according to the conversation. Although these questions require an overall understanding of the conversations, the first few sentences often "set the scene." In other words, the opening lines of the talk establish the time, place, and main topic. Read the opening lines of the extended conversation given below: Ml: (Answering phone) Hello? Fl: Hi, Rod, this is Rita-I'm in your nine o'clock class. I missed class because of a cold, and I was wondering if I could borrow your notes. Ml: I don't know if you could read my notes-I have terrible handwriting. But I can tell you what happened. Professor Phillips went over the material in Chapter 4, about different types of stars in our galaxy. And she talked about what the mid-term exam is going to be like. Fl: Uh-oh, you better tell me all about the mid-term-I really need to do well on it. From this portion of a conversation, we learn that · .. both of the speakers are students · .. they are probably taking a course in astronomy · .. the class is about halfway over (because they are taking mid-term exams) · .. the rest of the talk will probably deal with the material that will be on the examination Not all conversations begin with so much detail. However, it is important to concentrate on the opening lines to learn this kind of information. Exercise 12 Focus: Listening to the opening lines of extended conversations, and answering overview questions about the topics, settings, and speakers. Directions: Listen to the conversations and the questions about them. Then mark the answer choice-(A), (B), or (C)-that correctly completes the sentence. The first one is done as an example. -4))) Now start the audio. 1. ___ (A) Methods of predicting earthquakes. ~ (B) Ways to improve the man's presentation. ___ (C) The many new uses of computer graphics. 2. ___ (A) Statistics. _ _ _ (B) Computer science. ~ (C) Geology. Section 1 • Guide to Listening Comprehension 3. ___ (A) A language teacher and a student. ___ (B) A dean and a teacher. _ _ _ (C) A teacher and an assistant. 4. _ _ (A) The language of the deaf. _ _ _ (B) Methods of teaching German. ___ (C) Communication networks. 5. ___ (A) Professor Quinn's approach to teaching. _ _ _ (B) The process of getting a student identification card. ___ (C) Procedures for checking out reserve material. 6. ___ (A) At a university library. _ _ (B) In a psychology class. _ _ (C) In a laboratory. 7. _ _ (A) To ask for a job. _ _ _ (B) To get some advice. ___ (C) To discuss medical research. 8. ___ (A) Academic advisor. _ _ _ (B) Physician. ___ (C) Administrator. 9. _ _ (A) Helpful. ___ (B) Discouraging. ___ (C) Inconsiderate. 10. ___ (A) The art of raising dogs. _ _ (B) A softball game. _ _ (C) A dogsled race. 11. ___ (A) An archaeologist. _ _ _ (B) An anthropologist. ___ (C) A university student. 12. ___ (A) Their plans for the coming school year. ___ (B) Tina's volunteer position. _ (C) Tina's trip to Europe. 13._ . .0 (A) A clerk at a bookstore. __ (B) A librarian . (C) A publisher's sales officer. "_ _ 14. ___ (A) Before the spring term begins. _ __ (B) In the middle of the spring term. (C) After the spring term ends. _0.00_ 15. _ _ (A) In Nicholson Hall. ___ (B) In the Graduate Admissions Office. ___ (C) In the Financial Aid Office. 16. ___ (A) Requirements for graduate admission. ___ (B) Directions to another office. ___ (C) The woman's need for a scholarship. 63 64 Section 1 • Guide to Listening Comprehension LESSON 13 ANSWERING DETAIL QUESTIONS ABOUT EXTENDED CONVERSATIONS Most of the questions in Part B are detail questions that require an understanding of specific points in the conversation. A majority of these questions are factual questions, asking what, where, when, why, and how much. To answer the question, you need to listen carefully. Other questions are inference questions. As previously explained, the answers to inference questions are not directly stated, but are suggested by information in the lecture. Many of these questions begin, "What do the speakers imply about ..." or "What can be inferred from the conversation about ..." Remember that the order of detail questions follows the order of the conversation. In other words, the first detail question will be about something mentioned early in the conversation while the last one is about something mentioned near the end of the conversation. If anything in the conversation is emphasized, it will probably be asked about. In other words, if something one speaker says is repeated by the second speaker, or if one speaker talks about something in an emphatic tone of voice, there will probably be a question about that information, as in this section of a conversation: Ml: My project for my film-making class took me six weeks to finish. F1: Six weeks! I can hardly believe it. Doesn't the teacher realize you have other classes too? You can be fairly sure that there will be a question such as this: "How long did the man's project take to complete?" Exercise 13.1 Focus: Answering detail and inference questions based on specitk points in short portions of extended conversations. Directions: You will hear three extended conversations, each one divided into several short portions. After each portion, there will be a number of questions based on that part of the talk. Mark the best answer choice-(A), (B), or (C)-for each question. The first one is done as an example. l1li))) Now start the audio. l. _ _ (A) A doctor. .___ (B) A newspaper. ~ (C) A magazine. 2. ___ (A) It's too tiring. ___ (B) It can cause injuries. ____ (C) It's not demanding enough. 3. _ _ (A) Downhill skiing. _ _ (B) Jogging. ___ (C) Cross-country skiing. 4. _ . _ (A) It doesn't require much snow ____ (B) It is a recently developed sport. ___ (C) It can be done in flat areas . .:;. ___ (A) Use a cross-country ski machine. ___ (B) Travel to ski resorts. ___ (C) Take up jogging. Section 1 • Guide to Listening Comprehension 65 6. ___ (A) The expense. ___ (B) The weather conditions. _ _ (C) The danger. 7. ___ (A) He stayed up most of the night. ___ (B) He's been studying all morning. ___ (C) He took an exam last night. S. ___ (A) It was an improvement. _ _ _ (B) It was disappointing. ___ (C) It was unfair. 9. ___ (A) Undergraduate students. ___ (B) Teachers. ___ (C) Graduate students. lO. ___ (A) She learned how to do research. ___ (B) She was prepared for her sociology test. ___ (C) She learned teaching techniques. 11. ___ (A) Basic scientific research. ___ (B) Business management. _ _ (C) Test-taking skills. 12. ___ (A) In the library. ___ (B) In the Physics Tower. _ _ _ (C) In Staunton Hall. 13. ___ (A) Study for his next exam. _ _ _ (B) Go to the Study Skills Center. ___ (C) Get some sleep. 14. ____ (A) To buy something at an auction. ____ (B) To pay for employees' salaries. ___ (C) To improve their broadcasts. 15. ____ (A) Apply for a job at the station. ____ (B) Donate his services for the station's auction. ___ (C) Direct the construction of a new tower. 16. ___ (A) She must work on her own research project. _ _ (B) She has to help her parents. ___ (C) She must study for exams. 17. ___ (A) They seldom attend auctions. _ _ _ (B) They might bid on the man's services. ___ (C) They use the library often. Exercise 13.2 Focus: Answering detail and inference questions based on specific points in complete extended conversations. Directions: You will hear four extended conversations. After each conversation, there will be a number of questions based on it. Mark the best answer choice-(A), (B), or (C)-for each question. The first one is done as an example . .-))) Now start the audio. 66 Section 1 • Guide to Listening Comprehension 1. _ _ (A) North of Los Angeles. ~ (B) Between Los Angeles and San Diego. _ _ (C) East of San Diego. 2. ___ (A) They are a type of insect. ___ (B) They are a kind of fish. ___ (C) They are a type of bird. 3. ___ (A) In March. ___ (B) In early summer. _ _ _ (C) In October. 4. ___ (A) About 200 miles. _ _ (B) About 1,000 miles. _ _ (C) About 7,000 miles. 5. ___ (A) The swallows' arrival. _ _ (B) The parade. ___ (C) The swallows' departure. 6. ___ (A) Only during the first week of classes. ___ (B) Whenever students ask for them. _ _ _ (C) Only in the afternoon. 7. _ _ (A) A tour gUide. ___ (B) A classroom. _ _ (C) A map. 8. ___ (A) In the Science Building. ___ (B) In the Student Center Building. ___ (C) In the University Recreation Center. 9. ___ (A) A test in a composition class. _ _ (B) A road test. ___ (C) The written test for her driver's license. 10. ___ (A) He drove too fast. ___ (B) He couldn't park well. ___ (C) He made an improper turn. 11. ___ (A) Drive her to the test site. _ _ (B) Help her get ready for the road test. ___ (C) Sell her a car. 12. ___ (A) It doesn't belong to her. ___ (B) She's not a licensed driver. ___ (C) It isn't running right. Section 1 • Guide to Listening Comprehension 67 MINI-TEST 2: EXTENDED CONVERSATIONS Directions: Listen to the conversations and the questions about them. Decide which one of the four answer choices-(A), (B), (C), or (D)-is the best answer to the question. Then mark the appropriate answer on the answer sheet. liliiii))) Now start the audio. 1. ___ (A) To look up some terms. ~__ ___ ___ 2. ___ ___ ___ ___ (B) To meet Stanley (C) To get a snack. (D) To prepare for an exam. (A) His library card. (B) A statistics book. (C) Some index cards. (D) A notebook. 3. ___ ___ ___ ___ (A) Piles of note cards. 4. ___ ___ ___ ___ 5. ___ ___ ___ ___ (A) Behind the main desk. 6. ___ ___ ___ __ (A) A means of descending slopes. 7. ___ ___ ___ ___ (A) Educating people about geology. (B) The part of the library where journals are stored. (C) The part of the library where books are shelved. (D) A place to get something to eat. (B) The periodicals room. (C) A lost and found office. (D) The reference room. (A) The sport of mountain climbing. (B) Classes the man is taking. (C) An exhibit the man saw in a museum. (D) A new activity the man is involved in. (B) A method of climbing cliffs. (C) A way to clean walls. (D) A type of graffiti. (B) Cleaning up after careless people. (C) Photographing mountain peaks. (D) Rescuing people who are in danger. 8. ___ (A) Explore a cave with him. ___ (B) Take some photographs. _ _ _ (C) Attend a meeting. ___ (D) Examine a crystal. PAR T C Mini-Talks About Mini-Talks The third part of Section 1 consists of Mini-Talks. These are monologues (talks involving only one speaker). Each Mini-Talk lasts from thirty to ninety seconds. Like the Extended Conversations, the) are preceded by introductory comments. After each talk, there are from three to five questions. The questions arc separated by a twelve-second pause. You have to decide which of the four answer choices in the test book best answers the question, then mark that answer on your answer sheet. You are not allowed to take notes. There are three Mini-Talks. The Introductory Comments These comments tell you which questions the Mini-Talks refer to and provide some brief informatioll about the conversation: '"Vuestions 35 to 40: Listen to a lecture given at a botanical garden." "Questions 31 to 35: Listen to a talk about the university's housing policy." "Questions 35 to 3R: Listen to a lecture given in a history class." The introductory comments tell you how many questions in your test book to preview at one time and give you a general idea of the topic of the talk. 68 Section 1 • Guide to Listening Comprehension ... 69 Sample Item You will hear: M2: Listen to this lecture given in a university classroom. Ml: Students, this evening we'll have a chance to observe a phenomenon that we've discussed several times in class. Tonight there will be a lunar eclipse. As we've said, when an eclipse of the Moon occurs, the Earth passes between the Sun and the Moon. Therefore, the shadow of the Earth moves across the surface of the Moon and obscures it. Because you won't be looking at the Sun, it is not necessary to use the special lenses and filters that you need when observing a solar eclipse. You can observe a lunar eclipse with your unaided eye or with a telescope, and photograph it with an ordinary camera. So if the weather's not cloudy tonight, go out and take a look at this eclipse of the Moon. I'm sure you'll find it interesting. You will hear: M2: In what course is this lecture probably being given? You will read: (A) Philosophy. (H) Meteorology. (C) Astronomy. (D) Photography. The lecture concerns a lunar eclipse, a topic that would typically be discussed in an astronomy class. You will hear: M2: According to the speaker, which of the following occurs during a lunar eclipse? You will read: CA) The Earth's shadow moves across the Moon. (B) Clouds block the view of the Moon. (C) The Moon moves between the Earth and the Sun. CD) The Sun is too bright to be observed without special equipment. The speaker says "the shadow of the Earth moves across the Moon and obscures it." The best answer is therefore (A). ... The Talks The Mini-Talks are usually somewhat more formal in style than the Extended Conversations. Some of the talks resemble lectures given as part of a university course in history, literature, or biology, for example. Other Mini-Talks resemble talks you would hear at a university, but not in a classroom. For example, you might hear talks about campus organizations, registration procedures, or the services at a campus medical center. Still others have nothing to do with university life; you may hear a tour guide speaking to a group of tourists or a curator speaking to visitors at a museum. 70 Section 1 • Guide to Listening Comprehension The Questions As with the Extended Conversations, the questions about Mini-Talks may be overview questions or detail questions. The first question after the talk is usually an overview question. Overview questions require an understanding of the entire conversation. There are several types of overview questions: Topic/main ideal purpose questions "What is the lecture mainly about?" "What is the speaker mainly talking about?" "What is the main idea of this lecture?" "Why is the speaker giving this talk?" "What is the main point of this lecture?" Questions about setting (course, time, and location) "In what course was this lecture probably given?" "Where was this talk probably given?" "When was this talk probably given?" Questions about the speaker "Who is the speaker?" "What is the speaker's occupation?" You should listen carefully to the first few lines of the mini-talk to answer overview questions because this part of the talk often sets the scene. It often establishes the time and location of the conversation, the identity of the speaker, and the main idea of the rest of the conversation. Detail questions ask about specific points in the talk. Detail questions follow the order of information in the lecture. In other words, the first of these questions refers to a point made early in the lecture; the last asks about a point made near the end of the lecture. Most detail questions are factual questions; the answers are directly stated in the talk. Some factual questions begin with these phrases: According to the speaker, .. . According to the lecture, .. . A few are inference questions. The answers to these are not directly stated in the talk; they are only suggested. These questions usually contain some form of the words infer or imjJ~V or the word probably. What can be inferred about ... ? What does the speaker imply about ... ? What is probably true about ... ? The Answer Choices The four choices are all plausible answers for the question. Usually the answer choices are mentioned in some way in the talk. As with the conversations, it is better to read over the answer choices in the test book while listening so that you can preview the answer choices than to look away or close your eyes. In the next section of this Guide, you will practice previewing answer choices, listening to Part C Mini-Talks, and answering both overview and detail questions about the talks. Section 1 • Guide to Listening Comprehension 71 Tactic$foJ:.Nlini-Ta Iks • Be familiar with the directions, but remember that you cannot turn the page to look over answer choices while the directions are being read. • Pay attention to the introductory comment for each talk. These comments will tell you which items each talk refers to and may give you an idea of what the talk will be about. • Preview the answer choices while the talks are being read and during the pauses between questions. Try to guess what the questions and the topic will be. • Listen for overall concepts: • Who is giving the talk? • Where and when is the talk being given? • What is the main topic or purpose of the talk? The answers to these questions are often suggested in the first few lines of the talks. • You are not permitted to take written notes, but try to take "mental notes" on specif1c details: facts, figures, dates, places, and so on. You can sometimes check the information you think you hear against information you read in the answer choices while you are previewing. • Answer items right away. • Never leave any blanks on your answer sheet. Always guess. 72 Section 1 • Guide to Listening Comprehension LESSON 14 ANTICIPATING QUESTIONS ABOUT MINI-TALKS As in the other two parts of the Listening Comprehension section, you should preview the items in Part C and try to anticipate what the questions will be by the form of the answer choices. You may not he able to guess exactly what the questions will be, but you can guess what type of question will be asked. For example, jf the four answer choices are the names of places, a "Where ..." question will be asked; if the answer choices are times of day or dates, you will hear a "When ..." question. If you have some idea of what the question will be, you can focus your listening during the talks. Exercise 14 Focus: Guessing what Part C questions will be asked by looking at the answer choices. Directions: Look over the answer choices below. First try to guess the general type of question that will be asked about each item. Then look at the list of questions after each group of items and match the letter of the appropriate question with the item. One question in each set will not be used. The first one is done as an example. There is no audio material for this exercise. 1. (A) The life of Clara Barton. (B) A short history of the American Red Cross. (C) The role of nurses in the Civil War. CD) The writings of Clara Barton. Question: b 2. (A) In Virginia. (B) In Switzerland. (C) In Massachusetts. CD) In Texas. Question: (B) Superintendent of a hospital. (C) Clerk in a government office. (D) Diplomatic official. Question: To To To To Question: b) c) d) e) f) What was Clara Barton's first occupation? What is this talk mainly about? Where was Clara Barton born? When did Clara Burton found the American Red Cross? What was the original purpose of the American Red Cross? When was this lecture probably given? (B) To introduce a speaker. (C) To discuss the creative writing program. (D) To criticize a new book. Question: 7. (A) A science fiction novel. In 1845. During the American Civil War. During a trip to Europe. In 1881. Question: 5. (A) (B) (C) (D) a) 6. (A) To honor a student. 3. (A) Teacher. 4. (A) (B) (C) (D) Questions for items 1-5 help wounded soldiers. provide relief for hurricane victims. prevent famines. publish books about nursing. (B) Poetry. (C) Criticism. (D) A collection of short stories. Question: 8. (A) A cash prize. (B) Publication of his works. CC) A free trip. CD) A scholarship. Question: Section 1 • Guide to Listening Comprehension Questions jor items 6-8 a) How does the speaker feel about Jim McKee? b) What award did Jim McKee receive? c) What kind of writing has Jim McKee done? d) What is the main purpose of this talk? 9. (A) A radio announcer. (B) A waitress. (C) A television announcer. (D) A chef. Question: 10. (A) (B) (C) (D) Vegetarian food. Food from New Mexico. Food from Louisiana. Fresh seafood. Question: 11. (A) (B) (C) (D) On Atlantic Avenue. On a boat. On a dock. On First Street. 73 Question: 12. (A) Most of the dishes are reasonably priced. (B) All but a few of the dishes are delicious. (C) The service has improved lately. (D) It's not too crowded on weekday nights. Question: Questions jor Items 9-12 a) What does the speaker say about the Tangerine Cafe? b) What kind of food does the Tangerine Cafe mainly serve? c) What problem did the speaker have when she went to the Tangerine Cafe? d) Who is the speaker? e) Where is the Tangerine Cafe located? 74 Section 1 • Guide to Listening Comprehension LESSON 15 ANSWERING OVERVIEW QUESTIONS ABOUT MINI-TALKS After each talk in Part C, there are three or four questions. Usually the first question is an overview question. To answer this type of question, you need an understanding of the whole talk rather than of any specific point. Overview Questions for the ilfini-Talks • • • • • • • What is the main idea/main point/main topic of the lecture? What is the purpose of this talk? Where was this lecture given? When was this talk given? In what course was this lecture given? What is the speaker's occupation? Who is the audience for this talk? Main idea, main topic, and main point questions must correctly summarize the talk. Incorrect answers for these questions are usually too general, too specific, or incorrect according to the lecture. Although these questions require an overall understanding of the talks, the first few sentences often "set the scene." In other words, the opening lines of the talk frequently establish the time, place, and main topic. Read the opening lines of the Mini-Talk given below: Good morning, everyone. As you probably know, this class is a continuation of a course that began last term. Last term we focused on American writers of the nineteenth century. Today we'll begin our study of twentieth-century novelists with a look at Ernest Hemingway. From this introduction, we know that · .. · .. · .. · .. the the the the speaker is a teacher audience is a group of students course is in American literature talk will concern Ernest Hemingway Not all talks will begin with so much detail. However, it is important to concentrate on the opening lines to learn this kind of information. Exercise 15 Focus: Listening to the opening lines of Mini-Talks and answering overview questions about the main ideas, speakers and audiences, settings, and so on. Directions: Listen to the introductions and the questions about them. Then mark the answer choice-(A), (B), or (C)-that correctly completes the sentence. The first one is done as an example. -4»)) Now start the audio. 1. ___ (A) A teacher. ~ (B) A tour guide. ___ (C) A photographer. 2. ___ (A) A description of the wildlife preserve. ___ (B) Advice about outdoor photography. ___ (C) The scientillc classillcation of buffaloes. 3. ___ (A) To explain the traditions of handball. ___ (B) To give information about the rules of tennis. ___ (C) To discuss the rules of handball. 4. ___ (A) At the end of a tournament. ___ (B) Before an exhibition game. ___ (C) During a game. Section 1 • Guide to Listening Comprehension 5. ___ (A) Factory workers. ___ (C) Management trainees. 14. ___ (A) The fundamentals of skiing. _ _ _ (B) Championship skiers. ___ (C) The development of ski resorts. _~ 15. _ _ (A) Law. _ _ _ (B) Visitors to a factory. 6. 75 (A) The process of canning soft drinks. _ _ _ (B) Management-labor teamwork. ___ (C) The life cycle of plants. 7. ___ (A) The physical rewards of dancing. _ _ _ (B) The importance of the program to the university. ___ (C) The disadvantages of being in the program. S. ___ (A) Director of a dance program. _ _ (B) Professor of psychology. _ _ (C) Athletics coach. 9. _ _ (A) A host at a party. ___ (B) The president of a society. ___ (C) The captain of a ship. 10. ___ (A) The role of the State Historical Society. _~ (B) The history of New England. ___ (C) The story of some shipwrecks. 11. ___ (A) Composition. ___ (B) Fine arts. ___ (C) Architecture. 12. ___ (A) The use of blueprints. ___ (B) Methods of organization. _ _ (C) Editing papers. 13. ___ (A) Experienced skiers. ___ (B) Ski instructors. ___ (0 Beginning skiers. ___ (B) Economics. _ _ _ (C) Classical languages. 16. ___ CA) A few weeks after the beginning of class. ___ (B) In the first class meeting. _ _ _ (C) During the final exam. 17. _ _ (A) On a boat. _ _ _ (B) At an aquarium. _ _ (C) On a plane. IS. ___ (A) The habits of whales. _ _ (B) Types of whales. _ _ _. (C) Efforts to protect whales. 19. ___ CA) Students who own bicycles. _ _ _ (B) New members of the campus police force. _ _ _ (C) Pedestrians concerned about safety. 20. ___ (A) To descrihe some recent accidents. ~ _ (B) To introduce a new program. ___ (C) To provide safety and security hints. 76 Section 1 • Guide to Listening Comprehension LESSON 16 ANSWERING DETAll. QUESTIONS ABOUT MINI-TALKS Most of the questions about Part C talks are detail questions that ask about specific points in the talk. The majority of these questions are factual questions, asking about facts, reasons, places, or dates mentioned by the speaker. This type of question often begins, "According to the speaker, ..." Incorrect answers are often mentioned at some point in the talk but are not appropriate answers to the questions as asked. You are not permitted to take written notes while listening to the lecture. A few questions about the Part C talks are inference questions. Many of these questions begin, "What does the speaker imply about ..." or "What can be inferred from the lecture about ..." As in Part B, if a speaker emphasizes a point in the lecture by going back to it or repeating it, there will probably be a question about it. M: ... Now, in the days of the California Gold Rush, the journey by ship from the East Coast to San Francisco took about six months. Can you imagine that-gold-seekers spent six months at sea just getting to California! There will almost certainly be a question about how long it took to get from the East Coast to San Francisco during the Gold Rush. Exercise 16.1 Focus: Answering detail and inference questions based on specific points in Part C talks. Directions: You will hear three talks, each one divided into several short portions. After each portion, there will be a number of questions based on that part of the talk. Mark the best answer choice-(A), (B), or (C)-for each question. The first one is done as an example. -4))) Now start the audio. Talk A 1. ___ (A) It covers some difficult topics. ~ (B) It's unlike other biology courses. _ _ _ (C) It has never been offered by this department. 2. ___ (A) By visiting a coral reef. ___ (B) By going to the library. ___ (C) By going to a farm. 3. _ _ (A) Diving. _ _ (B) Photography. _ _ (C) Biology. 4. _ _ (A) As harmful. ___ (B) As easily damaged. _ _ _ (C) As frightening. 5. ___ (A) Transportation. _ _ (B) Housing. ___ (C) Basic equipment. 6. _ _ (A) Traveling by ship. ___ (B) Trying to get financial aid. ___ (C) Applying to another university. Talk B 7. ___ (A) They have such impressive appetites. ___ (B) They sometimes walk on two legs. _ _ (C) They frequently attack people. 8. ___ (A) They will eat anything except plants. _ _ (B) They eat only honey. ___ (C) They aren't limited to a few types of food. 9. _ _ (A) Five feet. _ _ _ (B) Ten feet. ___ (C) Fifteen feet. 10. _ _ (A) In Alaska. ___ (B) In Yellowstone National Park. ___ (C) Allover the United States. 11. _ _ (A) The kodiak bear. ___ (B) The grizzly bear. _ _ (C) The black bear. Section 1 • Guide to Listening Comprehension 12. ___ (A) They are sometimes not as friendly as they seem. _ _ _ (B) They are much more dangerous than grizzly bears. ___ (C) They look dangerous but are usually friendly. Talk C 13. _ _ (A) In the 1950's. _ _ (B) In the 1970's. _ _ (C) In the 1990's. 14. ___ (A) They fly off into deep space. _ _ _ (B) They remain in orbit forever. ___ (C) They burn up in the atmosphere. 15. _ _ (A) Three to four hundred. _ _ (B) Eight thousand. _ _ (C) Half a million. 16. ___ (A) They are too small. ___ (B) They are too far away. ___ (C) They are moving too fast. 17. ___ (A) A large booster rocket. _ _ _ (B) A piece of metal the size of an aspirin. _ _ (C) A tiny fleck of paint. 18. _ _ (A) Their high speed. _ _ (B) Their jagged shape. ___ (C) Their tremendous size. 19. _ _ (A) An aspirin. ___ (B) A piece of debris. _ _ _ (C) A model of the debris collector. 20. _ _ (A) They detect the debris. _ _ _ (B) They store the debris. _ _ (C) They collect the debris. 21. ___ (A) It has already been tested on Earth. _ _ _ (B) It has not been built yet. ___ (C) It has already been used on a spacecraft. Exercise 16.2 Focus: Answering detail questions about complete Part C talks. Directions: You will hear a number of Part C Mini-Talks. After each talk, there will be a number of questions based on that part of the talk. Mark the best answer choice-(A), (B), or (C)-for each question. The first one is done as an example. -4))) Now start the audio. 1. _ _ (A) Two years. ~ (B) Three years. _ _ (C) Five years. 2. ___ (A) Student fees. _ _ _ (B) Room-and-board charges at the dormitory. ___ (C) Student insurance rates. 77 78 Section 1 • Guide to Listening Comprehension 3. ___ (A) Its tuition rates are going up faster than the ones at Hambleton University. ___ (B) It has the highest tuition rates in the state. ___ (C) Its tuition rates are still lower than those at Hambleton University. 4. ___ (A) An executive on the Student Council. _ _ _ (B) A member of the Board of Regents. ___ (C) A spokesperson for the administration. S. ___ (A) A new dormitory will not be built. _ _ _ (B) The proposal to increase student services will not be adopted. ___ (C) The tuition will not be raised. 6. _ _ (A) A starfish. ___ (B) A salamander. _ _ (C) A mammal. 7. ___ (A) A snake sheds its skin and grows a new one. _ _ _ (B) An insect grows a new limb. _ _ (C) A baby gets its first set of teeth. 8. ___ (A) Embryonic cells. ___ (B) Specialized cells. ___ (C) Nerve cells. 9. ___ (A) To treat diseases among animals. ___ (B) To learn to speed up the process among lower animals. ___ (C) To apply what they learn to human medicine. 10. ___ (A) Before the Revolutionary War. ___ (B) During the Revolutionary War. ___ (C) After American independence. 11. ___ (A) His military service. _ _ _ (B) His political philosophy. ___ (C) His dictionary. 12. _ _ (A) T-H-E-A-T-R-E instead ofT-H-E-A-T-E-R. _ _ (B) L-A-B-O-U-R instead of L-A-B-O-R. ___ (C) N-I-F instead of K-N-I-F-E. Section 1 • Guide to Listening Comprehension MINI-TEST 3: MINI-TALKS Directions: Listen to the talks and the questions about them. Decide which one of the four answer choices-(A), (B), (C), or (D)-is the best answer to the question. Then mark the appropriate answer on the answer sheet. -4))) Now start the audio. 1. ___ (A) An improved toothbrush. ___ (B) Recent developments in genetics. ___ (C) New uses for bacteria. ___ (D) A means of fighting tooth decay. 2. ___ ___ ___ ___ (A) (B) (C) (D) They have many side effects. They occur naturally in people's mouths. They attack one type of bacteria. They cause tooth decay. 3. ___ (A) Microbiology. _ _ _ (B) Genetic engineering. _ _ (C) Dentistry. ___ (D) Civil engineering. 4. ___ ___ ___ ___ (A) (B) (C) (D) 5. _ _ (A) _ _ _ (B) ___ (C) ___ (D) PlanA includes dinner, but Plan B does not. Plan B is more expensive than Plan A. Plan B includes Sunday dinner, but PlanA does not. PlanA provides for three meals on most days while Plan B provides for only two. The Bengal Grill. Restaurants near campus. The Tiger's Lair. The dormitory cafeterias. 6. _ _ __ __ __ (A) (B) (C) (D) A receipt. A check. A student ID card. A friend. 7. ___ ___ ___ ___ (A) (B) (C) (D) Selling one's meal tickets to anyone else. Eating at a dormitory where one does not live. Eating at a cafeteria if one lives off campus. Going back for more than one serving of food. 8. _ _ ___ ___ __ (A) (B) (C) (D) They are open longer hours. They serve many more students. They offer a wider variety of foods. They charge lower prices. 79 80 Section 1 • Guide to Listening Comprehension 9. ___ ___ ___ ___ (A) Visitors to the Bronx Zoo. (B) Zoology students. (C) Visitors to the City Zoological Gardens. (D) New employees at the zoo. 10. ___ (A) They were usually sleeping when visitors were present. _ _ _ (B) They were uncomfortable because they were exposed to direct sunlight. ___ (C) They couldn't be observed because they were always in darkness. ___ (D) They couldn't sleep well and therefore became ill. 11. ___ (A) To make them visible to observers. _ _ _ (B) To put them to sleep. ___ (C) To simulate natural daylight. ___ (D) To allow them to see clearly. 12. ___ ___ ___ ___ (A) (B) (C) (D) Study marsupial mammals. Leave the zoo. Look at the nocturnal animals. Go to the World Down Under exhibit. Section 1 • Guide to Listening Comprehension 81 Mini-Lessons for Section 1 Idiomatic Expressions A knowledge of idioms is important for the Listening Comprehension section, especially Part A. These Mini-Lessons contain lists of some 300 expressions and their definitions as well as exercises to familiarize you with most of these expressions. Many of the expressions listed here have appeared on the Listening Comprehension sections of TOEFL exams in the past, some of them several times. Notes: 1. If a phrase contains a word in parentheses, that word is used only if the verb is followed by an noun or pronoun. f Example: catch UP! (with) You go ahead. I'll catch up later. (no noun or pronoun) I'll catch up with you later. (pronoun) 2. The words one and someone are used to indicate that any pronoun (or sometimes a noun) can be used in this expression. Example: on one's own How long have you been on your own? Tom's been on his own for several years. Mini-Lesson 1.1 about to almost ready to above all most importantly add up make sense; be logical all at once suddenly; without warning ! ' all of a sudden all at once; suddenly ~ a matter of fact in reality; actually as a rule generally; customarily ~ at ease not nervous; calm ~t the drop of a hat quickly; without any preparation time back out (of) withdraw an offer bank on depend on; count on be my guest do what you want; feel free; help yourself be rusty need practice or review beats me I don't know; I have no idea (often used in response to a question) 0- better off in an improved condition. \bite off more than one can chew take on more responsibility than one can handle C""' bound to certain to; sure to ~break down stop functioning (a machine, for example) ~!>reak in (on) interrupt ~reak the ice break through social barriers (as at a party) ~reak the news (to) inform; give bad news 'break up end (a meeting, for example) ~break up (with) stop being a couple (a boyfriend and girlfriend, for example) ( a breeze something very simple and easy to do S 82 Section 1 • Guide to Listening Comprehension ~. "hnng about L~bring , up cause to happen (1) raise (a child) (2) introduce (a topic, for example) review; study; practic~ bump into meet unexpectedly; run into by and large mostly; generally; on the whole by heart by memory; learned word for word &~y no means in no way; not at all C.:::: brush up on cy Exercise: Fill in the blanks in the sentences or dialogs with idioms from the list above. There will be one word per blank. It may be necessary to change the verb forms in order for the sentence to be grammatically correct. The first one is done as an example. 1. "Can you talk now?" "No, I'm about to go to the grocery store, but I'll call you as soon as 1 get back." 2. "You're probably too tired to play another game of racquetball, right?" "I'm not that tired. really. game." , I'd enjoy another 3. "Will you support my proposal at the meeting?" "Certainly. You can __ .________ my support." 4. 1 was talking to my aunt when suddenly my cousin George Ann our conversation. 5. "I understand Diane lost her job." "Yes, but she's actually ____ . She found a more interesting job with a higher salary." 6. _~ ____ , Carlos is very punctual, but he sure 'was late tonight. 7. My car _ _ _ _ _ _ _ _ last week, and 1 had to take the bus to work until it was repaired. 8. I was taking a quiet walk last night when, _ _ _ _ _ _ _ _ _ _ _ _ _ _ _ _ , there was a loud explosion. 9. Kent is fail that class if he doesn't start studying. 10. They _~________~ their children to be honest. 11. There were a Jew things I didn't like about Professor Wong's class, but _ _ _ _ _ _ __ ____ I enjoyed it. 12. "I think Matthew was cheating on that quiz." "That doesn't _____ . Why should the best student in the class cheat?" 13. "Can I have another sandwich'" "Sure. _________.... ______ . I made plenty." 14. Actors and actresses must know their lines _ _ _ _ _ _ __ IS. If you don't want to talk about this problem, why did you ____ it _ _ __ 16. Many accidents are _ _ _ _ _ _ _ _ by carelessness. 17. Their team won the game, but they close garne. dominated it. It was a very 18. "What a boring party. No one is talking to one another." "Maybe we should put on some music and start dancing. That might ___ ~ ._ _ __ 19. "Have you studied Spanish before?" "Yes, but it's been years since 1 took a Spanish class, so I'll need to _ .. ___ it before I go to Venezuela." 20. "You're taking five classes this term?" "Yes. and I'm havim! trouhle getting caught up. I'm afraid 1 .. _ .________ this time." - .. _._- -~--- Section 1 • Guide to Listening Comprehension '83 21. "You're all packed and ready to go, I see." "I could leave _ _ _ _ _ _ _ _ _ _ _ _ _ _ _ _ _ _ _ _ _ _ __ 22, "Do you know what the name of this street is?" _ _ _ _ _ _ _ _ . This is the first time I've ever been in this town." 23, "How's that biology class youre taking?" . We'..,., just been going over things I studied last semester." "So far, it's been 24. "What time did the part\ __ last night?" "I don't know. It was stU] buing on when 1 went home." 25. "I was awfully nervous when I gave that speech." "Really? You hid it well. 1 thought you were completely _ _ _ _ _ _ __ Mini..Lesson 1.2 call it a day stop working for the day; go home call off cancel call on visit cahn down relax care for (1) take care of (2) like; feel affection for catch on become popular catch on (to) understand; learn catch up (with) go as fast as; catch check in (or into) register (at a hotel) check out (00 (1) leave (a hotel) (2) take material (from a library, for example) cheer up become cheerful; be happy chip in (on/for) contribute clear up (1) clarify; make understandable (2) become nice and sunny (used to talk about the weather) conle across find; meet; encounter come around (to) begin to change one's opinion; begin to agree with come down with become sick with (an illness) come up with think of (an idea) cost an arm and a leg be very expensive count on depend on; rely on; bank on count out eliminate; no longer consider as a factor cut off stop; discontinue (a service, for example) cut out for have an aptitude for; be qualified for Exercise: Fill in the blanks in the sentences or dialogs with idioms from the list above. There will be one word per blank. It may be necessary to change the verb forms in order for the sentence to be grammatically correct. 1. The reception in the garden was because of a thunderstorm. 2. Don't get so excited. Just _ _ _ _ _ _ _ and tell us what happened. 3. I was looking up some information in the almanac when 1 _ _ _ _ _ _ _ _ an interesting fact. 4. I can ____ _ _ ___ my car. It's very dependable and never breaks down. S. How did you _ _ _ _ _ _ _ _ _ _ _ _ such a strange idea? 6. I arrived in town last night at seven-thirty and _._. __________ my hotel at around eight. This morning I plan to at about nine. ---- -- --- 84 Section 1 • Guide to Listening Comprehension 7. "Did you rent this videotape?" "No, I ~_____ it ______ the library." 8. You look tired. Why don't we____~ and finish up tomorrow? 9. It won't be too expensive to buy Professor McMillen a present if we all _ _ _ ~.. ___ 10. If Arthur doesn't pay his electric bill soon, the utilities company might_ . __ __ electricity. his 11. "I don't understand this theorem at all." "Talk to Professor Adler. I'll bet she can ______ your confusion." 12. Who ______ _ your cat while you were out of town? 15. A good stereo system doesn't have to __________ . _________________ __ . You can find one for a reasonable price. it at all. 14. Cauliflower isn't my favorite vegetable. In fact, I don't 15. "I'm depressed. I didn't do very well on the first quiz." _ _ _ !That quiz only counted for 10% of the total grade, and I'm sure you'll do better on the other tests." 16. "How did Eric do in the cross-country ski race?" "He got off to a bad start, so he never _ _ _ _ _ _ _ _ _ _ _ _ the leading skiers." 17. "Why did Brenda drop out of business school?" "She decided she wasn't _ _ _ _ _ _ _ _ _ _ _ _ a career in business. She's going to study art instead." 18. This song wasn't very popular when it was first recorded, but now it's starting to _ _ __ 19. "Your roommate is still planning to vote for Smithson for president of the Student Council?" "Yes, but I'm going to keep talking to him. I think eventually he'll ~~ __.. _________ our point of view and vote for Brannigan." 20. That saleswoman her clients at least once a month because she thinks personal contact is important. 21. "Is it still raining?" "No, the rain has stopped and it's starting to ________ Mini-Lesson 1.3 day in and day out constantly; for a long time die down become less severe; quiet down do over do again; repeat do without not have down the drain wasted: done for no reason (work, for example) dream up invent; think of; come up with drop (someone) a line send someone a letter drop in (on) visit informally drop off (1) leave something (a package, for example) (2) take (someone) home; let someone out of a car drop out (00 stop attending (classes, for example) easy as pie very simple; a piece of cake eyes bigger than one's stomach said of someone who takes more food than he or she can eat fall behind not move as quickly as; lag behind fall through fail to happen Section 1 • Guide to Listening Comprehension 85 a far cry from not similar to; not as good as fed up (with) not able to tolerate; disgusted with; annoyed by feel free do something if one wants feel like be inclined to; want to feel like a million dollars feel very good feel up to feel able to do something; ready to few and far between uncommon and infrequent figure out understand; solve Exercise: Fill in the blanks in the sentences or dialogs with idioms from the list above. There will be one word per blank. It may be necessary to change the verb forms in order for the sentence to be grammatically correct. 1. It took me hours to _________ how to record programs on my DVD player. 2. If you could _ _ ___ the laundry on the way to work, I'll pick it up on Monday. 3. "What a wond~Tful masquerade party!" "People certainly ___ """ ______ some interesting costumes, didn't they?" 4. "Do you _____ going out tonight?" "No, I'd rather stay home and read." 5" "Why have you _ _ _ _ in your French class?" "I was sick and I missed a few classes. But I'm studying hard to catch up." 6" "Why do we need to get gas now? We have quite a bit left." "This highway goes through some very empty country, and gas stations are _" _ _ _ _ __ 7. Howard's teacher wasn't satislled with the work he had done, so she asked him to _ _ _ _ it 8. I'm _ _ _ _ _ _ ""_________ my roommate's lack of responsibility. He never pays his bills or his share of the rt'nt on time" 9. "Isn't your class picnic today?" "No, our plans for the picnic" _ _ _ _ _ _ _"_ 10" "The food at that new restaurant isn't bad." "It's all right, but ___________ "____ ""______ it's the food at Mario's. Now that is a great restaurant!" 11. Ben had to " _ _ _ _ the university because of financial problems. 12. After blowing furiously all day, the wind fmally _ __ 13. "Do you telephone friends before you visit, or just __"___ _ ____ them?" "It depends. If they're close friends, I just visit them. If they're acquaintances, I generally call first." 14. "I'm tired of the same old routine." "I know how you feel. I get tired of doing the same things _ _ _ _ _ _ _ _ _ _ _ __ _ _ _ _ _ _ _ _ too." 15. "Ralph really loaded up his tray with food." "He'll never eat it all. Ralph's _ _ _ _ are _ _ _ _ _ _ _ _ his _____ 16. "I need to use a computer for a few hours." ____ to use my laptop computer. I don't need it this morning." 17. "I've missed James since he moved to Seattle." "You should him _____________ "_ _ and let him know how you're doing. I'm sure he'd love to get a letter from you." 86 Section 1 • Guide to Listening Comprehension Mini-Lesson 1.4 till in write in a hlank (on an application form, for example) nIl in (for) suhstitute for nIl one in provide missmg information f'ill out complete (an application form, for example) nnd out learn; discover a fish out of water someone not in his or her normal surroundings f'tx up repair; renovate follow in one's footsteps do what someone else did (especially a parent) for good permanently; forever for the time being temporarily; for now from out of the blue unexpectedly; without warning get a kick out of (doing something) enjoy; have fun doing something get along with have good relations with get carried away go too far; do too much; buy too much get in one's blood become a habit; hecome customary get in over one's head take on too much responsibility; bite off more than one can chew get in the way block; obstruct get in touch with contact get off leave (a vehicle) get off the ground start to he sllccessful get on board (a vehicle) get over recover from (a disease) get rid of discard; no longer have get under way begin; start give (someone) a cold shoulder act unfriendly toward someone; ignore give away distribute (for free) Exercise: Fill in the blanks in the sentences or dialogs with idioms from the list above. There will be one word per hlank. It may be necessary to change the verb forms in order for the sentence to be grammatically correct. 1. "How did you ____ ~.~~_ where Warren lives?" "I just looked it up in my address book." 2. ''I'm interested in the job that was advertised in the newspaper." ________ this application form." "Fine. Just _____ 3. Don't forget to ___~_ the date on your check. -J:. "Is Agnes still mad at you?"' "I suppose so. 1 saw her at a party last weekend, and she just _._._____ me _. 5. "Are YOll moving to Baltimore "No, just __ . __ ..__ ~_ ... _.. _ ?" I'll be back here in a month or two." 6. "That old paint that you have stored in your garage is a fire hazard." "You're right. 1 should_ _ ____.___ it." -, Some companies __ ... ___ consumers with them. ~. ___ free samples of new products in order to f;lmiliarize 8. "How's that advanced computer class you're taking, Polly?" "Not so good. 1 can't understand a word that the teacher or any of the students are saying. 1 really feel like ~__ _~ ___.._. . ___....... __ ..._......._._._ _._.._ _ <) ""Don't you just hate all this graffiti?" ·It is ugly, isn't it? I've never understood why people ... _...... _____ writing on walls. It doesn't seem like much fun to me." Section 1 • Guide to Listening Comprehension 10. When the train stopped, a mysterious looking woman in a black raincoat ___________ . train and found her seat. 11. "Maxwell's project will be very successful, I think" "Oh, I don't knOw. I'm not sure it will ever _____ __._ __________._.____ _._._. 87 (he 12. Do you _... your new roommaliC, or do you two argue? 13. "That run-down old house (iut navid bought looks terrific." "Yes, he's it _ beautifully, hasn't he?" 14. "Has Edward . ________ you lately?" "No, he hasn't. I don't think he has my new telephone number." 15. "Will the concert start soon?" "It should ______ _ _ _ _ _.___ any minute no\v." 16. Living by the ocean really ___ _ _ _ _ your ____ . Once you've lived here, you never want to leave. _____ the bus here. 17. This is the last stop. Everyone has to 18. Professor Dunbar came down with the flu, so her teaching assistant ________ ____ her for a few days. Mini-Lesson 1.5 get the hang of something learn how to do something give a hand applaud; clap give a hand (with) assist go easy on not punish SC\ trely go on (with) continue go overboard do too much; buy too much go with (1) accompany (2) look good together; complement (for example, two articles of clothing) go without saying be clear; be obvious grow up to mature; to become an adult hand in give back to; return hand out distribute hangon wait hard to come by difficult to find have a heart be compassionate; show mercy have a hunch have an intuitive feeling have a word with (someone) talk to someone briefly have on wear have one's hands full be very busy; have a challenging job have the time of one's life have fun; have a great time hear fIrsthand (from) get information directly from someone hear from be contacted by; be in touch with hear of know about; be familiar with hit it off become friendly (especially at a first meeting) hit the road leave; go away holdon wait hold on (to) grasp hold still not move hold up delay 88 Section 1 • Guide to Listening Comprehension Exercise: Fill in the blanks in the sentences or dialogs with idioms from the list above. There will be one word per blank. It may be necessary to change the verb forms in order for the sentence to be grammatically correct. 1. Everett was born in the South, but he _ _ _ _ _ _ _ _ in Michigan. 2. At the beginning of the class, the instructor students they had ten minutes in which to finish. the quizzes and told the 3. After ten minutes, the students ___ ._.__ their quizzes to the instructor. 4. "Have you ever William Carlos Williams?" "I believe so. He was a poet, wasn't he?" 5. "Hello, is Gina there?" "Yes, _ _ _ _ _ _ _ _ a minute and I'll get her." 6. "You're graduating next month, right?" "Yes, but I'm going to my studies in graduate school." 7. while I take your photograph. I don't want the picture to be blurry. 8. "I got stopped by the police for speeding. I have to pay a big fine." "Well, you could talk to the judge and ask him to reduce it. Since you've never been stopped for speeding before, maybe he'll .. _______________ you." 9. "Do you like this blouse?" "Yes, but I think the grey silk one would ______ better _____ your jacket." 10. "So, Dave is teaching you how to wind surf?" "Yes, and he's such a good teacher that I'm already _ _ _ _ _ _ _ _ _ _ _ _ _ _ _ _ it." 11. The audience __ ..__ the cast a big ____ after their wonderful performance. 12. "Do you think Iris will pass the history test?" "That . In fact, she'll probably have the best grade in the class." 13. "What _______ . your flight?" "There was a big snowstorm in Denver that delayed a lot of flights." 14. "Have you Maureen since she went to Hawaii?" "Yeah, I got a postcard from her yesterday. She said she's _ _ _ _ _ _ _ _ _ _ __ ____ her and never wants to come home." 15. Can you _ _ _ me _________ ._.. ___.__ this luggage? It's too heavy for me to carry myself. 16. The wind is starting to blow. You'd better ___.__________ your hat. 17. "Oh, you bought that new book by Richard Stone." "Yes, but that book is _ _ _ _ _ _ _ _ _ _ _ _ _ _ _ _ . I looked for it in three or four bookstores before I finally found it." 18. It's getting late. I'd better if I want to get home by midnight. 19. "Can 1_______ ______ you now, Professor Rivera?" "I've got to go to class right now. Drop by my office later, and we'll talk then." 20. "You sure bought a lot of groceries." "Yeah, I guess I ______ . I should never go grocery shopping when I'm hungry." Section 1 • Guide to Listening Comprehension 89 Mini-Lesson 1.6 in a nutshell in summary; in brief in favor of in agreement with; supporting in hot water in trouble in no time very soon: \'ery quickly in person face to face (not by telephone, letter, etc.) in store in the future; coming up in the dark not knowing: confused in the long run over a long period of time in the same boat in the same situation; having the same problem iron out solve (a problem) join the club have the same problem as other people jump to conclusions form opinions without sufficient evidence keep an eye on watch; take care of; look after keep an eye out (for) look for keep on (with) continue keep track of know where something or someone is keep up (with) maintain the same speed as kill time spend time doing unimportant things (before an appointment, for example) know like the back of one's hand be very familiar with layoff put out of work learn the ropes become Lli~;iliar with; get used to; get the hang of leave out not include; omit leave someone/something alone not disturb let someone down disappoint let up decline in intens;ty (rain ror example) look after take care of. mind look for try to locate look forward to anticipate (with pleasure) look into investigate Exercise: Fill in the blanks in the sentences or dialogs with idioms from the list above, There will be one word per blank. It may be necessary to change the verb forms in order for the sentence to be grammatically correct, 1. Will you ________,_. __ ,.. __ ._______,___.__ my dog while I go in the drugstore? 2. Kathy's daughter has such short legs that she has a hard time _ _"_ _ _________ the other children, 3. The store had to __________ a number of clerks because sales were down. 4, "You must be anxious to go on your vacation." _ ___ this trip." "I certainly am. I'm really ._____ ..______ 5, Don't ___________ , _. ________ . Maybe your jewelry wasn't stolen after all. 6. "1 need to find a new apartment." "There might be some vacancies in the building where I live. I'll _ _ _ _ _ _ _ _ ____ one." - The compal~Y may lose some money now, but _ _ _ _ _ _ _ _ _,_____._-' ______ , this is a good investment. 8. Alex complained that no one invited him to any social events and that he felt ____ ,_ 9. "Can you hurry mer here' I need to see you right away." "Sure. I'll be there , _______ ._.. 90 Section 1 • Guide to Listening Comprehension 10. "May I help you, sir?" "No, I'm just looking around and trying to .__.___ some _. ___ until my wife finishes shopping." 11. "Has it stopped raining yet?" "No, but it's beginning to ____ ._ ~_. _ _ a little." 12. "I can't go to Daryl's party this weekend. I have to study." . ._______. ___ ~..... _. I've got to study too." "Guess we're ___. _. 13. "Should we stop and spend the night at this motel?" driving for a few more miles." "No, let':-- __._ 14. If you find a baby animal in the woods, don't touch it. Just _ it 15. I al! over town . a good used car, but 1 couldn't tind one. 16. Stella's SI ;ter _____ ._ her baby while Stella is at work. 17, "Ifyou don't know how to use this software, why don't you ask Joanne to help?" "1 did ask her, but I'm still _ ______ _____ . I didn't understand a word she said." 18. The police are _______ .______ the crime. 19. "Did Amanda ever complete her project?" "She's almost finished. She just has a few minor problems left to __ . 20. Alfred is ~____ ._....... _.. _____ with his boss because he didn't finish an important project by the deadline. 2l. "Has Marilyn gotten used to her new job at the bank yet?' "It took her awhile, but I think she's finally _ _ _ _ _ _ _ _ _ _ ._~ there." 22. Are you .. __ . ___. . _.. __ this proposal or against it? 23. "Are you familiar with tl1is neighborhood?" "I grew up here, so 1 ~__ _ ___ my _ _ __ 24. "Is your roommate at home now?" "I have no idea. I can never ~_. ___. _~____~_ his comings and goings." 25. "I understand that you have a new dean over at the Business School." "Yes, his name is Dean Nishimura. He has a completely different philosophy of business education from the one Dean Woodford had, so I'm sure that some big changes arc _.______ ___ _ for us." Mini-Lesson 1.7 look like resemble look out (for) be careful look over examine; read look up (1) find information (especially in a reference book) (2) try to locate someone look up to respect; admire make a fool of oneself act embarrassingly make a point of make a special effort make ends meet balance a budget make sense (of) be logical and clear; understand make up invent; create make up one's mind decide make way for allow space for; provide a path for mean to intend to Section 1 • Guide to Listening Comprehension 91 mixed up confused music to one's ears something that sounds pleasant a nervous wreck someone who is very nervous next to nothing very little (money, for example); cheap no doubt about it certainly; definitely no harm done there was no damage done not at all not in any way; not to any Licgrce not believe one's ears (or eyes) be unable to believe what one hears (or sees) not think much of not like; not have a good opinion of odds and ends small, miscellaneous items an old hand (at) an experienced person on edge nervous on end consecutively, without a break (days on end, for example) on hand easily available on needles and pins nervous; an'inus on one's own independent' on second thought after reL.,blclering on the go always busy; alwaY",llJ"'clng on the tip of one's tongue :drnost able to remember on the whole in general out of (something) nO! j;,,\ing something out of one's tnind insane: illogical; irrational out of order broken; not functioning propcr!~ out of the question definitely not; impossihk: over and over again and again; repeatedly over one's head not understandable (a joke, for example); obscure Exercise: Fill in the blanks in the sentences or dialogs with idioms from the list above. There will be one word per blank. It may be necessary to change the verb forms in order for the sentence to be grammatically correct. l. "Have you _______ this contract yet''' "Not yet. I'll try to read it this weekend." 2. "I think I'll have the prime rib, waiter." "All right, sir." "Wait, no- ___ ~___ _____________ , I think I'll have the chicken." 3. Is this story rrue, or did you just __.. it 4. You can't get a soda from that machine. There', :, sign on it that sa\~" _~_______________ her lather because of all the help and good advice he's 5. Shern given her. 6. I don't have much cash machine. but I can get some from an automatic teller 7. What a confusing movie! T couldn't __ _ it. 8. "You did a fine job on this research paper. especially on the hlbliography." ___ getting the bibliography exactly right. I did "Thanks. T it ______ . 9. "What's Fntz's "It's ___________ ____ until it was perfect." c\H!~in'" name'" _____ my _______ , but I can't quite remember." 10. "Hello. I'd like to rest"ne ;t room' r this \yeekend." ____ _______ . The hotel is fully booked this 'Tm afraid that's __ ~~ ______ _ weekend:' 92 Section 1 • Guide to Listening Comprehension 11. "Do you han' ;U"· fresh reaches?" "Sorry, I'm __.__ ___ _ __ them. I just sold the last crate of peaches." 12. "Listen to the roar of the engines." "Yeah, it's _ _ _ _ _ ______ my _______ I just love going to these car races." 13. The plane was delayed for hours _______________ I thought we'd never get off the ground. 14. "Have you finished moving into your new apartment:'· "Almost. There are still a few ______ ____________ ____ move today." in myoid apartment that I need to 15. "Brad sure is busy, isn't he?" "Yeah, he's involved in so many activities that he's always _______ 16. There are so many interesting dishes on the menu that it's hard for ill': 'xhich one to ( ) f ( k r __________ alligators. 17. If YOLI go into the swamp, 18. "When will you be informed o( the test results?"' "Not until Monday, so I'll be ___ _ _________ to ________ my ______ all wed end," 19. "Do you know what the capital of Suurh Dakota is?" "I'm not sure. Let's it in this atlas." ________ a big new 20. They're going to tear down those old warehouses to __ hoteL 21. "I like that painting you bought. Did it cost much?" "No, the artist sold it to me for _____________________ _ 22. "How was your final exam, Laurie?" "A couple of questions were triCky, but ____ _ _ _ _ _____ it was pretty easy." 23. ''I'm sorry I knocked that vase over. I didn't mean to." ______ ._________________ . It wasn't damaged." 24. I must have been _ _ _ _ _ _ _ _ my _____ when I signed the lease on this apartment. can't afford this much rent. 25. He didn't break the plate; it was an accident. 26. "I didn't know ~·otl could play horseshoes so welL" "Oh, I'm ______________________________ horseshoes. I've been playing since I was a kid." 27. You must be ______ . This isn't River Street; it's Laurel Avenue. 28. "How long have you been living alone?" since I graduated from high schooL" "I've been ____ my -. \. \. 29. "Did you find that lecture boring?" __________ __________________ In fact, I thought it was fascinating." 30. "Christine is so funny, she should be a stand-up comic .. "I suppose, but a lot of her jokes go right _____ I'll Mini-lesson 1.8 part with no longer have; get rid of; not be in the company 01 pass up not accept; not choose pass with flying colors do very well (on a test) pat oneself on the back congratulate oneself pay attention (to) concentrate on; focus on pick out choose; select I just don't get them." Section 1 • Guide to Listening Comprehension pick up 93 (1) take something from a surface (for example, a floor) (2) go to a location and get someone or something (3) learn (especially without formal training) pick up the tab (for) pay for the picture of a perfect example of something play it by ear do something without a definite plan play it safe choose a cautious plan point out indicate a pretty penny a lot of money pull one's leg jokcwith someone; make up a story push one's luck to continue dl ling sumething too long; to keep taking chances put aside save for later; set ;l'·jl, Ie put away return something tf, its proper place put off delay; postpone put on begin to wear put together assemble put up with tolerate Exercise: Fill in the blanks in the sentences or dialogs with idioms from the list above. There will be one word per blank. It may be necessary to change the verb forms in order for the sentence to be grammatically correct. L Vanessa is allergic to tobacco smoke, so she can't __.____ . .. _ _ smoking. 2. "That conference you attended in San Diego must have been \cry expensive." "Yes, hut fortunately, the company I work for _____ ______________ .__ it:' 5. J a~ked my teacher to ____ ._._ . __________ the mistakes that I made in my essay so that I could correct them. 4. "What do you want to do tOl1lorro\,\T' "I don't know. Let's just ____ __ ______ _ 5. "You should get rid of that ole! leather jacket." "I know. but I hate to _____________ .___ it. I've had it for years." 6. "How did you learn hem to m;lke sllch beautiful pottery? Did you take a class in ceramics"" "No, I just ______ ,'11 nl\' own." 7. "Can you read that sign)' 'Just a minute. Let me 8. It took Linda week:- to . that thousand-piece jigsaw puzzle. 9. This bike cost ___ , but 1 think it was worth it. 10. "1 passed tht' llr"t two quizzes in this class, and f scarcely studied for them at alL" "WelL if I were you, I wouldn't _______ n1;_ _________ any further. You should study for the next quil l11'Cause it's going to be a lot harder." ____ the laundry that you dropped off at the cleaners this morning. 11. I'll 12. TllC child toy box. her toys from the floor and then 15. Bert and Marv had to wasn't feeling \\ell 14. "Whq them _ ____ .________ their dinner party until next weekend hecause Bert ________ that tie for you?" "No one:. I chose it mysC\f" 15. "Are you going to take that job-" "No. I \l(THini 10 it 16. ___. _....._ in her because I don't want to relocate." as I read the directions or you won't understand what to do. 94 Section 1 • Guide to Listening Comprehension 17. "1 finally finished collecting all the materials I need to write my report." "Great, but don't be too quick to _ _ .__. yourself ________________ . You still have to write the report and then word-process it." 1R. "How did vou do on your final exams'" "Great! I them all ___ .___ _. 19. I'm going to __ this magazine _. ___ . 20. I wouldn't belie' " :t t<.>r now and read it later. word Lynn told you.'-.llcs just _. __.____ . your _._. _____ . Mini-Lesson 1.9 right away immediately ring a bell (with) sound familiar to rough it experience somewhat difficult or primitive conditions rule out say something is impossible: eliminate run a temperature have a fever run for office try to get elected run into (1) meet unexpected I,: bump into (2) collide with run late be late; be in a hurry run of the mill ordinary run out of exhaust the supply of save one's breath don't bother asking someone search me I don't know; 1 have no idea; beats me see eye to eye (with someone) (on something) have the same opinion; be in agreement see (someone) off accompany (to an airport or train station, for example) see to take care of; check on; fix serve one right receive the proper punishment; get the penalty one deserves short for a nickname for show around orient; give a tour show off try to attract attention by unusual behavior show up arrive shut down close sign up (for) enroll (f<.>r a class, for example) sing another tune change one's opinion; feel differently size up measure; estimate sleep on it postpone a decision until the next day slowly but surely gradually; steadily but not quickly snowed under very busy so far, so good up until now, there are no problems sooner or later at some indefinite future time speak one's mind say what one is thinking speak up speak more loudly speak up for support verbally spell out (for) make something very clear; explain in detail Exercise: Fill in the blanks in the sentences or dialogs with idioms from the list above. There will be one word per blank. It may be necessary to change the verb forms in order for the sentence to be grammatically correct. I. "Has JOh;l ~"ne "Yes, I just _ back to :Vlinneapolis yet?" him _______ at the airport." Section 1 • Guide to Listening Comprehension 2. "This pipe is leaking again." "We'd hetter have a plumher ____ _ 95 it." 3. I was late because I _________________... ___ gasoline. 4. Write down your ideas ___._. __ ._ If you wait to write them down, you may forget them. 5. When the factory _____ _ _______ , hundreds of workers were laid off. 6. "Did you ____.____ ...._______..._ ._____ Professor Carmichael's class?" "No, I decided to take Professor Knudson's class instead." 7 "Vicki, how's that project you're working on coming along?" " ______ .___.___ ' __.________ , but the tricky part will be next week." 8. "Have YOll ever heard of an actor named Anthony ReecP" "Hmmm. I don't think so. The name doesn't me at all." 9. "Frank duesn'{ take a hint very well, dOl'~' he l " "No, you han' to things ___ ___ ______ Frank. lie likes even thing crystal clear." 10. I'd heard that the clothe., in this store "ere very nice, but I found them _ 11.- Bennet thought those stories Tina [(illl abollt )'Oll were pretty funny." "They weren't funny; they were emharrassing. Bennet would be _. _ _ _ _...________ if Tina had told that kind of story ahout him." 12. Norman doesn't like to _ _ _ _ _ _ _ _ when he goes on vacation. He prefers to stay at luxury hotels. 13. I don't like to go to parties too early. I'd rather. 14. It's expensive to _. ____ ._____ money. a little bit late. these days. Political campaigns cost a lot of 15. "So, do YOll plan to buy this motorcycle or not?" "I'm still not sure. Can I _ _ _ _ it and let you know tomorrow morning?" 16. "Have you seen the campus yet?" "Yes, my cousin Melissa is a student there, and she _______ me 17. "I'm collecting money for the Red Cross. I think I'll ask Pat to contribute." ., ______ your .. __. . __ . Pat never contrihutes to anything." 18. "Are you going to medical school?" ";'\Jot this year, hut I wouldn't _____ it 19. I ________ in the future." . _ myoId friend Leslie downtown yesterday. 1 hadn't seen her for months. 20. "1 undcrsta nd you're learning how to speak Russian." 'Yes, and it was really hard for me, especially at ttrst. Now, though, 1'm ______ _ ... ____ getting the hang of it." 21. "Your sister's name is Jessie?" "That's what everyone calls her-it\ ____ _._. ______ Jessica." 22. "Have a busy night at the restaurant· "We weren't just husy-we were _____ .! I've never seen so many customers'" 23. "I feel terrible. 1 have a terrihle cold or maybe even the flu," "Are you______ .___ ' If you have a fever, then you probably have the flu." 24. My brother and I agree on most issues, but 1 sure don't _ __ him ____.__ this proposal to build a new stadium. 96 Section 1 • Guide to Listening Comprehension Mini-lesson 1.10 spick and span extremely clean; spotless stack up against compare with stamp out eliminate; wipe out stand for (1) tolerate; pm lip with (2) symbolize represent stand out be noticeahle stay out not come hOIlH> stay out (or up) to all how·s come home (or go to bed) very late stay up not go to bed stick with not change; stay with stock up on get a large supply of something a stone's throw from not far away from; close to stop by visit informally; go to see straighten up clean up; make tidy stuck with have something one cannot get rid of take a break stop working for a short time take a lot of nerve require a lot of courage take a lot out of (someone) be hard on someOIW; drain energy from someone take advantage of lItilize; make use of; exploit take after resemble; look like (especially an older relative) take apart disassemhle take it easy relax; calm down Exercise: Fill in the blanks in the sentences or dialogs with idioms from the list above. There will be one word per blank. It may be necessary to change the verb forms in order for the sentence to be grammatically correct. 1. Vaccines have permitted doctors to virtually smallpox and polio. 2. "How late do you usually ____ >___ ?" "I'm normally in bed by eleven on weekdays." a number of diseases, including 3. "How late do you usually on weekends?" "1 sometimes don't come home uatil two or three in the morning." 4. "Do you __ _ "1 don't think I look mllLil __ your mother or father?" like either one of them." 5. Earl had no troub1e _____ the engine on the lawn mower, hut then he couldn't put it back together. 6. You look a little tired. Why don't you _____________ _> _______ and finish your homework later? _____ cheating. When she caught one student cheating on the 7. The teacher won't mid-term exam, she gave him a zero on that test. 8. "I tried and tried to find a huyer for this old car." "Looks like you're___ it for now." 9. "My brother is going to invest all his savings in a new business venture." "That _____ _____ ________ > I'd be afraid to take a risk like that." 10. "I give up. I can't solve this chemistry problem." ____ ______ it Eventually, you'll figure it out." 11. We have to leave the apartment ________ . The landlord said that if it wasn't clean when we moved out, we'd lose part of our security deposit> Section 1 • Guide to Listening Comprehension 97 12. Don was wearing jean-; and a T-shirt while all the other guests had on formal dinner wear. He really_. __ 13. ''I'm nervous ahout my interview." ., ______________ . You'll make a better impression if you're relaxed." 14. The fifty stars on the American flag __ ._._ ..___ the fifty states. \Ol!l" ;lpartment before guests ___._____... _ _. __ ..____ 1'· 15. "Do you ____ .. "A little hit. I don't mind if it's a little messy, but I don't want it to look like a dIsaster area." 00._ _ _ _ 16. "My new roommate is from Italy." "You should ______ ._..___ _ _ _ _ this opportunity to learn some Italian." 17. "I just heard on the news that the Florida orange crop was damaged by the hurricane last week, and that orange juice prices are going to go way up:' "If we had a big freezer, we could ____. __.... ___ .. ___.. frozen orange juice now and we wouldn't have to pay those prices." 18. "Do you live near Cecilia?" hers." "Oh, sure. My apartment building is just ______ . _ _ _._. 19. "So you ran in that ten-kilometer race?" "Yes. hut it _ _ _ _ _ _ _ _ _ _ __ me. I can hardly move." 20. "Tim, you've eaten at hoth these restaurants-how does Chez Michelle ____ the Oak Room?" "Oh, they're both good. I think the Oak Room has slightly better food, but the service is hetter at Chez Michelle." Mini-Lesson 1.11 take a short cut take a more direct or faster route than usual take off (1) remove (clothing, for example) (2) ascend (a plane, for example) (3) hecome popular quickly take over assume control or responsibility take part (in) participate in take the plunge finally take action; do something different take time off (from) take a vacation or a break from work or school take up begin to study some topic or engage in some activity talk down to speak to someone as if he or she were a child; patronize talk into persuade; convince talk out of dissuade; convince not to do something talk over discuss tear oneself away from something stop doing something interesting tear up rip into small pieces tell apart distinguish things are looking up the situation is improving think over consider throwaway discard; get rid of throw cold water on discourage; force to cancel (a plan, for example) throw the book at someone give someone the maximum punishment try on test clothing before buying (for size, style, and so on) try out test a product before buying try out (for) audition for (a role in a play, for example); attempt to join (a team, for example) 98 Section 1 • Guide to Listening Comprehension Exercise: Fill in the blanks in the sentences or dialogs with idioms from the list above. There will be one word per blank. It may be necessary to change the verb forms in order for the sentence to be grammatically correct. 1. You'd better these gloves before you buy them; they may not fit. 2. At first, 1 didn't want to go to the party, but I'm glad Annette me it. It was fun. 3. The spy the document so that no one else could read it. 4. The twins look so much alike that almost no one can them _ _ __ 5. "I need more excitement in my life." "Why don't you give up stamp collecting and _ _ _ _ _ _ _ _ sky-diving instead?" 6. "Who made the final decision?" "All of the people who were at the meeting _ _ _ _ _ _ _ _ _ _ _ _ the decision-making process." 7. Most air accidents take place when a plane is _ _ _ _ _ _ _ _ or landing. those computer games and do your 8. You've got to homework. 9. Dan was afraid the judge would _ _ _ _ _ _ _ _ _ _ _ _ _ _ _ _ him because he had been charged with the same offense several times. 10. _ _ _ _ _ _ _ _ your boots before you go into the house. 11. 1 wish I'd right. this calculator before 1 bought it. It doesn't seem to be working 12. Marvin was going to drop out of school, but his grandfather 13. I'm not sure if this is a suitable topic for a research paper. 1 need to my teacher before 1 start writing. him _ _ _ _ _ _ _ _ it. it ____ with 14. "I'm going to those old newspapers." "Don't just put them in the trash-recycle them instead." 15. "Are you going on the class trip?" "I need to it before 1 decide." 16. "How did you get home so quickly?" "I _ _ _ _ _ _ _ _ _ _ _ _ _ _ _ _ through the fields." 17. "So, Louisa, you decided to a part in the play?" "Yes, I've been interested in acting for quite a while, so 1 finally decided to _ _ _ _ _ _ __ 18. Akiko is going to ____ some _ _ _ _ _ _ _ _ _ _ _ _ teaching to finish writing her dissertation. Mini-Lesson 1.12 turn around face in a different direction turn down (1) reject an offer (2) decrease the intensity of ;...turn in (1) return; give back; hand in (2) go to bed change to; transform into turn off stop the operation (of an appliance, for example); shut off turn into Section 1 • Guide to Listening Comprehension 99 turn on start the operation (of an appliance, for example) turn out (1) result; end up; be the final product (2) produce (3) arrive; gather (for a meeting, for example) turn up (1) increase in intensity (2) arrive under the weather slightly ill use up usc completely wait on serve walk on air be very happy warm up (1) heat (2) practice; prepare for warm up (to) become friendly with; start to enjoy watch out (for) be alert; look out for wear out become no longer useful because of wear what the doctor ordered exactly what was needed; the perfect thing wipe out eliminate; stamp out without a hitch without a problem work out (1) exercise (2) bring to a successful conclusion; solve Exercise: Fill in the blanks in the sentences or dialogs with idioms from the list above. There will be one word per blank. It may be necessary to change the verb forms in order for the sentence to be grammatically correct. 1. The story of Dr. Jekyll and Mr. Hyde is about a scientist who drinking a chemical potion. a monster after 2. 3. It's warm in here. Could you _________ the heater a little? _ _ _ the radio a little? 4. I love that song. Could you ____. 5. I can't see a thing. Please. ________ the light. 6. This sweater looked nice when it was new, but now it's 7. "I'm hungry!" "Why don't you ____ _ __.____ some of the leftovers from lunch? You can use the microwave oven." 8. Not many people for the meeting last night. 9. It's been a rough day. I'm going to _ _... _ ... _. _ _ _ early and get a good night's sleep. 10. "How about a nice cup of hot tea?" "That's exactly what I'm in the mood for. It's just _____ ._ __ 11. "How was your presentation?" "Great, It went off _________ . ______ 12. 1"m going to the gym to __.._______._____ .. on the exercise machines. 13. If you're on a crowded bus or subway car, you must __.... _.____ .___ _ .____ pickpockets. 14. "A bear' I don't see a bear! Where is it?" __ ___ slowly. It's right behind you." 15. Maria had quite a few problems last year, but she ____ them all _ _ __ 16. Don't ask me to _ _ _ _ _ _ _ _ you! I'm not your servant. 17. Brian was offered the manager's job, but he _ _ _ it ____ He said he didn't want the responsibility. 100 Section 1 • Guide to Listening Comprehension 18. Before the game starts, the players need to _ _ _ _ _ _ __ 19. "This cake Holly baked for the wedding "Yes, indeed. It was delicious." very well, don't you think?" 20. "Mitchell looked pale and tired." "He told me he was feeling a little __~ ____________ 21. "You must be happy about getting that scholarship." "Are you kidding? I'm still _____ ~__________ ." T SECTION 2 Guide to Structure and Written Expression 101 102 Section 2 • Guide to Structure and Written Expression About Structure and Written Expression The second section of TOEFL tests your understanding of English grammar and usage. This section is divided into two parts: Structure and Written Expression. Structure (Sentence Completion) 15 items Written Expression (Error Identification) 25 items Totals 40 items Time 25 minutes Section 2 is important! Your best chance for improving your score on TOEFL in a short time is to improve your score on this section, and therefore pull up your total score. Although a wide range of grammar points are tested, there are certain points that appear again and again, and you can master these points with the information and practice this Guide provides. Also, there are fewer items in this part than in the other two, so each item that you answer correctly adds more to your total score. Section 2 may seem less stressful for you than Section 1 because you don't have to divide your attention between the audio and the test book. It may seem less stressful than Section 3 because it is easier to finish all the items before time is called. Section 2 sentences are generally about academic subjects: the physical sciences (such as astronomy or geology), the social sciences (such as psychology or economics), or the humanities (such as music or literature). You will not see sentences that deal with "controversial" subjects such as abortion, illegal drugs, or sensitive political issues. Any cultural references in the sentences are to the culture of the United States or Canada. Many of the sentences contain references to people, places, and institutions that you will not be familiar with. (In fact, many NorthAmericans are not familiar with these either!) It's not necessary to know these references; you should simply concentrate on the structure of the sentences. It's also not necessary to understand all the vocabulary in a sentence; you can often answer a question correctly without a complete understanding of that sentence. There are two possible approaches to Section 2 problems: an analytical approach and an intuitive approach. A test-taker who uses the analytical approach quickly analyzes the grammar of a sentence to see what element is missing (in Structure) or which element is incorrect (in Written Expression). Someone who uses the second approach simply chooses the answer that "sounds right" (in Structure) or the one that "sounds wrong" (in Written Expression). Although this Guide emphasizes the first approach, the second can be useful too, especially for people who learned English primarily by speaking it and listening to it rather than by studying grammar and writing. If you aren't sure which approach works best for you, take Section 2 of one of the Practice Tests using the first method, and Section 2 of another test using the second approach. Did one approach give you a significantly better score? You can also combine the two approaches: if you get "stuck" (unable to choose an answer) using one method, you can switch to another. Hint: An excellent way to prepare for Section 2 is to write your own Structure and Written Expression items. Write several items for each of the lessons in this part of the Guide. There's no better way to start thinking like a test-writer. Section 2 • Guide to Structure and Written Expression 103 • Be familiar with the directions for both parts. Don't waste time reading the directions or examples. Begin immediately with question 1. • You can spend an average of about thirty seconds on each item. If an item seems difficult, make a guess and lightly mark the item on your answer sheet so that you can come back to it later. Erase all such marks before time is up. Don't spend too much time working on difficult items; it's better to work on items that you will probably get correct. • Never answer any item too quickly even if it seems easy. Always consider all four answer choices. On both parts of this seclion, it's easy to make mistakes because of carelessness. • Never leave any answers blank. Always guess even if you have no idea what the answer is. • If you fmish before time is called, go back and work on items that you found difficult. You are not permitted go ahead to Section 3. Now begin your preparation for Section 2 by taking the Sample Structure and Written Expression Test. Be sure to observe the twenty-five-minute time limit. 104 Section 2 • Guide to Structure and Written Expression Sample Structure and Written Expression Test Time: 25 minutes This section tests your ability to recognize grammar and usage suitable for standard written English. This section is divided into two parts, each with its own directions. Structure Directions: Items in this part are incomplete sentences. Following each of these sentences, there are four words or phrases. You should select the one word or phrase-(A), (B), (C), or (D)-that best completes the sentence. 111en fill in the space on your answer sheet that matches the letter of the answer that you have selected. Example! Sample Answer Pepsin ____ an enzyme used in digestion. (A) that (B) is (C) of (D) being This sentence should properly read "Pepsin is an enzyme used in digestion." You should fill in (B) on your answer sheet. Example II Sample Answer C!DCDCD~ ____ large natural lakes are found in the state of South Carolina. (A) There are no (B) Not the (C) It is not (D) No This sentence should properly read "No large natural lakes are found in the state of South Carolina." You should fill in (D) on your answer sheet. As soon as you understand the directions, begin work on this part. @.i.".i·"hiUg:ii@t-. Section 2 • Guide to Structure and Written Expression 105 1. Martha Graham, of the pioneers of modern dance, didn't begin dancing until she was twenty-one. (A) who, as one (B) she was (C) one (D) was one 5. Many communities are dependent on groundwater from wells for their water supply. (A) that obtained (B) obtained (C) is obtained (D) obtain it 2. Tiger moths ____ wings marked with stripes or spots. (A) have (B) with (C) their (D) whose 6. experimental studies of the aging process, psychologist Ross McFarland determined that people could work productively much longer than had previously been thought. (A) In that (B) Through (C) Since (D) Into 7. often raise funds from the sale of stock. (A) For corporations to operate (B) The operations of corporations (C) Corporations operate by (D) To operate, corporations 3. Platinum is harder than copper and is almost as pliable _ _ __ (A) gold (B) than gold (C) as gold (D) gold is 4. Most ofAnnie]ump Cannon's career as an astronomer involved the observation, classification, and _ _ __ (A) (B) (C) (D) she analyzed stars the stars' analysis stars were analyzed analysis of stars 8. While all birds are alike in that they have feathers and lay eggs, great differences among them in terms of size, structure, and color. (A) there are (B) but are (C) if there are (D) to be IU.i·';'GJI;tj,(ii.@V. 106 Section 2 • Guide to Structure and Written Expression eeeeeeeeeeee 9. There were _ _ _ federal laws regulating mining practices until 1872. (A) none (B) not (C) no 13. In 1791 Quebec was divided into two sections, Upper Canada and Lower Canada, ____ were ruled by elected assemblies. (A) they both (B) both of them (D) nor 10. The Masters, one of the most important of all golf tournaments, every year in Augusta, Georgia. (A) has held (B) being held (C) is held (D) holding 11. Not only places of beauty; they serve scientific and educational purposes as well. 14. are a form of carbon has been known since the late eighteenth century. (A) Diamonds (B) Because diamonds (C) That diamonds (D) Diamonds, which 15. Designed by Frederic Auguste Bartholdi, (A) are botanical gardens (A) the United States was given the Statue (B) botanical gardens to be (C) botanical gardens are of Liberty by the people of France (B) the people of France gave the Statue of Liberty to the United States (C) the Statue of Liberty was given to the United States by the people of France (D) the French people presented the United States with a gift, the Statue of Liberty (D) to be botanical gardens 12. (C) in which both (D) both of which quicksand can be found all over the world, little was known about its composition until recently. (A) Except (B) Although (C) Even (D) Despite @.i."'i.i"t:j U43 i @t-. Section 2 .. Guide to Structure and Written Expression 107 eeeeeeeeeeee Written Expression Directions: The items in this part have four underlined words or phrases, (A), (B), (C) and (D). You must identify the one underlined expression that must be changed for the sentence to be correct. Then find the number of the question on your answer sheet and fijI in the space corresponding to the letter. Example! Sample Answer "COCDCQ) Lenses may to have either concave or convex shapes. ABC --0- This sentence should read "Lenses may have either concave or convex shapes." You should therefore select an~wer (A). Example II Sample Answer When painting a fresco, an artist is applied paint directly to the damp plaster of a wall. A B C 0 This sentence should read "When painting a fresco, an artist applies paint directly to the damp plaster of a wall." You should therefore select answer (B). As soon as you understand the directions, begin work on this section. lij.i.,;,(."UjU§!i.@t-. 108 Section 2 • Guide to Structure and Written Expression •••••••••••• 16. There are thousand of different types of roses. --A-- BCD 17. Since 1908 breeders set out to produce chickens that could survive Canada's cold climate. A --B- C D 18. From 1865 to 1875, a remarkable various of inventions was produced. -A- B C D 19. The simplest kind of plant, alike the simplest kind of anim:u, consists of only one cell. A --B- C D 20. The fIrst recorded use of natural gas to light streetlamps it was in the town of Frederick, A B --c- D New York, in 1825. 21. A museum in Denver, Colorado, chronicles black cowboys and his role in the history of the A -""Be- D American West. 22. Bluegrass music is a kind of country music who was pioneered by Bill Monroe of Rosine, A -BC 0 Kentucky. 23. A number of the materials used in manufacturing paint are potential dangerous if mishandled. A B C D 24. There are several races of giraffes, but there are only one species. --ABCD 25. Chestnuts were once the most common tree in the eastern United States, but almost of them A B --C- were destroyed by a disease called chestnut blight. D @.i.",t·"UWUq:ii@t-. Section 2 • Guide to Structure and Written Expression 109 eeeeeeeeeeee 26. Despite they are small, ponies are strong and have great stamina. A -C------n- -B- 27. Physical therapists help patients relearn how to use their bodies after disease or injure. A -B- -C- --D- 28. Liquids take the shape of any container which in they are placed. A C ~ --D- 29. The Sun supplies the light and the warmth that permit life on Earth existing. ~ B C D 30. For seventeen years, between 1932 to 1949, FredAllen was one of the most popular A "-B C comedians on radio. D 31. Boolean algebra is most often used to solve problems in logic, probability, and engineer. A B -C- D 32. Attorney Clarence Darrow is knowing for his defense of unpopular persons and causes. ABC --D- 33. Phi Beta Kappa is a honor society that encourages scholarship in science and art. ~ B ~ C 34. The French Quarter is the most famous and the most old section of New Orleans. A B C D 35. There was once a widespread believe that all lizards were poisonous. A B C D 36. In the late nineteenth century, many public buildings, especially that on college campuses, A -B- were built in the in the Romanesque Revival style of architecture. C D IA.i.,;it."U§iU§iii@t-. 110 Section 2 • Guide to Structure and Written Expression •••••••••••• 37. Sponges have neither heads nor separate body organs. ~ - B - """"C -D38. A wooden barrel is made from strips of wood called staves holding together with metal hoops. A -B- C -D- 39. Salt was once too scarce and precious that it was used as money. A ~ C D 40. Sharks acquire many sets of tooth during their lifetimes. -A- --B---C--o- THIS IS THE END OF SECTION 2. IF YOU FINISH BEFORE TIME IS CALLED, CHECK YOUR WORK ON SECTION 2 ONLY DO NOT READ OR WORK ON ANY OTHER SECTION OFTHETEST. ~TO' ~TO~ ~TO' ~TO' ~TO' ~TO' PAR T A About Structure Introduction This part of the test consists of fifteen incomplete sentences. Some portion of each sentence has been replaced hy a blank. Under each sentence, four words or phrases are listed. One of these completes the sentence grammatically and logically. T Sample Item Pepsin ____ an enzyme used in digestion. (A) that (B) is (C) of (D) heing The sentence consists of a single clause (Pepsin is an enzyme) and a reduced (shortened) adjective clause (used in digestion). Each clause must contain a subject and a verb. There is a subject but no main verb. (The verbal form used is NOT the main verb in this sentence; it is a past participle.) Therefore, a main verh is needed. Only choices (B) and (D) are verb forms. However, an -ing verb can never be used alone as a main verb. The only possible choice is (B), the main verb is. ... 111 112 Section 2 • Guide to Structure and Written Expression What Is Tested in This Section? In general, there are five main points tested. The chart below shows how these are presented in the next section of the Guide. :,..~ti~I~~~nts Sentence Structure Lesson Lesson Lesson Lesson Lesson Lesson Lesson 17: 18: 19: 20: 21: 22: 23: Incomplete Independent Clauses Incomplete Adjective Clauses Incomplete/Missing Participial Phrases Incomplete/MissingAppositives Incomplete Adverb Clauses Incomplete Noun Clauses Incomplete/Missing Prepositional Phrases Word Order Lesson 24: Word Order Items Verbs and Verbals Lesson 25: Items Involving Verb Problems Lesson 26: Incomplete/Missing Infinitive and Gerund Phrases Proper Style Lesson 27: Items Involving Parallel Structures Lesson 28: Items Involving Misplaced Modifiers Word Choice Lesson 29: Incomplete/Missing Comparisons Lesson 30. Missing Conjunctions Lesson 31: Missing Negative Words The points practiced in the first group of lessons (Lessons 17 to 23) are tested on almost every TOEFL exam. The points in the second group (Lessons 24 to 31) are tested on many TOEFL exams. What Is the Best Way to Answer Structure Items? If the answer choices are fairly short, you should begin by taking a quick look at the answer choices to get an idea of what to look for when you read the sentence. A look at the answer choices can often tell you that you are looking at a problem involving verb forms, word order, parallel structure, misplaced modifiers, or others. If the answer choices are long or complicated, begin by reading the stem. Don't analyze it word for word, but as you are reading, try to form a picture of the sentence's overall structure. How many clauses will there be in the complete sentence? Does each clause have a complete subject and verb? Is there a connecting word to join clauses? Are any other elements obviously missing? Then look at the answer choices. If you're not sure of the answer, try to elinlinate as many distractors as possible. Distractors in Structure are generally incorrect for one of the following reasons: • A necessary word or phrase is missing, so the sentence is still incomplete. • An unnecessary word or phrase is included. • Part of the answer choice is ungrammatical when put into the stem. Never choose an answer until you've read the sentence completely; sometinles an option seems to fit in the sentence unless you read every word. After you have eliminated as many answer choices as possible, read the sentence quickly to yourself with the remaining choice or choices in place of the blank. If an answer doesn't "sound right," it probably isn't. If you still can't decide, guess and go on. If you have time, come back to these more difficult items later. Section 2 • Guide to Structure and Written Expression 113 Punctuation clues can sometimes help you solve Structure problems. For example, if there are a series of items in a sentence separated by commas (A, B, and C), you will probably see a problem involving parallel structures. cTadiU·fOrStructure • If the answer choices are short, look them over before you read the sentence. Try to get an idea of what type of problem you are working with. • Read the sentence, trying to determine which elements are missing. Never choose an answer until you have read the entire sentence; sometimes an answer will seem to fit until you have read the last few words of the sentence. • Mark your choice immediately if the answer is obvious. If you're not sure, try to eliminate incorrect answers. • Read the sentence with the remaining answer choices in place of the blank. Choose the option that sounds best. • If you are still unable to decide on an answer, guess and and go on. Lightly mark these items on your answer sheet and come back to them if time permits. • Go on to the second part of this section (Written Expression) as soon as you've finished Structure. Don't spend so much time working on Structure that you don't have enough time to finish Written Expression. 114 Section 2 • Guide to Structure and Written Expression LESSON 17 INCOMPLETE INDEPENDENT ClAUSES The structures practiced in this lesson are the ones that are most often tested in the Structure section. About 20% of all problems in the section (usually three or four per test) involve incomplete independent clauses. A) Clauses All sentences consist of one or more clauses. A simple sentence consists of one clause. People need vitamins. The man took a vitamin pill. Judy lives in northern California. In the summer, Tom walks to his office. A compound sentence consists of two independent clauses joined by a coordinating conjunction (such as and and but). The man took a vitamin pill, and he drank a glass of orange juice. Judy lives in northern California now, but she was raised in Ohio. A complex sentence consists of an independent clause (called the main clause) and a dependent (subordinate) clause. Subordinate clauses may be adverb clauses, noun clauses, or adjective clauses. In the sentences below, the independent clauses are italicized. The man took a vitamin pill because he had a cold. (independent clause + adverb clause) I didn't realize that Nancy was here. (independent clause + noun clause) Tom walks to his office, which is located on Broadway, every day during the summer. (independent clause + adjective clause) All three types of subordinate clauses are commonly seen in the Structure part of the test, and each is considered in separate lessons (Lessons 18, 21, and 22). The emphasis in this chapter, however, is on the basic components of independent clauses. B) Missing Subjects, Verbs, Objects, and Complements All clauses have a subject and a verb. Clauses with an action verb often take a direct object as well. Subject People Verb need Object vitamins. The verb missing from an independent clause may be a single-word verb (need, was, took, had, walked) or a verb phrase consisting of one or more auxiliary verbs and a main verb (will need, has been, should take, would have had, had walked). The verbs may be active (need, take) or passive (was needed, is taken). The missing subject and direct object may be a noun (people, vitamins, Tom) a noun phrase (some famous people, a vitamin pill, my friend Tom) or a pronoun. (He, she, it, and they are subject pronouns; him, her, it and them are object pronouns.) After the verb to be and certain other non-action verbs, a subject complement is used rather than a direct object. (Subject complements are also known as predicate nominatives and predicate adjectives.) Section 2 • Guide to Structure and Written Expression Subject She The teacher Verb is seemed 115 Complement an architect. upset. In the Structure section of TOEFL, it is common for any of these elements or a combination of two or more of these elements to be missing from the stem. The most common problem in Structure involves a missing verb. A missing subject and a missing subject-verb combination are common as well. The missing element may also be part of rather than all of the verb or noun phrase. T Sample Items The art of storytelling _ _ _~ almost as old as humanity. (A) that is (B) is (C) it is (D) being The correct answer supplies the missing verb. Choice (A) is incorrect because the word that is used to connect a relative clause to a main clause; in this sentence, there is only one verb, so there can only be one clause. Choice (C) is incorrect because there is an unnecessary repetition of the subject (The art of storytelling it ...). Choice (D) is not correct because an -ing form (being) cannot be the main verb of a clause. _ _ _~ a few of the sounds produced by insects can be heard by humans. (A) Only (B) There are only (C) That only (D) With only The correct answer completes the noun phrase that is the subject of the sentence. The expletive There in choice (B) is incorrectly used. In (C), the word That creates a noun clause, but each clause must have its own verb. (Produced is used as a participle, not a main verb, in this sentence.) Choice (D) is incorrect because a preposition may not be used directly before the subject. ____ when lava cools very rapidly. (A) (B) (C) (D) Because pumice is formed To form pumice Pumice is formed Forming pumice The best answer supplies an independent clause to join to the adverb clause when lava cools very rapidly. Choice (A) consists of an adverb clause; two adverb clauses cannot be joined to form a complete sentence. Choices (B) and (D) are incorrect because they do not contain main verbs, and an independent clause must contain a main verb. (To form andforming are not main verbs.) Only choice (C) could serve as an independent clause because it contains a subject (Pumice) and a full verb, the passive verb is formed. 116 Section 2 • Guide to Structure and Written Expression Duke Ellington wrote ____ during his career. (A) that over a thousand songs (B) over a thousand songs (C) over a thousand songs were (D) there were over a thousand songs The direct object is missing from this sentence. In choice (A), the connecting word that is used unnecessarily. In (C), the verb were is used unnecessarily because there is only one clause and it has a verb (wrote). In choice (D) the phrase there were is not needed between a verb and its direct object. Choice (B) correctly supplies a noun phrase that serves as the direct object. Before the invention of the printing press, books _ _ __ (A) that were very rare (B) were very rarely (C) were very rare (D) as very rare Choice (A) incorrectly forms an adjective clause; an adjective must be joined to a main clause. Choice (B) contains an adverb; after the verb to be, an adjective is required. Choice (D) lacks a verb. Choice (C) correctly supplies a verb (were). ... C) Clauses with There and It Some clauses begin with the introductory words there or it rather than with the subject of the sentence. These introductory words are sometimes called expletives. The expletive there shows that someone or something exists, usually at a particular time or place. These sentences generally follow the pattern there + verb to be + subject: There are many skyscrapers in New York City. There was a good movie on television last night. The expletive it is used in a number of different situations and patterns: It is important to be punctual for appointments. (with the verb to be + adjective + infinitive) It was in 1959 that Alaska became a state. (with the verb to be + adverbial + noun clause) It takes a long time to learn a language. (with the verb to take + time phrase + infinitive) It was David who did most of the work. (with the verb to be + noun + relative clause) It and there, along with the verb and other sentence elements, may be missing from the stem. Section 2 • Guide to Structure and Written Expression .... 117 Sample Items In Michigan, ____ over six hundred feet deep. (A) salt deposits (B) where salt deposits are (C) having salt deposits (D) there are salt deposits Choice (D) correctly supplies an introductory word (there), a verb, and a subject. Choice (A) lacks a verb. Choice (B) contains a subordinator, used to introduce a clause; there is only one verb, however, so there can only be one clause. Choice (C) also lacks a main verb. ____ a tomato plant from 75 to 85 days to develop into a mature plant with ripe fruit. (A) It takes (B) To take (C) That takes (D) By taking Choice (A) correctly completes the sentence with the introductory word It and a verb. Choice (B) and (D) do not supply main verbs. Choice (C) incorrectly creates a noun clause. ... Exercise 17 Focus: Completing structure problems involving incomplete independent clauses. (Note: Three or four items in this exercise do not focus on missing subjects, verbs, complements, or introductory words; these items are marked in the answer key with asterisks.) Directions: Choose the one option-(A), (B), (C), or (D)-that correctly completes the sentences, and then mark the appropriate blank. The first one is done as an example. 1. In the United States, is generally the responsibility of municipal governments. ___ (A) for water treatment ~ (B) water treatment ___ (C) where water treatment ___ (D) in which water treatment 2. Crop rotation of preserving soil fertility. ___ (A) it is one method _ _ (B) one method ____ (C) a method is one _ _ (D) is one method 3. the dollar as its monetary unit in 1878. _ _ (A) Canada adopted _ _ (B) Adopted by Canada, ___ (C) It was adopted by Canada _ _ _ (D) The Canadian adoption of 118 4. Section 2 • Guide to Structure and Written Expression almost impossible to capture the beauty of the aurora borealis in photographs. _ _ (A) Being _ _ (B) His _ _ (C) There is _ _ (D) Is 5. Usually, political cartoons ______ on the editorial page of a newspaper. _ _ (A) appear _ _ _ (B) whose appearance _ _ (C) by appearing _ _ (D) when they appear 6. __ __ ___ __ two major art museums, the Fogg and the Sadler. (A) Harvard University has (B) At Harvard University (C) Harvard University, with its (D) There at Harvard University 7. American actress and director Margaret Webster ______ for her production of Shakespearean plays. ___ (A) who became famous ___ (B) famous as she became ___ (C) becoming famous ___ (D) became famous 8. gas tanks connected to welding equipment, one full of oxygen and the other full of acetylene. _ _ (A) It is two _ _ (B) Of the two _ _ (C) There are two _ _ (D) Two 9. is more interested in rhythm than in melody is apparent from his compositions. _ _ (A) That Philip Glass _ _ (B) Philip Glass, who _ _ (C) Philip Glass _ _ _ (D) Because Philip Glass lO. Compressed air ______ the power to drive pneumatic tools. _ _ (A) by providing _ _ _ (B) provides ___ (C) that provides ___ (D) the provision of 11. by cosmic rays. _ _ (A) The earth is constantly bombarded _ _ (B) Bombarded constantly, the earth _ _ (C) Bombarding the earth constantly ___ (D) The earth's constant bombardment 12. primary colors are red, blue, and yellow. ___ (A) There are three _ _ (B) The three _ _ (C) Three of them _ _ (D) That the three Section 2 • Guide to Structure and Written Expression 13. 119 who was elected the first woman mayor of Chicago in 1979. ___ (A) It was Jane Byrne _ _ (B) Jane Byrne _ _ (C) That Jane Byrne _ _ (D) When Jane Byrne 14. Every computer consists of a number of systems _ _ _ _ _ _ together. _ _ (A) by working _ _ (B) work _ _ (C) they work _ _ (D) that work 15. On the Moon, ______ air because the Moon's gravitational field is too weak to retain an atmosphere. _ _ (A) there is no _ _ (B) where no _ _ (C) no ___ (D) is no 16. The Glass Mountains of northwestern Oklahoma _ _ _ _ _ _ with flecks of gypsum, which shine in the sunlight. ___ (A) they are covered _ _ _ (B) covered them _ _ _ (C) that are covered _ _ _ (D) are covered 17. In some cases, _ _ _ _ _ _ to decide if an organism is a plant or an animal. _ _ (A) difficult if _ _ (B) it is difficult _ _ (C) the difficulty _ _ (D) is difficult 18. The first American novelist to have a major impact on world literature _ _ _ _ __ ___ (A) who was James Fenimore Cooper _ _ _ (B) James Fenimore Cooper was ___ (C) it was James Fenimore Cooper ___ (D) was James Fenimore Cooper 19. important railroad tunnel in the United States was cut through the Hoosac Mountains in Massachusetts. _ _ (A) At first _ _ (B) It was the first _ _ (C) The first _ _ (D) As the first of 20. Generally, in the valleys and foothills of the Pacific Coast ranges. _ _ (A) the California poppy grown _ _ (B) the growth of the California poppy _ _ (C) the California poppy grows _ _ (D) growing the California poppy 21. When bats are at rest, _ _ _ _ _ _ hang upside-down. _ _ (A) they _ _ (B) and _ _ (C) to _ _ (D)as 120 22. Section 2 • Guide to Structure and Written Expression that the capital of South Carolina was moved from Charleston to Columbia. _ _ (A) In 1790 was _ _ _ (B) There was in 1790 _ _ (C) In 1790 _ _ (D) It was in 1790 23. Although not as important as they once were, _ _ _ _ _ _ a major form of transportation in North America. ___ (A) there are still railroads _ _ _ (B) railroads, which are still _ _ (C) railroads are still ___ (D) railroads still being 24. The Loop, which is the commercial heart of Chicago, _ _ _ _ _ _ within a rectangular loop of elevated train tracks. ___ (A) that is enclosed ___ (B) enclosing it ___ (C) is enclosed _ _ (D) enclosed 25. amino acids that serve as the basic building blocks of all proteins. _ _ (A) About twenty _ _ (B) For about twenty of _ _ (C) About twenty are _ _ (D) There are about twenty Section 2 • Guide to Structure and Written Expression 121 LESSON 18 INCOMPLETE ADJECTIVE ClAUSES As mentioned in the previous lesson, there are three types of dependent clauses, all of which are tested in Structure. Adjective clauses-also called relative clauses-are the most commonly tested of the three. You will see one or two items involving adjective clauses on most tests. Adjective clauses are a way of joining two sentences. In the joined sentence, the adjective clause modifies (describes) a noun (called the head noun) in another clause of the sentence. It begins with an adjective clause marker. I wanted the book. The book had already been checked out. The book which I wanted had already been checked out. The adjective clause in this example begins with the marker which and modifies the head noun book. Adjective clause markers are relative pronouns such as who, that, or which or the relative adverbs when or where. . Ad~ :.;~~.c, ........ Use who Subject (people) A neurologist is a doctor who specializes in the nervous system. whom Object (people) This is the patient whom the doctor treated. whose Possessive (people) Mr. Collins is the man whose house I rented. which Subject/Object (things) That is a topic which interests me. (which as subject) That is the topic on which I will write. preposition) that Subject/Object (peop le/things) (which as object of Art that is in public places can be enjoyed by everyone. (that as subject) The painting that Ms. Wallace bought was very expensive. (that as object) where Adverb (place) Here is the site where the bank plans to build its new headquarters. when Adverb (time) This is the hour when the children usually go to bed. Like all clauses, adjective clauses must have a subject and a verb. In some cases the adjective-clause marker itself is the subject; in some cases, there is another subject. The painting was very expensive. Ms. Wallace bought it. The painting which Ms. Wallace bought was very expensive. The adjective-clause marker in the joined sentence replaces it, the object of the verb bought. In the joined sentence, the adjective clause keeps the subject-Ms. Wallace-that it had in the original sentence. 122 Section 2 • Guide to Structure and Written Expression This is a topic. It interests me. This is a topic that interests me. The adjective-clause marker in the joined sentence replaces it, the subject of the second original sentence. In the joined sentence, the marker itself is the subject of the adjective clause. Notice that the inclusion of the pronoun it in the joined sentences above would be an error. Incorrect: *The painting which Ms. Wallace bought it was very expensive. *This is a topic which it interests me. This type of mistake is sometimes seen in distractors. When the markers which, that, and whom are used as objects in relative clauses, they can correctly be omitted. The painting Ms. Wallace bought is very expensive. (which omitted) The adjective-clause markers which and whom can also be used as objects of prepositions: That is the topic. I will write on it. That is the topic on which I will write. You may also see sentences with adjective clauses used in this pattern: quantity word + of + relative clause He met with two advisors. He had known both of them for years. He met with two advisors, both of whom he had known for years. I read a number of articles. Most of them were very useful. I read a number of articles, most of which were very useful. Any part of a relative clause can be missing from the stem of Structure items, but most often, the marker and the subject (if there is one) and the verb are missing. Any word or phrase from another clause-usually the head noun-may also be missing from the stem. T Sample Items Cable cars are moved by cables ____ underground and are powered by a stationary engine. (A) they run (B) that they (C) run (D) that run run Choice (A) is incorrect because the pronoun they cannot be used to join two clauses. Choice (B) is not appropriate because the subject they is not needed in the adjective clause; the marker that serves as the subject of the clause. Choice (C) is incorrect because there is no marker to join the adjective clause to the main clause. The melting point is the temperature ____ a solid changes to a liquid. (A) which (B) at which (C) which at (D) at Section 2 • Guide to Structure and Written Expression 123 Choice (A) is incorrect because a preposition is needed before the adjective clause. Choice (C) is incorrect because the relative pronoun comes before the preposition. Choice (D) is incorrect because the relative pronoun has been omitted. There are six types of flamingos, all ____ have long legs, long necks, and beaks that curve sharply downward. (A) of them (B) that (C) of which (D) they Choices (A) and (D) do not contain connecting words needed to join clauses. Choice (B) does not follow the correct pattern of relative clauses after a quantity word (all). The correct pattern needed to complete this sentence is quantity word + of + marker. Only (C) follows this pattern. ... Exercise 18 Focus: Answering structure problems involving incomplete adjective clauses. (Note: One or two items in this exercise do not focus on adjective clauses; these items are marked in the answer key with asterisks.) Directions: Choose the one option-(A), (B), (C), or (D)-that correctly completes the sentence, and then mark the appropriate blank. The first one is done as an example. 1. Most folk songs are ballads ______ have simple words and tell simple stories. _ _ (A) what _ _ (B) although _ _ (C)when ~ (D) that 2. After its introduction in 1969, the float process ______ the world's principal method of manufacturing flat sheets of glass. ___ (A) by which it became _ _ _ (B) it became _ _ (C) became ___ (D) which became 3. In 1850, Yale University established Sheffield Scientific School, _ _ _ _ __ ___ (A) engineers were educated there _ _ _ (B) where engineers were educated _ _ _ (C) in which were engineers educated ___ (D) where were engineers educated 4. Many of Louise Nevelson's sculptures consisted of a number of large wooden structures ______ in complex patterns. ___ (A) which she arranged _ _ _ (B) she arranged them ___ (C) which arranged ___ (D) arranged them 124 Section 2 • Guide to Structure and Written Expression 5. In addition to being a naturalist, Stewart E. White was a writer ______ the struggle for survival on the American frontier. ___ (A) whose novels describe _ _ _ (B) he describes in his novels _ _ (C) his novels describe ___ (D) who, describing in his novels 6. Diamonds are often found in rock formations called pipes, ______ the throats of extinct volcanoes. _ _ (A) in which they resemble _ _ (B) which resemble ___ (C) there is a resemblance to _ _ (D) they resemble 7. William SamuelJohnson, president of Columbia College in 1787. _ _ (A) whom he had _ _ (B) and he had _ _ (C) who had _ _ (D)had 8. Seals appear clumsy on the land, most people can run. _ _ (A) but they _ _ (B) which they _ _ (C) they _ _ (D)which helped write the Constitution, became the flrst are able to move short distances faster than 9. The instrument panel of a light airplane has at least a dozen instruments _ _ _ _ __ ___ (A) the pilot must watch _ _ (B) what the pilot must watch _ _ (C) which the pilot must watch them ___ (D) such that the pilot must watch them 10. A keystone species is a species of plants or animals ______ absence has a major effect on an ecological system. _ _ (A) that its _ _ (B) its _ _ (C)whose _ _ (D) with its 11. The size and shape of a nail depend primarily on the function ______ intended. ___ (A) which it is _ _ _ (B) for which it is ___ (C) which it is for _ _ (D) for which is 12. In geometry, a tangent is a straight line _ _ _ _ __ a curve at only one pOint. _ _ (A) it touches _ _ (B) whose touching _ _ (C) its touching _ _ (D) that touches Section 2 • Guide to Structure and Written Expression I:). It was the ragtime pianist Scott Joplin _ _ _ _ _ _ the "Maple Leaf Rag, " perhaps the best known of all ragtime tunes. (A) wrote ___ (B) the writer of ___ (C) who wrote ___ (D) writing 14. There are oyer 2.000 varieties of snakes. _ _ _ _ _ _ are harmless to humans. ___ (A) mostly they ___ (B) most of them ___ (C) most of which ___ (D) which most I,). Smokejumpers are _ _ (A) firefighters ___ (D) when firefighters _ _. (C) who, as firefighters ___ (D) firefighters who descend into remote areas by parachute to fight forest fires. 16. Charlotte Gilman's best known book _ _ _ _ _ _ she urges women to become financially independent. _ _ (A) is U;r)11zell and Economics. in which ___ (ll) m'nnell and Economics. in which ___ (C) is W'i"mwn and Economics, which ___ (D) U;omen and Economics, which 125 126 Section 2 • Guide to Structure and Written Expression LESSON 19 INCOMPLETE OR MISSING PARTICIPIAL PHRASES Participial phrases generally occur after nouns. They are actually reduced (shortened) relative clauses. Present participles (which always end in -ing) are used to reduce adjective clauses that contain active verbs. Minnesota, which joined the Union in 1858, became the thirty-second state. (adjective clause with active verb) Minnesota, joini;;g the Union in 1858, became the thirty-second state. (participial phrase with a present participle) Most past participles end in -ed, but there are also many irregular forms. Past participles are used to reduce adjective clauses with passive verbs. William and Mary College, which wasfounded in 1693, is the second oldest university in the United States. (adjective clause with a passive verb) William and Mary College, founded in 1693, is the second oldest university in the United States. (participial phrase with a past participle) Participial phrases can also come before the subject of a sentence. joining the Union in 1858, Minnesota became the thirty-second state. Founded in 1693, William and Mary College is the second oldest university in the United States. Usually, the participle itself is missing from this type of Structure item, but any part of a participial phrase as well as parts of a main clause may be missing. .. Sample Item Natural resources provide the raw materials ____ to produce finished goods. (A) (B) (C) (D) needed are needed which need needing Option (B) is a passive verb; the sentence cannot contain two main verbs (are needed and prOl'ide) in the same clause. Choice (C) creates an adjective clause, but the verb in the clause is active and a passive verb is required. (However, a relative clause with a passive verb (which are needed) would be a correct answer.) Choice (D) is a present participle and has an active meaning; a past participle is needed. A Exercise 19 Focus: Completing structure problems involving incomplete or missing participial phrases. (Note: One or two items in this exercises do not focus on participial phrases; these items are marked on the answer key with asterisks.) Directions: Choose the one option-(A), (B), (C), or (D)-that correctly completes the sentence, and then mark the appropriate blank. The first one is done as an example. Section 2 • Guide to Structure and Written Expression 127 1. Aerodynamics is the study of the forces _ _ _ _ _ _ on an object as it moves through the atmosphere. ~ (A) acting _ _ (B) act ___ (C) are acting _ _ (D) acted 2. ___ ___ ___ ___ (A) (B) (C) (D) for their strong fiber include flax and hemp. Plants are grown Plants grown Plants that grow To grow plants ___ ___ _ __ ___ (A) (B) (C) (D) , Jose Limon's dance troupe often toured abroad. The u.s. State Department sponsored it Sponsored by the U.S. State Department The U.S. State Department, which sponsored it The sponsorship of the U.S. State Department 3. 4. Elfreth'sAlJey in Philadelphia is the oldest residential street in the United States, with _ _ _ _ _ _ from 1728. ___ (A) hOllses are dated ___ (B) the dates of the houses ___ (C) the dating of houses ___ (D) hOllses dating 5. In 1821 the city of Indianapolis, Indiana, was laid out in a design _ _ _ _ _ _ after that of Washington, D.C. ___ (A) patterned ___ (B) was patterned ___ (C) a pattern ___ (D) that patterned 6. ___ ___ ___ ___ (A) (B) (C) (D) in front of a camera lens changes the color of the light that reaches the film. Placed a filter A filter is placed A filter placed When a filter placed 7. The Massachusetts State House, the United States at that time. ___ (A) completing ___ (B) which was completed ___ (C) was completed _ _ (D) to be completed 8. Barbara McClintock _ _ (A) known ___ (B) who knows ___ (C) knowing ___ (D) is known in 1798, was the most distinguished building in for her discovery of the mobility of genetic elements. 9. The solitary scientist _ _ _ _ _ _ by himse1fhas in many instances been replaced by a cooperative scientific team. ___ (A) to make important discoveries ___ (B) important discoveries were made ___ (C) has made important discoveries _ _ _ (9) making important discoveries 128 Section 2 • Guide to Structure and Written Expression 10. Geometry is the branch of mathematics _ _ _ _ _ _ the properties of lines, curves, shapes, and surfaces. ___ (A) that concerned with ___ (B) it is concerned with ___ (C) concerned with ___ (D) its concerns are 11. an average of 4T\ inches of rain a year, Mount Waia1ea1e in Hawaii is the wettest spot in the world. ___ (A) It receives ___ (B) Receiving ___ (C) To receive ___ (D) Received 12. Amber is a hard, yellowish-brown of years ago. ___ (A) substance formed ___ (B) to form a substance ___ (C) substance has formed ___ (D) forming a substance ._ _ _ _ _ from the resin of pine trees that lived millions Section 2 • Guide to Structure and Written Expression 129 LESSON 20 INCOMPLETE OR MISSING APPOSITIVES An appositive is a noun phrase that explains or rephrases another noun phrase. It usually comes after the noun which it rephrases. It may also come before the subject of a sentence. Buffalo Bill, a famousfrontiersman, operated his own Wild West Show. (appositive following a noun) A famous frontiersman, Buffalo Bill operated his own Wild West Show. (appositive before the subject) Appositives are actually reduced adjective clauses. However, unlike adjective clauses, they do not contain a marker or a verb. Oak, which is one of the most durable hardwoods, is often used to make furniture. (adjective clause) Oak, one of the most durable hardwoods, is often used to make furniture. (appositive) Appositives are usually separated from the rest of the sentence by commas, but some short appositives (usually names) are not. Economist Paul Samuelson won a Nobel Prize in 1970. In Structure items. all or part of an appositive phrase may be missing. In addition, the noun that the appositive refers to or other parts of the main clause may be missing. .... Sample Item The National Road, of the first highways in North America, connected the East Coast to the Ohio Valley. (A) which one (B) it was one (C) one (D) was one Choice (A) is incorrect; there is no verb in the relative clause. Choice (B) has no connecting word to join the clause to the rest of the sentence. Choice (D) is incorrect because a verb cannot be used in an appositive phrase. Note: u'hich was one would also be a correct answer. ... Exercise 20 Focus: Completing structure problems involving appositives. (Note: The focus for one or two items in this exercises is not appositives; these sentences are marked in the answer key with asterisks.) Directions: Choose the one option-(A), (B), (C), or (D)-that correctly completes the sentences, and then mark the appropriate blank. The first one is done as an example. 1. The Democratic Party is older than the other major American political party, ___ (A) which the Republican party ~ (B) the Republican party ___ (C) it is the Republican party ___ (D) the Republican party is 130 2. Section 2 • Guide to Structure and Written Expression relations with friends and acquaintances, playa major role in the social development of adolescents. ___ (A) What are called peer group relations are ___ (B) Peer group relations are .___ (C) Peer group relations, the ___ (D) By peer group relations, we mean 3. Joseph Henry, advisor on scientific matters. _ _ (A) the _ _ (B) to be the _ _ (C) was the _ _ (D) as the first director of the Smithsonian Institute, was President Lincoln's 4. The Wassatch Range, extends from southeastern Idaho into northern Utah. ___ (A) which is a part of the Rocky Mountains, ___ (B) a part of the Rocky Mountains that ___ (C) is a part of the Rocky Mountains ___ (D) a part of the Rocky Mountains, it 5. ___ ___ ___ ___ (A) (B) (C) (D) Ruth St. Dennis turned to Asian dances to find inspiration for her choreography. It was the dancer The dancer That the dancer The dancer was 6. The organs of taste are the which are mainly located on the tongue. ___ (A) groups of cells, are taste buds ____ (B) taste buds, are groups of cells ___ (C) taste buds, these are groups of cells ___ (D) taste buds, groups of cells 7. In 1878 FrederickW Taylor invented a concept called scientific management, _ _ _ _ _ _ of obtaining as much efficiency from workers and machines as possible. ___ (A) it is a method ___ (B) a method which _ _ (C) a method ___ (D) called a method 8. A group of Shakers, settled around Pleasant Hill, Kentucky, in 1805. ___ (A) members of a strict religious sect which ___ (B) whose members of a strict religious sect ___ (C) members of a strict religious sect, ___ (D) were members of a strict religious sect 9. In physics, "plasma" refers to a gas which has a nearly equal number of positively and negatively charged particles. _ _ (A) the term _ _ (B) by the term ___ (C) is termed _ _ (D) terming Section 2 • Guide to Structure and Written Expression 10. Norman Weiner, development of the computer. ___ (A) who, as a _ _ (B) was a _ _ (C) whom a _ _ (D) a 131 mathematician and logician, had an important role in the 11. Jerome Kern's most famoLls work is Showbo({t, _ _ _ _ _ _ most enduring mLlsical comedies. _ (A) it is one of the i1nest, _ _ _ (B) one of the finest, ___ (C) the finest one ___ (D) as the finest of the 12. a marshland that covers over 7';0 square miles in North Carolina and Virginia. _ __ (A) In the Great Dismal Swamp, _ _ _ (B) The Great Dismal Swamp, which ___ (C) The Great Dismal Swamp, ___ (D) The Great Dismal Swamp is 132 Section 2 • Guide to Structure and Written Expression LESSON 21 INCOMPLETE ADVERB CLAUSES A) Full Adverb Clauses An adverb clause consists of a connecting word, called an adverb clause marker (or subordinate conjunction), and at least a subject and a verb. ~ The demand for economical cars increases when gasoline becomes more expensive. In this example, the adverb clause marker when joins the adverb clause to the main clause. The adverb clause contains a subject (gasoline) and a verb (becomes). An adverb clause can precede the main clause or follow it. When the adverb clause comes first, it is separated from the main clause by a comma. When gasoline becomes more expensive, the demand for economical cars increases. The following markers are commonly seen in the Structure section: u,~ because cause Because the speaker was sick, the program was canceled. since cause Since credit cards are so convenient, many people use them. although opposition (contrary cause) Although he earns a good salary, he never saves any money. even though opposition (contrary cause) Even though she was tired, she she stayed up late. while contrast Some people arrived in taxis while others took the subway. if condition lfthe automobile had not been invented, what would people use for basic transportation? unless condition I won't go unless you do. when time Your heart rate increases when you exercise while time Some people like to listen to music while they are studying. as time One train was arriving as another was departing. since time We haven't seen Professor Hill since she returned from her trip. until time Don't put off going to the dentist until you have a problem. once time Once the dean arrives, the meeting can begin. before time Before he left the country. he bought some traveler's checks. after time She will give a short speech after she is presented with the award. Section 2 • Guide to Structure and Written Expression 133 In Structure items, any part of a full adverb clause-the marker, the subject, the verb, and so on­ can be missing from the stem. B) Clause Markers with -ever Words that end with -ever are sometimes used as adverb clause markers. (In some sentences, these words are actually noun-clause markers, but they are seldom used that way in Structure items.) The three -ever words that you are likely to see in the Structure Section are given in the chart below: wherever any place that ... Put that box wherever you can find room for it. whenever any time that ... They stay at that hotel whenever they're in Boston. however anyway that. .. However you solve the problem, you'll get the same answer. C) Reduced Adverb Clauses When the subject of the main clause and the subject of the adverb clause are the same person or thing, the adverb clause can be reduced (shortened). Reduced adverb clauses do not contain a main verb or a subject. They consist of a marker and a participle (either a present or a past participle) or a marker and an adjective. When astronauts are orbiting the earth, they don't feel the force of gravity. (full adverb clause) When orbiting the earth, astronauts don't feel the force of gravity. (reduced clause with present participle) Although it had been damaged, the machine was still operational. (full adverb clause) Although damaged, the machine was still operational. (reduced clause with a past participle) Although he was nervous, he gave a wonderful speech. (full adverb clause) Although nervous, he gave a wonderful speech. (reduced clause with an adjective) You will most often see reduced adverb clauses with the markers although, while, if, when, before, after, and until. Reduced adverb clauses are NEVER used after because. Section 2 • Guide to Structure and Written Expression 133 In Structure items, any part of a full adverb clause-the marker, the subject, the verb, and so oncan be missing from the stem. B) Clause Markers with -ever Words that end with -ever are sometimes used as adverb clause markers. (In some sentences, these words are actually noun-clause markers, but they are seldom used that way in Structure items.) The three -ever words that you are likely to see in the Structure Section are given in the chart below: wherever any place that ... Put that box wherever you can fmd room for it. whenever any time that ... They stay at that hotel whenever they're in Boston. however anyway that ... However you solve the problem, you'll get the same answer. C) Reduced Adverb Clauses When the subject of the main clause and the subject of the adverb clause are the same person or thing, the adverb clause can be reduced (shortened). Reduced adverb clauses do not contain a main verb or a subject. They consist of a marker and a participle (either a present or a past participle) or a marker and an adjective. 'When astronauts are orbiting the earth, they don't feel the force of gravity. (full adverb clause) 'When orbiting the earth, astronauts don't feel the force of gravity. (reduced clause with present participle) Although it had been damaged, the machine was still operational. (full adverb clause) Although damaged, the machine was still operational. (reduced clause with a past participle) Although he was nervous, he gave a wonderful speech. (full adverb clause) Although nervous, he gave a wonderful speech. (reduced clause with an adjective) You will most often see reduced adverb clauses with the markers although, while, if, when, before, after, and until. Reduced adverb clauses are NEVER used after because. 134 Section 2 • Guide to Structure and Written Expression D) Prepositional Phrases with the Same Meaning as Adverb Clauses There are also certain prepositions that have essentially the same meaning as adverb-clause markers but are used before noun phrases or pronouns, not with clauses. because of because/since He chose that university because of its fine reputation. due to because/since The accident was due to mechanical failure. on account of because/since Visibility is poor today on account of air pollution. in spite of although/even though He enjoys motorcycle riding in spite of the danger. despite although/even though Despite its loss, the team is still in first place. during when/while Her father lived in England during the war. In structure items where the correct answer is an adverb-clause marker, one of these words often appears as a distract or. T Sample Items No one knows what color dinosaurs were _ _ _ _ _ _ no sample of their skin has survived. (A) (B) (C) (D) because of because that it is because because Choice (A) is incorrect; because of can only be used before nouns or pronouns. In choice (B), that is unnecessary. In (C), the phrase it is is used unnecessarily. _ _ _ _ _ _ rises to the surface of the earth, a volcano is formed. (A) Liquid magma (B) Whenever liquid magma (C) Liquid magma, which (D) That liquid magma Choice (A) creates two clauses, but there is no connecting word to join them. (C) creates a sentence with a main clause and an adjective clause, but the main clause has two subjects (liquid magma and a volcano). (D) creates a noun clause. In a correct sentence, when a noun clause begins a sentence, the clause itself is the subject of the verb in the main clause, but this sentence already has a subject (volcano). Section 2 • Guide to Structure and Written Expression 135 _ _ _ _ _ _ invisible to the unaided eye, ultraviolet light can be detected in a number of ways. (A) Although is Despite (C) Even though it (D) Although (B) The best answer completes a reduced adverb clause. In choice (A), the adverb clause lacks a subject and is not a correct reduction because it contains a verb. In choice (B), despite cannot be used with an adjective (only with a noun phrase or pronoun). (C) does not supply a verb for the adverb clause and is not a correct reduction because it contains a subject. Because _ _ _ _ _ _ , alabaster can be easily carved. (A) is soft (B) softness (C) of its softness (D) of soft Choice (A) lacks a subject in the adverb clause. Choice (B), a noun, could only be used with because of In (D), because of is followed by an adjective; to be correct, it must be followed by a noun phrase or pronoun. ... Exercise 21 Focus: Completing structure problems involving adverb clauses, reduced adverb clauses, and related prepositional expressions. (Note: Two or three items do not focus on one of these structures. These items are marked in the answer key with an asterisk.) Directions: Choose the one option-CA), (B), (C), or (D)-that correctly completes the sentences, and then mark the appropriate blank. The first one is done as an example. 1. Small sailboats can easily capsize _ _ _ _ _ _ they are not handled carefully. _ _ (A) but _ _ (B) ~ which (C) if _ _ CD) so 2. they are tropical birds, parrots can live in temperate or even cold climates. _ _ (A) Despite _ _ (B) Even though ___ CC) Nevertheless _ _ (D) But 3. added to a liquid, antifreeze lowers the freezing temperature of that liquid. _ _ (A) That _ _ (B) Asis ._ _ (C) It is _ _ (D) When 136 Section 2 • Guide to Structure and Written Expression 4. advertising is so widespread in the United States, it has had an enormous effect on American life. _ _ (A) Why ___ (B) The reason ___ (C) On account of ___ (D) Since 5. towards shore, its shape is changed by its collision with the shallow sea bottom. ___ CA) During a wave rolls ___ (B) As a wave rolls _ _ _ (C) A wave rolls ___ (D) A wave's rolling 6. are increasingly linked over long distances by electronic communications, but many of them still prefer face-to-face encounters. ___ CA) Although people _ _ (B) Despite people _ _ (C) Today people ___ CD) The fact that people 7. together in one place, they form a community. _ _ CA) When people who live _ _ (B) When people living _ _ _ (C) Whenever people live ___ CD) Whenever living people 8. managed by an independent governor and board of directors, the Bank of Canada is owned by the Canadian government. _ _ (A) And yet _ _ (B) In spite of it _ _ (C) Although _ _ (D) It is pieces of rope are of different thickness, the weaver's knot can be used to join 9. them. _ _ (A) Two of _ _ (B) What two _ _ (C) Two such _ _ CD) If two 10. , the seeds of the Kentucky coffee plant are poisonous. ___ (A) Until they have been cooked _ _ (B) Cooking them ___ (C) They have been cooked _ _ (D) Cooked until 11. Natural silk is still highly prized _______ similar artificial fabrics. _ _ CA) although is available ___ (8) despite there are available _ _ _ (C) in spite of the availability of ___ (D) even though an availability of 12. Cattle ranches are found almost _ _ _ _ _ _ in Utah. ___ CA) wherever _ _ CB) everywhere _ _ CC) overall ___ (D) somewhere Section 2 • Guide to Structure and Written Expression 137 13. through a prism, a beam of white light breaks into all the colors of the rainbow. ___ (A) When shines _ _ (B) It shines _ _ _ (C) It is shone ___ (D) When shone 14. most people think of freezing as a relatively modern method of food preservation, it is actually one of the oldest. _ _ (A) Even _ _ (B) As though _ _ _ (C) However _ _ (D) Even though 15. large bodies of water never freeze solid is that the sheet of ice on the surface protects the water below it from the cold air. ___ (A) Because _ _ (B) Why do ___ (C) The reason that ___ (D) For the reason 16. granted by the Patent Office, it becomes the inventor's property and he or she can keep it, sell it, or license it to someone else. ___ (A) Once a patent is _ _ _ (B) When a patent ___ (C) A patent, once ___ (D) A patent, whenever it 17. Owls can hunt in total darkness _ _ _ _ _ _ their remarkably keen sense of smell. ___ (A) since ___ (B) because of _ _ _ (C) the result _ _ (D) that [ H. three times ___ (A) _ _ (B) _ _ (C) ___ (D) most bamboo blooms every year, there are some species that flower only two or a century. Whenever That While However 138 Section 2 • Guide to Structure and Written Expression LESSON 22 INCOMPLETE NOUN ClAUSES Noun clauses are the third type of subordinate clause. They begin with noun-clause markers. Noun clauses that are formed from statements begin with the noun-clause marker that. Noun clauses formed from yes/no questions begin with the noun-clause markers whether or if. Those formed from information questions begin with wh- words: what, where, when, and so on. Dr. Hopkins' office is in this building. (statement) I'm sure that Dr. Hopkins' office is in this building. Is Dr. Hopkins' office on this floor? (yes/no question) I don't know if (whether) Dr. Hopkins' office is on this floor. Where is Dr. Hopkins' office? (information question) Please tell me where Dr. Hopkins' office is. Notice that the word order in direct questions is not the same as it is in noun clauses. The noun clause follows statement word order (subject + verb), not question word order (auxiliary + subject + main verb). Often one of the distractors for noun-clause items will incorrectly follow question word order. *1 don't know what is her name. (Incorrect use of question word order) I don't know what her name is. (Correct word order) *She called him to ask what time did his party start. (Incorrect use of question word order) She called him to ask what time his party started. (Correct word order) Noun clauses function exactly as nouns do: as subjects, as direct objects, or as complements after the verb to be. When the meeting will be held has not been decided. (noun clause as subject) The weather announcer said that there will be thunderstorms. (noun clause as direct object) This is what you need. (noun clause after to be) Notice that when the noun clause is the subject of a sentence, the verb in the main clause does not have a noun or pronoun subject. In Structure items, the noun-clause marker, along with any other part of the noun clausesubject, verb, and so on-may be missing from the stem, or the whole noun clause may be missing. Section 2 • Guide to Structure and Written Expression T 139 Sample Items _ _ _ _ _ _ was caused by breathing (~pure'air was once a common belief. '----(A) Malaria (B) That malaria (C) Why malaria (D) Because malaria Choice (A) is incorrect because there are two verbs (was caused and was) but only one subject. Choice (C) is incorrect because Why is not the appropriate noun-clause marker in this sentence; the noun clause is based on a statement, not on an information question. Choice (D) is incorrect because it forms an adverb clause, but the main clause lacks a subject. In the correct answer the noun clause itself (That malaria was caused by breathing impure air) is the subject of the verb was in the main clause. One basic question psychologists have tried to answer is _ _ _ _ __ (A) people learn (B) how do people learn (C) people learn how (D) how people learn Choice (A) is incorrect; there is no connector between the first clause and the second. Choice (B) incorrectly follows question word order. (C) is incorrect because how is in the wrong position. .. Exercise 22 Focus: Completing structure problems involving incomplete noun clauses. (Note: Two or three items in this exercise do not focus on noun clauses. These items are marked in the answer key with asterisks.) Directions: Choose the one option-(A), (B), (C), or (D)-that correctly completes the sentences, and then mark the appropriate blank. The first one is done as an example. 1. begin their existence as ice crystals over most of the earth seems likely. _ _ (A) Raindrops _ _ (B) If raindrops _ _ (C) What if raindrops ~ (D) That raindrops 2. Scientists cannot agree on _ _ _ _ _ _ related to other orders of insects. _ _ (A) that fleas are _ _ _ (B) how fleas are ___ (C) how are fleas ___ (D) fleas that are 3. It was in 1875 joined the staff ofthe astronomical observatory at Harvard University. _ _ (A) that Anna Winlock _ _ _ (B) Anna Winlock, who _ _ _ (C) as Anna Winlock _ _ (D) Anna Winlock then 140 Section 2 • Guide to Structure and Written Expression 4. is a narrow strip of woods along a stream in an open grassland. _ _ _ (A) Ecologists use the term "gallery forest" ___ (B) What do ecologists call a "gallery forest" _ _ _ (C) "Gallery forest" is the term ecologists use ___ (D) What ecologists call a "gallery forest" 5. developed very rapidly in Alabama primarily because of its rich natural resources. _ _ (A) That heavy industry ___ (B) Heavy industry _ _ _ (C) Heavy industry that was _ _ (D) When heavy industry 6. so incredible is that these insects successfully migrate to places that they have never even seen. _ _ _ (A) That makes the monarch butterflies' migration ___ (B) The migration of the monach butterflies is _ _ _ (C) Wnat makes the monarch butterflies' migration _ _ _ (D) The migration of the monarch butterflies, which is 7. Art critics do not all agree on what _ _ _ _ _ _ a painting great. _ _ _ (A) qualities make _ _ _ (B) are the qualities for making ___ (C) qualities to make ___ (D) do the qualities that make 8. In order to grow vegetables properly, gardeners must know _ _ _ _ __ _ _ _ (A) what the requirements for each vegetable are ___ (B) that the requirements for each vegetable _ _ _ (C) what are each vegetable's requirements ___ (D) that is required by each vegetable 9. When ___ ___ ___ ___ (A) (B) (C) (D) is not known. was the wheel invented the invention of the wheel inventing the wheel the wheel was invented 10. For many years people have wondered _ _ _ _ _ _ exists elsewhere in the universe. _ _ (A) that life _ _ (B) life which _ _ (C) whether life _ _ (D) life as it 11. of all modern domestic poultry is the red jungle fowl is widely believed. _ _ _ (A) The ancestor ___ (B) The ancestor is _ _ _ (C) How the ancestor ___ (D) That the ancestor 12. the right side of a person's brain is dominant, that person is left-handed. _ _ (A) That _ _ (B) If _ _ (C)Which _ _ (D) For Section 2 • Guide to Structure and Written Expression 141 LESSON 23 MISSING OR INCOMPLETE PREPOSITIONAL PHRASES A prepositional phrase consists of a preposition (in, at, with, fm; until. and so on) followed by a noun phrase or pronoun. which is called the prepositional object. Prepositional phrases often describe relationships of time and location, among others. In the auturnn, maple leaves turn red. Beacon Hill is one of the most famous neighborhoods in Boston. 'With luck, there won't be any more problems. This house was built by John's grandfather. Often, prepositional phrases come at the beginning of sentences, but they may appear in other parts of the sentence as well. The correct answer for this type of item may be a preposition, its object, or both, as well as other parts of the sentence. You may see prepositions in distractors, especially before the subject of a sentence. Remember, the object of a preposition cannot correctly be the subject of a sentence, as in these examples: *111 the aullllnn is my favorite season. *Without a pencil is no way to come to a test. ... Sample Items _ _ _ _ _ _ the unaided eye can see about 6,000 stars. (A) A clear night (B) It's a clear night (C) On a clear night (D) When a clear night Choice (A) is incorrect because there is no connector to join the noun phrase a clear night to the rest of the sentence. Choice (B) consists of an independent clause, but there is no connector to join it to the other clause. (D) seems to form a subordinate clause, but the clause lacks a verb. _ _ _ _ _ _ all the field crops grown in the United States are harvested with machines called combines. (A) Of nearly (B) Nearly (C) That nearly (D) Nearly of Choices (A) and CD) are incorrect because the subject of a sentence (all the field crops) cannot be the object of a preposition (of). Choice (C) creates a noun clause, but the noun clause lacks a verb. ... 142 Section 2 • Guide to Structure and Written Expression Exercise 23 Focus: Completing structure problems involving prepositional phrases. (Note: One or two items in this exercises do not focus on prepositional phrases. These items are marked in the answer key with asterisks.) Directions: Choose the one option-(A), (B), (C), or (D)- that correctly completes the sentence, and then mark the appropriate blank. The first one is done as an example. 1. Dynamite is ordinarily detonated _ _ _ _ _ _ called a blasting cap. _ _ _ (A) a device is used _ _ _ (B) that a device ~ (C) with a device ___ (D) the use of a device 2. seed of a flowering plant is covered by a dense protective coat. _ _ (A) On each _ _ (B) Each _ _ (C) Each of _ _ (D) That each 3. 1900 there were some 300 bicycle factories in the United States, and they produced over a million bicycles. _ _ (A) In ___ (B) Because in _ _ _ (C) It was in _ _ (D) That in 4. A thick layer of fat called blubber keeps whales warm even _ _ _ _ _ _ coldest water. _ _ _ (A) although the _ _ (B) in the _ _ (C) the _ _ (D) of the 5. the United States, the general movement of air masses is from west to east. _ _ _ (A) Across ___ (B) To cross ___ (C) They cross ___ (D) It's across 6. The bark of a tree thickens _ _ _ _ __ _ _ _ (A) with age _ _ (B) it gets older _ _ _ (C) as older _ _ (D) by age 7 A substance that is harmless to a person who has no allergies can cause mild to serious reactions in a person allergies. _ _ (A) has ___ (B) which having ___ (C) can have _ _ (D) with 8. In 1886 a number of national unions formed the American Federation of Labor _ _ _ _ __ _ _ _ (A) Samuel Gompers was its leader ___ (B) under the leadership of Samuel Gompers ___ (C) which, under Samuel Gompers' leadership ___ (D) Samuel Gompers led it Section 2 • Guide to Structure and Written Expression 9. Harmonicas, autoharps, and kazoos _ _ _ _ _ _ folk instruments. ___ (A) are examples _ _ (B) for example ___ (C) are examples of ___ (D) as examples of 10. charming shops and restaurants, Old Town is the most picturesque section of Albuquerque. ___ (A) With its _ _ (B) Its ___ (C) Because its ___ (D) For its 11. _ _ _ _ _ _ , such as banking and travel, in which computers are not a convenience but a necessity. ___ (A) Where some industries ___ (B) In some industries ___ (C) Some industries ___ (D) There are some industries 12. One of the oldest large suspension bridges still _ _ _ _ _ _ today is the George Washington Bridge between ~ewYork City and Fort Lee, New Jersey. ___ (A) uses ___ (B) is used ___ (C) the use of ___ (D) in use 143 144 Section 2 • Guide to Structure and Written Expression MINI-TEST 4: STRUCTURE Directions: The following sentences are incomplete. Beneath each of these sentences, there are four words or phrases marked (A), (B), (C), and (D). Choose the one word or phrase that best completes the sentence. 1. by Anna Baldwin in 1878. _ _~ (A) The invention of the vacuum milking machine _ _~ (B) That the vacuum milking machine was invented _ _~ (C) The vacuum milking machine, which was invented _ _~ (D) The vacuum milking machine was invented 2. Dry cleaning is the process _ _ _ _~ clothes are cleaned in liquids other than water. _ _ (A) hy ____ (B) which through ___ (C) hy which _ _ (D) through 3. Sand dunes are made of loose sand _ _ _ __ up by the action of the wind. _ _ (A) it huilds _ _ (8) huilds _ _ (C) is huilding _ _~ (D) built 1. hookjubiiee, which was hased on the life of her great-grandmother, Margaret Walker was awarded the Pulitzer Prize. _~ ___ (A) For her _ _ (B) Her _ _~ (C) It was her _ _ (D) That her 5. Job specialization takes place _ _ _ _ _~ of production is separated into occupations. _ _~ (A) whenever the work is _ _~ (B) when the work _ . _ (C) is when the work _ _~ (D) whenever working Section 2 • Guide to Structure and Written Expression 145 6. _ _ _ _ _ _ are hot is a common misconception. _ _ (A) All deserts _ _ _ (B) All deserts which ___ (C) Of all deserts ___ (D) That all deserts 7. imaginative stories about the origin of the game of chess. ___ (A) "'Ian)' of the _ _ (B) "'lany ___ (C) There are many _ _ CD) Of the many 8. one of Canada's greatest engineering projects, is a twenty-seven mile long waterway between Lake Erie and Lake Ontario. ___ (A) Because the Welland Ship Canal is _ _ (B) The Welland Ship Canal is _ _ (C) That the Welland Ship Canal is _ _ (D) The WeIland Ship Canal, 9. A deep-tissue massage is a type of massage therapy _ _ _ _ _ _ on one part of the body, such as the lower back. ___ (A) its concentration is _ _ _ (B) concentrating ___ CC) why it concentrates ___ CD) to be concentrated 10. One of the most powerful optical telescopes, the "Big Eye" at Mt. Palomar, _ _ _ _ _ _ a twohundred-inch mirror. _ _ (A) has _ _ (B) that has _ _ (C) with _ _ (D) which 11. Elfego Baca, Mexico in the late 1880's. _ _ (A) a ___ CB) who, as a ___ CC) was a ___ CD) and he was a legendary Mexican-American folk hero, was a lawman in New 12, _ _ _ _ _ _ relatively inexpensive, the metal pewter can be fashioned into beautiful and useful objects. ___ (A) Even it is _ _ (B) Despite ___ (C) Nevertheless, it is _ _ CD) Although 13. is a general category that includes all mental states and activities. ___ (A) What do psychologists call cognition _ _ _ (B) Psychologists call it cognition ___ CC) \Vhat psychologists call cognition ___ (D) Cognition, as it is called by psychologists, which 146 Section 2 • Guide to Structure and Written Expression 14. Nathaniel Hawthorne wrote four novels, _ _ _ _ _ _ The Scarlet Letter, became an American literary classic. ___ ___ ___ ___ 15. (A) (B) (C) (D) of which one, which one one of which, one was about four years for a new aircraft model to move from the preliminary design stage to the full-production stage. _ _ (A) It takes _ _ (B) Taking _ _ (C) That takes _ _ (D) To take Section 2 • Guide to Structure and Written Expression 147 LESSON 24 WORD ORDER ITEMS All of the answer choices for a Structure item involving word order contain more or less the same words, but they are arranged in four different orders. The word order is "scrambled" in three choices: one is correct. Most items consist of three or four words. (A) XY Z (B) Y X Z (C) ZY X (D) X ZY Word order problems are easy to identify because the answer choices are exactly-or almost exactly-the same length, so the answer choices form a rectangle. (A) so far away from (8) away so far from (C) from so far away (D) away from so far Many different types of structures are used in word order problems. One of the most common is a phrase with a superlative adjective or adverb. ,/) Word order items are the only Structure items in which the dis tractors can be ungrammatical. In other Structure problems, distractors are always correct in some context. However, at least two of the choices may be grammatical. The correct choice depends on the context of the sentence. It sometimes is easy to eliminate distractors in word order items by making sure they "fit" the rest of the sentence. If you are not sure which remaining answer is correct, use your "ear." Say the sentence to yourself (silently) to see which sounds best. Sometimes in word order problems, the answer that looks best doesn't always sound best. Don't, however, go just by the sound of the answer choices; you must consider them as part of the whole sentence. A special type of word order problem involves in.y:ersions~ This type of sentence uses question word order even though the sentence is not a question. When are inversio~s used? • When the negative words listed below are placed at the beginning of a clause for emphasis not only not until not once at no time by no means nowhere never seldom rarely scarcely no sooner Not once was he on time. Seldom have I heard such beautiful music. Not only did the company lose profits, but it also had to layoff workers. • When the following expressions beginning with only occur at the beginning of a sentence (with these expressions, the subject and verb in that clause are inverted) only in (on, at, by, etc.) only once only recently Only in an emergency should you use this exit. Only recently did she return from abroad. Only by asking questions can you learn. 148 Section 2 • Guide to Structure and Written Expression • When the following expressions beginning with only occur at the beginning of a sentence (with these expressions the subject and verb of the second clause are inverted) only if only because only until only when only after Only if you have a serious problem should you call Mr. Franklin at home. Only when you are satisfied is the sale considered final. • When clauses beginning with the word so + an adjective or participle occur at the beginning of a sentence So rare is this coin that it belongs in a museum. So confusing was the map that we had to ask a police officer for directions. • When clauses beginning with expressions of place or order occur at the start of a sentence (in these cases, the subject and main verb are inverted since auxiliary verbs are not used as they would be in most questions) In front of the museum is a statue. Off the coast of California lie the Channel Islands. First came a police car; then came an ambulance. ... Sample Items Andromeda is a galaxy containing millions of individual stars, but it is _ _ _ _ _ _ Earth that it looks like a blurry patch of light. (A) so far away from (B) away so far from (C) from so far away (D) away from so far Only choice (A) involves the correct word order for this sentence. Choices (B) and (D) are incorrect word orders in any sentence. Choice (C) could be correct in certain sentences, but is not correct in the context of this sentence. Not only _ _ _ _ _ _ shade and beauty, but they also reduce carbon dioxide. (A) do trees provide (D) trees provide (C) provide trees (D) trees do provide Only choice (A) correctly uses question word order after not on(y. Choices (B) and (C) do not use an auxiliary verb, which is required after not only. Choice (D) does not follow the correct word order: auxiliary + adjective + main verb. .... Section 2 • Guide to Structure and Written Expression 147 LESSON 24 WORD ORDER ITEMS All of the answer choices for a Structure item involving word order contain more or less the same words, but they are arranged in four different orders. The word order is "scrambled" in three choices; one is correct. Most items consist of three or four words. (A) XY Z (B) YX Z (C) ZYX (D) X ZY Word order problems are easy to identify because the answer choices are exactly-or almost exactly-the same length, so the answer choices form a rectangle. (A) (B) (C) (D) so far away from away so far from from so far away away from so far Many different types of structures are used in word order problems. One of the most common is a phrase with a superlative adjective or adverb. c":.) Word order items are the only Structure items in which the dis tractors can be ungrammatical. In other Structure problems, distractors are always correct in some context. However, at least two of the choices may be grammatical. The correct choice depends on the context of the sentence. It sometimes is easy to eliminate distractors in word order items by making sure they "fit" the rest of the sentence. If you are not sure which remaining answer is correct, use your "ear." Say the sentence to yourself (silently) to see which sounds best. Sometimes in word order problems, the answer that looks best doesn't always sound best. Don't, however, go just by the sound of the answer choices; you must consider them as part of the whole sentence. A special type of word order problem involves in"ersions~ This type of sentence uses question word order even though the sentence is not a question. When are inversio~s used? • When the negative words listed below are placed at the beginning of a clause for emphasis not only not until not once at no time by no means nowhere never seldom rarely scarcely no sooner Not once was he on time. Seldom have I heard such beautiful music. Not only did the company lose profits, but it also had to layoff workers. • When the following expressions beginning with only occur at the beginning of a sentence (with these expressions, the subject and verb in that clause are inverted) only in (on, at, by, etc.) only once only recently Only in an emergency should you use this exit. Only recently did she return from abroad. Only by asking questions can you learn. 148 Section 2 • Guide to Structure and Written Expression • When the following expressions beginning with only occur at the beginning of a sentence (with these expressions the subject and verb of the second clause are inverted) only if only because only until only when only after Only if you have a serious problem should you call Mr. Franklin at home. Only when you are satisfied is the sale considered final. • When clauses beginning with the word so + an adjective or participle occur at the beginning of a sentence So rare is this coin that it belongs in a museum. So confusing was the map that we had to ask a police officer for directions. • When clauses beginning with expressions of place or order occur at the start of a sentence (in these cases, the subject and main verb are inverted since auxiliary verbs are not used as they would be in most questions) In front of the museum is a statue. Off the coast of California lie the Channel Islands. First came a police car; then came an ambulance. ... Sample Items Andromeda is a galaxy containing millions of individual stars, but it is _ _ _ _ _ _ Earth that it looks like a blurry patch of light. (A) so far away from (B) away so far from CC) from so far away CD) away from so far Only choice CA) involves the correct word order for this sentence. Choices CB) and CD) are incorrect word orders in any sentence. Choice (C) could be correct in certain sentences, but is not correct in the context of this sentence. Not only _ _ _ _ _ _ shade and beauty, but they also reduce carbon dioxide. (A) do trees provide (B) trees provide CC) provide trees CD) trees do provide Only choice (A) correctly uses question word order after not only. Choices (B) and (C) do not use an auxiliary verb. which is required after not on~y. Choice CD) does not follow the correct word order: auxiliary + adjective + main verb. .. Section 2 • Guide to Structure and Written Expression 149 Exercise 24 Focus: Completing structure problems involving word order. (Note: All the items in this exercise focus on word order problems.) Directions: Choose the one option-(A), (B), (C), or (D)-that correctly completes the sentence, and then mark the appropriate blank. The first one is done as an example. 1. Hills known as land islands, or salt domes, are _ _ _ _ _ _ Louisiana's marshlands. ~ (A) extremely interesting features of _ _ _ (B) of extremely interesting features ___ (C) interesting extremely features of ___ (D) extremely interesting of features 2. ___ ___ ___ ___ (A) (B) (C) (D) of chamber music is the string quartet. The famous most form The most famous form The form most famous Most the form famous 3. Not until the seventeenth century _ _ _ _ _ _ to measure the speed of light. ___ (A) did anyone even attempt ___ (B) anyone did even attempt ___ (C) did anyone attempt even ___ (D) did even attempt anyone 4. Alfalfa is ___ (A) ___ (B) ___ (C) ___ (D) for livestock. a primarily grown crop grown primarily a crop a crop grown primarily a grown crop primarily 5. The Franklin stove, which became common in the 1790's, burned wood _ _ _ _ _ _ an open fireplace. ___ (A) efficiently more than much ___ (B) much more efficiently than _ _ _ (C) much more than efficiently ___ (D) more efficiently much than 6. Reinforced concrete is concrete that is strengthened by metal bars _ _ _ _ __ ___ (A) in it that are embedded ___ (B) embedded that are in it _ _ _ (C) are that it embedded in ___ (D) that are embedded in it 7. The type of clothing people wear tells others a lot about _ _ _ _ __ _ _ (A) who they are _ _ (B) are they who _ _ (C) they are who _ _ (D) who are they 8. Most Southern states had set up primary school systems by the late eighteenth century, but only in New England and open to all students. ___ (A) primary schools were free ___ (B) were primary schools free _ _ _ (C) free were primary schools ___ (D) were free primary schools 150 Section 2 • Guide to Structure and Written Expression 9. Fungi. _ _ _ _ _ _ , do not produce chlorophyll. ___ (A) as such mushrooms _ _ _ (B) mushrooms as such _____ (C) such as mushrooms ___ (D) mushrooms such as 10. Seldom more than twenty minutes a night. ___ (A) sleep giraffes _ _ _ (B) do giraffes sleep _ _ _ (C) giraffes do sleep ___ (D) giraffes sleep 11. of the early years of space exploration was the discovery of the Van Allen radiation belt in 19':;8. ___ (A) Perhaps the greatest triumph ___ (B) The triumph perhaps greatest ___ (C) The greatest perhaps triumph ___ (D) The triumph greatest perhaps 12. Today ___ ___ ___ ____ (A) (8) (C) (D) major new products \yithout conducting elaborate market research. corporations hardly introduce ever hardly tTer corporations introduce hardly ever introduce corporations corporations hardly ever introduce 1:). Across the Chesapeake Bay from the rest of the state _ _ _ _ _ _ , whose farms produce beans, tomatoes. and other garden vegetables. ___ (A) there lies Maryland's Eastern Shore ___ (B) lies Maryland's Eastern Shore ___ (C) Maryland's Eastern Shore lies there _ __ (D) Maryland's Eastern Shore lies 14. Acidophilus bacteria are _ _ _ _ _ _ in an acid medium. ___ (A) those that grow best ___ (8) those grow best that ___ (C) that those grow best ___ (D) grow best those that I .:;. of great apes, the gibbon is the smallest. ___ (A) Four of the types _ _ _ (B) The four of types _ _ _ (C) Four types of the _ _ _ (D) Of the f()Ur types 16. It is difficult through swamps because of tangled roots and shallow waterways. ___ (A) to navigate even for small boats _ (B) for even small boats to navigate ___ (C) even small boats for to navigate ___ (D) even to navigate for small boats P. A lodestone is ____ ___ ___ ___ ___ (A) (B) (C) (D) an occurring naturally magnet a magnet naturally occurring naturally a magnet occurring a naturally occurring magnet Section 2 • Guide to Structure and Written Expression 151 18. So complicated that consumers who use a product are seldom aware of where all its components come from. ___ (A) today trade is international ___ (B) today international trade is ___ (C) is international trade today ___ (D) international trade is today winter birds in Canada. 19. The snow bunting is ___ (A) one most of the common ___ (B) the most common one of ___ (C) one of the most common ___ (D) the one of most common 20. Nashville has the capital of country music. ___ (A) as long been known _ _ _ (B) been known long as ___ (C) long been known as ___ (D) long as been known 152 Section 2 • Guide to Structure and Written Expression LESSON 25 ITEMS INVOLVING VERB PROBLEMS The answer choices for this type of problem are all or almost all different forms of the same verb. From the context of the sentence stem, you'll have to decide which form works best in the sentence. Distractors are generally incorrect for one of these reasons: • The "verb" is not really a verb. Used alone, an infmitive, gerund, or participle cannot be a main verb. • The verb is active but should be passive, or it is passive but should be active. If the subject of the sentence performs the action, the verb must be in the active voice. If the subject of the sentence receives the action, the verb must be in the passive. The architect designed the building. (active verb) The building was designed by the architect. (passive verb) • The verb does not agree with its subject. Singular subjects require singular verbs; plural subjects require plural verbs. • The verb is not in the right tense. According to the time words or ideas in the sentence, the appropriate tense must be used. • An unnecessary element comes before the verb. Personal pronouns (he, she, it), relative pronouns (who, which, that, and so on), or conjunctions (and, but, and so on) may be used unnecessarily before verbs in some sentences. .... Sample Item Before the late eighteenth century, most textiles _ _ _ _ _ _ at home. (A) produced (B) was produced (C) producing (D) were produced Choice (D) is the best answer. (A) can be considered either an active verb in the past tense or a past participle; both are incorrect. An active verb is incorrect because a passive verb is needed; a past participle is incorrect because a past participle cannot serve as a main verb. (B) is incorrect because the plural subject textiles requires a plural verb, were. (C) is incorrect because, by itself, an -ing form can never be a main verb. ... Section 2 • Guide to Structure and Written Expression 153 Exercise 25 Focus: Completing structure problems involving verb forms. (Note: One or two items in this exercises do not focus on finite verb forms. These items are marked in the answer key with asterisks.) Directions: Choose the one option-(A), (B), (C), or (D)-that correctly completes the sentence, and then mark the appropriate blank. The first one is done as an example. 1. R. M. Bartlett of Philadelphia _ _ _ _ _ _ the first private business college in the United States in 1843. _ _ (A) founding ~ (B) founded _ _ (C) was founded _ _ (D) founds 2. In 1989 the space probe Voyager 2 ______ by the planet Neptune. _ _ (A) fly _ _ (B) having flown _ _ (C) flying _ _ (D) flew 3. A cupful of stagnant water may _ _ _ _ _ _ millions of microorganisms. ___ (A) contains ___ (B) to contain ___ (C) contain ___ (D) containing 4. Computers and new methods of communication _ _ _ _ _ _ revolutionized the modern office. _ _ (A) have ___ (B) to have _ _ (C) that have _ _ (D) has 5. Sarah Knight _ _ _ _ _ _ a fascinating account of a journey she made from Boston to New York in 1704. ___ (A) written ___ (B) write ___ (C) wrote ___ (D) writing 6. All animals _ _ _ _ _ _ on other animals or plants. _ _ (A) feed _ _ (B) feeds _ _ (C) fed _ _ (D) feeding 7. Chromium _ _ _ _ _ _ in the manufacture of stainless steel. _ _ (A) using ___ (B) is used _ _ (C) uses ___ (D) is using 154 Section 2 • Guide to Structure and Written Expression 8. The Baltimore and Ohio Railroad ______ the first air conditioning system for trains in 1931. _ _ (A) has installed _ _ (B) installed ___ (C) to have installed _ _ (D) installing 9. Porous rocks such as chalk and sandstone allow water _ _ _ __ _ _ (A) soaks _ _ (B) is soaked _ _ (C) to soak _ _ (D) can soak 10. By 1790, rice _ _ (A) being _ _ (B) has been _ _ (C) was _ _ (D) was being 11. Weavers are social birds that _ _ (A) build _ _ (B) are built _ _ (C) are building _ _ (D) built through them. an important crop in the South. complex nests housing hundreds of families. 12. The American dancer Maria Tallchief first ______ prominent in Europe. _ _ (A) to become _ _ (B) become _ _ (C) has become _ _ (D) became Section 2 • Guide to Structure and Written Expression 155 LESSON 26 ) INCOMPLETE OR MISSING INFINITIVE AND GERUND PHRASES i An infinitive is a verbal form that consists of the word to and the simple form of the verb: to be, to go, to give, to bUild. Infinitives are often followed by an object: to give directions, to build a house. Together, an infinitive and its object form an infinitive phrase. Infinitives can be used in a variety of ways. They may be the subjects or objects of verbs or used after to be + adjective. To read the directions is important. (infinitive as subject of a verb) He forgot to read the directions. (infinitive as object of a verb) It's important to read the directions. (infinitive after to be + adjective) Infinitives can be used as adjective phrases after noun phrases. You will often see this in Structure problems after noun phrases containing the word first. John Glenn was the first American to orbit the Earth. Infinitives can also be used to show purpose. In other words, they explain why an action takes place. (The phrase in order + infinitive also shows purpose.) These infinitive phrases often come at the beginning of a sentence, and are set off by commas. To learn how to dance, he took lessons. In order to learn how to dance, he took lessons. You may see Structure items that focus on passive infInitives. A passive infinitive consists of the word to + be + past participle. Roberta was the first person to be asked to speak at the meeting. A gerund is a verbal form that ends in -ing: being, going, giving, building. Like infinitives, gerunds are often followed by objects: giving directions, building a house. Together, a gerund and its object form a gerund phrase. Gerunds are verbal nouns, and are used as other nouns are used. You will generally see gerunds as subjects or objects of verbs or as objects of prepositions. (Note: Infmitives can also be subjects and objects but never objects of prepositions.) Playing cards is enjoyable. (gerund as subject of a verb) He enjoys playing cards. (gerund as object of a verb) He passes the time by playing cards. (gerund as object of a preposition) ... Sample Items _ _ _ _ _ _ the eggs of most birds must be kept warm. (A) Proper development (B) By properly developing, (C) They develop properly (D) To develop properly, The only one of these for phrases listed here that can show purpose is choice (D), an infinitive. This expression means in order to develop properly. 156 Section 2 • Guide to Structure and Written Expression In 1959 the political philosopher Hannah Arendt became the ftrst woman ______ a full professor at Princeton University. (A) to appoint (B) was appointed (C) to be appointed (D) an appointment as After a noun phrase such as the first woman an inftnitive is used as an adjective phrase. Because a passive form is needed (Hannah Arendt receives the action; she doesn't perform the action), choice (A) is not the correct inftnitive form. Choice (C), a passive inftnitive, is best. The ear is the organ of hearing, but it also plays a role in _ _ _ _ __ balance. (A) maintaining (B) it maintains (C) to maintain (D) maintained A gerund is used correctly after a preposition. Choices (B), (C), and (D) would not be appropriate after a preposition. ... Exercise 26 Focus: Completing structure problems involving inftnitive and gerund phrases. (Note: One or two of the items in this exercise do not focus on inftnitives or gerunds. These items are marked in the answer key with asterisks.) Directions: Choose the one option-(A), (B), (C), or (D)-that correctly completes the sentence, and then mark the appropriate blank. The ftrst one is done as an example. 1. for a career in dance generally begins at an early age. _ _ (A) People train _ _ (B) That people train _ _ (C) Iftraining (D) Training ~ 2. A baby's ftrst teeth ______ are generally the lower incisors. ___ (A) appearance _ _ (B) appear ~ (C) to appear _ _ (D) in appearing 3. A climbing helmet ______ protection for a rock-climber's head from falling rocks and other hazards. ___ (A) to provide ----'bL- (B) provides _ _ (C) providing _ _ (D) that provides Section 2 • Guide to Structure and Written Expression 4. Power tools require careful handling _ _ _ _ _ _ injuries. _ _ (A) by avoiding _ _ (B) they avoid _ _ _ (C) to avoid ___ (D) that avoid 5. An electromagnet is created _ _ _ _ _ _ electrical current through a coil of wire. _ _ (A) by passing _ _ (B) passes by ___ (C) to be passed _ _ (D) passed 6. at home requires only three types of chemicals, several pieces of simple equipment, and running water. ___ (A) For the development of film _ _ (B) To develop film ___ (C) When film is developed ___ (D) In developing film 7. The purpose of cost accounting is _ _ _ _ _ _ involved in producing and selling a good or service. ___ (A) as a determination of its costs _ _ _ (B) the costs determined _ _ _ (C) that determines the costs ___ (D) to determine the costs 8. ___ ___ ___ ___ (A) (B) (C) (D) was one of the most difficult tasks pioneers faced on their journeys west. Crossing rivers While crossing rivers Rivers being crossed By crossing rivers 9. Energy can be defined as the ability _ _ _ _ __ _ _ (A) do working _ _ _ (B) to do work _ _ (C) doing work ___ (D) work to be done 10. The process of _ _ (A) to bind books _ _ (B) binding books ___ (C) books are bound _ _ (D) bound books by hand has changed little since the fifteenth century. 11. A crescent wrench has adjustable jaws for _ _ _ _ _ _ a nut, bolt, or pipe. _ _ (A) to grip _ _ (B) they grip _ _ (C) gripping _ _ (D) gripped 12. Compressed air is ___ (A) used to powering ___ (B) to use powering _ _ _ (C) used to power _ _ _ (D) in use by powering air brakes, pneumatic tools, and other machinery. 157 158 Section 2 • Guide to Structure and Written Expression 13. Some people believe that the crystals of certain minerals _ _ (A) have _ _ (B) having _ _ (C) that have _ _ (D) to have 14. The narrow blades of speed skates allow ___ (A) for skaters maintaining ___ (B) skaters to maintain ___ (C) skaters maintain ___ (D) maintenance by skaters 15. The first library _ _ (A) to be established _ _ (B) was established _ _ (C) could establish _ _ (D) to establish curative powers. speeds of up to 30 miles per hour. in the Nebraska Territory was built in Fort Atkinson in 1870. Section 2 • Guide to Structure and Written Expression 159 LESSON 27 ITEMS INVOLVING PARALLEL STRUCTURES In certain Structure items, the correct use of parallel structures is tested. Parallel structures have the same grammatical form and function. Look at the following sentences: She spends her leisure time hiking, camping, and fishing. He changed the oil, checked the tire pressure, andfilled the tank with gas. Nancy plans to either study medicine or major in biology. Nancy plans to study either medicine or biology. All of the structures in italics are parallel. In the first, three gerunds are parallel; in the second, three main verbs; in the third, two simple forms; in the fourth, two nouns. Many other structures must be parallel in certain sentences: adjectives, adverbs, infinitives, prepositional phrases, noun clauses, and others. The most common situation in which parallel structures are required is in a sequence (A, B, and C) as in the first two sentences above. Parallel structures are also required with correlative conjunctions such as either . .. or or not on(y ... but also. (Correlative conjunctions are presented in Lesson 30.) T Sample Item San Francisco has a pleasant climate, _ _ _ _ _ _ and many fascinating neighborhoods. (A) exciting scenery, (B) has exciting scenery (C) that the scenery is exciting (D) the scenery is exciting, This sentence contains a series of three objects after the verb has: the first and third are noun phrases (a pleasant climate and many fascinating neighborhoods). To be parallel, the second object must also be a noun phrase. Therefore, choice (A) is the correct answer; (B), (C), and (D) are not parallel. ... Exercise 27 Focus: Completing structure problems involving parallelism. (Note: One or two items in the exercise do not focus on items involving parallel structures. These items are marked in the answer key with asterisks.) Directions: Choose the one option-(A), (B), (C), or (D)-that correctly completes the sentence, and then mark the appropriate blank. The first one is done as an example. 1. Insects provide many beneficial services, such as _ _ _ _ _ _ , breaking down deadwood, and pollinating plants. ___ (A) they condition soils _ _ _ (B) to condition soil ~ (C) conditioning the soil ___ (D) soil conditioned 160 Section 2 • Guide to Structure and Written Expression 2. Frozen orange juice must be packed, _ _ _ _ _ _ , and stored when the fruit is ripe. _ _ (A) be frozen _ _ _ (B) must be frozen _ _ (C) frozen ___ (D) it must be frozen 3. Sioux is a North American Indian language that is spoken not only _ _ _ _ _ _ Sioux but also by the Crow and Osage tribes. _ _ (A) by the _ _ (B) the _ _ (C) do the _ _ (D) and the 4. In 1900 electrically powered cars were more popular than gasoline powered cars because they were quiet, operated smoothly, and _ _ _ _ __ _ _ (A) handled easily _ _ (B) ease of handling _ _ (C) handling easily _ _ (D) easy to handle 5. Roger Williams was a clergyman, _ _ _ _ _ _ the colony of Rhode Island, and an outspoken advocate of religious and political freedom. _ _ (A) founded _ _ (B) the founder of ___ (C) was the founder of _ _ (D) he founded 6. Paint can be applied to a surface with rollers, _ _ _ _ _ _ , or spray guns. _ _ (A) brushes ___ (B) brushes can be used ___ (C) with brushes . _ _ (D) by brush 7. The use of labor-saving devices in homes, _ _ _ _ _ _ , and factories added to the amount of leisure time people had. ___ (A) at an office ___ (B) used in offices ___ (C) offices ___ (D) in offices 8. A dulcimer can be played by either striking its strings with a hammer or _ _ _ _ __ ___ (A) to pluck them with the fingers ___ (B) fingers are used to pluck them ___ (C) they are plucked with the fingers _ _ (D) plucking them with the fingers 9. Throughout history, trade routes have increased contact between people, _ _ _ _ _ _ , and greatly affected the growth of civilization. ___ (A) have resulted in an exchange of ideas _ _ _ (B) an exchange of ideas has resulted ___ (C) resulted in an exchange of ideas ___ (D) reSUlting in an exchange of ideas 10. Walt Disney made many technical advances in the use of sound, color, and _ _ _ _ _ _ in animated films. _ _ (A) photographing _ _ (B) using photography ~ (C) photography _ _ (D) use of photographs Section 2 • Guide to Structure and Written Expression 161 11. Artist Paul Kane traveled throughout Northwest Canada on foot, by canoe, and _ _ _ _ _ _ to sketch Native Canadians going about their ordinary lives. _ _ (A) on horseback ___ (B) riding a horse _ _ (C) horseback _ _ (D) by a horse 12. Barbara Jordan was the first woman in the South to win an election to the House of Representatives, as Congresswoman from Texas from 1973 to 1979. ___ (A) to serve _ _ (B) served _ _ (C) serving _ _ (D) has served 13. Photographers' choice of a camera depends on what kind of pictures they want to take, how much control they want over exposure, and they want to spend. ___ (A) the amount of money ___ (B) what money ___ (C) how much money ___ (D) so much money that 14. Atlanta is the commercial, financial, and _ _ _ _ _ _ of Georgia. ___ (A) center of administration ___ (B) administrative center ___ (C) center for administering ___ (D) administering center 15. Even after the Revolutionary War, American importers obtained merchandise from Britain because British merchants understood American tastes, offered attractive prices, and ___ ___ ___ ___ (A) (B) (C) (D) easy credit was provided because of easy credit easy credit provided easy credit 162 Section 2 • Guide to Structure and Written Expression LESSON 28 ITEMS INVOLVING MISPLACED MODIFIERS A misplaced modifier is a participial phrase or other modifier that comes before the subject, but does not refer to the subject. Look at this sentence: *Driving down the road, a herd of sheep suddenly crossed the road in front of Liza's car. (INCORRECn This sentence is incorrect because it seems to say that a herd of sheep-rather than Liza-was driving down the road. The participial phrase is misplaced. The sentence could be corrected as shown: As Liza was driving down the road, a herd of sheep suddenly crossed the her. (CORRECn ro~d in front of This sentence now correctly has Liza in the driver's seat instead of the sheep. The following sentence structures are often misplaced: present participle Walking along the beach, the ship was spotted by the men. Walking along the beach, the men spotted the ship. past participle Based on this study, the scientist could make several conclusions. Based on this study, several conclusions could be made by the scientist. appositive A resort city in Arkansas, the population of Hot Springs is about 35,000. A resort city in Arkansas, Hot Springs has a population of about 35,000. reduced adjective clause While peeling onions, his eyes began to water. While he was peeling onions, his eyes began to water. adjective phrases Warm and mild, everyone enjoys the climate of the Virgin Islands. Everyone enjoys the warm, mild climate of the Virgin Islands. expressions with like or unlike Like most cities, parking is a problem in San Francisco. Like most cities, San Francisco has a parking problem. Structure items with misplaced modifiers are usually easy to spot. They generally consist of a modifying element at the beginning of the sentence followed by a comma, with the rest or most of the rest of the sentence missing. The answer choices tend to be long. To find the answer, you must decide what subject the modifier correctly refers to. T Sample Item Using a device called a cloud chamber, _ _ _ _ __ (A) experimental proof for the atomic theory was found by Robert Millikin (B) Robert Millikin's experimental proof for the atomic theory was found (C) Robert Millikin found experimental proof for the atomic theory (D) there was experimental proof found for the atomic theory by Robert Millikin Section 2 • Guide to Structure and Written Expression 163 Choice (A) and (B) are incorrect because the modifier (Using a device called a cloud chamber . .. ) could not logically refer to the subjects (experimental proof and Robert Millikin's experimental proof). (D) is incorrect because a modifier can never properly refer to the introductory word there or it. ... Exercise 28 Focus: Completing structure problems involving misplaced modifiers. (Note: all the items in this exercise focus on misplaced modifiers.) Directions: Choose the one option-(A), (B), (C), or (D)-that correctly completes the sentences, and then mark the appropriate blank. The first one is done as an example. 1. Fearing economic hardship, ~ (A) many New Englanders emigrated to the Midwest in the 1820's ___ (B) emigration from New England to the Midwest took place in the 1820's _ _ _ (C) it was in the 1820's that many New Englanders emigrated to the Midwest ___ (D) an emigration took place in the 1820's from New England to the Midwest 2. Rich and distinctive in flavor, _ _ _ _ __ ___ (A) there is in the United States a very important nut crop, the pecan ___ (B) the most important nut crop in the United States, the pecan ___ (C) farmers in the United States raise pecans, a very important nut crop _~ (D) pecans are the most important nut crop in the United States 3. Orbiting from 2.7 to 3.6 billion miles from the sun, ___ (A) the astronomer Clyde Tombaugh discovered Pluto in 1930 ___ (B) Pluto was discovered by the astronomer Clyde Tombaugh in 1930 ___ (C) it was in 1930 that the astronomer Clyde Tombaugh discovered Pluto ___ (D) the discovery of Pluto was made by Clyde Tombaugh in 1930 4. A popular instrument, ___ (A) only a limited role has been available to the accordion in classical music ___ (B) there is only a limited role for the accordion in classical music ___ (C) classical music provides only a limited role for the accordion ___ (D) the accordion has played only a limited role in classical music 5. Unlike most birds, _ _ _ _ __ ___ (A) the heads and necks of vultures lack feathers ___ (B) feathers are not found on the heads and necks of vultures ___ (C) vultures do not have feathers on their heads and necks ___ (D) there are no feathers on vultures' heads and necks 6. Widely reproduced in magazines and books, _ _ __ ___ (A) AnselAdams depicted the Western wilderness in his photographs ___ (B) the Western wilderness was depicted in the photographs of Ansel Adams ___ (C) Ansel Adams's photographs depicted the Western wilderness ___ (D) it was through his photographs that Ansel Adams depicted the Western wilderness 7. Smaller and flatter than an orange, ___ (A) a tangerine is easy to peel and its sections separate readily ___ (B) the peel of a tangerine is easily removed and its sections are readily separated ___ (C) it's easy to peel a tangerine and to separate its sections ___ (D) to peel a tangerine is easy, and its sections can be readily separated 164 Section 2 • Guide to Structure and Written Expression 8. Like the federal government, ___ (A) taxation provides most of the funds for state and local governments as well _ _ _ (B) state and local governments obtain most of their funds through taxation ___ (C) through taxation is how state and local governments obtain most of their funds ___ (D) funds are provided from taxation for state and local governments 9. Originally settled by Polynesians around 700 A.D., _ _ _ _ __ ___ (A) Hawaii received its first European visitor in 1778, when Captain James Cook landed there _ _ _ (B) Hawaii's first European visitor, Captain James Cook, landed there in 1778 ___ (C) in 1778 the first European, Captain James Cook, visited Hawaii _ _ _ CD) the first European to visit Hawaii was Captain James Cook, landing there in 1778 10. Unlike most modernist poets, based on ordinary speech. ___ (A) Robert Frost's poems were ___ (B) the works of Robert Frost were ___ (C) Robert Frost wrote poems that were ___ (D) the poetry written by Robert Frost was 11. Named for its founder, in Ithaca, New York. ___ (A) in 1865 Ezra Cornell established Cornell University _ _ _ (B) Cornell University was established in 1865 by Ezra Cornell ___ (C) it was in 1865 that Cornell University was established by Ezra Cornell ___ (D) Ezra Cornell established Cornell University in 1865 the Creole people of Louisiana. 12. While living in New Orleans, ___ (A) a book of folklore, Bayou Folk, was written by Kate Chopin about _ _ _ (B) Bayou Folk, a book of folklore, was written by Kate Chopin about _ _ _ (C) the subject of Kate Chopin's book Bayou Folk was the folklore of ___ (D) Kate Chopin wrote Bayou Folk, a book about the folklore of Section 2 • Guide to Structure and Written Expression 165 LESSON 29 INCOMPLETE OR MISSING COMPARISONS You may see sentences in the Structure section that contain comparisons. Many of these involve the comparative forms of adjectives. On the average, the Pacific Ocean is deeper than the Atlantic. Rhonda is a more experienced performer than Theresa. This show is less interesting than the one we watched last night. Be sure that the sentence compares similar things or concepts. ·The ears ofAfrican elephants are bigger than Indian elephants. (INCORRECn The ears ofAfrican elephants are bigger than those of of Indian elephants. (CORRECn The first sentence above is incorrect because it compares two dissimilar things: an African elephant's ears and an Indian elephant. In the second, the word those refers to ears, so the comparison is between similar things. Another type of comparison involves the phrase as . .. as. The lab lasted as long as the class did. There weren't as many people at the meeting as I had thought there would be. The words like/alike and unlike/not alike can also be used to express comparison: Like A, B .. . A, like B, .. . A is like B. Unlike X, Y .. . X,unlikeY, .. . X is unlikeY A and B are alike. X and Yare not alike. In these sentences, similar-looking words such as likely, likewise, dislike, and unlikely may appear as distractors. Other phrases can be used in making comparisons: A is the same as B A and B are the same A is sin1ilar to B X is different from Y X and Yare different X differs from Y A special kind of comparison is called a proportional statement. A proportional statement follows this pattern: The more A ... the more B. The higher the humidity, the more uncomfortable people feel. .... Sample Items Subtropical zones _ _ _ _ _ _ temperate zones, but they still have distinct summer and winter seasons. (A) (B) (C) (D) that are warmer warmer are warmer as are warmer than 166 Section 2 • Guide to Structure and Written Expression Choice (A) creates a relative clause, which is not appropriate in this sentence; also, the choice lacks the word than. Choice (B) lacks both a verb and the word than. (C) incorrectly uses as in place of than. _ _ _ _ _ _ other mammals, whales do not have a sense of smell. (A) Not alike (B) Unlike (C) Unlikely (D) Dislike Choice (A) is used only in the pattern "A and B are not alike." (C) and (D) are not used in comparisons; unlikely is an adjective meaning "not probable"; dislike is a verb meaning "not enjoy, not admire." _ _ _ _ _ _ the diameter of a circle, the larger its circumference is. (A) Greater than (B) The greater (C) Great as (D) As great as The correct pattern for a proportional statement is the more A ... the more B. Only choice (B) follows this pattern. ... Exercise 29 Focus: Completing structure problems involving comparisons. (Note: All the items in this exercise focus on comparisons.) Directions: Choose the option-(A), (B), (C), or (D)-that correctly completes the sentence, and then mark the appropriate blank. The first one is done as an example. 1. Sea bass freshwater bass. ~ (A) are larger than _ _ _ (B) the larger the ___ (C) are as large ___ (D) are larger 2. Wild strawberries are _ _ _ _ _ _ as cultivated strawberries. ___ (A) not so sweet _ _ _ (B) not as sweet ___ (C) less sweeter ___ (D) not as sweeter 3. Automobiles, airplanes, and buses use more energy per passenger _ _ _ _ __ ___ (A) as do trains ___ (B) than trains do ___ (C) trains do ___ (D) like trains Section 2 • Guide to Structure and Written Expression 4. The larger a drop of water, _ _ _ _ _ _. freezing temperature. _ _ (A) the higher its _ _ (B) its higher _ _ (C) higher than its ___ (D) higher of its San Diego and San Francisco, Los Angeles has no natural harbor. 5. _ _ _ (A) Dissimilar _ _ (B) Unlike _ _ (C) Dislike ___ (D) Different 6. The water of the Great Salt Lake is _ _ _ _ _ _ seawater. _ _ (A) saltier than that of ___ (B) as salty as that of _ _ (C) saltier than _ _ (D) so salty as 7. A psychosis is a severe mental disorder, _ _ _ _ _ _ than a neurosis. _ _ _ (A) the most serious _ _ _ (B) as serious _ _ _ (C) more serious than ___ (D) as though serious 8. The social system of bumblebees is not as complex _ _ _ _ __ _ _ (A) than honeybees ___ (B) as honeybees _ _ _ (C) that honeybees are ___ (D) as that of honeybees 9. The administration of private colleges is nearly _ _ _ _ _ _ that of public colleges. _ _ (A) same _ _ (B) just as _ _ _ (C) the same as _ _ (D) similar 10. __ __ ___ __ (A) (B) (C) (D) a river on land, an ocean current does not flow in a straight line. Alike Like Likewise Likely 11. The skin temperature of humans is _ _ _ _ _ _ their internal temperature. _ _ (A) not high as _ _ (B) not so high _ _ (C) as low _ _ (D) lower than 12. A butterfly a moth in a number of ways. ___ (A) is different from _ _ (B) is different ___ (C) the difference is _ _ (D) differing from 167 168 Section 2 • Guide to Structure and Written Expression LESSON 30 MISSING CONJUNCTIONS Conjunctions are connecting words; they join parts of a sentence. In this lesson, we'll look at two types of conjunctions. Coordinate conjunctions are used to join equal sentence parts: single words, phrases, and independent clauses. When two full clauses are joined, they are usually separated by a comma. The coordinate conjunctions you will most often see in Structure problems are listed in the chart below: and addition Hereford cows are brown and white. He washed his car and cleaned out the garage. or choice, possibility This plant can be grown in a house or in a garden. Her action was very brave or very foolish. but contrast, opposition He brought his wallet but forgot his checkbook. The book discussed some interesting ideas, but it wasn't very well written. nor negation He's never taken a class in sociology, nor does he intend to. so effect It was a bright day, so she put on her sunglasses. (The conjunction so is used to join only clauses-not single words or phrases.) Conjunctive adverbs (moreover, therefore, however, nevertheless, and so on) are also used to join clauses, but in Structure problems, these words are most often used as distractors-they seldom appear as correct answers. Correlative conjunctions are two-part conjunctions. Like coordinate conjunctions, they are used to join clauses, phrases, and words. both ... and addition Both wolves and coyotes are members of the dog family. not only ... but also addition Dominic studied not only mathematics but also computer science. either ... or choice, possibility We need either a nail or a screw to hang up this picture. neither ... nor negation (not A and not B) Neither the television nor the stereo had been turned off. Section 2 • Guide to Structure and Written Expression ..... 169 Sample Items The automobile began as a toy _ _ _ _ _ _ developed into a powerful force for social change. (A) it (B) but (C) when (D) or There is a contrast in this sentence; the role of the automobile as a toy in its early days is contrasted with its later role as a force for social change. The only word among the four choices that indicates contrast is choice (B), but. Singer Marian Anderson trained _ _ _ _ _ _ in the United States and abroad. (A) not just (B) and . (C) both (D) not only The correct structure for this sentence is both . .. and. Choices (A), (B), and (D) do not follow this pattern. (In choice (D), not only must be paired with but also to be correct.) ... Exercise 30 Focus: Completing structure problems involving conjunctions. (Note: One or two of the items in this exercise do not focus on conjunctions. These items are marked in the answer key with asterisks.) Directions: Choose the one option-(A), (B), (C), or (D)-that correctly completes the sentence, and then mark the appropriate blank. The first one is done as an example. 1. Blindfish, which spend their whole lives in caves, have _ _ _ _ _ _ eyes nor body pigments. _ _ (A) not any ~ (B) neither _ _ (C) nor _ _ _ (D) without 2. Specialty stores, unlike department stores, handle only one line of merchandise _ _ _ _ _ _ a limited number of closely related lines. _ _ (A) either _ _ (B) but _ _ (C) instead _ _ (D) or 3. Thomas Eakins studied not only painting _ _ _ _ _ _ anatomy when he was training to become an artist. _ _ _ (A) moreover _ _ (B) but also _ _ (C) as well _ _ (D) and 170 Section 2 • Guide to Structure and Written Expression 4. Although topology is the youngest branch of geometry, sophisticated. _ _ (A) but it ___ (B) so it _ _ (C) it ___ (D) however it is considered the most 5. In 1923 Jean Toomer wrote a book titled Cane which combined fiction poetry to describe the experience of being black in the United States. _ _ (A) and _ _ (B) to _ _ (C) also _ _ (D) or 6. Endive can be used _ _ _ _ _ _ as a salad green or as a cooking vegetable. _ _ (A) such _ _ (B) both _ _ _ (C) either ___ (D) neither 7. Glucose does not have to be digested, _ _ _ _ _ _ it can be put directly into the bloodstream. _ _ (A) so _ _ (B) while _ _ _ (C) and since _ _ (D) nor 8. Natural __ ___ __ fiber comes from either animal _ _ _ _ _ _ plant sources. (A) or (B) otherwise (C) and _ _ (D) nor 9. Paint is used to protect wood. _ _ _ (A) not only the substance _ _ _ (B) the substance which is not only _ _ _ (C) not only a substance which is _ . _ (D) not the only substance 10. An acoustic engineer's purpose in designing a factory is to suppress sound, _ _ _ _ _ _ his or her purpose in designing a concert hall is to transmit sound faithfully. _ _ (A) or _ _ (B) so _ _ (C) but .___ (D) which 11. Demographers believe most metropolitan areas will continue to grow in _ _ _ _ __ population and area in the future. _ _ _ (A) moreover _ _ (B) both _ _ (C) together ___ (D) besides 12. Most crustaceans live in the sea, _ _ _ _ _ _ some live in fresh water and a few have ventured onto land. _ _ (A) both _ _ (B) also _ _ (C) but _ _ (D)and Section 2 • Guide to Structure and Written Expression 171 LESSON 31 MISSING NEGATIVE WORDS The answer choices for this type of item are four negative expressions, such as the ones listed below: no adjective not any There was no milk in the refrigerator. none pronoun not one They took a lot of pictures, but almost none of them turned out. nothing pronoun not anything There was nothing in his briefcase. no one pronoun not anyone No one arrived at the meeting on time. nor conjunction and ... not He's never been fishing, nor does he plan to go. without preposition not having She likes her coffee without milk or sugar. never adverb at no time I've never been to Alaska. The negative word not is used to make almost any kind of word or phrase negative: verbs, prepositional phrases, infinitives, adjectives, and so on. Both no and not can be used before nouns, depending on meaning. There is no coffee in the pot. This is not coffee. (It's tea.) (It's empty.) The adjective no is also used before the word longer to mean "not anymore." I no longer read the afternoon paper. .... Sample Item There is almost ______ vegetation in the Badlands, a barren region of South Dakota. (A) (B) (C) (D) not nor none no Choices (A), (B), and (C) cannot be used before nouns as adjectives. ... By the way, probably the most common correct answer for this type of problem is the adjective no. 172 Section 2 • Guide to Structure and Written Expression Exercise 31 Focus: Completing structure problems involving negative words. (Note: All the items in this exercise focus on negative words.) Directions: Choose the one option-(A), (B), (C), or (D)-that correctly completes the sentence, and then mark the appropriate blank. The first one is done as an example. 1. Old Faithful is the most famous but _ _ _ _ _ _ the most powerful geyser in Yellowstone National Park. _ _ _ (A) none of - - ( B ) no _ _ (C) nothing ~(D)not 2. Early carpenters, having _ _ _ _ _ _ nails, had to use wooden pegs to secure their constructions. _ _ (A) no _ _ (B) not ___ (C) without _ _ (D) neither 3. Joseph Priestly, the discoverer of oxygen, had little _ _ _ _ _ _ interest in science until he met Benjamin Franklin in Paris. _ _ (A) and not _ _ _ (B) or no _ _ (C) but not ___ (D) nor any 4. Mobile homes were _ _ _ _ _ _ counted as permanent houses until the 1960 census. _ _ (A) not _ _ (B) nor _ _ (C) no _ _ (D) none 5. Most solo musicians play _ _ _ _ _ _ sheet music in front of them. _ _ _ (A) without _ _ _ (B) not having _ _ (C) lacking _ _ (D) and no 6. Desertification is the creation of deserts where _ _ _ _ _ _ had existed before. _ _ _ (A) never _ _ _ (B) no one _ _ (C) none ___ (D) not one 7. Glass snakes are actually legless lizards, _ _ _ _ _ _ snakes. _ _ (A) no _ _ (B) not _ _ (C) nor ___ (D) none 8. There is _ _ _ _ _ _ truth to the old expression "Lightning never strikes the same place twice:' _ _ (A) without _ _ _ (B) none _ _ (C) no _ _ (D) not Section 2 • Guide to Structure and Written Expression 9. single person can be said to have invented the automobile. _ _ (A) There was not a _ _ (B) Nora ___ (C) Not one of _ _ (D) No 10. A serious study of physics is impossible _ _ _ _ _ _ some knowledge of mathematics. _ _ _ (A) not with _ _ (B) no _ _ (C) not having ___ (D) without 11. two fingerprints have ever been found to be exactly the same. _ _ _ (A) No _ _ _ (B) Never _ _ (C) Not ___ (D) None 12. One of the few stands of forest on the East Coast of the United States that has _ _ _ _ __ been harvested is Hutcheson Forest in New Jersey. _ _ (A) no _ _ (B) never _ _ (C) none _ _ (D) nothing 13. Customers could, until the 1960's, open small savings accounts at U.S. Post Offices, but that service is offered. _ _ (A) no longer _ _ _ (B) not longer _ _ (C) no long _ _ _ (D) not along 14. the reptiles alive today is capable of flight. _ _ (A) No _ _ (B) None of _ _ (C) Not one _ _ (D)Not 173 174 Section 2 • Guide to Structure and Written Expression MINI-TEST 5: STRUCTURE Directions: The following sentences are incomplete. Beneath each of these sentences, there are four words or phrases, marked (A), (B), (C), and (D). Choose the one word or phrase that best completes the sentence. 1. , an organism must be able to adapt to changing factors in its environment. _ _ (A) If survival _ _ _ (B) For surviving ___ (C) To survive ___ (D) It survives 2. The art of landscape architecture is ______ that of architecture itself. _ _ _ (A) almost as old as _ _ (B) as almost old ___ (C) almost as old than ___ (D) old as almost 3. The Mummers' Parade has ______ every year in Philadelphia on New Year's Day since 1901. _ _ (A) holding _ _ (B) been holding _ _ (C) held _ _ (D) been held 4. Rarely ___ ___ ___ ___ more than ftfty miles from the coast. (A) redwood trees grow (B) redwood trees do grow (C) grow redwood trees (D) do redwood trees grow 5. Microorganisms live in extreme conditions of heat and cold where ______ other organisms can survive. _ _ (A) not _ _ _ (B) never _ _ (C) no _ _ (D) none 6. The higher one rises in the atmosphere, ______ the temperature generally becomes. _ _ (A) colder than _ _ (B) the colder ___ (C) the colder as ___ (D) is colder Section 2 • Guide to Structure and Written Expression 175 7. Medical researchers are constantly looking for ways to control, _ _ _ _ _ _ , and cure diseases. ___ (A) prevention _ _ (B) preventing _ _ (C) prevent _ _ _ (D) to prevent 8. Nerve cells, or neurons, in the human body. ___ (A) the most complex cells are _ _ _ (B) are the most complex cells ___ (C) most complex the cells are ___ (D) most are the complex cells 9. Released in 1915, ___ (A) D. W Griffith made an epic fUm about the Civil War, Birth of a Nation _ _ _ (B) the Civil War was the subject of D. W Griffith's epic ftlm, Birth of a Nation ___ (C) D. W Griffith's epic film Birth of a Nation was about the Civil War ___ (D) the subject of D. W Griffith's epic ftlm Birth of a Nation was the Civil War 10. ___ ___ ___ ___ on barren slopes can help prevent erosion. (A) Planting trees (B) For trees to be planted (C) In order to plant trees (D) Trees are planted 11. Vermont is the only state in New England an Atlantic coastline. ___ (A) without _ _ (B) not with _ _ (C) which not having ___ (D) doesn't have 12. In 1867, Hiram R. Revels ___ (A) becoming _ _ _ (B) became ___ (C) to have become _ _ (D) has become the first black to be elected to the u.s. Senate. 13. Jupiter's moons can be easily seen through _ _ _ _ _ _ binoculars or a small telescope . ........,--- (A) either _ _ (B) if _ _ (C) whether _ _ (D)or 14. The Colorado beetle is a beautiful insect, _ _ _ _ _ _ it causes a great deal of damage to food crops. _ _ (A) but _ _ (B) what - - ( C ) or _ _ (D) that 15. Judge Francis Hopkins is probably best known as a signer of the Declaration of Independence, but he also excelled as a poet, , and an orator. ___ (A) as a musician _ _ (B) by playing music ___ (C) a musician _ _ _ (D) he played music PAR T B Written Expression About Written Expression In this part of the test, there are twenty-five sentences. In each sentence, four expressions-single words or two- or three-word phrases-are underlined. Your job is to identify which of these phrases must be rewritten (it can't simply be omitted) in order for the sentence to be correct. All the errors involve grammar or usage-never punctuation or spelling. T Sample Items Music, dramatic, and art contribute to the culture of any community. A B C J) This sentence should correctly read "Music, drama, and art contribute to the culture of any community." Choice (A) would have to be rewritten to correct the sentence, so (A) is the best answer. 176 Section 2 • Guide to Structure and Written Expression 177 Lenses may having either concave or convex shapes. --AB C --D- The correct verb form after a modal auxiliary is the simple form have. This sentence should read "Lenses may have either concave or convex shapes." The best answer is (A). .. What's the Best Way to Answer Written Expression Items? You should begin with a quick reading of each sentence to find any obvious errors. Don't simply read the underlined portions, because in most items the underlined expression is incorrect only in the context of the sentence. Don't answer the question until you've read the entire sentence. Easier questions can be answered after the first reading; mark your answer and go on. If you don't find the error immediately, re-read the sentence, now concentrating on the underlined expressions. You can't use the same techniques for reading these items as you would to read other materials, such as newspapers or magazine articles. Usually, a person's eyes move very quickly over "little words" like articles and prepositions because these words don't contain much information. However, in this part of the test, these expressions may be used incorrectly. It would be helpful, in fact, if you could read these sentences aloud, but of course that isn't permitted. You can, however, train your eyes to move slowly and to pronounce the sentences in your mind exactly as if you were speaking them. If you haven't identified the error after a careful reading of the sentence, go through a mental checklist of the most common errors: word form, word choice, verb error, parallelism, pronoun errors, and singular/plural noun errors. Do the underlined expressions seem to fit into any of these categories? If you still can't find an error, eliminate expressions that seem to be used correctly, and then make a guess from any items that remain. Type of Error Frequency of Errors in Written Expression* Percentage Word Form 21 Word Choice 15 Verbs 10 Parallel Structures 9 Pronouns 8 Singular/Plural Nouns 8 Verbals (Infinitives, Gerunds, and Participles) 6 Prepositions 6 Articles 5 Word Order 5 Comparatives and Superlatives 3 Conjunctions 2 Other Types of Errors 2 • Based on an analysis of 20 different exams that test-takers were alJo:vved to keep after Disclosed Test Admissions. Section 2 • Guide to Structure and Written Expression 178 In this section of the Guide, the lessons follow the same order as given in the chart above. The most common error, which involves incorrect word forms, is considered ftrst, and so on. Each type of error is explained, and examples are provided. There are exercises to help you practice identifying all these types of errors. When taking actual exams, once you've found an error, don't worry about how to correct it. However, while working the exercises in this Guide, it's important to understand why an item is incorrect, so most of the exercises ask you not just to identify errors but to supply corrections as well. • Skim each sentence, looking for obvious errors. o If you haven't found the error, read the sentence again carefully, concentrating on the underlined parts. Go through a mental checklist of the most common types of errors (those involving word form, word choice, parallelism, verbs, pronouns, and singular/plural nouns) to see if any of the underlined expressions seem to fall into those categories. • If you are still unable to fmd an error, try eliminating options that seem to be correct. If more than one option remains, put a mark by the number of that item on your answer sheet, and then take a guess and go on. o If you fmish before time is called, go back and work on Section 2 problems that you marked as difficult. Make sure you have an answer for every problem. Don't go on to Section 3. Section 2 • Guide to Structure and Written Expression 179 LESSON 32 ERRORS WITII WORD FORMS By far the most common type of Written Expression error involves word forms. As many as eight or nine items per test may be word-form problems. Most errors of this type involve using one part of speech in place of another. Both the incorrect word and the correction come from the same root (rapid and rapidly, for example, or inform and information). The four parts of speech generally involved are verbs, nouns, adjectives, and adverbs. The most common problems are adjectives in place of adverbs and adverbs in place of adjectives. Nouns in place of adjectives and adjectives in place of nouns are also commonly seen. In some word-form problems, different forms of the same part of speech may be involved. For example, a noun that refers to a person (leader) may be used in place of the field (leadership). A gerund (a verbal noun) may also be used in place of an ordinary noun (judging andjudgment, for example). Parts of speech can often be identified by their suffixes (word endings): Common Nount:miing$· .:'::,::;:,{":';;',;; -tion information -sion provision -ship scholarship -ence existence -tude multitude Mance acceptance -ism capitalism -ity -ery recovery democracy creativity -cracy -hood childhood -logy biology -dom wisdom -ness happiness -th health -er explorer -ee employee -or sailor -ic comic Mist psychologist -ian technician student -ant attendant -ent -ment Common Verb ~ndin9$ , ,~. , experiment . -ize realize -ify satisfy -en shorten -ate incorporate -er recover 180 Section 2 • Guide to Structure and Written Expression Endings -ate moderate -y sunny -ous dangerous -ic economic -al normal -ical logical -ial remedial -ory sensory -able comfortable -less hopeless -ible sensible -ive -ish sluggish -ly friendly -ant resistant -ful colorful quickly -ally -ly competitive historically A) Adjective! Adverb Errors The most common type of word-form problem involves the use of an adverb in place of an adjective or an adjective in place of an adverb. A few points to keep in mind: • Adjectives modify nouns, noun phrases, and pronouns. • Adjectives often come before nouns. an important test a quiet evening a long letter • They often answer the question What kind? She is a brilliant doctor. (What kind of a doctor is she? A brilliant one.) • Adjectives also follow the verb to be and other linking verbs. The glass was empty. That song sounds nice. They look upset. • Adverbs may modify verbs, participles, adjectives, prepositions, adverb clause markers, and other adverbs. Ann eagerly accepted the challenge. (adverb modifying the main verb accepted) It was a rapidly changing situation. (adverb modifying the present participle changing) She wore a brightly colored scarf. (adverb modifying the past participle colored) Ted seemed extremely curious about that topic. (adverb modifying the adjective curious) We arrived at the airport shortly before our flight left. (adverb modifying the adverb-clause marker before) We arrived at the airport shortly before noon. (adverb modifying the preposition before) The accident occurred incredibly quickly. (adverb modifying the adverb quickly) Section 2 • Guide to Structure and Written Expression 181 • Sometimes adverbs are used at the beginning of sentences, often followed by a comma. These adverbs sometimes modify the entire sentence rather than one word in the sentence. Generally, I like my classes. Usually Professor Ingram's lectures are more interesting. • Most adverbs tested in this section are adverbs of manner. They are formed by adding the suffix -ly or -ally to an adjective. qUick comfortable quickly comfortably comic historic comically historically • Adverbs of manner answer the question How? She treated her employees honestly. (How did she treat her employees? Honestly.) • A few adverbs (fast, hard, high, for example) have the same form as adjectives. He bought a fast car. (adjective) He was driving so fast that he got a speeding ticket. (adverb) • Well is the irregular adverb form of the adjective good. Juan is an exceptionally good student. He did very well on the last test. • Some adjectives also end in -ly: friendly, yearly, costly, and lively, for example. That was a costly mistake. I found Houston a very friendly city. ..... Sample Items The Black Hills of South Dakota are covered with densely pine forests. 1\ IB --cD The best answer is (D). An adjective, dense, not an adverb, is required to modify the noun phrase pine forests. During solar storms, the amount of radiation reaching the Earth is A B C abnormal high. D The adverb abnormally is needed to modify the adjective high. ... B) Incorrect Forms of Words Connected with Certain Fields This error involves a confusion between the names of fields (biology, for example) and the name of a person who practices in that field (biologist), or between one of those terms and the adjective that describes the field (biological). 182 Section 2 • Guide to Structure and Written Expression ~ Sample Item First specializing in industrial photography, Margaret Bourke-White later A B became a famous news photographer and editorial . C 0 The adjective editorial is used to describe the field of editing. However, a noun referring to a person (editor) is needed in this sentence. ... C) Other Word-Form Problems There are many other word-form problems. Some examples are given here: ~ Sample Items Corn played an important role in the cultural of the cliff-dwelling A --BC 0 Indians of the Southwest. The noun culture, not the adjective cultural, is needed. The galaxy Andromeda is the most distance object visible to observers A B --C-- 0 in the Northern Hemisphere. The adjective distant is needed in place of the noun distance. Scientists belief that the continents once formed a single continent --A-- B surrounded by an enormous sea. C 0 In this sentence, the verb believe is needed in place of the noun belief Bunsen burners are used to hot materials in a chemistry lab. A -n C 0 The verb heat is needed in place of the adjective hot. A sudden freezing can destroy citrus crops. ABC --0-- Rather than the gerund (-ing) form, the noun freeze is required. ... Section 2 • Guide to Structure and Written Expression 183 Exercise 32.1 Focus: Correctly providing word forms for parts of speech commonly confused in Written Expression problems. Directions: Fill in the lines in the blanks below with the appropriate word forms. In some cases, there may be more than one correct answer. The first one is done as an example. Noun Verb 1. differ difference Adjective different differently inventive 2. 3. comEete 4. fertilize deeply 5. decision 6. 7. beautify 8. prohibit 9. originate 10. emphatic 11. inconvenient glory 12. 13. Adverb mystify equally 14. general 15. simply 16. 17. familiar 18. pure 19. free 20. restrict Exercise 32.2 Focus: Providing word forms related to the names of fields, to adjectives describing those fields, and to people involved in those fields. Directions: Fill in the blanks in the chart below with the appropriate form. The first one is done as an example. 184 Section 2 • Guide to Structure and Written Expression Field I. music Person musician poetic 3. electrician 4. administration -----_._-financial 6. photographer 7. 8. theory athletic 9. 10. editing II. philosopher 12. criminal political 13. lawyer 14. 15. musical surgeon 2. 5. Adjective humor Exercise 32.3 Focus: Identifying errors and recognizing correct use of adjectives and adverbs. Directions: Underline the form that correctly completes the sentence. The first one is done as an example. 1. In any animal community, herbivores (great/~) outnumber carnivores. 2. Floods cause billions of dollars worth of property damage (annual/annually). 3. (Regular/Regularly) airmail service in the United States began in 1918. 4. Writer Ernest Hemingway was known for his (simple/simply) language and his lively dialogue. 5. The tiny coral snake is (beautiful/beautifully) but deadly. 6. Skyscrapers developed (simultaneous/simultaneously) in Chicago and New York City. 7. (General/Generally), bauxite is found near the surface, so it is relatively (simple/simply) to mine. 8. A good proofreader (painstaking/painstakingly) examines a manuscript for errors in spelling and grammar as well as for factual mistakes. 9. The colony of New Hampshire was (permanent/permanently) separated from the Massachusetts Bay Colony in 1692. 10. The most numerous and (wide/widely) distributed of all insectivorous animals are the shrews. The endocrine system functions in (close/closely) relationship with the nervous system. A gap in the Coast Range of California provides (easy/easily) access to the San Francisco Bay area. Mushrooms are found in an (incredible/incredibly) range of sizes, colors, and shapes. Some airplanes have an automatic pilot that is connected to the airplane's controls and (automatic/automatically) keeps the plane on course. 15. Winslow Homer, who had no (formal/formally) training in art, became famous for his paintings of the sea. 11. 12. 13. 14. Section 2 • Guide to Structure and Written Expression 185 16. 17. 18. 19. The potter's wheel was an invention of (profound/profoundly) importance. The nuclear-powered cargo ship Savannah proved (commercial/commercially) impractical. Sojourner Truth spoke (persuasively/persuasive) in opposition to slavery. In 1948 Stan Getz made a (masterful/masterfully) solo recording of the song "Early Autumn" which (deep/deeply) influenced younger musicians. 20. The planet Venus was once believed to be two (distinct/distinctly) objects: the morning star Phosphorous and the evening star Hesperus. Exercise 32.4 Focus: Identifying which parts of speech are appropriate in sentences. Directions: Underline the form that correctly completes the sentence. Then identify the parts of speech of the words in parentheses. You can use these abbreviations for parts of speech: N=noun V = verb PN = "person" noun G = gerund (-ing) noun ADJ = adjective ADV= adverb The first one is done as an example. 1. Sinclair Lewis' novel Babbitt is set in the (fiction/fictional) town of Zenith. ( N / AD. I ) 2. By-products from chicken eggs are used by (industry/industrial) in manufacturing such (produces/products) as soap and paint. ( / ) ( / ) 3. The daylily is an attractive, (fragrance/fragrant) flower. ( _ _ _ _ _ / _ _ _ _ _ ) 4. An equation is a (mathematics/mathematical) statement which says that two expressions are (equal/equality). ( / )( / ) 5. The Supreme Court has ruled that (evidence/evident) obtained from (illegal/illegally) searches cannot be used in court. ( / )( / ) 6. The Richter Scale measures the (severely/severity) of earthquakes. ( _ _ _ _ _ / _ _ _ _ _ ) 7. Justin Winsom promoted the (developing/development) of libraries throughout the United States in the nineteenth century. ( / ) 8. Pipelines (transportation/transport) huge quantities of natural gas and liquid petroleum products. ( / ) 9. Scientists (differ/different) in their opinions of how snow crystals (originate/origin). ( / )( / ) 10. Harry Blackstone was a famous (magic/magician). ( _ _ _ _ _ / _ _ _ _ _ ) 11. Glass sponges are found in oceans at a (deep/depth) of 300 feet or more. ( / ) 12. Colorado shares with Wyoming the (distinction/distinctly) of having four (perfect/perfectly) straight borders. ( / )( / ) 13. Yale's Peabody Museum has a world-famous (collection/collecting) of fossils. ( / ) 14. Pronghorns, which are American antelopes, are (present/presence) in large numbers on the (open/openly) plains of Wyoming. ( / )( / ) 186 Section 2 • Guide to Structure and Written Expression 15. The President's (chooses/choices) for the members of the Cabinet must be (approved/approval) by the Congress. ( / )( / ) 16. Rose Han Lee wrote a number of (scholar/scholarly) accounts about the effects of (immigrant! / ) immigration) on mining towns in the western United States. ( ( / ) 17. Most snails venture out to look for (feed/food) only after sunset or on (rain/rainy) days. ( / )( / ) 18. Hats may (symbolize/symbol) social status or (occupation/occupational) as well as being fashion items. ( / )( / ) 19. Analgesics are used to (relieve/relief) pain and reduce fever. ( _ _ _ _ _ / _ _ _ _ _ ) 20. The process of (respire/respiration) in plants involves a complex series of (chemistry/chemical) reactions. ( / )( / ) 21. A (member/membership) of the Paiute tribe of Nevada, Sarah Winnemuca worked as a guide and (interpret/interpreter). ( / )( / ) 22. The (strong/strength) of a rope is (direct/directly) proportional to its cross-sectional area. ( / )( / ) 23. The Nassau grouper is a (tropics/tropical) fish that is noted for its (able/ability) to change color. ( / )( / ) 24. Alpha rays (loss/lose) energy (rapidity/rapidly) as they pass through matter. ( / )( / ) 25. The cherry is one of the only fruits that will not (ripe/ripen) if it is removed from the tree. ( / ) Exercise 32.5 Focus: Identifying errors involving word-form problems. (Note: One or two items in this exercise do not focus on word-form errors. These are marked in the answer key with an asterisk.) Directions: Decide which of the four underlined words or phrases-(A), (B), (C), or (D)-would not be considered correct, and write the letter of the expression in the blank. Then, in the line at the end of the sentence, write the correction for the underlined phrase. The first one is done as an example. ~ 1. Liberal arts colleges cultivate general intellectually abilities rather than technical or ABC professional skills. intellectllal o 2. Goats are extremely destruction to natural vegetation, and are often responsible for soil ABC 0 erosion. _ _ _ _ __ 3. Wild plants were of considerable important to early settlers, and many are still used A B C medicinally and as foods. _ _ _ _ __ o Section 2 • Guide to Structure and Written Expression 187 4. One important branch of linguistics is semantics, which analysis the meaning of words. A B C 0 5. Unlike folk dancers, which are the product ofa single culture, ballet is an international A B C 0 art form. ______ 6. The Earth's out shell is divided into sections called plates, which are constantly in 1\ motion. ______ B C o 7. Black bears can move rapidly when necessary and are skillful tree-climbers for their size A B --C- and weigh. _ _ _ _ __ --0- 8. In an arboretum, trees are cultivated for scientific and educational purpose. A B C 0 9. In most Western states, the first major industry was mining, which was gradually A B supplemented by ranches. _ _ _ _ __ C 0 lO. Peach trees grow good in a variety of soil types, but do best in sandy loam. -A--B- -0- C 11. The unit of measuring called the foot was originally based on the length of the human -;;:- B C --0- foot. 12. Philosopher Theodore A. Langerman was interested in the fields of literary and music. -0- A B C 13. Pure nitric acid is colorless, but it acquires a yellow color when it is exposed of air. -A- B -C- 14. A chemical react that absorbs heat is called endothermic. ______ A B C o 0 188 Section 2 • Guide to Structure and Written Expression ___ 1 '5. One characteristic of the poems of Emily Dickinson is the sharp of her images. A B C D 16. Luther Gulick was a teacher and physician who spent much of his live promoting A B -C physicalfitness. _ _ __ D P. A dog should be checked regularly by a veterinarian to ensure that it remains in good A B C healthy. D I H. Southwestern Boston is made up of Hyde Park, West Roxbury, and other f>1easant A B residential neighbors. _ _ _ _ __ C D ___ 19. Hunting and fishing techniques were highly developed among the North American A Indians, particularly in regions where agriculture was less success. _ _ _ _ __ B C D 20. Science requires the careful collect and organization of data. _ _ _ _ __ A --B---C- D 21. The Natchez Trace was an important commercial and military route between Nashville, A B C Tennessee to Natchez, MiSSissippi. _ _ _ _ __ D 22. Some games rely mainly on skill and practice while others primarily involve lucky. --A- -B- C -D- _ __ 23. In the absent of natural enemies, the gypsy moth has become a serious pest in North A B C 0 America. 24. Huey Long and his brother Earl were the two most powerful polities in the history of A B C D Louisiana. _ _ _ _ __ 2.,. To make candles, pioneers twisted string into wicks, dipped the wicks into hot fat, then A 13 hung the candles to cool and hard. _ _ _ _ __ -(-:- --0- Section 2 • Guide to Structure and Written Expression 189 LESSON 33 ERRORS IN WORD CHOICE Word-choice errors involve the incorrect use of one word in place of another. These two words may be related forms (otber and anotber, for example), or they may be completely different (do and make, for example). Descriptions of some of the most common word choice errors are given below. A) Wrong Choice of Make or Do The verb to do is often used in place of to make, and to make in place of to do. In its basic sense, to make means to produce, to create, to construct, while to do means to perform, to act, to accomplish. These verbs are also used in a number of set expressions: make advances make an offer make an attempt make a plan make a comparison make a point make a contribution make a prediction make a decision make a profit make a distinction make a promise make a forecast make a sound/noise make an investment make a suggestion make a law be made of (= be composed of) make up (= compose) To make is also used in this pattern: make + someone + adjective (The gift made her happy). do an assignment do a job (errand, chore) do business with do research do one's duty do one's work do someone a favor The auxiliary verb do is used rather than repeat main verbs (My computer doesn't operate as fast as theirs does). Anytime you see the verb make or do underlined in the Written Expression section, suspect a wordchoice error. 190 Section 2 • Guide to Structure and Written Expression ... Sample Items Cement is done from varying amounts of limestone, clay, and gypsum. ABC D The verb done is incorrect in this sentence. The correct word choice is made. Small town newspapers often urge readers to make business with local merchants. A B C The phrase should read do business. D ... B) Wrong Choice of Like or Alike and Like or As The word alike is incorrectly used in place of like, or like is used in place of alike. These words are used correctly in the following patterns: LikeA, .. . A, like B, .. . A is like .. . A and B are alike ... Like birds, mammals are warm-blooded. Birds, like mammals, are warm-blooded. Birds are like mammals in that they are both warm-blooded. Birds and mammals are alike in that they are both warm-blooded. Whenever you see the words alike or like underlined, you should suspect a word-choice error. The word like is also sometimes confused with the word as. When like is used in a comparison, it is followed by a noun or pronoun. When as is used in a comparison, it is followed by a clause containing a subject and a verb. I did my experiment just as Paul did. My results were much like Paul's. The word as is also used before nouns when it means in place of or in the role of. This is particularly common after certain verbs: serve, function, and use, among others. The Vice-President served as President when the President was sick. ... Sample Items Alike their close relative the frogs, toads are amphibians. -A- B C D Choice (A) doesn't follow the pattern Like A, B ... Asters, as most perennial plants, bloom once a year. A -B-_c-D- The word like should be used in place of the word as before a noun phrase (most perennial plants). ... Section 2 • Guide to Structure and Written Expression 191 C) Wrong Choice of So, Such, Too, and As The words so, such, and too are used in the following patterns: so + adjective + that clause These boxes are so heavy that we can't lift them. (So is also used with many . .. that and much . .. that.) There were so many people in the auditorium that we could barely get in the front door. such + adjective + noun phrase + that clause It was such a pretty view that he took a photograph. too + adjective + infinitive It's too cold to go swimming today. Notice that so and such are both followed by that clauses, but too is followed by an infinitive. The words as and so are also sometimes confused: *Jane did so well as I did on the economics exam. (lNCORRECn *The coffee was as hot that I couldn't drink it. (lNCORRECn In the first sentence, the word as should be used in place of so; in the second, so should be used in place of as. Also look for so much or too much used in place of so or too. T Sample Items The sun is so bright to look at directly. ----;\ B --cD The correct pattern too + adjective + infinitive. In much of Alaska, the growing season is as short that crops can't be raised. A B C -0- The correct pattern is so + adjective + that clause. The giant squid is so an elusive animal that at one time it was believed to be A B C purely mythical. o Before an adjective + noun + that clause, the word such should be used. The mineral grains in basalt are so much small that they cannot be seen A B C with the unaided eye. The phrase should read so small rather than so much small. 0 192 Section 2 • Guide to Structure and Written Expression D) Wrong Choice of Another or Other Use of Another and Other another + singular noun (Have another sandwich.) other + plural noun (I wonder if there is life on other planets.) determiner + other + noun (There may be life on some other planets.) another (Thanks. I'll have another.) determiner + other ("I have one book." "I have the other.") Another means "one more, an additional one." It can be used as an adjective before a singular nouns or alone as a pronoun. He needs another piece of paper. I have one class in that building, and another in the building across the quadrangle. Other is used as an adjective before a plural noun. It is also used as an adjective before a singular noun when preceded by a determiner such as the, some, any, one, or no. It can also be used alone as a pronoun when preceded by a determiner. There are other matters I'd like to discuss with you. One of the books was a novel; the other was a collection of essays. There's no other place I'd rather visit. .... Sample Items Willa Cather is known for My Antonia and another novels A B of the American frontier. c D Before a plural noun, other must be used. An understudy is an actor who can substitute for other actor A B -c- in case of an emergency. D Other is used incorrectly in place of another before a singular noun. .... E) Wrong Choice of Because or Because Of; and Similar Expressions or Although; During or WhenIWhile Certain expressions, such as because, are adverb clause markers and are used only before clauses. Other expressions, such as because oj, are prepositions and are used before noun phrases or pronouns. Section 2 • Guide to Structure and Written Expression Adverb ·ctauseMarkers (Used with Clauses) Prepositions (Osedwith Noun Phrases) because because of although when despite in spite of while during T 193 Sample Items Because migration to the suhurhs, the population of ma~l large A B C American cities declined hetween 1950 and 1960. o -e CD CD CD) Before a noun phrase (migration), the preposition because o!must he used. Despite most people consider the tomato a vegetahle, botanists classify A B C it as a fruit. D Before a full clause (most people consider the tomato a vegetable), the adverh marker although must he used. ... F) Wrong Choice of Much or Many and Similar Expressions Certain expressions can only be used in phrases with plural nouns; others can he lIsed in expressions only with non-count nouns. Used with Plural Nouns Used with Non-count Nouns many much few, a few little, a little fewer, the fewest less, the least number amount T Sample Items Pearls are found in much colors, including cream, blue, lavender, and hlack. ABC Many must be used with a plural noun (colors). D 194 Section 2 • Guide to Structure and Written Expression Even during economic booms, there is a small number of unemployment. A B C D The word amount must be used to refer to a non-count noun such as unemployment. ... -------------------------------------------- G) Other Word-Form Problems Other pairs of words are sometimes confused in Written Expression, including those listed below. All of the sentences with asterisks are examples of errors and are INCORRECT. no not Used as an adjective before nouns; means "not any." Also used in the expression no longer. Used to make all other words negatives. *Not gasoline was left in the tank. "This is no the station I usually listen to. *1 not longer listen to that station. most almost Used in superlative adjective phrases; also used to mean "the majority." Used as an adverb to mean "nearly." *This is the almost interesting chapter in the book. "I've read almost of the chapters in the book. *I've solved most all of the problems in the book. twice double Used as an adjective to mean "two times." Used as an adjective to mean "make twice as large." "Henry has double as much money as he did before he invested it. "Henry twice his money. earliest soonest Used as a superlative adjective to mean "most distant in time." Used as a superlative adverb to mean "most promptly." *These are the soonest examples of the artist's works. (You will probably not see earliest used incorrectly in place of soonest.) percent percentage Used after a number. Not used after a number. *Fifty percentage of the people voted in favor of the initiative. *The percent of people who approve of the initiative has been steadily growing. after afterwards Used as a preposition before a noun or as an adverb clause marker before a clause. Used as an adverb, means "after that." "We'll go to dinner afterwards the play. "We'll go to dinner afterwards the play is over. *First the performer played the guitar and after she played the flute. ago before Used to talk about a time earlier than the present. Used to talk about a time earlier than some other point in time. *Harold won a gold medal in the Olympics last year, and four years ago that, he won a silver medal. Section 2 • Guide to Structure and Written Expression 195 (You will probably not see before used incorrectly in place of ago.) tell Used with an object; also used in certain set expressions: tell a story, say Used without an object. tell the truth, tell a secret. *Mr. Hunter said us that he'd had a good trip. *Joe said a wonderful story. *Mr. Hunter told that he'd had a good trip. ever never Means "at any time." Used with not to mean "never." Also used in some set expressions such as ever since and hardly ever. Means "at no time." Not used with a negative word. *He hardly never goes to that club. (You will probably not see ever used incorrectly in place of never.) alive live Used after a verb. Used before a nOlln. 'Sue likes to have alive plants in her apartment. *Although she forgot to water it for a week, the plant was still live. around round Used as a preposition to mean "in a circular path." Used as an adjective to mean "circular in shape." *The new office building will be around glass tower. (You will probably not see round used incorrectly in place of around.) age old Used as a noun, often in these patterns: at the age of twenty-one twenty-one years of age Used as an adjective, often in this pattern: twenty-one years old *Harriet will be thirty years age next week. 'Operators of motor vehicles must be thirty years of old in this state. near nearly Used as an adjective; means "close to." Used as an adverb; means "almost." Lynn is looking for an apartment nearly the Medical Center. The two-bedroom apartment she looked at cost near a thousand dollars a month. some somewhat Used as a determiner before a noun to mean "an indefinite amount." Used as an adverb to mean "slightly." *This bicycle is some more expensive than the one I looked at yesterday. (You will probably not see somewhat used incorrectly in place of some.) You can practice the distinctions between many of these words in Exercise 44.7. Note: The distinctions between words such as desert and dessert, stationary and stationery, capital and capitol, which are really spelling problems, are NOT tested on TOEFL. 196 Section 2 • Guide to Structure and Written Expression Exercise 33.1 Focus: Choosing correctly between do and make. Directions: Underline the word that correctly completes each sentence below. The first one is done as an example. 1. The tips of high-speed dental drills are (done/made) oftungsten steel and often contain diamonds. 2. A cottage industry is a form of manufacturing (done/made) at home. 3. Margaret Mead (did/made) fundamental contributions to both the theory and field work of anthropology. 4. Many universities receive grants to (do/make) research for the federal government. 5. Research in genetics in the early nineteenth century (did/made) much to improve agriculture. 6. Futurologists study current trends to (do/make) predictions about the future. 7. Filmmaker George Lucas has (done/made) many advances in the production of motion pictures, especially in the use of special effects. 8. The distinction between wildflowers and weeds is one that is often difficult to (do/make). 9. The helicopter can (do/make) jobs that no other aircraft can. 10. Yeast is added to dough to (do/make) bread light and porous. Exercise 33.2 Focus: Choosing correctly between like and alike and like and as. Directions: Underline the word that correctly completes each sentence below. The first one is done as an example. 1. The government of the United States and that of Canada are (alike/like) in that both conduct a complete census every ten years. 2. Fashion design, (as/like) all types of design, has been greatly aided by computers. 3. (Alike!Like) stars, galaxies tend to congregate in clusters. 4. Fungi are the most important decomposers of forest soil, just (as/like) bacteria are the most important decomposers of grassland soiL 5. The spinal column is (alike/like) the brain in that its main functions can be classified as either sensory or motor functions. 6. A peanut is not actually a nut but a legume (alike/like) peas and beans. 7. The stately Government House in Annapolis serves (as/like) the residence of the Governor of Maryland. 8. The cetosaur was a dinosaur that looked much (as/like) the whales of today. 9. Fats are made up of carbon, hydrogen, and carbon atoms just (like/as) carbohydrates are. 10. One way pumpkins and watermelons are (like/alike) is that both grow on vines trailing along the surface of the ground. Section 2 • Guide to Structure and Written Expression 197 Exercise 33.3 Focus: Choosing correctly between so, such, too, and as. Directions: Underline the word that correctly completes each sentence below. The first one is done as an example. 1. The mineral talc is ~such) soft that it can be scratched with a fingernail. 2. Oceanographers use rohots and unmanned submarines to explore parts of the ocean that are (so/too) deep for people to explore safely. 3. (So/As) much paper money was printed during the Revolutionary War that it became almost worthless. 4. The walking stick is an insect with (so/such a) close resemblance to a twig that it escapes the notice of its enemies. 5. At present, solar cells are (so/too) expensive and inefficient to be used in the commercial generation of electricity. 6. Acrylic plastics are very hard and are (so/as) clear as glass. 7. Founded in ]682, Norfolk developed (so/such a) prosperous sea trade that it quickly became the largest town in the colony of Virginia. 8. Continental islands are (so/so much) close to continents that their plant and animal life are identical to life on the mainland. 9. Timberline is the elevation on a mountainside above which temperatures he come (so/too) cold for most trees to grow. 10. A few people have (such/too) good eyesight that they can actually see the brightest stars during full daylight. Exercise 33.4 Focus: Choosing correctly between other and another. Directions: Underline the word that correctly completes each sentence below. The first one is done as an example. 1. Lightning is a rush of electrical current from a cloud to the ground or from one cloud to (another/other). 2. A ballet dancer's techniques and skills are very different from those of (another/other) dancers. 3. The commercial center of New York City, the island of Manhattan is joined to the (another/other) boroughs by bridges and tunnels. 4. The legal surrender of a criminal suspect from one state or country to (another/other) is called extradition. 5. Rocky Mountain spotted fever is one type of disease that is carried by ticks, and Colorado tick fever is (another/other). 6. The art of photography has often been influenced by-and has influenced-(another/other) fine arts. 7. William 0. Douglas was a Supreme Court justice for thirty-six years, longer than any (another/other) justice in the history of the Court. 8. In physics, diffusion is the spread of one substance's molecules or atoms through those of (another/other). 198 Section 2 • Guide to Structure and Written Expression 9. A basketball player may advance the ball by dribbling it or passing it to (another/other) player. 10. Limkins are water birds that eat snails and (another/other) mollusks. Exercise 33.5 Focus: Choosing correctly between because of or because and similar expressions. Directions: Underline the word that correctly completes the sentences below. The first one is done as an example. 1. (Although/Despite) cats cannot see in complete darkness, their eyes are much more sensitive to light than humans' eyes. 2. (Because!Because of) cheese is essentially a concentrated form of milk, it contains the same nutrients as milk. 3. (Although/In spite of) its frightening appearance, the octopus is shy and completely harmless. 4. (Because!Because of) its acute sense of smell, the bloodhound is often used in tracking. 5. (When/During) the female oriole is absent from the nest, the male oriole stands guard. 6. (Although/Despite) their light weight, aluminum alloys can be very strong. 7. (Although/In spite of) Adlai Stevenson was never elected preSident, he was one of the preeminent American politicians of the mid-twentieth century. 8. Snakebirds were not given their name because they eat snakes, but (because!because of) their long, slender necks resemble snakes. 9. In the sixteenth century, it was thought that a compass needle pointed north (because!because of) some mysterious influence of the stars. 10. (Although/Despite) it can occur in adults, chicken pox is classified as a disease of childhood. 11. Opinion polls are often used (while/during) political campaigns to find out how voters feel about candidates and issues. 12. Geneticists often experiment with bacteria and viruses (because!because ot) those organisms reproduce so quickly. Exercise 33.6 Focus: Choosing correctly between much or many and similar words. Directions: Underline the word that correctly completes each sentence below. The first one is done as an example. 1. (Many/Much) industrial products can be made from soybeans. 2. Desert plants compete fiercely for the (few/little) water that is available. 3. The American designer Louis Comfort Tiffany took (many/much) of his inspiration from nature. 4. A (few/little) simple precautions can prevent accidents at home and on the job. 5. In a formal debate, the same (number/amount) of persons speak for each team, and both teams are granted an equal (number/ amount) of time in which to make their arguments. 6. Bats do (few/little) damage to people, livestock, or crops. 7. Even small (numbers/amounts) of zinc can have a significant effect on the growth of plants. · II . I: ·• Section 2 • Guide to Structure and Written Expression 199 8. The adrenal glands, one on top of each kidney, secrete (many/much) important hormones. 9. (Many/Much) of the stories in John Weems' biography of George Washington are difficult to believe. 10. Folk artists have (few/little) or no formal art training. Exercise 33.7 Focus: Choosing correctly between other commonly confused words. Directions: Underline the word that correctly completes each sentence below. The first one is done as an example. 1. At eight weeks of ~old), red foxes begin to get their adult markings. 2. The Missouri River is about (double/twice) as long as the Colorado River. 3. Catherine Esther Beacher established schools in Connecticut and Ohio, and (after/afterwards) founded the American Women's Educational Association. 4. (Most/Almost) antibiotics are antibacterial agents, but some are effective against fungal, protozoal, or yeast infections. 5. In 1941, nylon was first used to make stockings, and the year (ago/before), it was first used to make toothbrush bristles. 6. Chuck Berry was one of the (soonest/earliest) and most influential performers of rock music. 7. Long before Columbus, various thinkers believed that the Earth was (around/round). 8. Apricots, which are (some/somewhat) smaller than peaches, are known for their delicate taste. 9. Huge radio telescopes aimed into space may someday (say/tell) us whether intelligent life exists elsewhere in the universe. 10. One of Canada's most beautiful botanical gardens is Butchart Gardens (near/nearly) Victoria, British Columbia. 11. Since 1945, the average size of American farms has more than (doubled/twice). 12. When the Hopi Indians perform the Snake Dance, the dancers handle (alive/live) rattlesnakes. 13. Around eighty-five (percentage/percent) of the bauxite produced in the United States is mined in Arkansas. 14. Artist Clementine Hunter continued to paint until she was over 100 years (age/old). 15. The period immediately (after/afterwards) the Civil War is known as Reconstruction. 16. (No/Not) plant has a nervous system, and most respond very slowly to stimuli in their environment. 17 . (Most/Almost) every county in the United States has agricultural extension agents who provide help to farmers. 18. Murals (say/teU) narrative stories through visual images. 19. Forests cover (near/nearly) half the land area of Tennessee. 20. Giraffes hardly (ever/never) sleep more than twenty minutes a night. 200 Section 2 • Guide to Structure and Written Expression Exercise 33.S Focus: Identifying a variety of word-choice errors. (Note: One or two items in this exercise do not focus on word-choice errors. These are marked in the answer key with an asterisk.) Directions: Decide which of the four underlined words or phrases-(A), (B), (C), or (D)-would not be considered correct, and write the letter of the expression in the blank. Then, in the line at the end of the sentence, write the correction for the underlined phrase. The first one is done as an example. ~ 1. When a spacecraft is operating beyond the atmosphere, its fins and wings not longer --A-- B serve to stabilize it. C _...1n.u.oL.lJ..<{o,,-,n':9"@eWr_ _ D 2. The University of Chicago is unlike most other US. universities in that it has emphasized --A- B graduate student programs so much as undergraduate programs ever since it opened. C D 3. The mass production of paper bags cut costs so much that a bag soon became a routine A B C part of near every purchase. ______ ]) 4. A person must be at least thirty years age in order to serve as a US. senator. A --B- c: D 5. No other state receives as few rainfall as the state of Nevada. _ _ _ _ __ A -B----cD 6. Because of refraction, the water in a tank ever looks as deep as it actually is. A B C D 7. Molds grow on bread, fruit, paper and much other substances. _ _ _ _ __ -A- -B---C-_- D 8. The lei, which is made of flowers, shells, and other materials, is presented to visitors A -B- as a symbolize of Hawaiian hospitality. _ _ _ _ __ C D 9. The botanists Katherine Hunter and Emily Fose spent many difficult months making A -B- research in the Rocky Mountains. _ _ _ _ __ D C Section 2 • Guide to Structure and Written Expression 201 10. Early explorers in Utah named the cliffs they encountered "reefs" because they thought A B C these cliffs looked alike coral formations. I) 11. Today oysters are grown and harvested much like any another crop. ______ -A- -B- ----c D 12. Walter Hunt invented an enormous amount of devices, including the safety pin and a A B machine for making nails. _ _ _ _ __ C D 13. Connecticut, like the other New England states, are dotted with many little lakes. -:\ -BC -D- 14. The soonest parachutes were made of canvas, but later, silk and then nylon were used. A B -C- 15. When vigorous exercise, muscles require a much greater amount of oxygen than A B C when they are at rest. D 16. One should never throw water on an alive electrical fire. -B- C D D 202 Section 2 • Guide to Structure and Written Expression LESSON 34 ERRORS WITH VERBS Whenever the verb is underlined in a Written Expression problem, you should check for the common verb errors outlined in this lesson. A) Errors in Subject-Verb Agreement If a subject is singular, the verb must be singular. If the subject is plural, the verb must be plural. Most problems involving subject-verb agreement on TOEFL are simple, but a few are tricky. T Sample Items Minerals in seawater exists in the same proportions in A -BC all of the oceans of the world. -D- The plural subject minerals requires a plural verb, exist. You might have found this question tricky because the singular noun seawater comes between the subject and the verb, and you may have mistaken that word for the true subject. Bowling, one of the most popular indoor sports, are popular all over A B C the United States and in other countries. D The subject of the sentence is bowling, not sports. The singular verb form is should therefore be used. There are some special rules about subject-verb agreement that you should be familiar with: • A sentence with two subjects joined by and takes a plural verb. The chemistry lab and the physics lab are . .. • Some words end in -s but are singular in form. Many of these words are the names of fields of study (economics, physics, and so on). News is another word of this kind. Economics is . . . The news was . . . • Irregular plurals (children, feet, mice, and so on) do not end in -s but take plural verbs. The women were . .. His feet are . .. • When a clause begins with the expletive there, the verb may be singular or plural, depending on the thematic subject. There was a loud noise ... There were a few problems ... Section 2 • Guide to Structure and Written Expression 203 • Subjects with each and every take singular verbs. (This includes compound words like everyone and everything.) Each state has . .. Each of the representatives was . .. Every person was . . . Everyone wants . . . • The verb in relative clauses depends on the noun that the relative pronoun refers to. The house that was built ... The students who were selected ... • The phrase the number of + plural noun takes a singular verb. The phrase a number of + plural noun takes a plural verb. The number of trees is . .. A number of important matters have . .. • Singular subjects used with phrases such as along with, accompanied by, together with, as well as, and in addition to take singular verbs. The mayor, along with the city council, is . .. Together with his friends, Mark has . .. • Quantities of time, money, distance, and so on usually take a singular verb. Five hundred dollars was . .. Two years has . . . Ten miles is . . . 8) Errors Involving Tense Most tense errors involve the simple present tense, the simple past tense, and the present perfect tense. • The simple present tense is a general-time tense. It usually indicates that a condition is always true or that an action always occurs. It may also indicate that an action regularly occurs. The atmosphere surrounds the earth. Dana often stays at this hotel. Generally, the lectures in this class are very interesting. • The simple past tense indicates that an action took place at a specific time in the past. They moved to Phoenix five years ago. This house was built in the 1920's. Dinosaurs lived millions of years ago. • The present perfect tense usually indicates that an action began at some time in the past and continues to the present. It may also indicate that an action took place at an unspecified time in the past. Mr. Graham has worked for this company since 1990. She hasn't been to a doctor for a year. Jennifer has recently returned from Europe. 204 Section 2 • Guide to Structure and Written Expression ~ Sample Items The most important period of physical growth in humans occurred A B C during their first two years. -0- Choice (C) is best. The simple present tense, not the past tense, should be used because the situation described in this sentence always occurs. Personal taxes for Americans rose sharply since 1945. A If -C- 0 Option (C) is again best. The time phrase since 1945 means from 1945 until now. Therefore, the present perfect (have risen) is required in place of the past tense. ... C) Incorrect Verb Forms Some of the verb errors are errors in form. An -ing form may be used in place of a part participle, a simple form in place of an -ing form, an infinitive in place of a simple form, and so on. Some involve irregular verbs that have different forms for the past tense and the past participle-took and taken, for example. The following information may help you choose the correct form of the main verb. • The simple form follows all modal auxiliaries. might be must know can remember could go should study may follow Certain similar auxiliary verbs require infinitives. ought to attend used to play have to hurry • The past participle is used after a form of have in all perfect forms of the verb. has done have run had called will have read should have said could have made • The -ing form is used after a form of be in all progressive forms of the verb. is sleeping was working has been writing had been painting should have been wearing will be waiting • The past participle is used after a form of be in all passive forms of the verb. is worn is being considered were told would have been lost might have been cancelled has been shown had been promised will have been missed Verb-form problems may also involve auxiliary verbs: has may be used in place of did, is in place of does, and so on. Section 2 • Guide to Structure and Written Expression ... 205 Sample Items The first bicycle race on record in the United States A B taken place in 1883. C 0 The correct verb is the past tense form (took), not a past participle. The Michigan Dunes, located on Lake Michigan's eastern A B shore, may to reach a height of 200 feet. e 0 After a modal auxiliary, the simple form of the verb (reach) should be used in place of the full infinitive (to reach). Dextrose does not taste as sweet as table sugar is . -A--B- e 0 The correct auxiliary verb in this sentence is does, not is. The auxiliary does replaces the present tense verb tastes. ... Exercise 34.1 Focus: Choosing correct verb forms to achieve subject-verb agreement. Directions: Underline the form that correctly completes each sentence. The first one is done as an example. 1. Ethics @fare) the study of moral duties, principles, and values. 2. The first bridge to be built with electric lights (was/were) the Brooklyn Bridge. 3. There (is/are) two types of calculus, differential and integral. 4. George Gershwin, together with his brother Ira, (was/were) the creator of the first musical comedy to win a Pulitzer Prize. 5. In a chess game, the player with the white pieces always (moves/move) first. 6. The Earth and Pluto (is/are) the only two planets believed to have a single moon. 7. A number of special conditions (is/are) necessary for the formation of a geyser. 8. Each of the Ice Ages (was/were) more than a million years long. 9. The battery, along with the alternator and starter, (makes/make) up the electrical system of a car. 10. Teeth (is/are) covered with a hard substance called enamel. 11. The more-or-less rhythmic succession of economic booms and busts (is/are) referred to as the business cycle. 12. The number of protons in the nucleus of an atom (varies/vary) from element to element. 13. All trees, except for the tree fern, (is/are) seed-bearing plants. 206 Section 2 • Guide to Structure and Written Expression 14. Fifteen hundred dollars a year (was/were) the per capita income in the United States in 1950. 15. Everyone who (goes/go) into the woods should recognize common poisonous plants such as poison ivy and poison oak. Exercise 34.2 Focus: Recognizing and correcting errors in verb tense and form. Directions: If the underlined form is correct, mark the sentence C If the underlined form is incorrect, mark the sentence X and write a correction for the underlined form in the blank at the end of the sentence. The first one is done as an example. ~ 1. Coal, grain, steel, and other products are often shipping by barge on inland waterways. ), Bh;nned 2. The first cotton mill in Massachusetts has built in the town of Beverly in 1787. 3. Physician Alice Hamilton is known for her research on industrial diseases. ______ 4. When scientists search a site for fossils, they begin by examining places where the soil has wore away from the rock. _ _ _ _ __ 5. The popularity of recreational vehicles has been grown over the last few decades. 6. Experts have estimated that termites cause as much property damage every year as fire has. _ _ _ _ __ --- 7. In music, a chord is the sound of two or more notes that are playing together. 8. The white pine ~ the most commercially important forest tree in North America until the beginning of the twentieth century. _ _ _ _ __ 9. In 1846 the Swiss naturalist LouisAgassiz come to the United States to give a series of lectures. ______ 10. Parrots and crows are considered the most intelligent birds. _ _ _ _ __ 11. Portable fire extinguishers generally containing liquid carbon dioxide. _ _ _ _ __ 12. The first experimental telegraph line in the United States run from Baltimore to Washington, a distance of forty miles. _ _ _ _ __ 13. The first seven American astronauts were chose in 1959. _ _ _ _ __ 14. Since ancient times, farmers used scarecrows to protect their crops from hungry birds. 15. In the late nineteenth century, many important theories in both the biological and the physical sciences have been produced. _ _ _ _ __ Exercise 34.3 Focus: Identifying and correcting errors involving verb forms. (Note: One or two items in this exercise do not focus on word-form errors. These are marked in the answer key with an asterisk.) Directions: Decide which of the four underlined words or phrases-(A), (B), (C), or (D)-would not be considered correct, and write the letter of the expression in the blank. Then, in the line at the end of the sentence, write a correction for the underlined phrase. The first one is done as an example. 207 Section 2 • Guide to Structure and Written Expression 1. Medical students must to study both the theory and practice of medicine. A stl/dy -B- D --C- 2. The seal, like the sea lion and the walrus, is a descendant of ancestors that once live --::\ B C ----0ontheland. ___________ 3. The top layer of the ocean stores as much heat as does gases in the atmosphere. ----r> A B C 4. Every one of the body's billions of cells require a constant supply of food and oxygen. A B C 0 5. In science, the results of an experiment are not generally accepted until they had been A B 4C duplicated in other laboratories. ________ o 6. In colonial times, flax and wool required months of preparation before they could be A B -cdyed and spin into cloth. _ _ _ _ __ I> 7. Although some people find bats terrifying, they are actually beneficial because they ~ B C ate harmful insects. _ _ _ _ __ D 8. Each of the four types of human tooth are suited for a specific purpose. _ _ _ _ __ -B- -A- C 0 9. Mathematicians taken centuries to develop the methods that now are used in A B C arithmetic. _ _ _ _ __ o ___ 10. Electric milking machines have made dairy farming a much easier job than it once did. A B C \) 11. Playwright Frank Shin has often describes the lives of Chinese Americans in his dramas. A B -C- 0 208 Section 2 • Guide to Structure and Written Expression 12. Cans of paint must be shaking to mix the pigments with the medium in which they are A ~ C suspended. ____________ o ___ 13. Beavers continuously repair the dams they have build. ____________ A -B- -C- --0- 14. The emphasize on team sports has become even stronger in this century than it was A B C 0 in the past. __________ ___ 15. Sheep are often dip in liquid chemicals to eliminate ticks and other external parasites. A B C -0- Section 2 • Guide to Structure and Written Expression 209 LESSON 35 ERRORS WITH PARALLEL STRUCTURES Written Expression items involving errors with parallel structures are similar to those in the Structure part of the test (Lesson 24). These sentences most often contain a series of three expressions: X, Y, and Z. One of these expressions is not grammatically parallel to the other two items in the series. Structures that are often involved in parallelism are nouns, adjectives, verbs, prepositional phr--ases, gerunds, and infmitives. Some problems with parallelism are actually word-form problems similar to those in Lesson 32. T Sample Item As a young man, George Washington liked boating, to hunt, ---,:- B C and fishing. --0- Option (C) is not parallel with the other items in the series: to hunt is an infinitive, while the other items are gerunds. You may have considered the other options that are part of the series, (B) and (D), but if you rewrote only one of these, the three expressions would still not be parallel. In general, errors involving parallelism are easy to identify. Exercise 35.1 Focus: Identifying and correcting errors involving parallelism. Directions: If the underlined form is parallel to other forms in the sentence, mark the sentence C. If the underlined form is not parallel, mark the sentence X and write a correction for the underlined form in the blank at the end of the sentence. The first one is done as an example. ~ 1. Because of their hardness, industrial diamonds can be used for cutting, grind, and drilling. @rindin@ 2. Sacramento is the commercial, industry, and financial center of California's Central Valley, as well as being the state capital. ______ 3. Philosophers are concerned with questions about nature, human behavior, society, and reality. _ _ _ _ __ 4. When taking part in winter sports, one should wear clothing that is lightweight, warmth, and suitable for the activity. ______ 5. Folklore consists of the beliefs, customs, traditions, and telling stories that people pass from generation to generation. ______ 6. Major sources of noise pollution include automobiles and other vehicles, industrial plants, and heavy construction equipment. ______ 7. Steel is alloyed with manganese to increase its strength, hardness, and resistance to wear. _ _ _ _ __ 8. Scholar John Fiske wrote on history, religious, and social issues. ____- - 210 Section 2 • Guide to Structure and Written Expression 9. Electricity is used to light, hot, and cool buildings. _ _ _ _ __ 10. T. S. Eliott was equally distinguished as a poet, he wrote criticism, and a dramatist. 11. Jute is a glossy fiber that is strong, does not easily stretch, and inexpensive. 12. Wetlands were once considered useless areas, but they have been found to purify water, nurture wildlife, and flood control. _ _ _ _ __ Exercise 35.2 Focus: Identifying and correcting errors with parallel structures. (Note: One or two items in this exercise do not focus on errors involving parallel structures. These are marked in the answer key with an asterisk.) Directions: Decide which of the four underlined words or phrases-(A), (B), (C), or (D)-would not be considered correct, and write the letter of that expression in the blank. Then, in the line at the end of the sentence, write a correction for the underlined phrase. The first one is done as an example. ~ 1. Computers are often used to control, adjustment, and correct complex industrial A B C operations. 0 _----"a:ucy.y.iu;I!f2:2.Jt"----_ 2. The bellflower ~ a wildflower that grows in shady fields, in marshes, and mountain A B C 0 slopes. _ _ _ _ __ 3. Eggs may be boiled in the shell, scrambled, fried, and cooked in countless another A B C D ways. _ _ _ _ __ 4. Many places of history, scientific, cultural, or scenic importance have been designated A --c- B national monuments. _ _ _ _ __ o 5. R. Buckminster Fuller was a design, an architect, an inventor, and an engineer. 0 A B C 6. Modern motorcycles are lighter, faster, and specialized than motorcycles of 25 years A --B- C 0 ago. _ _ _ _ __ 7. Many people who live near the ocean depend on it as a source of food, recreation, and A B C to have economic opportunities. _ _ _ _ __ o Section 2 • Guide to Structure and Written Expression 211 8. Large commercial fIshing vessels are equipped to clean, packaging, and freeze the fish A B C that they catch at sea. _ _ _ _ __ D 9. As a breed, golden retrievers are intelligent, loyally, and friendly dogs. _ _ _ _ __ A B C D 10. Mathematics can be considered a language, an art, a science, a tool, or playing a game. ABC [) 11. Paper may contain vegetable, minerals, or man-made fIbers. _____ A B C [) 12. According to Susan Sontag, our concepts of art, beauty, and nature has been changed A B --C-O by photography. ___ 13. The economist Kenneth Boulding proposed a single social science that would unify A B economic, sociology, and political science. _ _ _ _ __ C D 14. The teeth front are llsed to bite food, the canines to tear it, and the molars to grind it. ABC ___ 15. An ant's antennae provide it with a sense of hear, smell, touch, and taste. A B -c: D [) 212 Section 2 • Guide to Structure and Written Expression LESSON 36 ERRORS~THPRONOUNS Pronoun errors in Written Expression involve several types of pronouns: • Personal pronouns (he, she, it, they, and so on) • Reflexive pronouns (himself, herselj; itself, themselves, and so on) • Relative pronouns (adjective clause markers) (who, whose, which, that, and so on) • Demonstrative pronouns (this, that, these, those) For the purposes of this lesson, possessive adjectives (his house, their bicycles) are considered personal pronouns and demonstrative adjectives (that book, those horses) are considered demonstrative pronouns. The greatest number of errors involve personal pronouns. A) Errors in Pronoun/Noun Agreement A pronoun must agree with the noun to which it refers (the pronoun's referent). Most agreement errors with personal pronouns, reflexive pronouns, and demonstrative pronouns consist of a singular pronoun referring to a plural noun or a plural pronoun referring to a singular noun. Agreement errors with relative pronouns usually involve the use of who to refer to things or which to refer to persons. (Note: The relative pronoun that can be used in certain sentences to refer to both persons and things.) Another error involves the use of this or these in place of that and those. (This and these are used to refer to things that are perceived as close in time or space; that and those are used to refer to things that are perceived as distant in time or space.) T Sample Items Jackrahhits have powerful rear legs that enable it to leap long distances. -1\H C J) The pronoun referring to the plural noun jackrabbits must be plural. The best way for children to learn science is to perform experiments A B C himself. [) The referent is plural (children), so the reflexive pronoun must also be plural (themselves) to agree with it. Therefore, the best answer is (D). Section 2 • Guide to Structure and Written Expression 213 The Canadian Shield is a huge, rocky region who curves around Hudson B A C Bay like a giant horseshoe. D The referent for the pronoun who is region. To agree with the referent, the relative pronoun which or that must be used. The pronoun lrho can refer only to a person. Trademarks enable a company to distinguish its products from these of --AIf -c-,another company. \) The demonstrative these cannot be used to refer to the products of another company. The demonstrative those should be used instead. ... B) Errors in Pronoun Form These errors involve personal pronouns. A subject form like he might be used in place of an object form like him, or a possessive pronoun like hers might be used in place of a possessive adjective like her. This type of pronoun error is usually easy to spot. T Sample Item Herman Melville gathered material for A hi~ novels, including Mob)! Dick, B during his years at sea. C 0 The possessive form his. not the object form him. is required ... C) Incorrect Type of Pronoun In some sentences, the wrong type of pronoun is used. For example, a reflexive pronoun might be used when a personal pronoun is needed, or a personal pronoun used when a relative pronoun is required. T Sample Items As larvae. barnacles are free-swimming, but as adults they attach them A 8 C to stones, docks, and hulls of ships. D The reflexive pronoun is required because the subject and object are the same entity: they attach themselves. 214 Section 2 • Guide to Structure and Written Expression A barometer is a device it is used to measure atmospheric pressure. A -nC 0 A personal pronoun (it) cannot be used to connect an adjective clause to the rest of the sentence. A relative pronoun (which or that) must be used instead. D) Incorrect Inclusion of Pronouns Some errors involve the unnecessary use of pronouns. Often, this type of error occurs when a personal pronoun is used as a subject in a sentence that already has a noun subject. It may also involve a personal pronoun used unnecessarily in a relative clause. In a few items, a relative pronoun is used unnecessarily. ... Sample Items Block Island in Long Island Sound it is surrounded by cold, A -n- c dangerous waters. o The subject of the sentence is Block Island; the personal pronoun it is an unnecessary repetition of the subject. Dutch elm disease, which it is caused by a fungus, can destroy a A B C tree within four weeks. --0- The relative pronoun which is the true subject of the relative clause; the personal pronoun it is not needed. Certain types of turtles that may live as long as 100 years. A B C -0- The relative pronoun that is unnecessary in this sentence because there is only one verb (may live). A sentence that contains a relative clause must have a verb in each clause. Exercise 36.1 Focus: Identifying and correcting errors of pronoun agreement. Directions: If the underlined form is correct, mark the sentence C. If the underlined form is incorrect, mark the sentence X and write a correction for the underlined form in the blank at the end of the sentence. The first one is done as an example. Section 2 • Guide to Structure and Written Expression ~ 1. Unlike other cats, the cheetah cannot fully retract their claws. 215 its 2. One cannot see through translucent materials, but light can pass through it. 3. Investment banking is concerned with the sale of government bonds, and he also deals with corporate stocks and bonds. _ _ _ _ __ 4. The oldest known forms of fossils are those of microscopic plants similar to algae. 5. Gene Krupa had one of the few big bands who was centered around a drummer. 6. Emeralds get its beautiful green color from titanium and chromium impurities in the stone. _ _ _ _ __ 7. The viola is larger and heavier than the violin, and she has a darker, somewhat nasal tone. _ _ _ _ __ 8. The Ringling Brothers were five brothers which built a small group of performers into the world's largest circus. _ _ _ _ __ 9. 10. 11. 12. 13. 14. Storms on the planet Saturn may be larger than the planet Earth itself. _ _ _ _ __ The molecules of a liquid are held together tighter than that of a gas. _ _ _ _ __ Ducks make nests out of leaves and its own feathers. _ _ _ _ __ The clipper ship Flying Cloud was one of the fastest ships of their kind. _ _ _ _ __ There are thousands of kinds of bacteria, many of whom are beneficial. _ _ _ _ __ When babies reach the age of one, her growth begins to slow down. _ _ _ _ __ 15. The arrangement of keys on the keyboard of a personal computer is almost the same as those on a standard typewriter. _ _ _ _ __ Exercise 36.2 Focus: Identifying and correcting errors involving incorrect types and forms of pronouns. Directions: If the underlined form is correct, mark the sentence C. If the underlined form is incorrect, mark the sentence X and write a correction for the underlined form in the blank at the end of the sentence. The first one is done as an example. -----X---- 1. Artist Margaret LengTan combined dance and piano playing in hers performances. her 2. Over the years, the intensive breeding of domestic rabbits has given their softer, finer fur than wild rabbits. _ _ _ _ __ 3. New England poet EdwinA. Robinson moved to New York City in 1896 and devoted himself to his writing. _ _ _ _ __ 4. Yellow journalism was a form of news reporting it emphasized the spectacular aspects of the news. _ _ _ _ __ 5. There are between 100 and 400 billion stars in ours galaxy, the MilkyWay. 6. The atoms of a crystal always arrange them into a specific array, called a lattice. ___ 7. Fred Astaire and Gene Kelly were basically tap dancers, but their both added some ballet movements to their dance steps. _ _ _ _ __ 216 Section 2 • Guide to Structure and Written Expression 8. The Pritzken Prize is given every year to architects their work benefits humanity and the environment. ___ 9. Charleston, South Carolina, has preserved to a remarkable degree it historic houses and famous gardens. _ _ _ _ __ 10. Ice fishermen sometimes build small, movahle huts to protect them from the cold winds. Exercise 36.3 Focus: Identifying errors involving the incorrect inclusion of pronouns. Directions: If the sentence contains a pronoun that is incorrectly included. mark that sentence X and underline the pronoun. If the sentence does not contain an incorrect pronoun inclusion. mark that sentence C. The first one is done as an example. ~ 1. The first great public library in the United States it was founded in Boston in the 1830's. 2. Floods which cause billions of dollars worth of property damage in the United States annually. 3. As a class, percussion instruments such as drums that arc the simplest in construction of any musical instruments. 4. Richard G. Hatcher of Gary, Indiana, he was one of the first black mayors of a sizahle American city. 5. Active stocks are stocks which are frequently bought and sold. 6. There are many species of plants and animals that they are peculiar to Hawaii. 7. Pipettes are glass tubes, open at both ends, which chemists use them to transfer small volumes of liquid. 8. When molten basalt cools, it forms six-sided columns. 9. Elizabeth Peabody, founder of the first American kindergarten, she helped gain acceptance of that institution as a regular part of public education. ___ 10. Today meteorologists obtain the information which they use to make weather predictions chiefly from satellites. 11. Cells often obtain water through which the process of osmosis. 12. The lighting of large outdoor arenas it first became feasible in the 1930's. Exercise 36.4 Focus: Identifying and correcting errors involving pronouns. (Note: One or two items in this exercise do not focus on pronoun errors. These are marked in the answer key with an asterisk.) Directions: Decide which of the four underlined words or phrases-(A), (13), (C), or (D)-would not be considered correct, and write the letter of the expression in the blank. Then, in the line at the end of the sentence, write a correction for the underlined phrase. The tlrst one is done as an example. Section 2 • Guide to Structure and Written Expression ------L- 217 1. A caricature is a picture in which the subject's distinctive features they are deliberately A B C exaggerated. are o 2. A beaver uses its strong front teeth to cut down trees and peel off its bark. A B C ___ 0 3. Ants are blind to red light, so it is possible to observe themselves in an artificial nest -:A B without disturbing their activities. ______ C 0 4. An auger is a tool which a carpenter uses it to bore holes in wood. _ _ _ _ __ --B- -A- C 0 5. The glaciers in Olympia National Park are unusually because they are found at altitudes A -B- lower than those at which glaciers are usually found. _ _ _ _ __ -c- ___ --0- 6. In his novels, Sinclair Lewis drew critical portraits of Americans who thought of them A -B-----c: -0as model citizens. _ _ _ _ __ ___ 7. Jaguars which resemble leopards, but they are larger and are marked with rosettes A --B- C rather than spots. _ _ _ _ __ o 8. Most bacteria have strong cell walls much like that of plants. _ _ _ _ __ ~ ___ -B- """""C"" ] ) 9. Bees collect pollen, which furnishes protein for its diet. _ _ _ _ __ --A--BC D 10. A small business often limits their operations to a single neighborhood or a group of -A- -B- C neighboring communities. _ _ _ _ __ o 11. Louisa May Alcott, she is best known for her books for children, served as a nurse ---:A B C during the Civil War. _ _ _ _ __ o 12. The principles used in air-conditioning are basically the same as those used by the A B C human body to cool himself. ______ o 218 Section 2 • Guide to Structure and Written Expression ___ 13. In that age of computers, it is difficult to imagine how tedious the work of accountants --;:13 C and clerks must have been in the past. _ _ _ _ __ o 14. In general, the only kind of cells that cannot replace itself are nerve cells. A -B- --C- D ___ 15. The naturalist Edwin Teal illustration his books with photographs he had taken himself. A B C D Section 2 • Guide to Structure and Written Expression 219 LESSON 37 ERRORS WITH SINGUlAR AND PLURAL NOUNS A) Plural Nouns in Place of Singular Nouns and Singular Nouns in Place of Plural Nouns Underlined nouns in the Written Expression section may be incorrect because they are plural but should be singular, or because they are singular but should be plural. Sometimes it is clear that a singular subject is incorrectly used because the verb is plural, or that a plural noun is used incorrectly because the verb is singular. In this type of item, the verb will not be underlined, because this is not a verb error. Sometimes it is obvious that a plural or a singular noun is needed because of the determiners that precede the noun. Certain determiners are used only bef()fe singular nouns while other determiners are used only before plural nouns. , , .. "",--,-,-, rs Used with SingtjiJ@rNouns ers Used with a/an two, three, four, etc. one dozens of a single hundreds of each thousands of every a few (of) this many (of) that a number of the number of a couple (of) everyone of each one of each of one of these those Each contestant won a prize. Each of the contestants won a prize. Thisjlower is a yellow rose. Thesejlowers are yellow roses. I attended only one game this season. It was one of the most exciting games that I ever attended. 220 Section 2 • Guide to Structure and Written Expression T Sample Items Several of Washington Irving's story have become classics in A -BC American literature. D In this item, both the determiner before the noun (Several oj) and the plural verb (have) indicate that a plural noun (stories) should be used. Mauna Loa, an active volcano on the island of Hawaii, usually has A 13 -c- one eruptions every three years. D A singular noun must be used after the determiner one. One of the most beautiful state capitol is the Utah State Capitol, A B C located in Salt Lake City. D The correct pattern is one of the + superlative adjective + plural noun. The plural noun capitols must therefore be used. A B) Errors Involving Irregular Plurals Most plural nouns in English end in -s, but a few are irregular. Only the most common irregular plurals are tested onTOEFL. (Irregular plurals that come to English from Latin or Greek-data, cacti, alumnae, or phenomena, for example-will NOT be tested on TOEFL.) child children man men woman women foot feet tooth teeth mouse mice fish fish Section 2 • Guide to Structure and Written Expression T 221 Sample Item As childs grow older, their hones become thicker and longer. --A --c--n~I-l- The correct plural form of cbild is children. • C) Errors with Plural Forms of Non-Count Nouns In some items a non-count noun (such asfurniture, research, sunshine, information, or bread) is incorrectly given as a plural noun. T Sample Item Some encyclopedias deal with specific fields, such as music or A B C philosophy, and provide informations on only those subjects. D in(i:)rmatioll is an uncountable noun and cannot be pluralized. • D) Errors with Plural Compound Nouns Compound nouns consist of two nouns used together to express a single idea: grocerJ! store, travel agent, dinner party, and house cat, for example. Only the second noun of compounds is pluralized: grocery stores, travel agents, dinner parties, and house cats. (There are rare exceptions to this rule-sports cars and women doctors, for example-but these won't be tested.) T Sample Item Raymond Chandler's detectives stories are admired by both critics A B and general readers. c [) The correct plural form ofthis compound noun is detective stories. • E) Errors Involving Plural Forms of Numbers and Measurement Some errors involve numbers + measurements: They went for a six-mile walk. They walked six miles. In the first sentence, the number + measurement is used as an adjective, and the measurement is singular. In the second, the measurement is a noun and is therefore plural. 222 Section 2 • Guide to Structure and Written Expression Numbers like hundred, thousand, and million may be pluralized when they are used indefinitely-in other words, when they do not follow other numbers. thousands millions of dollars seven thousand five million dollars T Sample Items The U.S. president serves a maximum of two four-years terms. A --B- C D When used before a noun, a number + measurement is singular. Thousand of antibiotics have been developed, but only about A --c- B thirty are in common use today. D The plural form Thousands should be used. Exercise 37 Focus: Identifying and correcting errors involving singular and plural nouns. (Note: One or two items in this exercise do not focus on singular-plural errors. These are marked in the answer key with an asterisk.) Directions: Decide which of the four underlined words or phrases-(A), (B), (C), or (D)-would not be considered correct, and write the letter of the expression in the blank. Then, in the line at the end of the sentence, write a correction for the underlined phrase. The first one is done as an example. ~ 1. The male mandril baboon is one of the most colorful of all mammal. _--"-mu..a""m.I.1LLm""au.::/s2....-_ A ~ o C 2. Zoonoses are diseases that can be transmitted to humans beings by animals. A B C ___ 0 3. Many championship automobiles and motorcycle races take place in Daytona Beach, -A- B C -0- Florida. ______ ___ 4. The Newberry Award is granted every years to the authors of outstanding books for -AB --c-children. _ _ _ _ __ D ___ 5. The major source of air pollution vary from city to city. _ _ _ _ __ ----:;\ --8- C ----n Section 2 • Guide to Structure and Written Expression 223 6. Around seventy-five percents of the Earth's surface is covered by water. _ _ _ _ __ A B C ___ D 7. All college and universities get their funds from a variety of sources. ______ A --C- -B- D 8. Russell Cave in northeastern Alabama was the home of cliff-dwelling Indians thousand of A -B- C years ago. ______ D ___ 9. In 1792 a corporation constructed a sixty-miles toll road from Philadelphia to Lancaster, A B C -D- Pennsylvania. ______ 10. The mathematician and astronomer David Rittenhouse was one of the first man of -C- A science in the American colonies. ______ D 11. Publishers of modern encyclopedias employ hundreds of specialists and large editorials A B C D staffs. _ _ _ _ __ 12. The electric toaster was one of the earliest appliance to be developed for the kitchen. ---::\ --B- C D ___ 13. Tornadoes can pick up objects as heavy as automobiles and carry them for hundreds of A B C foot. _ _ _ _ __ ]) 14. Many kinds of vegetables are growth in California's Imperial Valley. _ _ _ _ __ -A- BCD ___ 15. In typical pioneers settlements, men, women, and children worked from morning until A B C night at farm and household tasks. _ _ _ _ __ I> 16. Some engineers have predicted that, within twenty years, automobiles will be make A B C almost completely of plastic. _ _ _ _ __ D ___ 17. The pine tree is probably the more important lumber tree in the world. _ _ _ _ __ ~ -B- C]) 224 Section 2 • Guide to Structure and Written Expression 18. Mary Lyon founded Mount Holyoke College, the fIrst permanent institution of higher A B learning for woman in the United States. _ _ _ _ __ C 0 ___ 19. Adult humans have more than a trillions cells in their bodies. _ _ _ _ __ A B -C- 0 20. Phytoplankton is found only in the upper layers of of the ocean, where sunlights can -A- reach. _ _ _ _ __ -B- -c- 0 Section 2 • Guide to Structure and Written Expression 225 MINI-TEST 6: WRITTEN EXPRESSION Directions: The sentences below have four underlined words or phrases, (A), (B), (C), and (D). Identify the one underlined expression that must be changed for the sentence to be correct. Then find the number of the question on your answer sheet and fill in the space corresponding to the letter. Time: 12 minutes 1. Bricks can be made from many difference types of clay. --A-- --B-- -uo C 2. Despite most mushrooms are edible, some species cause serious poisoning. A _____ ----c- B D 3. Judges in dog shows rate dogs on such points as their colorful, posture, shape, and size. A B C D 4. The medicine of prehistoric people probably consisted of a mixture of scientific A B practices, superstitions, and religious believes. C D 5. The game backgammon has been playing for thousands of years. A B C D 6. One of the greatest of An~erican automobile designer was Harley Earl. A _____ C l:I 7. Before the late eighteenth century, most textiles were done at home. --A-- ___ ---0- B -c-,- D 8. Political science. alike the other social sciences, is not an exact science. --A- --B-- C D 226 Section 2 • Guide to Structure and Written Expression ___ 9. About 8,000 years ago, people began using animals to carry themselves and their A B C 0 belongings. 10. Storks constantly rearrange their nests to keep their eggs safety, dry, and warm. A B C D 11. In its purely state, hydrochloric acid is a gas, but!! is almost always used as a solution BCD --A- in water. ___ 12. Animals that hibernate usually eat large numbers of food in the autumn. 0 A B C ___ 13. Many folk songs have been written about railroads and railroads workers. -A- -B- C 0 14. Some plants and insects exhibit so high degree of interdependence that the elimination A B of one results in the elimination of the other. --Co ___ 15. Lightly, sandy soil absorbs water more quickly than clay or loam. --A- B C D 16. Fannie Farmer, an educator and cooking expert, she wrote the first distinctively 0 A B C American cookbook. ___ 17. The rhesus monkey has been widely used in biological, psychological, and medicine T--BC 0 research. 18. During the Depression of the 1930's, many artists were giving jobs by the Federal Arts A c-----O- -B- Project. ___ 19. Crocodiles are sometimes confused with alligators, but are different from they in A B C a number of ways. o ___ 20. As a concert violinist, conductor, and he composed both serious and popular music, A B Leonard Bernstein achieved a series of remarkable successes. o --c-- Section 2 • Guide to Structure and Written Expression 227 21. It is a chemical called capsaicin that gives hot peppers their spice flavor. A B -C- --n___ 22. Flying snakes can launch itself from the top of one tree and glide to another. A -BC n ___ 23. A basic knowledge of social studies such as history and geography are considered an -A- B <= important part of the education of every child. -n- 24. The black walnut tree is grown principally for its lumber, which is used for cabinets and A B C furnitures. n ___ 25. Plymouth was the soonest of the five colonies established by the Pilgrims in A B C D Massachusetts. 228 Section 2 • Guide to Structure and Written Expression LESSON 38 ERRORS WITH VERBALS Verbals are participles, gerunds, infinitives, and-for the purpose of this lesson-simple forms of the verb. Participles are verbal adjectives. In this part of the test, participles are often seen before nouns as one-word adjectives. Present participles end with -ing. When used before a noun, present participles have an active meaning. Past participles of regular verbs end in oed; the past participles of many common verbs are irregular. Before nouns, past participles have a passive meaning. It was an exhausting ten-kilometer race. (present participle) The exhausted runners were too tired to move after the race. (past participle) In the first sentence, the race exhausts the runners. The race "performs" the action. In the second sentence, the runners are exhausted by the race. They receive the action. Participles are also used in phrases after nouns as reduced (shortened) relative clauses. Again, present participles imply an active idea, past participles a passive one. The man stealing the money was arrested. (present participle; means "who stole") The money stolen from the bank was recovered. (past participle; means "which was stolen") Gerunds are verbal nouns. Like present participles, gerunds end in -ing. They can be the subjects of verbs, the objects of prepositions, and the objects of certain verbs. (See the chart on the following page.) Dancing is good exercise. (gerund as subject) You can solve this problem by using a calculator. (gerund as object of a preposition) He enjoys going to good restaurants. (gerund as object of a verb) All two- and three-word verb phrases that can be followed by verbals are used with gerunds, not infinitives. This is true even when the verb phrase ends with the word to. (This can be tricky because infinitives always begin with the word to.) I'm looking forward to going to New Orleans. John is opposed to our participating. Infinitives consist of the word to and the simple form of the verb. Like gerunds, infinitives can be the subjects of verbs and the objects of certain verbs (see chart). Unlike gerunds, infinitives can never be the objects of prepositions. To help others is rewarding. (infinitive as subject) He attempted to swim across the river. (infinitive as object of a verb) Infinitives are used in several other ways: It's important to change the oil in your car frequently. (infinitive after an adjective) The first man to land on the moon was Neil Armstrong. (infinitive used as an adjective after a noun) She must take this class to graduate. (infinitive used to show purpose) Section 2 • Guide to Structure and Written Expression 229 Simple forms are the base forms of verbs; they consist of the infinitive without the word to. Simple forms are used after the causative verbs have, make, and let; He had the carpenter repair the door. His father makes him study hard. She let her son go on the trip. Verblr8ed with . ·Gerunds admit agree have avoid allow let deny arrange make enjoy attempt finish cause justify choose quit decide recommend enable suggest hope understand instruct know (how) learn (how) permit persuade require seem teach (how) tell usc warn Watch for the following errors involving verbals: A) Incorrect Choice of Verbal Any of these verbals-participle, gerund, infinitive, or simple form-may be incorrectly used when another one of them is required. Section 2 • Guide to Structure and Written Expression 229 Simple forms are the base forms of verbs; they consist of the infinitive without the word to. Simple forms are used after the causative verbs have, make, and let: He had the carpenter repair the door. His father makes him study hard. She let her son go on the trip. ·····lf~~i~;edWith .~etptW:$ admit agree have avoid allow let deny arrange make enjoy attempt finish cause justify choose quit decide recommend enable suggest hope understand instruct know (how) learn (how) permit persuade require seem teach (how) tell use warn Watch for the following errors involving verbals: A) Incorrect Choice of Verbal Any of these verbals-participle, gerund, infinitive, or simple form-may be incorrectly used when another one of them is required. 230 Section 2 • Guide to Structure and Written Expression T Sample Items The writer Edgar Allan Poe is usually credited with invent the ---A-- ---c-- B short story . o After a preposition (with), a simple form cannot be used. The correct form is a gerund (inventing). A single-lens reflex camera allows a photographer seeing exactly what A B C the camera will photograph. o After the verb allow, a gerund (seeing) cannot be used. An infinitive (to see) is correct. .. B) Incorrect Choice of Participle You may see past participles used incorrectly for present participles or present participles used incorrectly for past participles. You may also see a main verb used when a participle is required. T Sample Items There are probably around 3,000 languages speaking in the world. A B C 0 A past participle (spoken) is required because the idea is passive. The sentence means, "... 3,000 languages which are spoken . .." For decades, journalist Theodore H. White wrote books described A --B- C American presidential elections. D The past participle described is used incorrectly because the idea is active: the books described the elections. Therefore, a present participle, describing, should be used. (The sentence could also be corrected by adding a relative pronoun: that described.) Nutmeg, widely is used as a spice, is actually the kernel of a ---A-- -B- C tropical nut. o Rather than the main verb is used, a past participle (used) is required. Section 2 • Guide to Structure and Written Expression 231 C) Incorrect Forms of Infinitives Incorrect infinitive forms such as for go or to going may be used in place of the correct form, to go. T Sample Item Viral infections are generally more difficult to treating than bacterial A B C infections. D The correct form of the infinitive is to treat. ... Exercise 38.1 Focus: Identifying incorrect and correct forms of gerunds, infinitives, and simple forms. Directions: Underline the form that best completes each sentence. The first one is done as an example. 1. Sport parachutes are relatively easy (controlling/to control). 2. Sleeve bearings allow pistons (to move/move) back and forth. 3. One of the most important steps in (producing/to produce) a motion picture is film editing. 4. An opera singer is required (having/to have) a powerful and beautiful voice. 5. The Wampanoag Indians taught the Pilgrims how (growing/to grow) corn. 6. Frogs and certain kinds of birds use their tongues (to catch/catch) insects. 7. Isadora Martinez invented a knee implant that lets people with arthritis (to bendlbend) their knees easily. 8. Smells can be more effective than any other sensory stimuli in vividly (bringing!bring) back memories. ·•j ,. · i 1 9. Modems permit computers (communicating/to communicate) with one another over telephone lines. 10. A sudden sound can make a golfer (to miss/miss) a shot. 11. Heavy spring snows may cause the branches of trees (snap/to snap). 12. Modern race cars store fuel in rubber bladders that are almost impossible (rupturing/to rupture). !! 13. New words are constantly being invented (describe/to describe) new objects and concepts. - 14. Dr. Mary Walker, a surgeon in the Union Army during the Civil War, was the first and so far only woman (to be awarded/awarding) the Congressional Medal of Honor. ~ ~ • ~ • OJ · 15. In 1957 Ralph Abernathy founded an organization devoted to (achieve/achieving) racial equality for black Americans. 232 Section 2 • Guide to Structure and Written Expression Exercise 38.2 Focus: Identifying incorrect and correct forms of participles. Directions: Underline the form that best completes each sentence. The first one is done as an example. 1. The largest (knowing/known) insects are found in tropical rain forests. 2. A bill of lading is a (writing/written) receipt for goods that are sent by public transportation. 3. A hummingbird's heart beats at the (astonishing/astonished) rate of 615 beats per minute. 4. At the peak of his jump, a pole vaulter performs a series of (twisting/twisted) body motions to clear the bar. 5. Anyone (working/worked) under conditions that cause a heavy loss of perspiration can suffer heat exhaustion. 6. A mosquito (filled/is filled) with blood is carrying twice its own body weight. 7. The state of Wisconsin has seventy-two counties, many (naming/named) after Indian tribes. 8. Sunspots occur in cycles, with the greatest number generally (appearing/are appearing) every eleven years. 9. A delta is a more-or-less triangular area of sediments (depositing/deposited) at the mouth of a river. lO. William H. Kilpatrick was a philosopher and scholar now generally (regarding/regarded) as the father of progressive education. 11. Checkerboard Mesa in Utah features a strangely (cracked/cracking) expanse of stone. 12. It has been known since the third century that coffee has a (stimulated/stimulating) effect. Exercise 38.3 Focus: Identifying and correcting a variety of errors involving verbals. (Note: One or two items in this exercise do not focus on errors involving verbals. These are marked in the answer key with an asterisk.) Directions: Decide which of the four underlined words or phrases-(A), (B), (C), or (D)-would not be considered correct, and write the letter of that expression in the blank. Then, in the line at the end of the sentence, write a correction for the underlined phrase. The first one is done as an example. ~ 1. The most widely used material for package consumer goods is cardboard. A B C D packa@in@ 2. One of the latest methods of quarrying stone is to cutting the stone with a jet torch. A B C D 3. In 1944 biologist Charles Michener devised a system for to classify the approximately A B C 20,000 species of bees. _ _ _ _ __ D Section 2 • Guide to Structure and Written Expression 233 4. Most candles are made of paraffin wax mix with compounds that have higher melting A B points to keep them from melting in hot weather. ______ C ___ 0 5. Machines used to harvest tree crops such as cherries or almonds can be classified either --A- B C as shakers and as pick-up machines. ______ o 6. Fishing cats, found in Southeast Asia, are distinguished by their webbed feet, which -A- B C enable them catching fish. _ _ _ _ __ o ___ 7. Geothermal energy is energy to obtain by using heat from the Earth's interior. A B -C- 0 8. Bathe in mineral water has long been believed to have beneficial effects. ______ B -A- ___ C --0- 9. It is the facets cut into a diamond that make it to sparkle. ______ A B C 0 lO. Ralph Blakelock specialized in paint wild, lonely nighttime landscapes, usually with A -B- C black trees silhouetted against the moon. ______ o 11. The Farallon Islands are a group of uninhabited islands lying about 40 mile west of San A B --C- 0 Francisco. ______ ___ 12. The crushing leaves of yarrow plants can serve as a traditional medicine for cleansing A B C 0 wounds. ______ ___ 13. RobertA. Moog developed an electronic device that could be used for play synthesized A B C 0 music. ______ 14. Hypnosis is sometimes employed as a means of helping people to quit to smoke. A -B- C 0 ___ 15. Throughout his long career, Pete Seeger has been a lead figure in reviving folk music. A B --c:- 0 234 Section 2 • Guide to Structure and Written Expression LESSON 39 ERRORS WITH PREPOSITIONS Errors with prepositions are among the most difficult errors to catch. Preposition use in English is very complex. For every rule, there seems to be an exception. Recently, there have been more errors involving prepositions in the Written Expression part of TOEFL, and the errors have been more difficult to spot. Prepositions are used in the following ways: • In adverbial phrases that show time, place, and other relationships in the morning on Pennsylvania Avenue to the park by a student a reason for a solution to • After certain nouns a cause of • After certain adjectives and participles different from aware of disappointed in rely on refer to • After certain verbs combine with • In phrasal prepositions (two- or three-word prepositions) according to together with instead of in general on occasion • In certain set expressions by far at last Note: In Mini-Lessons for Section 2, found at the end of this section of the Guide, all of these uses for prepositions are explained and practice exercises are provided. There are two main types of preposition errors that you may see in the Written Expression part of the test: A) Errors in Preposition Choice The wrong preposition is used according to the context of the sentence. Some of the rules for choosing the correct prepositions are given in the Mini-Lessons, but you will never be able to memorize all the rules for preposition use in English. The more you practice, though, the more you will develop a "feel" for determining which preposition is correct in any given situation. There are two particular situations involving preposition choice that are often tested in Written Expression: • Errors with from . .. to and between . .. and Both these expressions are used to give the starting time and ending time. They can also be used to show relationships of place and various other relationships. He lived in Seattle from 1992 to 1997. He lived in Seattle between 1992 and 1997. Route 66 ran from Chicago to Los Angeles. Route 66 ran between Chicago and Los Angeles. Section 2 • Guide to Structure and Written Expression 235 Errors usually involve an incorrect pairing of those words, or the incorrect use of other prepositions: *between A to B *between A with B *from X andY *since X toY • Errors with since, for, and in Since is used before a point in time with the present perfect tense-but never with the past tense. For is used before a period of time with the present perfect and other tenses. In is used before certain points in time (years, centuries, decades) with the past tense and other tenses-but never with the present perfect tense. He's lived here since 1995. He's lived here jor two years. He moved here in 1995. Errors involve the use of one of these prepositions for another: *He's lived here in 1995. *He's lived here since two years. *He lived here since 1995. .... Sample Items The pitch of a tuning fork depends of the size and shape of its arms. ABC D The correct preposition after the verb depend is on, not of The Alaskan Pipeline runs between Prudhoe Bay on the Arctic A B Coastal Plain to the port of Valdez, a distance of 789 miles. ----c- D The correct pattern isjrom . .. to. Candles were mankind's chief source of illumination since A B -C- at least 2,000 years. ---0 Before a period of time (2,000 years) the prepositionjor should be used. B) Incorrect Inclusion or Omission of Prepositions A preposition is used when one is not needed, or not used when one is needed. ... 236 Section 2 • Guide to Structure and Written Expression ... Sample Items According many critics, MarkTwain's novel Huckleberry Finn is his A greatest work, and is one of the greatest American novels ever written. B C D The preposition to has been omitted from the phrase According to. Some of the most of spectacular waterfalls in the eastern B -c-,- -A- United States are found in the Pocono Mountains of Pennsylvania. D The preposition of should not be used in this phrase. (When most means "majority," it can be used in the phrase most of the. "Most of the people agree ... ," for example. However, in this sentence, most is part of the superlative form of the adjective spectacular, and so cannot be used with of) ... Exercise 39.1 Focus: Identifying correct and incorrect preposition choice. Directions: Underline the prepositions that correctly complete the sentences below. The first one is done as an example. 1. (EQrffo) an injection to be effective (on/against) tetanus, it must be administered (by/within) seventy-two hours (Qfjfor) the injury. 2. Wage rates depend (in/on) part (from/on) the general prosperity (of/for) the economy. 3. The invention (of/for) the hand-cranked freezer opened the door (for/to) commercial ice cream production, and (for/since) then, the ice-cream industry has grown (in/into) a four-billion-dollara-year industry. 4. (At/On) the time (of/in) the RevolutionaryWar, the North American colonies were merely a long string (with/ot) settlements (along/among) the Atlantic Coast (between/from) Maine and Georgia. 5. The probability (of/for) two people (in/on) a group (of/for) ten people having birthdays (in/on) the same day is about one (in/ot) twenty. 6. Showboats were floating theaters that tied up (at/to) towns (in/on) the Ohio and Mississippi rivers to bring entertainment and culture (to/at) the people (on/in) the frontier. 7. Scrimshaw, the practice (of/for) carving ornate designs (in/on) ivory, was first practiced (by/ot) sailors working (by/with) sail needles while (in/on) long sea voyages. 8. Assateague Island, (off/ot) the coast (off/ot) Virginia, is famous (for/to) its herds (of/with) wild ponies. 9. an/On) order (for/to) an object to be visible, light must travel (from/for) that object (at/to) a person's eyes. 10. an/On) the 1930's and 1940's, when train travel was (on/at) its peak, passengers could look forward (for/to) wonderful meals (on/at) trains. Section 2 • Guide to Structure and Written Expression 237 11. (In/Since) the 1960's, op art, which was based (in/on) scientific theories (of/for) optics, employed patterns (of/in) lines and colors that seemed to change shape as the viewer looked (on/at) them. 12. The first national convention devoted (for/to) the issue (of/with) women's rights, organized partly (oflby) Elizabeth Cady Stanton, was held (in/on) her hometown (in/of) Seneca Falls, New York, (in/on) 1848. 13. (In/Since) 1716 a party (of/for) explorers led (by/with) Lieutenant Governor Spotswood (of/in) Virginia tried (in/on) vain to find a route (through/of) the Appalachian Mountains. 14. Dolphins rely (in/on) echolocation, a form (of/for) navigation similar (with/to) the sonar systems used (on/at) submarines. 15. Analytical geometry, (in/on) which algebraic ideas are used (for/to) the description (of/for) geometric objects, has been (in/on) use (for/since) the seventeenth century. Exercise 39.2 Focus: Identifying and correcting errors involving the inclusion or omission of prepositions. Directions: If there is a preposition unnecessarily included in a sentence, mark that sentence X and underline the preposition. If there is a preposition incorrectly omitted from a sentence, mark that sentence X, underline the word before and after the missing preposition, and write the correct preposition on the line at the end of the sentence. If the sentence is correctly written, mark that sentence C. The first one is done as an example. ~ 1. According polls taken throughout the twentieth century, Lincoln and Washington are the preeminent American presidents. to 2. Today, many varieties of fruit are available all year thanks improved storage and shipping techniques. _ _ _ _ __ 3. The origin of the Moon remains a mystery. _ _ _ _ __ 4. Traffic jams can cause of pollution, delays, and short tempers. _ _ _ _ __ 5. The Sun's rays heat the Earth's surface, on which then radiates the heat into the air. 6. A warm-blooded animal is one that keeps the same body temperature regardless the air temperature. _ _ _ _ __ 7. Charlie Parker, considered by many the greatest improviser in the history of jazz, influenced many other jazz musicians. _ _ _ _ __ 8. Most of people are aware of the need to visit dentists regularly. _ _ _ _ __ 9. Muscle fibers are attached bones by tendons. _ _ _ _ __ 10. In his essay "Self-Reliance," Ralph W Emerson told to his readers why they should not depend on the ideas of others. _ _ _ _ __ 11. The crayfish is a freshwater crustacean related the lobster. _ _ _ _ __ 12. Charles Goren was an expert the game of bridge. _ _ _ _ __ 13. Stomata are the tiny openings in the leaves of plants through which oxygen and carbon dioxide pass. _ _ _ _ __ 14. Ducks have small oil glands by which keep their feathers oily and repel water. 238 Section 2 • Guide to Structure and Written Expression 15. The tail of a comet always points away the Sun. _ _ _ _ __ 16. Lichens grow in extreme environments in where no other plants can exist. 17. Not all of waterfalls are formed in the same way. _ _ _ _ __ 18. The pulmonary artery carries blood from the right side the heart to the lungs. ___ 19. In addition to the twelve constellations of the zodiac, thirty other constellations were familiar people of ancient times. ______ 20. Rainbows always appear in that part of the sky opposite the Sun. _ _ _ _ __ Exercise 39.3 Focus: Identifying and correcting preposition errors. (Note: One or two items in this exercise do not focus on preposition errors. These are marked in the answer key with an asterisk.) Directions: Decide which of the four underlined words or phrases-(A), (B), (C), or (D)-would not be considered correct, and write the letter of that expression in the blank at the beginning of the sentence. Then, in the blank at the end of the sentence, write a correction for the underlined phrase. The first one is done as an example. ~ 1. Water polo is a game in which is played in the water by two teams, each with seven A B C players. 0 which 2. Dynamics is a branch of physics that deals for the relationship between motion A B C and force. ______ o ___ 3. Many of radio stations began broadcasting baseball games during the 1920's. A ___ --B-- C D 4. The economy of Maine is based to a great extent in its forests, which cover eighty ---s A -C- percent of its surface area. _ _ _ _ __ o 5. The removal of waste materials is essential to all forms of live . _ _ _ _ __ A ___ B C o 6. John Diefenbaker, Prime Minister of Canada during 1957 to 1963, is given much of the A B C credit for the adoption of the Canadian Bill of Rights. _ _ _ _ __ o ___ 7. The first stage on the manufacturing of all types of clothing is the cutting of the A -BC 0 material. _ _ _ _ __ Section 2 • Guide to Structure and Written Expression 239 8. All of the wheat grown throughout the world belongs one of fourteen species. -A- ___ B C D 9. There are approximately 600 different species of trees native of the continental United A B C D States. ______ 10. Waterwheels, which appeared on the fourth century B.C., were probably the first --A- If C machines not powered by humans or animals. ______ D 11. Since centuries, Southwestern Indian tribes have valued turquoise and have used it -A- --B- -c- in jewelry. _ _ _ _ __ D 12. Loggerhead turtles lay thousands eggs at a single time, but only a a few of survive A B C toadulthood. _ _ _ _ __ D ___ 13. In nowadays, commercial bakeries use complex, automated machines, but the basic A principles of baking have changed little for thousands of years. _ _ _ _ __ B -c- D 14. It takes over four years for light from the nearest star reaching the earth. _ _ _ _ __ -AB -cD ___ 15. In the mid-1900's, an increasing number of jobs in the United States have involved the A B C handling of information. ______ D 240 Section 2 • Guide to Structure and Written Expression LESSON 40 ERRORS WITH ARTICLES Like errors with prepositions, errors with articles are sometimes hard to catch. This is partly because of the complexity of the article system in English, and partly because articles, like prepositions, are "small words" and one's eye tends to skip over errors involving these words. The basic uses of articles are explained in the chart: A or an is used before singular nouns when one does not have a specific person, place, thing, or concept in mind: The is used before singular, plural, and non-count nouns when one has a specific person, place, thing, or concept in mind: No article is used hefore non-count nouns or plural nouns when one does not have specific persons, places, things, or concepts in mind: an orange the oranges (2) orange a chair the oranges (2) oranges the fruit (2) fruit the chair (2) chair the chairs (2) chairs the furniture (2) furniture The indefinite article a is used before words that begin with a consonant sound (a chair, a book); an is used before words that begin with a vowel sound (an orange, an ocean liner). Before words that begin with the letters h- and U-, either a or an can be used, depending on the pronunciation of the words. an honor a hat an umbrella a university There are also some specific rules for using (or not using) articles that you should be aware of. • An indefinite article can be used to mean "one." It can also used to mean "per." a half, a quarter, a third, a tenth a mile a minute (one mile per minute) an apple a day (one apple per day) • A definite article is used when there is only one example or the thing or person, or when the identity of the thing or person is clear. The Moon went behind some clouds. (There's only one Moon.) Please open the door. (You know which door I mean.) Section 2 • Guide to Structure and Written Expression 241 • A definite article is usually used before these expressions of time and position. the morning the afternoon the evening" the past the present the future the the the the the front back center top bottom the beginning the middle the end *No article is used in the expression "at night." • A definite article comes before a singular noun that is used as a representative of an entire class of things. This is especially common with the names of animals, trees, inventions, musical instruments, and parts of the body. The tiger is the largest cat. My favorite tree is the oak. The Wright Bothers invented the airplane. The oboe is a woodwind instrument. The heart pumps blood. • A definite article is used before expressions with a ordinal number. No article is used before expressions with cardinal numbers. the first the fourth chapter the seventh volume one Chapter Four Volume Seven • A definite article is used before decades and centuries. the 1930's the fifties the 1800's the twenty-first century • A definite article is usually used before superlative forms of adjectives. the widest river the most important decision • A definite article is used in quantity expressions in this pattern: quantifier + of + the + noun. many of the textbooks some of the water all of the people not much of the paper most of the students a few of the photographs These expressions can also be used without the phrase of the. many textbooks some water all people not much paper most students a few photographs • A definite article is used before the name of a group of people or a nationality. No article is used before the name of a language. The Swedish are proud of their ancestors, the Vikings. She learned to speak Swedish when she lived in Stockholm. • A definite article is used when an adjective is used without a noun to mean "people who are ...." Both the young and the old will enjoy this movie. The poor have many problems. 242 Section 2 • Guide to Structure and Written Expression • A definite article is used before a non-count noun or a plural noun when it is followed by a modifier. No article is used when these nouns appear alone. The rice that I bought today is in the bag. Rice is a staple in many countries. Trees provide shade. The trees in this park are mostly evergreens. • A definite article is used before the name of a field of study followed by an of phrase. If a field is used alone, or is preceded by an adjective, no article is used. literature the literature of the twentieth century the history of the United States American history • Definite articles are used before the "formal" names of nations, states, and cities. (These usually contain of phrases.) No articles are used before the common names of nations, states, and cities. the United States of America the state of Montana the city of Philadelphia America Montana Philadelphia • Definite articles are used before most plural geographic names: the names of groups of lakes, mountains, and islands. No article is used before the names of individual lakes, mountains, and islands. the Great Lakes the Rocky Mountains the Hawaiian Islands Lake Powell Mount Washington Long Island In the Written Expression section, there are three main types of errors involving articles: A) Incorrect Article Choice One of the most common errors is the use of a in place of an or vice versa. Fortunately, this is also the easiest type of error to detect. Another error is a or an used in place of the, or the in place of a or an. .... Sample Items A eclipse of the Sun may be either total or partial. A B -C- --0- An must be used before a noun beginning with a vowel sound such as eclipse. Rose Bird was a first woman in the history of California to serve -A- B C on the State Supreme Court. D In a phrase with an ordinal number (such as first) the definite article the must be used. £. Section 2 • Guide to Structure and Written Expression 243 B) Incorrect Omission or Inclusion of an Article Sometimes an article is used when none is needed, or one is omitted when one is required. ... Sample Items Slag consists of waste materials and impurities which -A- -H- rise to top of melted metals. -c- D The definite article the should not be omitted from the phrase to the top of The most asteroids are beyond the orbit of the planet Mars. ABC D Definite article are used only before quantity expressions that contain of phrases. (Most asteroids or Most of the asteroids are both correct in this sentence.) .. C) Use of a Definite Article in Place of a Possessive A definite article may be incorrectly used in place of a possessive word-its, his, her, or their. ... Sample Item The Ozark Mountains of Arkansas are famous for the rugged beauty. ABC 0 The should correctly read their because the sentence refers to the beauty belonging to the Ozark Mountains. .. Exercise 40.1 Focus: Identifying the correct and incorrect use of articles. Directions: Underline the forms that correctly complete the sentence. The first one is done as an example. 1. Only about (the one/one) percent of (the water/water) on Earth is (the fresh/fresh) water. 2. (The mineral/Mineral) phosphate is (the most/most) common ingredient of all types of (the fertilizers/fertilizers) . 3. (The/A) process of refming minerals requires (a/an) huge amount of (an electrical/electrical) energy. 4. (A humor/Humor) runs through (the American/American) literature from (the earliest/earliest) times until (the present/present). 244 Section 2 • Guide to Structure and Written Expression 5. (The ozone/Ozone) layer acts as (a/an) umbrella against (the most/most) of (the Sun's/Sun's) dangerous rays. 6. In (the early/early) 1800's, Sequoia, (a Cherokee/Cherokee) leader, created (the/a) ftrst written form of (a North/North) American Indian language. 7. (The Goddard/Goddard) family of (the New/New) England produced some of (the/a) ftnest furniture made in (the United/United) States in (the seventeenth/seventeenth) century. 8. (The popcorn/Popcorn) has (a/the) same food value as any other kind of (a corn/corn). 9. One of (the most/most) important tools for (a research/ research) in social science is (a wellwritten/well-written) questionnaire. 10. Native to (the American/American) West, (the/a) coyote came east early in (the twentieth/twentieth) century when its chief natural competitor, (the/a) woIf, died out. 11. (The nineteenth/Nineteenth) century astronomer Alvin G. Clarke built hundreds of (the refracting/refracting) telescopes during (theihis) lifetime. 12. (The Hawaiian!Hawaiian) Islands are among (the most/most) geographically isolated islands in (the world/world). Exercise 40.2 Focus: Identifying and correcting errors with articles. (Note: One or two items in this exercise do not focus on article errors. These are marked in the answer key with an asterisk.) Directions: Decide which of the four underlined words or phrases-(A), (B), (C), or (D)-would not be considered correct, and write the letter of the expression in the blank. Then, in the line at the end of the sentence, write a correction for the underlined phrase. The first one is done as an example. ~ 1. The most butterfly eggs are coated with a sticky substance that holds them to plants. A -B-C -0Most 2. A number of large insurance companies have the headquarters in Hartford, A B C D Connecticut. _ _ _ _ __ ___ 3. To be effective, an advertisement must first attract an attention. _ _ _ _ __ A B --c0 4. Virgin Islands National Park features a underwater preserve with coral reefs and colorful A -B- C tropical fish. _ _ _ _ __ o ___ 5. Arthritis, a painful swelling of the joints, is often associated with elderly people, but A B C can afflict young as well. _ _ _ _ __ o Section 2 • Guide to Structure and Written Expression 245 6. Wilmington is an only large city in the state of Delaware. _ _ _ _ __ 13 A C 0 7. About the third of the earth's land surface is covered by relatively flat plains. A B -C D 8. In the 1920's, gasoline companies began giving away free road maps to the customers. A ___ -c: B D 9. The Tropic of Cancer is imaginary line that marks the northern boundary of the Earth's A B C D tropical zone. _ _ _ _ __ 10. Hereford cows are one of most common breeds of cattle raised for beef. _ _ _ _ __ -c-:- B A D 11. American soprano Kathleen Battle taught music in elementary school before beginning A B the career as a professional singer. _ _ _ _ __ C D 12. In 1891, first state law to help local communities pay for highways was passed in New 7:"" -BC D Jersey. _ _ _ _ __ ___ 13. Lumber is dried and seasoned in an heated chamber called a dry kiln. _ _ _ _ __ A B C D 14. Grandfather Mountain, a highest mountain in the Blue Ridge mountain range, is in A B C D North Carolina. _ _ _ _ __ ___ 15. The term "baritone" refers to the range of male voice that lies between tenor to bass. A B -CD ___ 16. It was around 1925 that accurate, convenient system for recording the choreography of A B C ballet was developed. _ _ _ _ __ D 246 Section 2 • Guide to Structure and Written Expression ___ 17. Richard Byrd was the first person in the history to fly over the North Pole. A B -C-O ___ 18. At beginning of the Civil War, Matthew Brady was authorized to accompany A B the Union Army and take photographs. _ _ _ _ __ C 0 ___ 19. In 1878 in San Francisco, KateWiggins open the first kindergarten on the West Coast. A -B- C D 20. The tulip tree, the tallest broadleaf plant in the eastern United States, may reach height A B C of over 200 feet. _ _ _ _ __ -D- Section 2 • Guide to Structure and Written Expression 247 LESSON 41 ERRORS WITH COMPARATIVES AND SUPERLATIVES Most adjectives have three forms: the absolute (the basic adjective form), the comparative, and the superlative. Comparatives are used to show that one item has more of some quality that another does. George is taller than his brother. Superlatives are used to show that one item in a group of three or more has the greatest amount of some quality. He was the tallest man in the room. The chart explains how comparatives and superlatives are formed: One-syllable adjectives warm warmer the warmest Two-syllable adjectives ending with l' funny funnier the funniest Other two-syllable adjectives common more common the most common Adjectives with three or more syllables important more important the most important Some two-syllable adjectives have two correct forms of both the comparative and the superlative: narrower more narrow clever more clever polite more polite narrowest most narrow cleverest most clever politest most polite A "negative" comparison can be expressed with the words less and least. Less and least are used no matter how many syllables an adjective has. less bright the least bright less expensive the least expensive The absolute form of a few adjectives ends in -er (tender, bitter, slender, clever, and so on). Don't confuse these with the comparative forms (more bitter or bitterer, for example). Many adverbs also have comparative and superlative forms. The comparative and superlative forms of all -ly adverbs are formed with more and most. more brightly most brightly more importantly most importantly A few adjectives and adverbs have irregular comparative and superlative forms: good/well better the best badlbadly worse the worst far farther the farthest further the furthest 248 Section 2 • Guide to Structure and Written Expression (Far has two comparative and superlative forms, depending on how the word is used, but the distinction between these two forms will not be tested.) There are two main types of errors involving comparatives and superlatives: A) Incorrect Choice of the Three Forms Any of the three forms-absolute, comparative, or superlative-may be incorrectly used in place of one of the other forms. ... Sample Items Basketball is played at a much fast pace than baseball. ABC 0 The comparative formfaster is needed because two concepts-the pace of basketball and the pace of baseball-are being compared. The deep oceans contain some of the stranger of all living creatures. A B C D This sentence does not compare two groups; a superlative form (strangest) is required. .... B) Incorrect Forms of Comparatives and Superlatives Incorrect forms such as more bigger or most hot may appear. ... Sample Item The most small vessels in the circulatory system are capillaries. A B C D The correct form is smallest because small is a one-syllable adjective. .... Exercise 41 Focus: Identifying and correcting errors in comparatives and superlatives. Directions: If the underlined form is correct, mark the sentence C. If the underlined form is incorrect, mark the sentence X and write a correction for the underlined form in the blank at the end of the sentence. The first one is done as an example. ~ 1. The period is probably the most easiest punctuation mark to use. the easieet 2. When metal replaced wood in the construction of ships' hulls, more strong and large ships could be built. _ _ _ _ __ 3. Charcoal is the most commonly used cooking fuel in the world. _ _ _ _ __ Section 2 • Guide to Structure and Written Expression 249 4. Soft solder melts at a low temperature than ordinary solder. ______ 5. Many of the nation's most important documents are stored in the National Archives Building in Washington, D.C. _ _ _ _ __ 6. The surfboards used twenty-five years ago were more heavier than the ones used by surferstoda~ ______ ___ 7. Few American politicians have spoken more eloquently than William Jennings Bryan. ___ 8. Subterranean termites are the more destructive type of termites in the United States. 9. Prince Edward Island is the less populous of Canada's ten provinces. ______ 10. During a depression, economic conditions are far worst than they are during a recession. 11. One of the most basic American contributions to technology was the so-called "American system" of interchangeable machine parts. ______ 12. The horse chestnut has a stronger, bitter taste than other chestnuts. ______ 13. Chicago's Field Museum is one of the largest and better known natural history museums in the United States. _ _ _ _ __ 14. Baltimore has one ofthe world's most frnest natural harbors. ______ 15. The Kennedy-Nixon race of 1960 was the closest presidential election of last century. 16. The finback whale is the fastest of all whales, and only the blue whale is largest. 17. The cello is shorter and more slender than the double bass. _ _ _ _ __ 18. Oil floats on water because oil is less denser than water. ______ 250 Section 2 • Guide to Structure and Written Expression lESSON 42 ERRORS IN WORD ORDER Most word-order errors in Written Expression consist of two words in reverse order. Some of the most common examples of this type of error are given below: Noun + adjective drivers careful careful drivers Noun + possessive clothing women's women's clothing Main verb + auxiliary finished are are finished Adjective + adverb a basic extremely idea an extremely basic idea Verb + subject (in an indirect question or other wh- clause) Tell me where is it. I spoke to John when was he here. Tell me where it is. I spoke to John when he was here. Preposition/adverb clause marker + adverb after immediately immediately after Participle + adverb baked freshly bread freshly baked bread Relative pronoun + preposition the house which in she lives the house in which she lives adverb, adjective, or quantifier + almost totally almost late almost all almost almost totally almost late almost all enough + adjective* enough good good enough *Enough can correctly be used before nouns: enough money, enough time. Enough may also be used before an adjective when the adjective comes before a noun. (There weren't enough good seats at the concert.) .. Sample Items Goods such as flowers fresh and seafood are often shipped by air. --A- B C D The adjectivefresh must come before the noun flowers: fresh flowers. Visitors to Vancouver often comment on how beautiful its A B setting is and on how clean is it. C l) The correct word order is subject + verb: it is. .... Section 2 • Guide to Structure and Written Expression 251 Exercise 42.1 Focus: Identifying and correcting word order in sentences. Directions: If the word order of the underlined form is correct, mark the sentence C. If the word order is incorrect, mark the sentence X and write a correction in the blank at the end of the sentence. ~ 1. The Douglas fir is the source chief of lumber in the state of Oregon. chief so' wce 2. The painted turtle is a colored brightly, smooth-shelled turtle. _ _ _ _ __ 3. Trained in Europe, John Sargent became an extremely successful portrait painter in the United States. _ _ _ _ __ 4. Insects lived on the Earth before long the first mammal appeared. _ _ _ _ __ 5. The freezing point is the temperature which at a liquid becomes a solid. _ _ _ _ __ 6. Ammonia, a compound of nitrogen and hydrogen, has many industrial uses. ___ 7. The Atlantic coastline of the United States is about 400 longer miles than the Gulf coastline. _ _ _ _ __ 8. Zoos provide an opportunity to study a wide range of animals, often in their habitats natural. _ _ _ _ __ 9. A test pilot tries out new kinds of aircraft to determine if are they safe. _ _ _ _ __ 10. The air of the upper atmosphere is just enough dense to ignite meteors by friction. 11. More pigs are raised in Iowa than in other any state. _ _ _ _ __ 12. Cirrus clouds are composed entirely almost of ice crystals. _ _ _ _ __ 13. Many sailboats are equipped with small engines for times when there is not enough wind. _______ 14. Few of the doctors in the thirteen colonies had any training formal as physicians. ___ 15. Margaret Wise Brown was a successful writer of books children's. _ _ _ _ __ Exercise 42.2 Focus: Identifying errors involving word order. (Note: One or two items in this exercise do not focus on word-order errors. These are marked in the answer key with an asterisk.) Directions: Decide which of the four underlined words or phrases-(A), (B), (C), or (D)-would not be considered correct, and write the letter of the expression in the blank. Then, in the line at the end of the sentence, write the correction for the underlined phrase. The first one is done as an example. ~ 1. During pioneer times, the Allegheny Mountains were a barrier major to transportation. A maior barrier -B- C 0 252 Section 2 • Guide to Structure and Written Expression 2. In Philadelphia's Franklin Institute, there is a working model of a human heart A B C enough large for visitors to walk through. _ _ _ _ __ D ___ 3. The task of the cartographer is to represent the Earth's surface at a reduced greatly scale. --A- -B- C D 4. Mutualism is a relationship between animal species which in both benefit. ABC D 5. To grow well, a tree must be well-suited to the area where is it planted. ______ ABC -0- 6. The development of transistors made possible it to reduce the size of many A B C electronic devices. _ _ _ _ __ D ___ 7. Twelve drawings usually have to be prepared for second each of animated film. ABC D 8. Today many garden vegetables have few resemblance to the wild plants from which A B C D they come. _ _ _ _ __ ___ 9. Sloths are moving slow, shaggy mammals that are often seen hanging upside down from A B C tree limbs. _ _ _ _ __ D 10. Carbohydrates as such sugar or starches are important energy sources for A B C humans and animals. _ _ _ _ __ D 11. On nights when is the sky clear and the air calm, the Earth's surface rapidly radiates heat A B C D into the atmosphere. _ _ _ _ __ 12. Frank Lloyd Wright is known for his original highly methods of harmonizing buildings A B C with their surroundings. ______ D Section 2 • Guide to Structure and Written Expression 253 ___ 13. Some algae are microscopic and consist of one only cell, but others are large plants A B C containing many cells. _ _ _ _ __ o 14. A fully grown male mountain lion may be eight long feet. _ _ _ _ __ A B C 0 ___ 15. Stone fruits are fruits such as peaches and plums in which a hard pit surrounded is by A B C soft pulp. _ _ _ __ o 16. Job enrichment is a technique used to increase satisfaction workers' by giving them A B C more responsibilities. ______ o 17. The first permanent European settlement in what is now Mississippi was a A B -C- center trading in Biloxi. ______ o 18. In the early 1800's, a dispute developed among geologists how about rocks are formed. A B C 0 ___ 19. Most country music songs are deeply personal and deal with themes oflove, lonely, and A B C -0- separation. ______ 20. An umbra is a shadow's darkest central part from which is light totally excluded. A B --C- 0 254 Section 2 • Guide to Structure and Written Expression LESSON 43 ERRORS WITH CONJUNCTIONS You may encounter errors with either correlative conjunctions or coordinate conjunctions. A) Errors with Correlative Conjunctions Correlative conjunctions are two-part conjunctions. Errors usually involve an incorrect combination of the two parts, such as neither . .. or or not only . ... and. Anytime you see a sentence containing correlative conjunctions, you should be on the lookout for this type of error. This is an easy error to spot! either ... or neither ... nor both ... and not only ... but also whether ... or Another error is the use of both . .. and to join three elements. T Sample Items X rays have important applications, not only in medicine -A- B and in industry. C D The correct pattern is not only . .. but also. The air that surrounds our planet is both odorless, ~ --B- C D colorless, and invisible. Because both . . . and can be used to join only two elements, the word both must be eliminated to correct the sentence. .. B) Errors with Coordinate Conjunctions The conjunction and is correctly used to show addition; or is used to show choice between alternatives; but is used to show contrast or opposition. Section 2 • Guide to Structure and Written Expression 'Y 255 Sample Item Brakes and clutches serve very different functions A B in an automobile, and their principles of operation -c are nearly the same. -0- The first clause discusses how brakes and clutches are different; the second clause discusses how they are the same. Therefore, the conjunction joining them must show contrast. Choice (C) should read but. ... Exercise 43 Focus: Identifying and correcting errors involving conjunctions. Directions: If the underlined form is correct, mark the sentence C. If the underlined form is incorrect, mark the sentence X and write a correction for the underlined form in the blank at the end of the sentence. The first one is done as an example. ~ 1. Model airplanes can be guided both by control wires or by radio transmitters. either 2. Information in a computer can be lost because it is no longer stored or because it is stored but cannot be retrieved. _ _ _ _ __ 3. John Lancaster Spaulding was not only a religious leader and also a social reformer. 4. Although fish can hear, they have neither external ears or eardrums. _ _ _ _ __ 5. In all animals, whether simple and complex, enzymes aid in the digestion of food. 6. The two most common methods florists use to tint flowers are the spray method or the absorption method. ______ ___ 7. Beekeepers can sell either the honey and the beeswax that their bees produce. 8. The alloys brass and bronze both contain copper as their principal metals. _ _ _ _ __ 9. The human brain is often compared to a computer, and such an analogy can be misleading. ______ 10. Rust both corrodes the surface of metal but also weakens its structure. _ _ _ _ __ 11. A work of science fiction generally uses scientific discoveries and advanced technology, either real or imaginary, as part of its plot. _ _ _ _ __ ___ 12. Community theater both provides entertainment for local audiences but also furnishes a creative outlet for amateurs interested in drama. ______ 256 Section 2 • Guide to Structure and Written Expression 13. The heron is a long-legged wading bird that preys on both frogs, fish, and eels. 14. For over twenty years after winning the World Chess Championship in 1972, Bobby Fischer played in either a tournament nor an exhibition game. _ _ _ _ __ ___ 15. Designing fabric requires not only artistic talent but also a knowledge of fiber and of textile machinery. _ _ _ _ __ Section 2 • Guide to Structure and Written Expression 257 MINI-TEST 7: WRITIEN EXPRESSION Directions: The sentences below have four underlined words or phrases, (A), (B), (C), and (D). You must identify the one underlined expression that must be changed for the sentence to be correct. Then find the Ilumber of the question Oil your answer sheet and fill in the space corresponding to the letter. Time: 15 minutes 5. Commercial bakeries can make thousands of loaves of bread on one time by using A -B- C -r-)- automated equipment. 2. North America is a third largest of the seven continents. ABC ___ 0 3. Neither humans or dogs can hear as well as cats. A Il c:---O 4. The astronomer George Hale was a pioneer in the art of photograph the Sun. A i l e 0 5. Trucks can be used transport a wide variety of cargoes. --A- B --C- --0- 6. Pikes Peak, named for explorer Zebulon Pike, is Colorado's most famous but not its A B C most highest mountain. D ___ 7. Identical colors may appear to be quite different when are they viewed against different A backgrounds. -B-·- C D 258 Section 2 • Guide to Structure and Written Expression S. Oceanography is not a singl~ A science and rather a group of disciplines wit~ a common n c focus. -0- 9. Until 1960, Maine was unique in that it held presidential and congressional elections on <= --n- A September, two months earlier than the rest of the nation. o 10. Snowshoes allow a person walking on snow without sinking into it because they A B C distribute the person's weight over a wide area. I) 11. Fuel injection engines employ injectors instead a carburetor to spray fuel into the A B C 0 cylinders. 12. Ocean currents have a enormous effect on life on this planet. --A- --c- B D ___ 13. Hydraulic elevators are still used in some old buildings. but all almost new buildings are --n-- B- C equipped with electrical elevators. o 14. Diaries and journals writing during colonial times provide the best records of that era. D A B C 15. Frequently, the combination of several spices will result of a more pleasing flavor than A B C J) the use of just one. ___ 16. Gold topaz is much rare than either white or blue topaz. A -n- --C->----0 ___ 17. A mosaic is picture made from small bits of colored glass or tile. A -BC D IS. Reclamation is the successful attempt to A mak~ deserts, marshlands, or other unusable B land suitable of farming or building. C 0 ___ 19. A successful salesperson must have an intuitive understanding of psychology human. A B C 0 Section 2 • Guide to Structure and Written Expression 259 20. The University of Wisconsin was the first school to make a serious effort teach students A c B public administration. D 21. Some underground water is enough safe to drink, but all surface water must be treated. -A- B [) D 22. A feeding animal will usually allow competitors approaching only within a certain A B --c- distance, the boundaries of which are called its feeding territory. D ___ 23. Most familiar type of pump in use today is the piston pump. A B C D 24. The term "forgetting" refers to the loss, whether temporary and long-term, of material A B C that has previously been learned. D 25. Dieticians urge people to eat the banana a day to get enough potassium in their diet. -A- B C D 260 Section 2 • Guide to Structure and Written Expression MINI-LESSONS FOR SECTION 2 Preposition Use It is important for the Structure and especially the Written Expression parts of the test that you be familiar with the correct usage of prepositions. The mini-lessons for this section consist of lists of prepositions in combination with other words and in various set expressions. The exercises allow you to practice using these prepositions in sentences. Cover the lists with a piece of paper as you complete the exercises. Mini-Lesson 2.1 Adjectives/Participles + Prepositions, Part A acceptable to accustomed to adequate for afraid of aware of based on capable of characteristic of close to composed of contrary to dependent on different from disappointed in/with eligible for equal to equipped with essential to/for familiar with famous for Exercise: Fill in the blanks in the sentences below with the correct prepositions. The first one is done as an example. 1. The Medical Center is close to campus. 2. I was disappointed the grade I received on my last essay. 3. Now that she has graduated, Anne is no longer dependent her parents for financial support. 4. Catherine became accustomed spicy foods when she was traveling. 5. Table salt is composed two elements, sodium and chlorine. 6. Is your bicycle equipped a light? 7. This computer isn't capable running that software. 8. Bluegrass music is somewhat different other types of country music. 9. Washington State is famous its apples. 10. Was your choice of research topic acceptable your instructor? 11. People who are afraid heights are called acrophobes. 12. Water is essential all life. 13. 14. 15. 16. 17. 18. 19. 20. the regulation against smoking in this area? Were you aware Tepees are characteristic the Indian tribes of the Great Plains. Will this office be adequate your needs? I'm not familiar that song. One meter is approximately equal ____ a yard. This movie is based a true story. This summer, he'll be eligible a three-week vacation. What he said is contrary common sense. Section 2 • Guide to Structure and Written Expression 261 Mini-Lesson 2.2 Adjectives/Participles + Prepositions, Part B free of independent of inferior to married to native to necessary for/to next to opposed to opposite of* perfect for possible for preferable to related to relevant to satisfied with suitable for surprised at/by typical of 'Opposite oj is used for words or concepts that are completely different, such as large and small. When opposite means "across from," it is not used with of. "The bank is opposite the post office on Cedar Street." Exercise: Fill in the blanks in the sentences below with the correct prepositions. 1. The art museum is located next the museum of natural history on State Street. 2. Many vegetables, induding tomatoes, potatoes, and corn, are native the New World. the colonial period. 3. This style of architecture is typical 4. Oxygen, fuel, and heat are all necessary ____ combustion. ';. Two people would find this apartment too crowded, but it's perfect ____ one. 6. These tools are inferior 7. Were you surprised the ones I usually use. the grade you received? 8. The opposite oldJashioned is modern. 9. House cats are distantly related lions and tigers. 10. Is this type of soil suitable growing tomatoes? 11. The point that Murray brought up wasn't really relevant ____ the discussion. 12. Organically raised crops are free ____ chemical pesticides and herbicides. 13. A grade of A- is preferable 14. If you're not satisfied one ofB+. your essay, then I suggest that you rewrite it. 15. Abolitionists were people who were opposed the practice of slavery. 16. The United States became independent England in 1776. 17. Is it possible ____ me to get an appointment sometime next week? 18. President James Madison was married ____ one of the most famous of all first ladies, Dolley Madison. 262 Section 2 • Guide to Structure and Written Expression Mini-Lesson 2.3 Nouns + Prepositions approach to attention to cause of component of contribution to cure for decrease in demand for effect oflon* example of exception to experience with expert on form of group of improvement in increase in influence on interest in native of origin of part of price of probability of quality of reason for reliance on result of solution to supply of *effect + of + cause effect + on + thing or person affected (fhe effect of heat on rocks ... ) Exercise: Fill in the blanks in the sentences below with the correct prepositions. 1. A decrease the supply a good usually results in an increase ____ the price that good. 2. Once scientists fully understand the cause ____ a disease, it becomes easier for them to find a cure ____ it. 3. Professor Lyle noticed a distinct improvement ____ the quality ____ her students' work. 4. It is believed that sunspots have an influence ____ the Earth's weather patterns. 5. Have you had much experience ____ computers? 6. I'm sorry; I wasn't paying close attention what you said. 7. The professor gave us several examples that phenomenon. S. Do you know the reason the delay? 9. Interest physical fitness increased during the 19S0's. 10. The Dorothy Chandler Pavilion is part ____ the Los Angeles Music Center. II. Linguists have many theories about the origin ____ language. 12. This is an exception ____ the general rule. this problem? 13. What approach should I take 14. The Ivy League is a group ____ eight prestigious universities in the Northeast. 15. People's reliance automobiles as their chief form transportation has increased over the years. 16. I've tried and tried, but I simply can't find a solution this dilemma. 17. The demand personal computers continues to grow. IS. Only a native the United States can serve as President. the economy. 19. Economists don't agree on what effects government spending has 20. Margaret Knight designed some of the components the rotary engine. Section 2 • Guide to Structure and Written Expression 263 Mini-Lesson 2.4 Verbs + Prepositions account for adjust to agree with/on* attach to attribute to begin with believe in belong to combine with insist on interfere with participate in plan on refer to rely on result in search for compete with concentrate on consist of contribute to cooperate with deal with depend on devote to engage in 'Agree with is used with people. Agree on is used with an issue, plan, etc. (I agreed with Mary on that issue.) Exercise: Fill in the blanks in the sentences below with the correct prepositions. 1. Do you belong any campus clubs or organizations? 2. Cytology is the branch of biology that deals the structure, form, and life of cells. 3. Maybe you should begin your speech ____ some jokes. 4. Let's concentrate solving this problem before we discuss the other ones. 5. People want friends they can rely _ _ __ 6. Most essays consist 7. We didn't plan an introduction, a body, and a conclusion. such a long delay. S. If you are engaged _ _ _ _ any extracurricular activities, you should mention that fact on your resume. 9. Iron combines ____ oxygen to form rust. 10. After several hours of discussion, the council finally agreed _ _ _ _ a plan. 11. The accident resulted several minor injuries. 12. Storms on the Sun can interfere radio broadcasts on the Earth. 13. By the late 1940's, television had begun to seriously compete advertisers. 14. That theatrical company can always be depended 15. James didn't have any trouble adjusting in the South. radio for audience and to deliver a good performance. the climate in Atlanta because he'd grown up 16. William insists _ _ _ _ getting up early, even on weekends. 17. What does this symbol refer _ _ __ IS. Occupational physicians search _ _ _ _ the causes of injury and sickness at the workplace. 19. Many companies participated 20. How do you account the trade fair. this discrepancy? 21. Do you believe any superstitions? 22. Workaholics devote too much of their time _ _ _ _ their jobs. 264 Section 2 • Guide to Structure and Written Expression Mini-Lesson 2.5 Phrasal Prepositions according to ahead of along with because of by means of due to except for in favor of in spite of instead of on account of prior to regardless of thanks to together with Exercise: Fill in the blanks in the sentences below with the correct prepositions. 1. Work on the new highway will be finished ahead 2. spite schedule. the warnings, Phil dove off the cliff. 3. The chairman, along his staff, attended the conference. 4. According _ _ _ _ my dictionary, you're mispronouncing that word. 5. Prior _ _ _ _ her wedding, Nicole's last name was Brooks. 6. Are you _ _ _ _ favor _ _ _ _ that amendment or against it? 7. account _ _ _ _ a lack of funds, the university library will now close at nine instead _ _ _ _ at eleven. 8. Sunsets may appear more colorful because _ _ _ _ air pollution. 9. Due _ _ _ _ a computer error, $100,000 was transferred into]udy's checking account. 10. Regardless the final score, I'm sure this will be an exciting game. 11. Glider pilots can actually increase their altitude called thermals. 12. Thanks means hot air currents the financial aid he received, he was able to attend the university. Mini-Lesson 2.6 In, On, and At, Part A Expressions of Time + + + in + + + century (in the eighteenth century) decade (in the 1990's) year (in 1975) season (in the summer) month (in july) parts of the day (in the morning, in the evening, in the afternoon) + days of the week (on Wednesday) on + dates (on October 7) + time of day (at 6 EM.; at noon) at + night Section 2 • Guide to Structure and Written Expression 265 Expressions of Place + continent (in Africa) + country (in Mexico) + state (in Pennsylvania) in + city (in Los Angeles) + huilding (in the bank) + room (in the auditorium) + the world + street (on Maxwell Street) on + floor of a building (on the f()urtb floor) + Earth at + address (at 123 Commonwealth Avenue) Exercise: Fill in the hlanks in the sentences helow with the preposition in, on, or at. 1. John F Kennedy was the first President of the United States to be born century. 2. Fruit trees generally bloom the twentieth April or May. 3. Gettysburg, the greatest battle ever fought North America, took place ____ July 1863. 4. I like to shower ____ the morning, hut my roommate likes to shower _ _ _ _ night. 1. The President lives the White House, which is located _ _ _ _ 1600 Pennsylvania Washington, D.C. Avenue 6. Many advertising agencies are located Madison Avenue New York City. 7. Jazz was so popular 8. Leaves turn red and gold the 1920's that the decade is sometimes called the Jazz Age. the autumn. 9. Most college foothall games are played 10. Both Washington and Lincoln were horn February 22, Lincoln February 12. Saturdays. February. Washington was born _ _ __ 11. About 90% of all the people New Jersey live 12. The New Year is celehrated midnight 13. Dean Hughes' office is 14. Quebec is the largest province cities. January 1. the Administration Building Canada. the third floor. 266 Section 2 • Guide to Structure and Written Expression Mini-Lesson 2.7 In, On, and At, Part B The prepositions in, on, and at are also used in a number of set expressions: in a book/magazine newspaper in charge (of) in common (with) in danger (of) in detail in existence in the front/middlelback in general in part in the past/future in practice in a row in style in theory at best/worst at birth at death at first/last at least/most at the moment at once at the peak (of) at present at random on a bus/train/etc. on fire on the other hand on purpose on radio/television on the whole Exercise: Fill in the blanks in the sentences below with the preposition in, on, or at. 1. Did you hear that news television or read it 2. The members of a jury are chosen the newspaper? random from a list of voters. 3. Videophones are not practical present, but they may be charge of the marketing department. 4. Mr. Grigsby is 5. Please come here the near future. once. 6. I'm sure he didn't break the window ____ purpose. 7. Air travel is its peak the summer. the whole, I enjoyed the movie, but there were a few scenes I didn't like. 8. 9. Mark is a good friend, but we don't have much common. 10. Claudia has gotten so many speeding tickets that she's danger of losing her driver's license. 11. The story ____ the magazine described the incident ____ great detail. 12. There is least one vowel in virtually every English word. 13. Five oak trees were planted 14. This type of music is no longer a row front of the school. style. 15. ____ general, I found zoology to be an easier subject than botany. 16. The American Constitution has been existence for over two hundred years. 17. Like many other baby birds, ducklings are blind 18. This book is based ____ part on fact. birth. 19. The ship was ____ fire. 20. ____ theory, this idea is quite difficult to understand; ____ the other hand, it is quite simple ____ practice. Section 2 • Guide to Structure and Written Expression Mini-Lesson 2.8 Other Prepositions By is often used with forms of communication and trdnsportation: by car, by plane, by phone, by e:xjJress mail ('Jote: If the noun is plural or is preceded by a determiner, the preposition in or on must be used: in cars, on a boat, on tbe telepbone, in a taxi.) By is also used with gerunds to show how an action happened: How did you get an appointment with Dr. Blish? By calling his secretary. With is used to indicate the idea of accompaniment or possession: Melanie came to the party with her friend. He wanted a house witb a garage. Without indicates the opposite relationship: Melanie came to the party witbout ber friend. He bought a house without a garage. With also indicates that an instrument was used to perform an action: He opened the door witb a key. Without indicates the opposite relationship: He opened the door witbout a key. By andfiJr are also llsed in the following expressions: by chance by far by hand for example for free for now }"or is sometimes used to show purpose; it means "to get." She went to the store for toothpaste and shampoo. Exercise: Fill in the blanks in the sentences below with the correct prepositions. 1. Magnetic compasses are not very useful on ships steel hulls. 2. Penicillin was discovered more or less ____ chance. 3. Legal aid organizations provide legal advice for poor people for small fees or ____ free. 4. Alaska is far the largest state. '). Amoebas are so small that they can't be seen a microscope. 6. Fruits, vegetables. and other agricultural products are generally shipped ____ truck . . . , Semaphore operators communicate ____ using flags. 8. The most expensive rugs are made hand. 9. Ice hockey is played a hard rubber disk called a puck. 10. They traveled to the island ____ a boat. 267 T SEC TI 0 N 3 Guide to Reading Comprehension 269 270 Section 3 • Guide to Reading Comprehension About Reading Comprehension This part of the exam tests your ability to read and answer questions about passages written in formal written English. It usually contains five passages. After each passage there are usually from eight to twelve questions referring to that passage for a total of fifty questions. The passages vary in length from about seven lines to about thirty-five. Sample Items (line) (5) (0) Like mammals, birds claim their own territories. A bird's territory may be small or large. Some birds claim only their nest and the area right around it, while others claim far larger territories that include their feeding areas. Gulls, penguins, and other waterfowl nest in huge colonies, but even in the biggest colonies, each male and his mate have small territories of their own immediately around their nests. Male birds defend their territory chiefly against other males of the same species. In some cases, a warning call or threatening pose may be all the defense needed, but in other cases, intruders may refuse to leave peacefully. 1. What is the main topic of this passage? (A) Birds that live in colonies (8) How birds defend their territory (C) The behavior of birds (D) Territoriality in birds Choice (A) deals with particular types of birds-gulls, penguins, and others-but the passage concerns all birds. Choice (8) deals only with the concept of defending a territory. This is the topic of the second paragraph, but not of the passage as a whole. Choice (C) is too general; there are many types of bird behavior that this passage does not examine. Choice (D) is best, because all the aspects of the passage deal with some factors of birds' territories. 2. According to the passage, male birds defend their territories primarily against (A) female birds (B) birds of other species (C) males of their own species (D) carnivorous mammals Lines 9-10 state that male birds" defend their territories chiefly against other males of the same species." There is no mention in the passage of any of the other options. Section 3 • Guide to Reading Comprehension 3. It can be inferred from the passage that gulls and penguins (A) do not claim a feeding area as part of their territories (B) share their territories with many other birds (C) leave their colonies during their nesting season (D) do not build nests The passage says that birds which claim their feeding areas have large territories compared to those which do not. Birds living in colonies have "small territories ... immediately around their nests," indicating that their feeding areas would not be part of their territories. Choice (B) contradicts lines 7-8, which state that "each male and his mate ... have small territories of their own." Choice (C) is unlikely because the passage indicates that these birds' nests are part of large colonies; they would not leave during nesting season. Choice (D) is incorrect because these birds' nests are part of their territories. 4. In line 5, the word "it" refers to (A) a nest (B) an area (C) a bird (D) a territory Substitute all four answer choices for the word it in line 3; as you'll see, only (A) is a logical choice. 5. The word "intruders" in line 12 is closest in meaning to (A) invaders (B) youngsters (C) defenders CD) guests It is clear from the passage that the "intruders" mentioned in this line are male birds that "refuse to leave peacefully." Of the four answer choices, only "invaders" (persons or creatures that attack from the outside) could be substituted for the word "intruders" with no major change in meaning. ... The Passages The passages cover a wide range of topics, but in general can be classified as follows: 1. Science and technology (40%) Includes astronomy, geology, chemistry, physics, mathematics, zoology, botany, medicine, engineering, mechanics, and so on. 2. North American history, government, geography, and culture (20%) 3. Art (15%) Includes literature, painting, architecture, dance, drama, and so on. 4. Social science (10%) sociology, and so on. 5. Biography (15%) Includes anthropology, economics, psychology, urban studies, 271 272 Section 3 • Guide to Reading Comprehension Some passages may be classified in more than one way. For example, a biography might be about the life of a historical figure, an artist, or a scientist. If there is a national context for any of the passages, it is American or occasionally Canadian. Therefore, if a passage is about history, it will be about the history of the United States or Canada. Although the passages deal with various topics, the style in which they are written is similar, and they usually follow fairly simple patterns of organization. The vocabulary used in the Reading Comprehension portion is fairly sophisticated. There will almost certainly be words that you do not recognize. Sometimes you can guess the meaning of these words by context. However, it is not necessary to understand all the vocabulary in the passages in order to answer the questions. The Questions The main types of questions asked can be categorized as shown below: Main Ideal Main Topicl Main Purpose 00%) These ask you to identify an answer choice that correctly summarizes the author's main idea, the subject of the whole passage, or the author's reason for writing the passage. "What is the main idea of the passage?" "What is the passage primarily about?" "Why did the author write the passage?" Factual Questions These ask you to locate and identify answers to questions about specific information and details in the passage. "According to the passage, where did ... ?" "According to the author, why did ... ?" "Which of the following is true, according to the passage?" These ask which of the answer choices is NOT discussed in the passage. "Which of the following is NOT true about ..." "All of the following are true (24%) Negative Questions (9%) EXCEPT ..." Scanning Questions (4%) Inference Questions 02%) These ask you to find the place in the passage that some topic is mentioned. "Where in the passage does the author first discuss ..." These ask you to draw conclusions based on information in the passage. 'The author implies that which of the following is true?" "Which of the following can be inferred from the passage?" Section 3 • Guide to Reading Comprehension Vocabulary-in-Context Questions (26%) Reference Questions (10%) 273 These ask you identify the meaning of a word or phrase as used in the passage. "The word ' - - ' in line 5 is closest in meaning to .. :' These ask you to identify the noun to which a pronoun or other expression refers. "The word "it" in line 15 refers to ..." "In line 20, the word "there" refers to which of the following?" There are a few other types of questions that are occasionally asked about the readings. These make up the remaining 5% of the question types. About half the Reading items have closed stems; they begin with direct questions. The others have open stems; they begin with incomplete sentences. Closed Stem Which of the following is the main topic of the passage? Open Stem The main topic of the passage is The Answer Choices For all questions in this part, the four options are equally grammatical and, in most cases, would be equally logical choices for someone who has not carefully read the passage. Incorrect choices are generally based on information that is found in the passage, but does not correctly answer the question. What Is the Best Way to Approach the Passages? First, take a quick look at the questions (not at the answer choices) to see what will be asked about. Try to keep these in the back of your mind as you read the passage. Then read the article at a comfortable speed. Try not to read one word at a time, like this: A bird's territory may be small nest and the area right around or it. large. Some birds clain1 only their Word-by-word reading slows you down and interferes with your comprehension. Try to read in units of thought. In other words, group words into related phrases. A bird's territory may be small or large. Some birds claim only their nest and the area right around it. The paragraph is the primary unit of meaning in all reading. Most passages consist of from one to five paragraphs. If you can form a clear idea of what each paragraph is about, you can put these ideas together and come up with the main idea of the entire passage. The main idea of each paragraph is often contained in one sentence, called the topic sentence. Try to locate topic sentences. Often, the topic sentence is the first sentence of a paragraph, but it may also be the last sentence. Occasionally, a paragraph has no topic sentence; the topic is stated indirectly. If there is more than one paragraph in the passage, try to understand the relationship between the paragraphs. Form a mental "map" of the passage. Watch for words that signal special relationships and tranSitions, such as however, therefore, first, next, then, and so on. 274 Section 3 • Guide to Reading Comprehension What Is the Best Way to Answer the Questions? It depends on the type of question. The best tactics for answering each type of question are given in the next five lessons. In the exercises, you will read passages similar to the ones found on actual TOEFL tests to develop the reading skills needed to do well on this section. What Are Some Important Factors in Scoring High on This Section? Timing is an important factor. Most test-takers find this the hardest to complete because reading the passages takes up so much time. Near the end of the test, you may realize that you won't have enough time to finish. Don't panic' When there is only about four or five minutes remaining, this is what you should do: If you have not read one of the passages, skim it over very quickly to get the main idea. Answer the first question about it (which will probably be a main idea or topic question). Then answer any of the questions that provide line numbers (vocabulary-in-context and reference questions) because these require less time to locate the information you need. Then answer any remaining questions, referring back to the passage as little as pOSSible. If you can't find the information needed to answer the question in about ten seconds, just pick the choice that seems the most logical. Then, in the last few seconds, fill in any empty blanks with your "guess answer." On the other hand, if you do finish the test before time is called, go back and work on items that you had trouble with the tirst time. Don't just sit back and relax, and don't turn back to Sections 1 or 2. Concentration is another important factor. This is the last section of the test. It's also the longest and, for some people, the most difficult. You may be starting to get tired. However, you've got to maintain your concentration until the end of the test and not waste any time. What Is the Best Way to Prepare for This Part of TOEFL? After completing the exercises in the Guide and taking the practice tests in this book, read as much on your own as you can. A recent study showed that extracurricular reading (readings done outside of classes) was the single most important factor in improving overall TOEFL scores. Material from textbooks, magazines, newspapers, and encyclopedias will be useful. You might also want to try writing TOEFL-style questions about passages that you read. This technique not only increases your comprehension, but it also helps you think like the people who write TOEFL tests. 3 • As with the other sections, be familiar with the directions and examples for Section 3 so that you can begin work immediately. • For each passage, begin by briefly looking over the questions (but not the answer choices). Try to keep these questions in mind during your reading. • Read each passage at a comfortable speed. • Answer the questions, referring to the passage when necessary. • Eliminate answers that are clearly wrong or do not answer the questions. If more than one option remains, guess. • Mark difficult or time-consuming answers so that you can come back to them later if you have time. Erase all these marks before the end of the test. • Don't spend more than about 10 minutes on anyone reading and the questions about it. Section 3 • Guide to Reading Comprehension o o 275 When only a few minutes remain, don't start guessing blindly, Skim the remaining passage or passage quickly; then answer the first question in each set, Then answer any questions with line numbers. After that, read the remaining questions, and if you can't find the question quickly, choose the one that seems most logical to you. When there are only a few seconds left, fill in all remaining blanks with your guess letter. 276 Section 3 • Guide to Reading Comprehension 00 Sample Reading Comprehension Test Time: 55 minutes This section of the test measures your ability to understand the meaning of words and to comprehend written materials. Directions: The remainder of this section contains several passages, each followed by a number of questions. Read the passages and, for each question, choose the one best answer-(A), (B), (C), or (D)-based on what is stated in the passage or on what can be inferred from the passage. Then fill in the space on your answer sheet that matches the letter of the answer that you have selected. Read the following passage: (line) (5) (10) Like mammals, birds claim their own territories. A bird's territory may be small or large. Some birds claim only their nest and the area right around it, while others claim far larger territories that include their feeding areas. Gulls, penguins, and other waterfowl nest in huge colonies, but even in the biggest colonies, each male and his mate have small territories of their own immediately around their nests. Male birds defend their territory chiefly against other males of the same species. In some cases, a warning call or threatening pose may be all the defense needed, but in other cases, intruders may refuse to leave peacefully. Example! What is the main topic of this passage? (A) Birds that live in colonies (B) Birds' mating habits (C) The behavior of birds CD) Territoriality in birds Sample Answer Section 3 • Guide to Reading Comprehension 277 The passage mainly concerns the territories of birds. You should fill in (D) on your answer sheet. Example!! Sample Answer According to the passage, male birds defend their territory primarily against (A) (B) (C) (D) female birds birds of other species males of their own species mammals The passage states that "Male birds defend their territory chiefly against other males of the same species." You should fill in (C) on your answer sheet. As soon as you understand the directions, begin work on this part. lij.i.,;,t."UjU43 i @t. • 278 Section 3 • Guide to Reading Comprehension Questions 1-12 The technology of the North American colonies did not differ strikingly from that of Europe, but in one respect, the colonists enjoyed a great advantage. Especially by comparison with (line) Britain, Americans had a wonderfully plentiful supply of wood. (5) The first colonists did not, as many people imagine, find an entire continent covered by a climax forest. Even along the Atlantic seaboard, the forest was broken at many points. Nevertheless, all sorts of fine trees abounded, and through the early colonial period, those who pushed westward encountered new (10) forests. By the end of the colonial era, the price of wood had risen slightly in eastern cities, but wood was still extremely abundant. The availability of wood brought advantages that have seldom been appreciated. Wood was a foundation of the economy. Houses (15) and all manner of buildings were made of wood to a degree unknown in Britain. Secondly, wood was used as a fuel for heating and cooking. Thirdly, it was used as the source of important industrial compounds, such as potash, an industrial alkali; charcoal, a component of gunpowder; and tannic acid, used for (20) tanning leather. The supply of wood conferred advantages but had some negative aspects as well. Iron at that time was produced by heating iron ore with charcoal. Because Britain was so stripped of trees, she was unable to exploit her rich iron mines. But the American (25) colonies had both iron are and wood; iron production was encouraged and became successful. However, when Britain developed coke smelting, the colonies did not follow suit because they had plenty of wood and besides, charcoal iron was stronger than coke iron. Coke smelting led to technological innovations and was (30) linked to the emergence of the Industrial Revolution. In the early nineteenth century, the former colonies lagged behind Britain in industrial development because their supply of wood led them to cling to charcoal iron. 1. What does the passage mainly discuss? (A) The advantages of using wood in the colonies (B) The effects of an abundance of wood on the colonies (C) The roots of the Industrial Revolution (D) The difference between charcoal iron and coke iron lij.i.",t."njU43 i @t-. Section 3 • Guide to Reading Comprehension 279 2. The word "strikingly" in line 2 is closest in meaning to (A) realistically (B) dramatically (C) completely (D) immediately 3. Which ofthe following is a common assumption about the forests of North America during the colonial period? (A) They contained only a few types of trees. (B) They existed only along the Atlantic seaboard. (C) They had little or no economic value. (D) They covered the entire continent. 4. The use of the word "abounded" in line 8 indicates that the trees were (A) present in large numbers (B) restricted to certain areas (C) cut down (D) cultivated 5. According to the passage, by the end of the colonial period, the price of wood in eastern cities (A) rose quickly because wood was becoming so scarce (B) was much higher than it was in Britain (C) was slightly higher than in previous years (D) decreased rapidly because of lower demand for wood 6. What can be inferred about houses in Britain during the period written about in the passage? (A) They were more expensive than American houses. (B) They were generally built with imported materials. (C) They were typically smaller than homes in NorthAmerica. (D) They were usually built from materials other than wood. 7. Why does the author mention gunpowder in line 19? (A) To illustrate the negative aspects of some industrial processes (B) To give an example of a product made with wood (C) To remind readers that the Colonial era ended in warfare CD) To suggest that wood was not the only important product of the Colonies 8. The word "conferred" in line 21 is closest in meaning to (A) consulted (B) gathered (C) provided (D) restricted 9. The phrase "follow suit" in line 27 means (A) do the same thing (B) make an attempt (C) have the opportunity (D) take a risk (ij.i.';'(.ilUjU§3.@t-. 280 Section 3 • Guide to Reading Comprehension 10. According to the passage, why was the use of coke smelting advantageous? (A) It led to advances in technology. (B) It was less expensive than wood smelting. (C) It produced a stronger type of iron than wood smelting. (D) It stimulated the demand for wood. 11. The phrase "cling to" in line 33 is closest in meaning to (A) try to develop (B) avoid (C) continue to use (D) reconsider 12. Where in the passage does the author begin to discuss in detail the disadvantages that an abundant supply of wood brought to the colonies? (A) Lines 1-3 (B) Lines 5-7 (C) Lines 13-14 (D) Lines 21-22 lij.i.,;.tfjlUj U43 i @t-. Section 3 • Guide to Reading Comprehension Questions 13-22 The Peales were a distinguished family of American artists. Charles Willson Peale is best remembered for his portraits of leading figures of the American Revolution. He painted portraits (line) of Franklin and Jefferson, and over a dozen of George Washington. His life-size portrait of his sons Raphaelle and Titian was so (5) realistic that George Washington reportedly once tipped his hat to the figures in the picture. Charles Willson Peale gave up painting in his middle age and devoted his life to the Peale Museum, which he founded in (10) Philadelphia. The world's first popular museum of art and natural science, it featured paintings by Peale and his family as well as displays of animals in their natural settings. Peale found the animals himself and devised a method of taxidermy to make the exhibits more lifelike. The museum's most popular display was the skeleton of a mastodon-a huge, extinct elephant-which Peale (15) unearthed on a New York farm in 180l. Three of Peale's seventeen children were also famous artists. RaphaeUe Peale often painted stilllifes of flowers, fruit, and cheese. His works show the same luminosity and attention to (20) detail that the works of the Dutch masters show. In the late eighteenth century, however, portraiture was the rage, and so RaphaeUe Peale found few buyers for his stilllifes at the time. His brother Rembrandt studied under his father and painted portraits of many noted people, including one of George (25) Washington. Another brother, Rubens Peale, painted mostly landscapes and portraits. James Peale, the brother of Charles Willson Peale, specialized in miniatures. His daughter Sarah Miriam Peale was probably the first professional female portrait painter in America. 13. What is the main topic of the passage? (A) The life of Charles Willson Peale (B) Portraiture in the 18th century (C) The Peale Museum (D) A family of artists 14. The author probably mentions that Washington "tipped his hat to the figures in the painting" (lines 6- 7) to indicate that (A) Charles Willson Peale's painting was very lifelike (B) Washington respected Charles Willson Peale's work (C) Washington was friendly with Raphaelle and Titian Peale (D) the painting of the two brothers was extremely large 15. The word "settings" in line 12 is closest in meaning to which of the following? (A) Environments (B) Categories (C) Positions (D) Requirements @",u'G1'HinAiti¥t- • 281 282 Section 3 • Guide to Reading Comprehension ,----- - - ~-- --- - - -- --------------------- - --------- 1() For which of the following terms does the author give a definition in the second paragraph? (A) Natural SCi(,IlCl' (B) Skeleton (C) 'Llxidermy (I) :vlastmlon 1"7_ Which of the following questions about the Peale ,\1useum does thc passage NOT supply enough information to answer' (A) Who found and prepared its animal exhibits? In what city \\-a~ It lucatedr (e) \Vhcrc did it" m,)sf popular exhihit COlB, trmn? (D) III \V\1;lt YC:ll \. ~\s Ii t<H1l1ded? on I K_ The word 1I1le.lfrhnl' in linc I () is closest ill mcanmg to (:\) displa\nl (B) dug up (C) located (D) jooked over Il)_ -Which of the following words could best be substituted for the word "rage" in line 2l? (A) Fashion (B) Anger (C) (D) Conflict Desire 20. According to the p,(ssage, Rembrandt Peale and his father both painted C-\) miniatures (B) portraits of George \Vashington (C) paintings of flowers. fruit, and cheese (D) pictures of animals 21. \Vhich of the fo\lmving is NOT one of the children of Charles Willson Peale? (A) Titian Peale (B) Ruhens Peale (C) Raphadle Peak (1)) Sarah \1iriam Peale 22. The author's attitude toward the Peales is generally (A) envious (B) puzzled (C) admiring (D) disappointed lijoi.,;,t.ilUj U43 i ¥t--' I ~-~- Section 3 • Guide to Reading Comprehension 283 Questions 23-34 ---------------------------------------------------------------- According to the best evidence gathered by space probes and astronomers, Mars is an inhospitable planet, more similar to Earth's Moon than to Earth itself-a dry, stark, seemingly (line) lifeless world. Mars' air pressure is equal to Earth's at an faltitude of 100,000 feet. The air there is 95% carbon dioxide. (5) Mars has no ozone layer to screen out the Sun's lethal radiation. Daytime temperatures may reach above freezing, but because the planet is blanketed by the mere wisp of an atmosphere, the heat radiates back into space. Even at the equator, the temperature (10) drops to -50°C (-60°F) at night. Today there is no liquid water, although valleys and channels on the surface show evidence of having been carved by running \vater. The polar ice caps are made of frozen water and carbon dioxide, and water may be frozen in the ground as permafrost. (15) Despite these difficult conditions, certain scientists believe that there is a possibility of transforming Mars into a more Earth-like planet. Nuclear reactors might be used to melt frozen gases and eventually build up the atmosphere. This in turn could create a "greenhouse effect" that would stop heat from radiating (20) back into space. Liquid water could be thawed to form a polar ocean. Once enough ice has melted, suitable plants could be introduced to build up the level of oxygen in the atmosphere so that, in time, the planet would support animal life from Earth and even permanent human colonies. "This was once thought to be (25) so far in the future as to be irrelevant," said Christopher McKay, a research scientist at the National Aeronautics and Space Administration. "But now it's starting to look practical. We could begin work in four or five decades." The idea of "terra-forming" Mars, as enthusiasts call it, has (30) its roots in science fiction. But as researchers develop a more profound understanding of how Earth's ecology supports life, they have begun to see how it may be possible to create similar conditions on Mars. Don't plan on homesteading on Mars any time soon, though. The process could take hundreds or even thousands of years to complete, and the cost would be staggering. 23. With which of the following is the passage primarily concerned? (A) The possibility of changing the Martian environment (B) The challenge of interplanetary travel (C) The advantages of establishing colonies on Mars (D) The need to study the Martian ecology 24. The word "stark" in line 3 is closest in meaning to (A) harsh (B) unknown (C) dark (D) distant ''d.,.",t." hi .t§:ii@(&. 284 Section 3 • Guide to Reading Comprehension 25. The word "there" in line 5 refers to (A) a point 100 miles above the Earth (B) the Earth's Moon (C) Mars (D) outer space 26. Which of the following does the author NOT list as a characteristic of the planet Mars that would make colonization difficult? (A) There is little liquid water. (B) Daytime temperatures are dangerously high. (C) The Sun's rays are deadly. (D) Nighttime temperatures are extremely low. 27. According to the passage, the Martian atmosphere today consists mainly of (A) carbon dioxide (B) oxygen (C) ozone (D) water vapor 28. It can be inferred from the passage that the "greenhouse effect" mentioned in line 19 is (A) the direct result of nuclear reactions (B) the cause of low temperatures on Mars (C) caused by the absence of green plants (D) a possible means of warming Mars 29. The word "suitable" in line 21 is closest in meaning to (A) resistant (B) altered (C) appropriate (D) native 30. According to Christopher McKay, the possibility of transforming Mars (A) could occur only in science fiction stories (B) will not begin for hundreds, even thousands of years (C) is completely impractical (D) could be started in 40 to 50 years 31. As used in line 29, the term "terra-forming" refers to (A) a process for adapting plants to live on Mars (B) a means of transporting materials through space (C) a method of building housing for colonists on Mars (D) a system of creating Earth-like conditions on other planets 32. The phrase "more profound" in lines 30-31 is closest in meaning to (A) deeper (B) more practical (C) more up-to-date (D) brighter @.i.,;,t."Uj;t§:J.@t-. Section 3 • Guide to Reading Comprehension 285 33. According to the article, the basic knowledge needed to transform Mars comes from (A) the science of astronomy (B) a knowledge of Earth's ecology (C) data from space probes (D) science fiction stories 34. The word "staggering" in line 35 is closest in meaning to (A) astonishing (B) restrictive (C) increasing (D) unpredictable ,§t.I·'I,t·j'bi .t§:J'UW t- . 286 Section 3 • Guide to Reading Comprehension Questions 35-43 Another critical factor that plays a part in susceptibility to colds is age. A study done by the University of Michigan School of Public Health revealed particulars that seem to hold true for (line) the general population. Infants are the most cold-ridden group, faveraging more than six colds in their first year. Boys have more (5) colds than girls up to age three. After the age of three, girls are more susceptible than boys, and teenage girls average three colds a year to boys' two. The general incidence of colds continues to decline into (10) maturity. Elderly people who are in good health have as few as one or two colds annually. One exception is found among people in their twenties, especially women, who show a rise in cold infections, because people in this age group are most likely to have young children. Adults who delay having children until (15) their thirties and forties experience the same sudden increase in cold infections. The study also found that economics plays an important role. As income increases, the frequency at which colds are reported in the family decreases. Families with the lowest income suffer (20) about a third more colds than families at the upper end. Lower income generally forces people to live in more cramped quarters than those typically occupied by wealthier people, and crowding increases the opportunities for the cold virus to travel from person to person. Low income may also adversely influence diet. (25) The degree to which poor nutrition affects susceptibility to colds is not yet clearly established, but an inadequate diet is suspected of lowering resistance generally. 35. The paragraph that precedes this passage most probably deals with (A) minor diseases other than colds (B) the recommended treatment of colds (C) a factor that affects susceptibility to colds (D) methods of preventing colds among elderly people 36. Which of the following is closest in meaning to the word "particulars" in line 3? (A) Minor errors (B) Specific facts (C) Small distinctions (D) Individual people 37. What does the author clainl about the study discussed in the passage? (A) It contains many inconsistencies. (B) It specializes in children. (C) It contradicts the results of earlier studies in the field. (D) Its results apparently are relevant for the population as a whole. 4,.@t-. lij.i."'i."nj U Section 3 • Guide to Reading Comprehension 287 38. It may be inferred from the passage that which of the following groups of people is most likely to catch colds? (A) Infant boys (B) Young girls (C) Teenage boys (D) Elderly women 39. There is information in the second paragraph of the passage to support which of the following conclusions? (A) Men are more susceptible to colds than women. (B) Children infect their parents with colds. (C) People who live in a cold climate have more colds than those who live in a warm one. (D) People who don't have children are more susceptible to colds than those who do. 40. The phrase "people in this age group" (line 13) refers to (A) infants (B) people in their twenties (C) people in their thirties and forties (D) elderly people 41. The author's main purpose in writing the last paragraph of the passage was to (A) explain how cold viruses are transmitted (B) prove that a poor diet causes colds (C) discuss the relationship between income and frequency of colds (D) discuss the distribution of income among the people in the study 42. The word "cramped" in line 21 is closest in meaning to (A) cheap (B) crowded (C) depressing (D) simple 43. The author's tone in this passage could best be described as (A) neutral (B) humorous (C) tentative (D) critical @.i.';'i·"Uj,i4=J.¥t-. 288 Section 3 • Guide to Reading Comprehension Questions 44-50 About flfty years ago, plant physiologists set out to grow roots by themselves in solutions in laboratory flasks. The scientists found that the nutrition of isolated roots was quite (line) simple. They required sugar and the usual minerals and vitamins. (5) However, they did not require organic nitrogen compounds. These roots got along flne on mineral inorganic nitrogen. Roots are capable of making their own proteins and other organic compounds. These activities by roots require energy, of course. The process of respiration uses sugar to make the high energy compoundATp, (10) which drives the biochemical reactions. Respiration also requires oxygen. Highly active roots require a good deal of oxygen. The study of isolated roots has provided an understanding of the relationship between shoots and roots in intact plants. The leaves of the shoots provide the roots with sugar and vitamins, (15) and the roots provide the shoots with water and minerals. In addition, roots can provide the shoots with organic nitrogen compounds. This comes in handy for the growth of buds in the early spring when leaves are not yet functioning. Once leaves begin photosynthesizing, they produce protein, but only mature (20) leaves can "export" protein to the rest of the plant in the form of amino acids. 44. What is the main topic of the passage? (A) The relationship between a plant's roots and its shoots (B) What can be learned by growing roots in isolation (C) How plants can be grown without roots (D) What elements are necessary for the growth of plants 45. The word "themselves" in line 2 refers to (A) plant physiologists (B) solutions (C) laboratory flasks (D) roots 46. According to the passage, what isATP? (A) A biochemical process (B) The tip of a root (C) A chemical compound (D) A type of plant cell 47. The word "intact" in line 13 is closest in meaning to (A) mature (B) wild (C) whole (D) tiny @.i.';'i."UjU§iiiUWV. Section 3 • Guide to Reading Comprehension 48. The use of the phrase "comes in handy" in line 17 indicates that the process is (A) useful (B) predictable (C) necessary (D) successful 49. It can be inferred from the passage that, in the early spring, the buds of plants (A) "export" protein in the form of amino acids (B) do not require water (C) have begun photosynthesizing (D) obtain organic compounds from the roots 50. Which of the following best describes the organization of the passage? (A) The results of two experiments are compared. (B) A generalization is made, and several examples of it are given. (C) The fIndings of an experiment are explained. CD) A hypothesis is presented, and several means of proving it are suggested. THIS IS THE END OF THE SAMPLE READING COMPREHENSION SECTION. IF YOU FINISH BEFORE TIME IS CALLED, GO BACK AND CHECK YOUR WORK. ~TO' ~TO' ~TO' ~TO' ~TO' ~TO' 289 290 Section 3 • Guide to Reading Comprehension LESSON 44 OVERVIEW QUESTIONS A) Main Idea, Main Topic, and Main Purpose Questions After almost every passage, the first question is an overview question about the main idea, main topic, or main purpose of a passage. Main idea questions ask you to identify the most important thought in the passage. Answer choices are complete sentences. Sample Questions • What is the main idea of the passage? • The primary idea of the passage is ... • Which of the following best summarizes the author's main idea? When there is not a single, readily identified main idea, main topic questions may be asked. These ask you what the passage is generally "about." Answer choices are noun phrases. Sample Questions • The main topic of the passage is ... • What does the passage mainly discuss? • The passage is primarily concerned with ... Main purpose questions ask why an author wrote a passage. The answer choices for these questions usually begin with infinitives. Sample Questions • • • • The author's purpose in writing is to ... What is the author's main purpose in the passage? The main point of this passage is to ... Why did the author write the passage? Sample Answer Choices • • • • To define .. . To discuss .. . To illustrate .. . To distinguish between ____ and _ _ __ • • • • To relate .. . To propose .. . To support the idea that ... To compare and Don't answer the initial overview question about a passage until you have answered the other questions. The process of answering the detail questions may give you a clearer idea of the main idea, topic, or purpose of the passage. The correct answers for main idea, main topic, and main purpose questions correctly summarize the main points of the passage; they must be more general than any of the supporting ideas or details, but not so general that they include ideas outside the scope of the passages. Distractors for this type of question have one of these characteristics: 1. They are too specific. 2. They are too general. 3. The are incorrect according to the passage. 4. They are irrelevant (unrelated) to the main idea of the passage. Section 3 • Guide to Reading Comprehension 291 If you're not sure of the answer for one of these questions, go back and quickly scan the passage. You can usually infer the main idea, main topic, or main purpose of the entire passage from an understanding of the main ideas of the paragraphs that make up the passage and the relationship between them. B) Other Overview Questions A number of other questions are asked that require an overall understanding of the passage. These are often the last question in a set of questions. Tone questions ask you to determine the author's feelings about the topic by the language that he or she uses in writing the passage. Look for vocabulary that indicates if the author's feelings are positive, negative, or neutral. Sample Questions • What tone does the author take in writing this passage? • The tone of this passage could best be described as ... Sample Answer Choices • • • • • • • Positive Favorable Optimistic Amused Pleased Respectful Humorous • Negative • Critical • Unfavorable • Neutral • Objective • Impersonal • Angry • Defiant • Worried • Outraged If you read the following sentences in passages, would the tone of those passages most likely be positive or negative? A) That was just the beginning of a remarkable series of performances by this brilliant actress. B) Despite some minor problems, this device has a number of admirable features. C) This practice is a waste of time and money. D) At the time his poems were first published, they were very popular, but today most critics find them simplistic and rather uninteresting. The italicized words in sentences A) and B) show a positive tone; in C) and D). the italicized words indicate a negative attitude. Notice that sentence B) contains negative words ("minor problems"), but the overall meaning of the sentence is positive. Sentence D) contains positive language ("very popular"), but overall, the tone is negative. (Words like despite, but, although, however, and similar words can "reverse" the tone of the passage.) Most TOEFL Reading passages have a neutral tone, but sometimes an author may take a position for or against some point. However, answer choices that indicate strong emotion-angry, outraged, sad, and so forth-will seldom be correct. Attitude questions are very similar to tone questions. Again. you must understand the author'S opinion. The language that the author uses will tell you what his or her positJOn is. What is the author's attitude toward smoking on airplanes as expressed in the sentence below? Although some passengers may experience a slight discomfort from not smoking on I'ng flights, their smoking endangers the health of all the passengers and crew. The author opposes smoking during flights. He admits that there is some ;lrgllll1ent in favor of smoking-some passengers may feel discomfort-but this is not as important as the fact that smoking can be dangerous to everyone on the flight. The use of the word although shmvs this. 292 Section 3 • Guide to Reading Comprehension Sample Questions • • • • What is the author's attitude toward ... The author's opinion of is best described as ... The author's attitude toward could best be described as one of ... How would the author probably feel about ... Another type of attitude question presents four statements and asks how the author would feel about them. • Which of the following recommendations would the author most likely support? • The author would be LEAST likely to agree with which of the following statements? • The author of the passage would most likely be in favor of which of the following policies? Organization questions ask about the overall structure of a passage or about the organization of a particular paragraph. Sample Question • Which of the following best describes the organization of the passage? Sample Answer Choices • • • • • • • • A general concept is defined, and examples are given. Several generalizations are presented, from which a conclusion is drawn. The author presents the advantages and disadvantages of _ _ __ The author presents a system of classification for _ _ __ Persuasive language is used to argue against _ _ __ The author describes _ _ __ The author presents a brief account of _ _ __ The author compares and _ _ __ Questions about previous or following paragraphs ask you to assume that the passage is part of a longer work: What would be the topic of the hypothetical paragraph that precedes or follows the passage? To fmd the topic of the previous paragraph, look for clues in the first line or two of the passage; for the topic of the following passage, look in the last few lines. Sometimes incorrect answer choices mention topics that have already been discussed in the passage. Sample Questions • With what topic would the following/preceding paragraph most likely deal? • The paragraph prior to/after the passage most probably discusses ... • It can be inferred from the passage that the previous/next paragraph concerns ... • What most likely precedes/follows the passage? Exercise 44.1 Focus: Identifying correct answers and recognizing distractors in main idea/main topic/main purpose questions. Directions: Read the passages. Then mark each answer choice according to the following system: S G X I C Too specific Too general Incorrect Irrelevant Correct The first one is done as an example. Section 3 • Guide to Reading Comprehension 293 There are two main types of cell division. Most cells are produced by a process called mitosis. In mitosis, a cell divides and forms two identical daughter cells, each with an identical number of chromosomes. Most one-celled creatures reproduce by this method, as do most of the cells in multicelled plants and animals. Sex cells, however, are formed in a special type of cell division called meiosis. This process reduces the number of chromosomes in a sex cell to half the number found in other kinds of cells. Then, when sex cells unite, they produce a single cell with the original number of chromosomes. 1. What is the main topic of this passage? ~ (A) The method by which one-celled organisms reproduce ~ (B) A comparison between mitosis and meiosis ~ (C) Meiosis, the process by which identical cells are produced The last gold rush belongs as much to Canadian history as it does to American. The discovery of gold along the Klondike River, which flows from Canada's Yukon Territory into Alaska, drew some 30,000 fortune hunters to the north. The Yukon became a territory, and its capital of the time, Dawson, would not have existed without the gold rush. The gold strike furnished material for a dozen of Jack London's novels. It inspired Robert Service to write "The Shooting of Dan McGrew" and other poems, and it provided the background for the wonderful Charlie Chaplin movie, The Gold Rush. It also marked the beginnings of modern Alaska. 2. This author's main purpose in writing is to ___ (A) discuss the significance of mining in Canada and the United States _ _ _ (B) show the influence of the Klondike gold strike on the creative arts ___ (C) point out the impact of the Klondike gold strike The keystone arch was used by almost every early civilization. To build a keystone arch, stones are cut so that the opposite sides taper toward each other slightly. The upper and lower surfaces are carved so that when several stones are placed side by side, the upper and lower surfaces meet in smooth, continuous curves. Some form of scaffolding is built under the arch and shaped to accept the curved underside of the stones. Then the stones are fitted in place one by one. The keystone is the top center stone, the last to be dropped into position. Afterwards, the scaffolding is removed and the arch is self-supporting. 3. The passage mainly concerns the ___ (A) basic principles of building keystone arches ___ (B) uses of arches in modern architecture ___ (C) role of scaffolding in building keystone arches Circumstantial evidence is evidence not drawn from the direct observation of a fact. If, for example, there is evidence that a piece of rock embedded in a wrapped chocolate bar is the same type of rock found in the vicinity of the candy factory, and that rock of this type is found in few other places, then there is circumstantial evidence that the stone found its way into the candy during manufacture and suggests that the candy-maker was negligent. Despite a popular notion to look down on the quality of circumstantial evidence, it is of great usefulness if there is enough of it and if it is properly interpreted. Each circumstance, taken singly, may mean little, but a whole chain of circumstances can be as conclusive as direct evidence. 4. What is the author's main purpose in this passage? ___ (A) To show that a manufacturer's negligence can be shown by direct evidence only ___ (B) To define circumstantial evidence and show its usefulness ___ (C) To demonstrate that circumstantial evidence can be very useful in science 294 Section 3 • Guide to Reading Comprehension The Northwest Ordinance was passed by Congress in 1787. It set up the government structure of the region north of the Ohio River and west of Pennsylvania, then called the Northwest Territory. It set the conditions under which parts of the Territory could become states having equality with the older states. But the ordinance was more than just a plan for government. The law also guaranteed freedom of religion and trial by jury in the Territory. It organized the Territory into townships of 36 square miles and ordered a school to be built for each township. It also abolished slavery in the Territory. The terms were so attractive that thousands of pioneers poured into the Territory. Eventually, the Territory became the states of Ohio, Indiana, Illinois, Michigan, and Wisconsin. 5. What is the main topic of this passage? ___ (A) The structure of government ___ (B) The provisions of an important law ___ (C) The establishment of schools in the NorthwestTerritory The story of the motel business from 1920 to the start of World War II in 1941 is one of uninterrupted growth. Motels spread from the West and the Midwest all the way to Maine and Florida. They clustered along transcontinental highways such as U.S. routes 40 and 66 and along the north-south routes running up and down both the East and West coast. There were 16,000 motels by 1930 and 24,000 by 1940. The motel industry was one of the few industries that was not hurt by the Depression ofthe 1930's. Their cheap rates attracted travelers who had very little money. 6. What does the passage mainly discuss? ___ (A) How the Depression hurt U.S. motels _ _ _ (B) The impact of transcontinental highways ___ (C) Two decades of growth for the motel industry An old proverb states, "Beware of oak, it draws the stroke." This saying is handy during thunderstorm season. In general, trees with deep roots that tap into groundwater attract more lightning than do trees with shallow, drier roots. Oaks are around 50 times more likely to be struck than beeches. Spruces are nearly as safe as beeches. Pines are not as safe as these two, but are still much safer than oaks. 7. What is the author's main point? ___ (A) Old proverbs often contain important truths. ___ (B) Trees with shallow roots are more likely to avoid lightning than those with deep roots. ___ (C) The deeper a tree's roots, the safer it is during a thunderstorm. Alternative history is generally classified as a type of science fiction, but it also bears some relation to historical fiction. This type of writing describes an imaginary world that is identical to ours up to a certain point in history, but at that point, the two worlds diverge; some important historical event takes place in one world but not in the other, and they go in different directions. Alternative histories might describe worlds in which the Roman Empire had never fallen, in which the Spanish Armada had been victorious, or in which the South had won the Civil War. Or they may suppose that some technology had been introduced earlier in the world's history than actually happened. For example: What if computers had been invented in Victorian times? Many readers fmd these stories interesting because of the way they stimulate the imagination and get them thinking about the phenomenon of cause and effect in history. 8. What is the main idea of this passage? ___ (A) Alternative histories describe worlds in which history has taken another course. _ _ _ (B) Alternative histories are a type of historical novel. ___ (C) Science fiction writers have accurately predicted certain actual scientific developments. Section 3 • Guide to Reading Comprehension 295 Until the late 1700's, metal could not be turned on a lathe to make it uniformly smooth and round. The operator could not guide the cutting tool evenly by hand against the turning piece. This problem was solved by David Wilkinson of Pawtucket, Rhode Island. In 1798 he invented a machine in which the cutter was clamped into a movable slide that could be advanced precisely, by hand crank, parallel to the work. The slide rest, as it came to be called, has many uses. It permits the manufacture of parts so uniform that they can be interchanged. Without it, mass production would not have been possible. As it turns out, the great English machinist Henry Maudsley had developed nearly the same mechanism a few years before, but this was unknown to Wilkinson and does not diminish his accomplishment. 9. Why did the author write this passage? ___ (A) To prove that Wilkinson's invention was based on Maudsley's ___ (B) To demonstrate the importance of mass production to American society ___ (C) To show the usefulness of Wilkinson's invention Almost every form of transportation has given someone the idea for a new type of toy. After the Montgolfier brothers flew the first balloon, toy balloons became popular playthings. In the nineteenth century, soon after railroads and steamships were developed, every child had to have model trains and steamboats. The same held true for automobiles and airplanes in the early twentieth century. Toy rockets and missiles became popular at the beginning of the space age, and by the 1980's, there were many different versions of space-shuttle toys. 10. The main idea of this passage is that ___ (A) inventors have been inspired by toys to build new forms of transportation ___ (B) toy automobiles and airplanes were very popular in the early 1900's ___ (C) toy design has followed developments in transportation Exercise 44.2 Focus: Answering a variety of overview questions about short passages. Directions: Read the passages and mark the best answer choice-(A), (B), (C), or (D). The first one is done as an example. American folk music originated with ordinary people at a time when the rural population was isolated and music was not yet spread by radio, audios, compact disks, or music DVDs. It was transmitted by oral tradition and is noted for its energy, humor, and emotional impact. The major source of early American folk songs was music from the British Isles, but songs from Africa as well as songs of the American Indians have a significant part in its heritage. Later settlers from other countries also contributed songs. In the nineteenth century, composer Stephen Foster wrote some of the most enduringly popular of all American songs, which soon bec,me part of the folk tradition. Beginning in the 1930's, Woody Guthrie gained great popularity by adapting traditional melodies and lyrics and supplying new ones as well. In the 1950's and 1960's, singer-composers such as Pete Seeger, Bob Dylan, and Joan Baez continued this tradition by creating "urban" folk music. Many of these songs dealt with important social issues, such as racial integration and the war in Vietnam. 1. The primary purpose of this passage is to ~ (A) trace the development of American folk music _ _ _ (B) explain the oral tradition ___ (C) contrast the styles of folk musicians ___ (D) point out the influence of social issues on "urban" folk music 296 Section 3 • Guide to Reading Comprehension Every scientific discipline tends to develop its own special language because it finds ordinary words inadequate, and psychology is no different. The purpose of this special jargon is not to mystify non-psychologists; rather, it allows psychologists to accurately describe the phenomena they are discussing and to communicate with each other effectively. Of course, psychological terminology consists in part of everyday words such as emotion, intelligence, and motivation, but psychologists use these words somewhat differently. For example, laymen use the term anxiety to mean nervousness or fear, but most psychologists reserve the term to describe a condition produced when one fears events over which one has no control. 2. The main topic of this passage is ___ (A) effective communication _ _ (B) the special language of psychology ___ (C) two definitions of the word anxiety ___ (D) the jargon of science Gifford Pinchot was the first professionally trained forester in the United States. After he graduated from Yale in 1889, he studied forestry in Europe. In the 1890's he managed the forest on the Biltmore estate in North Carolina (now Pisgah National Forest) and became the first to practice scientific forestry. Perhaps his most important contribution to conservation was persuading President Theodore Roosevelt to set aside millions of acres in the West as forest reserves. These lands now make up much of the national parks and national forests of the United States. Pinchot became the Chief Forester of the U.S. Forest Service in 1905. Although he held that post for only five years, he established guidelines that set forest policy for decades to come. 3. The passage primarily deals with ___ (A) Gifford Pinchot's work on the Biltmore Estate ___ (B) the practice and theory of scientific forestry ___ (C) the origin of national parks and national forests in the United States _ _ _ (D) the contributions Gifford Pinchot made to American forestry Off-Broadway theater developed in New York City in about 1950 as a result of dissatisfaction with conditions on Broadway. Its founders believed that Broadway was overly concerned with producing safe, commercially successful hit plays rather than drama with artistic quality. Off-Broadway producers tried to assist playwrights, directors, and performers who could not find work on Broadway. Off-Broadway theaters were poorly equipped, had limited seating, and provided few conveniences for audiences. But the originality of the scripts, the creativity of the performers, and the low cost of tickets made up for these disadvantages, and off-Broadway theater prospered. However, by the 1960's, costs began to rise and by the 1970's, off-Broadway theater was encountering many of the difficulties of Broadway and had lost much of its vitality. With its decline, a experimental movement called off-off-Broadway theater developed. 4. What is the main idea of this passage? ___ (A) After initial success, off-Broadway theater began to decline. ___ (B) Off-Broadway theaters produced many hit commercial plays. _ _ _ (C) Theaters on Broadway were not well equipped. ___ (D) Off-Broadway plays were highly creative. 5. The paragraph that follows this passage most likely deals with ___ (A) the help off-Broadway producers provided directors, playwrights, and performers _ _ _ (B) methods off-Broadway theaters used to cope with rising prices ___ (C) the development of off-off-Broadway theater ___ (D) the decline of Broadway theater Section 3 • Guide to Reading Comprehension 297 At the time of the ftrst European contact, there were from 500 to 700 languages spoken by North American Indians. These were divided into some 60 language families, with no demonstrable genetic relationship among them. Some of these families spread across several of the seven cultural areas. The Algonquin family, for instance, contained dozens of languages and occupied a vast territory. Speakers of Algonquin languages included the Algonquins of the Eastern Woodland, the Blackfoots of the Plains, and the Wiyots and Yuroks of California. Other language families, like the Zuni family of the Southwest, occupied only a few square miles of area and contained only a single tribal language. 6. What is the main idea of this passage? ___ (A) Each of the cultural areas was dominated by one of the language families. _ _ _ (B) The Zuni language is closely related to the Algonquin language. ___ (C) There is considerable diversity in the size and the number of languages in language families of the North American Indians. ___ (D) Contact with Europeans had an extraordinary effect on the languages of the Indian tribes of North America. Other major changes in journalism occurred around this time. In 1846, Richard Hoe invented the steam cylinder rotary press, making it possible to print newspapers faster and cheaper. The development of the telegraph made possible much speedier collection and distribution of news. Also in 1846, the frrst wire service was organized. A new type of newspaper appeared around this time, one that was more attuned to the spirit and needs of the new America. Although newspapers continued to cover polities, they came to report more human interest stories and to record the most recent news, which they could not have done before the telegraph. New York papers and those of other northern cities maintained corps of correspondents to go into all parts of the country to cover newsworthy events. 7. The main purpose of the passage is to ___ (A) present a brief history of American journalism _ _ _ (B) outline certain developments in mid-19th-century journalism ___ (C) explain the importance of the steam cylinder rotary press ___ (D) present some biographical information about Richard Hoe 8. What is the most probable topic of the paragraph preceding this one? ___ (A) Other types of rotary presses _ _ _ (B) Alternatives to using wire services ___ (C) Newspapers that concentrated on politics ___ CD) Other developments in journalism 9. The tone of the passage could best be described as _ _ (A) objective _ _ _ (B) optimistic _ _ (C) angry ___ (D) humorous In the western third of North America, the conv<iluted folds of the Earth's surface and its fractured geologic structure tend to absorb the seismic energy of an earthquake. Even if an earthquake measuring 8.5 on the Richter scale struck Los Angeles, its force would fade by the time it reached San Francisco, some 400 miles away. But in the eastern two thirds of the continent the same energy travels more easily. The earthquake that struck New Madrid, Missouri, in 1811, estimated at 8 on the Richter scale, shook Washington, D.C., about 800 miles away, and was felt as far as Boston and Toronto. 298 Section 3 • Guide to Reading Comprehension 10. Which of the following best expresses the main idea of this passage? ___ (A) If a major earthquake strikes Los Angeles, it will probably damage San Francisco as well. _ _ _ (B) The New Madrid earthquake of 1811 was felt in Boston and Toronto. _ _ _ (C) The geology of the western United States is much more complex than that of the East. ___ (D) Earthquakes travel farther in the East than in the West. There has never been an adult scientist who has been half as curious as any child between the ages of four months and four years. Adults sometimes mistake this superb curiosity about everything as a lack of ability to concentrate. The truth is that children begin to learn at birth, and by the time they begin formal schooling at the age of five or six, they have already absorbed a fantastic amount of information, perhaps more, fact for fact, than they will learn for the rest of their lives. Adults can multiply by many times the knowledge children absorb if they appreciate this curiosity while Simultaneously encouraging the children to learn. 11. With which of the following statements would the author probably agree? ___ (A) Children lack the ability to concentrate. _ _ _ (B) Young children have a much greater curiosity than adult scientists do. ___ (C) The first few years of school are the most important ones for most children. ___ (D) Adults can use children's intense curiosity to help children learn more. 12. The paragraph following this one most likely deals with ___ (A) ways in which adults can help children learn by stimulating their curiosity _ _ _ (B) the learning habits of children over the age of four ___ (C) the methods adult scientists use to study the curiosity of young children ___ (D) ways in which adults can become as curious as children about their environments Settlement houses were institutions established to improve living conditions in poor city neighborhoods in the late 1800's and early 1900's. They offered health, educational, recreational, and cultural activities. The first to open in the United States was University Settlement in New York City. It was established by the social reformer Stanton Coit in 1886. The most famous example was Hull House, established by the well-known reformer Jane Addams in Chicago in 1890. Settlement houses were usually staffed by idealistic young college graduates who were eager to improve the condition of the poor. 13. The passage mainly discusses ___ (A) U.S. cities in the late nineteenth century _ _ _ (B) the idealism of college graduates ___ (C) settlement houses in the late 1800's and early 1900's ___ (D) the life of several American social reformers The dancer Isadora Duncan was a daring, dynamic innovator in dance. While she was not very successful in teaching her highly personal style of dance to others, she taught a generation of dancers to trust their own forms of expression. She rebelled against the rigid, formal style of classical ballet. Inspired by the art of Greece, she usually danced barefoot in a loose, flowing Greek tunic. She found further inspiration in nature and used dance movements to mirror the waves of the sea and passing clouds. Isadora Duncan was born in San Francisco in 1878. She gave her first performance in 1899. Early failures gave way to triumphant performances in Budapest, Berlin, London, and [mally, in 1908, back in the United States. She lived in Europe most of her life, establishing dancing schools for children there. She died in 1927 near Nice, France, in a freak accident, her long scarf being caught in the wheel of an open sports car in which she was riding. Section 3 • Guide to Reading Comprehension 299 14. The author's attitude toward Isadora Duncan could best be described as one of ___ (A) displeasure ___ (B) admiration ___ (C) compassion ___ (D) amazement 15. Which of the following best describes the organization of the passage? ___ (A) The author first discusses Isadora Duncan's style of dance and then her life history. _ _ _ (B) The first paragraph deals with Isadora Duncan's role as a teacher, the second her role as a performer. ___ (C) The author first discusses Isadora Duncan's shortcomings and then her positive points. ___ (D) First there is an analysis of Isadora Duncan's influences and then of her lasting contributions to dance. Through the centuries, the dream of medieval alchemists was to discover how to turn lead and other "base" metals into gold. Some were fakes, but many were learned men with philosophical goals. Their quest was based on the ancient idea that all matter consists of different proportions of just four substances-earth, water, fire, and air. They believed that it was possible to adjust the proportions of the elements that made up lead by chemical means so that it turned into gold, a process called transmutation. Their experiments were concerned with finding the substance-which they called the philosopher's stone-that, when added to lead, would cause this astonishing change to take place. Alchemists also searched for the elixir of life, a substance that could cure diseases and prolong life. They failed on both counts. However, their techniques for preparing and studying chemicals helped lay the foundation for the modern science of chemistry. 16. Which of the following statements best summarizes the author's attitude toward medieval alchemists? ___ (A) Although they were all fakes, they made important contributions to science. _ _ _ (B) Their discovery of the philosopher's stone was more important than the achievements of modern chemists. ___ (C) Although their theories were sound, they lacked the equipment needed to accomplish their goals. ___ (D) They were unable to realize their goals, but they helped prepare the way for modern chemistry. Exercise 44.3 Focus: Understanding the meaning of multi-paragraph passages by identifying the main point of each of the paragraphs. Directions: Read the following passages and the questions about them. Decide which of the choices best answers the question, and mark the answer. The first one is done as an example. Questions 1-3 In most of Europe, farmers' homes and outbuildings are generally located within a village, and tools and animals are housed there. Every morning, the farmers and farm laborers leave their village to work their land or tend their animals in distant fields, and return to the village at the end of the day. Social life is thus centripetal; that is, it is focused around the community center, the village. Only in certain parts of Quebec has this pattern been preserved in North America. Throughout most of North America, a different pattern was established. It was borrowed from northern Europe, but was pushed even further in the NewWorld where land was cheap or even free. 300 Section 3 • Guide to Reading Comprehension It is a centrifugal system of social life, with large isolated farms whose residents go to the village only to buy goods and procure services. The independence associated with American farmers stems from this pattern of farm settlement. The American farmer is as free of the intimacy of the village as the urbanite. 1. The main topic of the first paragraph is ___ (A) European farm products ___ (B) social life in Quebec ~ (C) the European pattern of rural settlement 2. The main topic ___ (A) the _ _ _ (B) the ___ (C) the of the second paragraph is relative isolation of North American farm families relationship between farmers and urbanites in North America low cost of farmland in North America 3. The main topic of the entire passage is ___ (A) a comparison of farming in northern and southern Europe _ _ _ (B) the difference between farming in Quebec and the rest of North America ___ (C) European influence on American agriculture ___ (D) a contrast between a centripetal system of rural life and a centrifugal system Questions 4-7 While fats have lately acquired a bad image, one should not forget how essential they are. Fats provide the body's best means of storing energy, far more efficient energy sources than either carbohydrates or proteins. They act as insulation against cold, as cushioning for the internal organs, and as lubricants. Without fats, there would be no way to utilize fat-soluble vitamins. Furthermore, some fats contain fatty acids that contain necessary growth factors and help with the digestion of other foods. An important consideration of fat intake is the ratio of saturated fats to unsaturated fats. Saturated fats, which are derived from dairy products, animal fats, and tropical oils, increase the amount of cholesterol in the blood. Cholesterol may lead to coronary heart disease by building up in the arteries of the heart. However, unsaturated fats, derived from vegetable oils, tend to lower serum cholesterol if taken in a proportion twice that of saturated fats. The consumption of a variety of fats is necessary, but the intake of too much fat may lead to a variety of health problems. Excessive intake of fats, like all nutritional excesses, is to be avoided. 4. The main idea of the first paragraph is that ___ (A) fats have a bad image ___ (B) fats serve important functions in the body _ _ _ (C) fats store food more effiCiently than proteins or carbohydrates 5. What is the main idea of the second paragraph? ___ (A) Unsaturated fats may reduce cholesterol levels. _ _ _ (B) The consumption of any type of fat leads to heart disease. ___ (C) Fats taken in the proper proportion may reduce serum cholesterol. 6. The main idea of the third paragraph is that _ _ (A) people are eating less and less fat today ___ (B) fats should be gradually eliminated from the diet ___ (C) excessive consumption of fats may be dangerous to one's health 7. With which of the following is the whole passage primarily concerned? ___ (A) The role of fats in human health _ _ _ (B) The dangers of cholesterol ___ (C) The benefits of fats in the diet ___ (D) The importance of good nutrition Section 3 • Guide to Reading Comprehension 301 Questions 8-10 The term weathering refers to all the ways in which rock can be broken down. It takes place because minerals formed in a particular way (say at high temperatures, in the case of igneous rocks) are often unstable when exposed to various conditions. Weathering involves the interaction of the lithosphere (the earth's crust) with the atmosphere and hydrosphere (air and water). It occurs at different rates and in different ways, depending on the climactic and environmental conditions. But all kinds of weathering ultimately produce broken minerals and rock fragments and other products of the decomposition of stone. Soil is the most obvious and, from the human point of view, the most important result of the weathering process. Soil is the weathered part of the Earth's crust that is capable of sustaining plant life. The character of soil depends on the nature of rock from which it is formed. It also depends on the climate and on the relative "age" of the soil. Immature soils are little more than broken rock fragments. Over time, immature soil develops into mature soil, which contains quantities of humus, formed from decayed plant matter. Mature soil is darker, richer in microscopic life, and more conducive to plant growth. 8. The first paragraph primarily describes ___ (A) the process by which rocks are broken down ___ (B) the weathering of igneous rocks ___ (C) gradual changes in the earth's weather patterns 9. The main topic of the second paragraph is ___ (A) a description of immature soil ___ (B) the growth of plants ___ (C) the evolution of soil 10. The main idea of the entire passage is that ___ (A) weathering breaks down rocks and leads to the development of soil ___ (8) soils may be classified as mature or immature ___ (C) the process of soil development is more important to humans than that of weathering ___ (D) the Earth's crust is constantly changing Questions 11-15 The first Dutch outpost in New Netherlands was made at Fort Orange (now Albany) in 1624; it became a depot of the fur trade. But the most important settlement was at the southern tip of Manhattan, commanding the great harbor at the mouth of the Hudson River. Peter Minuit, first governor-general of New Netherlands, "purchased" title to the island from the Canarsie Indians for the equivalent of twenty-four dollars worth of trinkets. However, the Canarsie Indians might be described as tourists from Brooklyn; Minuit had to make a later payment to the group that was actually resident there. In 1626 engineers from Holland arrived in Manhattan to construct FortAmsterdam. Within its rectangular walls, permanent houses were built, replacing the thatched dwellings of the original Manhattanites. The fort became the nucleus of the town of New Amsterdam. Soon Manhattan had its first skyline: the solid outline of the fort, the flagstaff, the silhouette of a giant windmill, and the masts of trading ships. The Dutch West India company established dairy farms in the vicinity of New Amsterdam. Each morning, the cattle were driven to the "Bouwerie" (now the Bowery), a large open common in the city. Just southwest of the Bouwerie was the Bowling Green, a level area where the burghers played ninepins, the ancestor of modern bowling. The Bowling Green became the site of a cattle fair where livestock were marketed; beer and sausage was available from booths; cheese, lace, and linen were sold by farmers' wives; and Indian women sold baskets and other handicrafts. These colorful gatherings and other aspects of everyday life in New Amsterdam are described in Washington Irving's rollicking book, Diedrich Knickerbocker's History o/New York. 302 Section 3 • Guide to Reading Comprehension The last and most powerful governor-general of New Netherlands was Peter Stuyvesant. famolls for his temper and his wooden leg. He annexed the Swedish colony of Delaware and ordered the streets of ;\few Amsterdam laid out in an orderly manner and numbered. He did his best to obtain military and financial aid from Holland against the British. When the British sent emissaries demanding the surrender of the colony, he wanted to fight. Four British warships, commanded by Colonel Richard Nicolls, sailed into the harbor in 1664. The fort was long out of repair, and there was a shortage of ammunition. Stuyvesant had no choice hut to surrender. New Netherlands became the British colony of New York. and Ne\v Amsterdam became New York City. 11. What is the main topic of the first paragraph;> ___ (A) The first Dutch settlement in New Netherlands ____ (B) Peter Minuit's acquisition of Manhattan ___ (C) Tourism in Manhattan 12. The second paragraph deals primarily with ___ (A) the establishment of Fort Amsterdam ___ (B) the skyline of Manhattan ___ (C) the thatched houses of the Indians 10). The third paragraph mainly describes ___ (A) aspects of everyday life in New Amsterdam ___ (B) the origin of the game of modern bowling ___ (C) Washington Irving's book about New Amsterdam I L What does the fourth paragraph mainly discuss' ___ (A) The annexation of the Swedish colony of Delaware ___ (B) The ordering of the streets in New Amsterdam ___ (C) A description of Peter Stuyvesant and his accomplishments 15. What is the ___ (A) ___ (B) ___ (C) ___ (D) primary topic of the entire passage? A history of the British colony of New York The origin and importance of the Cattle Fair European colonization in the NewWorid Forty years of Dutch rule in New Amsterdam Section 3 • Guide to Reading Comprehension 303 LESSON 45 FACTIJAL QUESTIONS, NEGATIVE QUESTIONS, AND SCANNING QUESTIONS A) Factual Questions Factual questions ask about explicit facts and details given in the passage. They often contain one of the wh- question words: who. what, whell, where, why, holl' much. Factual questions often begin with the phrases "According to the passage, ..." or "According to the author, ..." When you see these phrases, you know that the information needed to answer the question is directly stated somewhere in the passage (unlike answers for inference questions). To answer factual questions, you have to locate and identify the information that the question asks about. If you are not sure from your tIrst reading where to look for specific answers, use the following scanning techniques: • Focus on one or two key words as you read the stem of each question. Lock these words in your mind. • Scan the passage looking for the key words or their synonyms. Look only for these words. Do not try to read every word of the passage. • It may help to use the eraser end of your pencil as a pointer to focus your attention. Don't reread the passage completely-just look for these words. • When you tInd the key words in the passage, carefully read the sentence in which they occur. You may have to read the sentence preceding or following that sentence as well. • Compare the information you read with the four answer choices. The order of detail questions about a passage almost always follows the order in which ideas are presented in the passage. In other words, the information you need to answer the first detail question will usually come near the beginning of the passage; the information for the second will follow that, and so on. Knowing this should help you locate the information you need. Correct answers for detail questions are seldom the same, word for word, as information in the passage; they often contain synonyms and use different grammatical structures. There are generally more factual questions-twelve to eighteen per Reading section-than any other type except (on some tests) vocabulary-in-context questions. B) Negative Questions These questions ask you to determine which of the four choices is not given in the passage. These questions contain the words NOT, EXCEPT. or LEAST (which are always capitalized). • According to the passage, all of the following are true EXCEPT • Which of the following is NOT mentioned in the passage? • Which of the following is the LEAST likely ... Scan the passage to find the answers that ARE correct or ARE mentioned in the passage. Sometimes the three distractors are clustered in one or two sentences; sometimes they are scattered throughout the passage. The correct answer, of course. is the one that does not appear. Negative questions often take more time than other questions. Therefore, you may want to guess and come back to these questions if you have time. There are generally from three to six negative questions per Reading section. 304 Section 3 • Guide to Reading Comprehension C) Scanning Questions These questions ask you to find where in the passage some particular information or transition is located. They are easy to identify: the answers are usually line numbers. They are usually easy to answer too. Scanning questions are often the last question in a set of questions about a passage. To answer these questions, use the same techniques for scanning given in Part A about factual questions. Sample Questions • • • • • In what line does the author shift his focus to ____ Where in the passage does the author first discuss ____ A description of can be found in ... Where in the passage does the author specifically stress ____ In what paragraph does the author first mention the concept of _ _ __ In each Reading section, there are generally from one to three scanning questions. Exercise 45.1 Focus: Scanning passages to locate answers for factual and scanning questions. Directions: For each question, locate that part of the passage in which the answer will probably be found, and write down the line numbers in the blank at the end of the passage. Don't worry about answering the question itself, only about finding the information. The first one is done as an example. Do these scanning exercises as fast as you can. Questions 1-7 (line) (';) (10) (1 ';) (20) Antlers grow from permanent knob like bones on a deer's skull. Deer use their antlers chiefly to fight for mates or for leadership of a herd. Among most species of deer, only the males have antlers, but both male and female reindeer and caribou have antlers. Musk deer and Chinese water deer do not have antlers at all. Deer that live in mild or cold climates lose their antlers each winter. New ones begin to grow the next spring. Deer that live in tropical climates may lose their antlers and grow new ones at other times of year. New antlers are soft and tender. Thin skin grows over the antlers as they develop. Short, fine hair on the skin makes it look like velvet. Full-grown antlers are hard and strong. The velvety skin dries up and the deer rubs the skin off by scraping its antlers against trees. The antlers fall off several months later. The size and shape of a deer's antlers depend on the animal's age and health. The first set grows when the deer is from one to two years old. On most deer, the first antlers are short and straight. As deer get older, their antlers grow larger and form intricate branches. 1. How do deer primarily use their antlers? 2-3 2. In what way are reindeer and caribou different from other types of deer? ____ .1. When do deer that live in temperate climates begin to grow their antlers? ____ 4. According to the article, which of the following does the skin on deer's antlers most closely resemble? _____ Section 3 • Guide to Reading Comprehension 305 5. Which of the following factors influences the size and shape of a deer's antlers? ___ _ 6. At what age do deer get their first antlers? _ _ __ 7. What happens to a deer's antlers as the deer grow older? _ __ Questions 8-13 (line) (')) (10) (15) The trumpet player Louis Armstrong, or Satchmo as he was usually called, was among the first jazz musicians to achieve international fame. He is known for the beautifuL clear tone of his trumpet playing and for his gruff, gravelly singing voice. He was one ofthe first musicians to sing in the scat style, using rhythmic nonsense syllables instead of lyrics. Armstrong was born into a poor family in New Orleans. He first learned to play the cornet at the age of 13, taking lessons while living in a children's home. As a teenager, he played in a number of local jazz bands in New Orleans' rollicking nightlife district, Storyville. In 1922, Armstrong moved to Chicago to play in Joe "King" Oliver's band. Two years later, he joined Fletcher Henderson's band. Then, from 192') to 1928, Armstrong made a series of records with groups called the Hot Five, the Hot Seven, and the Savoy Ballroom Five. These records rank among the greatest recordings in the history of jazz. They include "Cornet Chop Suey,""Potato Head Blues," and "West End Blues." Armstrong led a big band during the 1930's and 1940's, but in 1947 returned to playing \vith small jazz groups. He performed all over the world and made a number of hit records, such as" I !clIo, Dolly" and "Mack the Knife." Armstrong also appeared in a number of movies, first in New Orleans in 1947, High Society in 1956, and Hello, Dolly in 1969. 8. What was Armstrong's nickname? _ _ __ 9. Which of the following phrases best describes Armstrong's singing voice? _ _ __ 10. Where did Armstrong first learn to play the cornet? _ _ __ II. In what city was Joe "King" Oliver's band based' _ _ __ 12. During what period did Armstrong record some of jazz's greatest records? _ _ __ 13. What was the first movie Armstrong appeared in? _.____ Questions 14-23 (line) (5) (10) (15) In 1862, during the Civil War, President Lincoln signed the Morrill Act. The measure was named for its sponsor, Congressman (later Senator) Justin S. Morrill of Vermont. Popularly called the Land Grant Act, it provided each state with 30,000 acres of public land for each senator and each representative it had in Congress. It required that the land be sold, the proceeds invested, and the income used to create and maintain colleges to teach agriculture and engineering. Although not all states used the money as planned in the act, some thirty states did establish new institutions. Purdue University, the University of Illinois, Texas A & M, Michigan State, and the University of California all trace their roots to the Morrill Act. Eighteen states gave the money to existing state universities to finance new agricultural and engineering departments. A few gave their money to private colleges. For example, Massachusetts used much of its funds to endow the Massachusetts Institute of Technology. One state changed its mind. Yale University was chosen to be funded in Connecticut, but farmers protested, and the legislature moved the assets to the University of Connecticut. Most students chose to study engineering. Agriculture was not even considered a science until it had been dignified by the work of research stations. These were established at land-grant institutions in 1887 by the Hatch Act. Gradually, universities broke away from the narrow functions Congress had assigned them and presented a full range of academic offerings, from anthropology to zoology. Today there are some 69 land-grant institutions in all 50 states, the District of Columbia, and Puerto Rico. About one in five college students in the United States attends land-grant schools. 306 Section 3 • Guide to Reading Comprehension 14. When was the Morrill Act signed? _ _ __ IS. Who sponsored the Morrill Act? _ _ __ 16. What position did the sponsor of the Morrill Act have at the time it was passed' _ _ __ 17. How much land did each state receive under the Morrill Act? _ _ __ lB. How many states used the money in the way it was intended by Congress? _ _ __ 19. Which of these states used its money to fund a private university? 20. Who objected to the way the Connecticut legislature initially decided to spend its funds? 21. What was one effect of the Hatch Act of 1BS7? _ _ __ 22. How many land-grant institutions are in operation at present? _ _ __ 23. What percent of college students in the United States currently attend land-grant institutions? Exercise 45.2 Focus: Answering factual, negative, and scanning questions about reading passages. Directions: Read the following passages and the questions about them. Decide which of the choices-(A), (B), (C), or (D)-best answers the question, and mark the answer. The first one is done as an example. Questions 1-9 (line) (5) (10) (15) (20) (25) Mesa Verde is the center of the prehistoric Anasazi culture. It is located in the high plateau lands near Four Corners, where Colorado, Utah, New Mexico, and Arizona come together. This high ground is majestic but not forbidding. The climate is dry, but tiny streams trickle at the bottom of deeply cut canyons, where seeps and springs provided water for the Anasazi to irrigate their crops. Rich red soil provided fertile ground for their crops of corn, beans, squash, tobacco, and cotton. The Anasazi domesticated the wild turkey and hunted deer, rabbits, and mountain sheep. For a thousand years the Anasazi lived around Mesa Verde. Although the Anasazi are not related to the Navajos, no one knows what these Indians called themselves, and so they are commonly referred to by their Navajo name, Anasazi, which means "ancient ones" in the Navajo language. Around 550 A.D., early Anasazi-then a nomadic people archaeologists call the Basketmakersbegan constructing permanent homes on mesa tops. In the next 300 years, the Anasazi made rapid technological advancements, including the refinement of not only basket-making but also potterymaking and weaving. This phase of development is referred to as the Early Pueblo Culture. By the Great Pueblo Period (1100-1300 A.D.), the Anasazi population swelled to over 5,000 and the architecturally ambitious cliff dwellings came into being. The Anasazi moved from the mesa tops onto ledges on the steep canyon walls, creating two and three story dwellings. They used sandstone blocks and mud mortar. There were no doors on the first floor and people used ladders to reach the first roof. All the villages had underground chambers called kivas. Men held tribal councils there and also used them for secret religious ceremonies and clan meetings. Winding paths, ladders, and steps cut into the stone led from the valleys below to the ledges on which the villages stood. The largest settlement contained 217 rooms. One might surmise that these dwellings were built for protection, but the Anasazi had no known enemies and there is no sign of conflict. But a bigger mystery is why the Anasazi occupied these structures such a short time. By 1300, Mesa Verde was deserted. It is conjectured that the Anasazi abandoned their settlements because of drought, overpopulation, crop failure, or some combination of these. They probably moved southward and were incorporated into the pueblo villages that the Spanish explorers encountered two hundred years later. Their descendants still live in the Southwest. Section 3 • Guide to Reading Comprehension 307 I. The passage does l\OT mention that the Anasazi hunted _ _ (A) sheep ~ (B) turkeys ___ (C) deer _ _ _ (D) rabbits 2. The name that the Anasazi used for themselves ____ (A) means "basketmakers" in the l\avajo language ___ (B) is unknown today _ _ _ (C) was given to them by archaeologists ._ _ _ (D) means "ancient ones" in the Anasazi language 3. How long did the Early Pueblo Culture last? . _ _ (A) 200 years ____ (B) 300 years _ _ (C) 5'50 years _ _ _ (D) 1000 years 4. Where did the Anasazi move during the Great Pueblo Period? ___ (A) To settlements on ledges of canyon "valls _ _ _ (B) To pueblos in the south _ _ _ (C) Onto the tops of the mesas _ _ _ (D) Onto the floors of the canyons 5. According to the passage, the Anasazi buildings were made primarily of ___ (A) mud _ _ _ (B) blocks of wood _ _ _ (C) sandstone _ _ (D) the skins of animals 6. According to the passage, the Anasazi entered thdr buildings on the ledges __ ~ (A) by means of ladders _ _ _ (B) from underground chambers ___ (C) by means of stone stairways _ _ (D) through doors on the first floor 7. According to the passage, kivas were used for all the following purposes EXCEPT _ _ (A) clan meetings _ _ _ (B) food preparation _ (C) religiOUS ceremonies _ _ _ (D) tribal councils 8. According to the passage, the LEAST likely reason that the Anasazi abandoned ~\1esa Verde was _ _ _ (A) drought _ _ _ (B) overpopulation _ _ . (C) war ____ (D) crop failure 9. Where in the passage does the author mention specific accomplishments of the Basketmakersr ___ (A) Lines 7-9 ___ (B) Lines 10-13 ___ (C) Lines 14-15 _ _ _ (D) Lines 23-25 308 Section 3 • Guide to Reading Comprehension Questions 10-15 (line) (5) (10) Dulcimers are musical instruments that basically consist of wooden boxes with strings stretched over them. In one form or another, they have been around since ancient times, probably originating with the Persian santir. Today there are two varieties: the hammered dulcimer and the Appalachian, or mountain dulcimer. The former is shaped like a trapezoid, has two or more strings, and is played with wooden mallets. It is the same instrument played in a number of Old World countries. The Appalachian dulcimer is classified by musicologists as a box zither. It is a descendant ofthe Pennsylvania Dutch scheitholt and the French epinette. Appalachian dulcimers are painstakingly fashioned by artisans in the mountains of West Virginia, Kentucky, Tennessee, and Virginia. These instruments have three or four strings and are plucked with quills or the fingers. They are shaped like teardrops or hourglasses. Heart-shaped holes in the sounding board are traditional. Most performers play the instruments while seated with the instruments in their laps, but others wear them around their necks like guitars or place them on tables in front of them. Originally used to play dance music, Appalachian dulcimers were popularized by performers such as John Jacob Niles and Jean Ritchie during the folk music revival of the 1960's. 10. According to the passage, a hammered dulcimer is made in the shape of ___ (A) an hourglass _ _ _ (B) a heart _ _ _ (C) a trapezoid ___ (D) a teardrop 11. According to the passage, which of the following is NOT an ancestor of the Appalachian dulcimer? ___ (A) The box zither ___ (B) The santir ___ (C) The scheitholt ___ (D) The epinette 12. According to the passage, how many strings does the Appalachian dulcimer have? ___ (A) One or two ___ (B) Three or four ___ (C) Four or five ___ (D) Six or more 13. According to the author, most performers play the Appalachian dulcimer ___ (A) while sitting down ___ (B) with the instrument strapped around their neck ___ (C) while standing at a table ___ (D) with wooden hammers 14. According to the author, what are John Jacob Niles and Jean Ritchie known for? ___ (A) Played dance music on Appalachian dulcimers ___ (B) Are artisans who design Appalachian dulcimers ___ (C) Helped bring Appalachian dulcimers to the public's attention ___ (D) Began the folk music revival of the 1960's 15. Where in the passage does the author describe the hammered dulcimer? _ _ (A) Lines 1-2 _ _ (B) Lines 4-5 ___ (C) Lines 9-] 0 _ _ (D) Lines 13-14 Section 3 • Guide to Reading Comprehension 309 Questions 16-20 (line) (5) ( 10) (15) Humanitarian Dorothea Dix was born in Hampden, Maine, in 1802. At the age of 19, she established a school for girls, the Dix Mansion School, in Boston, but had to close it in 1835 due to her poor health. She wrote and published the first of many books for children in 1824. In 1841, Dix accepted an invitation to teach classes at a prison in East Cambridge, Massachusetts. She was deeply disturbed by the sight of mentally ill persons thrown in the jail and treated like criminals. For the next 18 months, she toured Massachusetts institutions where other mental patients were confined, and reported the shocking conditions she found to the state legislature. When improvements followed in Massachusetts, she turned her attention to the neighboring states and then to the West and South. Dix's work was interrupted by the Civil War; she served as superintendent of women hospital nurses for the federal government. Dix saw special hospitals for the mentally ill built in some 15 states. Although her plan to obtain public land for her cause failed, she aroused concern for the problem of mental illness all over the United States as well as in Canada and Europe. Dix's success was due to her independent and thorough research, her gentle but persistent manner, and her ability to secure the help of powerful and wealthy supporters. 16. In what year was the Dix Mansion School closed? _ _ (A) 1802 _ _ (B) 1824 _ _ (C) 1835 _ _ _ (D) 1841 17. Why did Dorothea Dix first go to a prison? ___ (A) She taught classes there. _ _ _ (B) She was sent there by the state legislature. ___ (C) She was convicted of a crime. ___ (D) She was doing research for a book. 18. Where was ___ (A) ___ (B) ___ (C) ___ (D) Dorothea Dix first able to bring about reforms in the treatment of the mentally ill? Canada Massachusetts The West and South Europe 19. Dorothea Dix was NOT successful in her attempt to ___ (A) become superintendent of nurses _ _ _ (B) publish books for children _ _ _ (C) arouse concern for the mentally ill ___ (D) obtain public lands 20. At what point of the passage does the author discuss specific reasons for Dix's success? _ _ (A) Lines 1-3 ___ (B) Lines 4-5 ___ (C) Lines 10-11 _ _ (D) Lines 15-16 310 Section 3 • Guide to Reading Comprehension Questions 21-26 (line) (5) (10) (15) A quilt is a bed cover made of squares of material pieced together. Each square consists of two layers filled with a layer of wool or cotton cloth, feathers, or down. Often, the squares are decorated with fancy stitches and designs. According to legend, the earliest pieced quilt was stitched in 1704 by Sarah Sedgewick Everett, wife of the governor of the Massachusetts colony. By 1774 George Washington was buying quilts in Belvoir, Virginia, to take back to Martha in Mount Vernon. As the frontier moved westward, quilting went along. In addition to sleeping under them, homesteaders kept out drafts by hanging quilts over doors and windows. And if the money ran out, quilts were used to pay debts. For isolated pioneer women, quilts were a source of comfort. Mary Wilman, whose family moved to Texas from Missouri in 1890, recalled the first time she and her mother had to spend a week alone and a dust storm came up. "The wind blew for three days and the dust was so thick that you couldn't see the barn. My mother quilted all day, and she taught me how to quilt. If it hadn't been for quilting, I think we would have gone crazy." Quilting provided an important social function for the women of the frontier as well. At quilting bees, women met to work on quilts and to share the latest news. Today, however, the homely quilt has become a costly cultural phenomenon. The International Quilt Festival in Houston, Texas, the "world's fair of quilting;' attracted only 2,500 people and displayed only 200 quilts when it began a dozen years ago. This year there were over 20,000 visitors and 5,000 quilts, some of which sold for as much as $50,000. 2l. According to legend, who made the first American quilt? ___ (A) Sarah Sedgewick Everett ___ (B) the governor of the colony of Massachusetts ___ (C) Martha Washington _ _ (D) MaryWilman 22. Which of the following is NOT mentioned in the passage as one of the benefits of quilts for pioneers? ___ (A) They could be used to pay debts. ___ (8) They could be used to help insulate houses. ___ (C) They could provide psychological comfort. ___ (D) They could be worn as warm clothing. 23. According to the passage, what is a "quilting bee?" ___ (A) A type of insect ___ (8) A gathering where women socialized and made quilts _ _ (C) A type of quilt ___ (D) A place where people buy and sell quilts 24. In what state is the International Quilt Festival held? ___ (A) Massachusetts ___ (B) Texas _ _ _ (C) Virginia ___ (D) Missouri 25. How many quilts were displayed at the first International Quilt Festival? _ _ (A) 200 _ _ (B) 2,500 _ _ (C) 5,000 _ _ (D) 20,000 Section 3 • Guide to Reading Comprehension 311 26. Where in the passage does the author first begin to discuss the way in which the public's perception of quilts has changed in modern times? _ _ (A) Lines 6-8 _ _ (B) Line 9 _ _ (C) Lines 14-15 _ _ (D) Line 16 Questions 27-32 (line) (5) (10) Ambient divers, unlike divers who go underwater in submersible vehicles or pressure-resistant suits, are exposed to the pressure and temperature of the surrounding (ambient) water. Of all types of diving, the oldest and simplest is free diving. Free divers may use no equipment at all, but most use a face mask, foot fins, and a snorkel. Under the surface, free divers must hold their breath. Most free divers can descend only 30 to 40 feet, but some skilled divers can go as deep as 100 feet. Scuba diving provides greater range than free diving. The word scuba stands for self-contained underwater breathing apparatus. Scuba divers wear metal tanks with compressed air or other breathing gases. When using open-circuit equipment, a scuba diver simply breathes air from the tank through a hose and releases the exhaled air into the water. A closed-circuit breathing device, also called a rebreather, filters out carbon dioxide and other harmful gases and automatically adds oxygen. This enables the diver to breathe the same air over and over. In surface-supplied diving, divers wear helmets and waterproof canvas suits. Today, sophisticated plastic helmets have replaced the heavy copper helmets used in the past. These divers get their air from a hose connected to compressors on a boat. Surface-supplied divers can go deeper than any other type of ambient diver. 27. Ambient divers are ones who ___ (A) can descend to extreme depths _ _ _ (B) use submersible vehicles _ _ _ (C) use no equipment _ _ _ (D) are exposed to the surrounding water 28. According to the passage, a free diver may use any of the following EXCEPT ___ (A) a rebreather _ _ _ (B) a snorkel ___ (C) foot fins _ _ _ (D) a mask 29. According to the passage, the maximum depth for free divers is around _ _ (A) 40 teet _ _ (B) 100 feet _ _ (C) 200 feet _ _ (D) 1,000 feet 30. When using closed-circuit devices, divers ___ (A) exhale air into the water _ _ (B) hold their breath ___ (C) breathe the same air over and over ___ (D) receive air from the surface 31. According to the passage, surface-supplied divers today use helmets made from _ _ (A) glass _ _ (B) copper ___ (C) plastic ___ (D) canvas 312 Section 3 • Guide to Reading Comprehension 32. Where in the passage does the author mention which type of diver can make the deepest descents? _ _~ (A) Lines 2-3 _ _ (B) Lines 6-8 _ _ (C) Lines 9-10 _ _ (D) Lines 14-15 Section 3 • Guide to Reading Comprehension 313 LESSON 46 INFERENCE QUESTIONS AND PURPOSE QUESTIONS A) Inference Questions As in the Listening Comprehension section. there are questions in the Reading Comprehension section that require you to make inferences. The answers to these questions are not directly provided in the passage-you must "read between the lines." In other words, you must make conclusions based indirectly on information in the passage. Many test-takers find these questions the most difficult type of Reading question. Inference questions may be phrased in a number of ways. Many of these questions contain some form of the words infer or imply. • Which of the following can be inferred from the passage? • It can be inferred from the passage that ... • The author implies that ... • Which of the following does the passage imply? • Which of the following would be the most reasonable guess about _ __ • The author suggests that ... • It is probable that ... There will probably be from five to eight of these questions per Reading section. ... Sample Item A star very similar to the Sun is one of the nearest stars to Earth. That star is Alpha Centauri, just 4.3 light-years away. Other than our own Sun, the nearest star to the Earth is a tiny red star, not visible without a telescope, called Proxima Centauri. It can be inferred from this passage that (A) Proxima Centauri is similar to the Earth's Sun. (R) Proxima Centauri is the closest star to the Earth. (C) Alpha Centauri is invisible from the Earth. (D) Proxima Centauri is less than 4.3 light-years from the Earth. Choice (A) is not a valid inference; Alpha Centauri is similar to the Sun, but Proxima Centauri is "a tiny red star." Choice (B) also cannot be inferred; the closest star to the Earth is our own Sun. Nor can (C) be inferred; Proxima Centauri is invisible, but there is no information as to whether Alpha Centauri is. Since Alpha Centauri is 4.3 light-years away, it can be inferred that Alpha Centauri, the closest star, is less than that. ... B) Purpose Questions These questions ask why the author of a passage mentions some piece of information, includes a quotation from a person or a study, or uses some particular word or phrase. 314 Section 3 • Guide to Reading Comprehension Sample Questions • • • • Why does the author mention ____ ? The author refers to to indicate that ... The author quotes in order to show ... The phrase _____ in line is mentioned to illustrate the effect of ... Sample Answer Choices • • • • • To To To To To strengthen the argument that _ _ __ provide an example of _ _ __ challenge the idea that _ __ contradict _ _ __ support the proposal to _ _ __ There are usually from one to four purpose questions per Reading section. Exercise 46.1 Focus: Identifying valid inferences based on sentences. Directions: Read each sentence; then mark the one answer choice-(A), (B), or (C)-that is a valid inference based on that sentence. The first one is done as an example. 1. Cities founded around the turn of the eighteenth century, such as Williamsburg, Annapolis, and especially Philadelphia, were laid out on a regular grid with public squares, while cities laid out in the mid-seventeenth century, such as Boston, remain chaotic to this day. ___ (A) Philadelphia is today laid out more regularly than either Williamsburg or Annapolis. ~ (B) Boston was not originally laid out according to a logical plan. ___ (C) Philadelphia, Williamsburg, and Annapolis were founded before Boston~ 2. When apple growers talk about new varieties of apples, they don't mean something developed last month, last year, or even in the last decade. ___ (A) Apple growers haven't developed any new varieties in recent decades. ___ (B) Some varieties of apples can be developed in a short time, but others take a long time. ___ (C) New varieties of apples take many years to develop. 3. Blood cholesterol used to be thought of as a problem only for adults. ___ (A) Blood cholesterol is no longer a problem for adults. ___ (B) Only children have a problem with blood cholesterol. ___ (C) Blood cholesterol affects both adults and children. 4. A metal-worker of 3,000 years ago would recognize virtually every step of the lost-wax process used to cast titanium for jet engines. ___ (A) Titanium has been forged for thousands of years. ___ (B) The lost-wax method of casting is very old. _ _ _ (C) Metal working has changed very little in 3,000 years. 5. There is more quartz in the world than anyone kind of feldspar, but the feldspars as a group are five times more common than quartz. ___ (A) One type of quartz is five times more plentiful than feldspar. ___ (B) Quartz is less common than the feldspars. ___ (C) The most common type of feldspar is as plentiful as quartz. Section 3 • Guide to Reading Comprehension 315 6. Compared with the rest of its brain, the visual area of a turtle's brain is comparatively small since turtles, like all other reptiles, depend on senses other than sight. ___ (A) No reptile uses sight as its primary sense. ___ (B) Animals that depend on sight all have larger visual areas in their brain than turtles do. ___ (C) The visual areas of other reptile brains are comparatively smaller than those of turtles. 7. Contrary to popular belief, there is no validity to the stories one hears of initials carved in a tree by a young boy becoming elevated high above his head when he visits the tree as an old man. ___ (A) Trees don't grow the way many people think they do. ___ (B) If a child carves initials in a tree, it won't grow. ___ (C) Over time, initials that are carved into a tree will be elevated. 8. Illegible handwriting does not indicate weakness of character, as even a quick glance at the penmanship of George Washington, Franklin D. Roosevelt, or John Kennedy reveals. ___ (A) Washington, Roosevelt, and Kennedy all had handwriting that was difficult to read. ___ (B) A person's handwriting reveals a lot about that person. ___ (C) The author believes that Washington, Roosevelt, and Kennedy all had weak characters. 9. William Faulkner set many of his novels in and around an imaginary town, Jefferson, Mississippi, which he closely patterned after his hometown of Oxford, Mississippi. ___ (A) William Faulkner wrote many of his novels while living in Jefferson, Mississippi. ___ (B) The town of Oxford, Mississippi, exists only in Faulkner's novels. ___ (C) Faulkner actually wrote about his hometown but did not use its real name. 10. Most fish take on, to a certain degree, the coloration of their natural surroundings, so it is not surprising that the fish inhabiting warm, shallow waters around tropical reefs are colored all the brilliant tints of the rainbow. ___ (A) Tropical fish are unlike other fish because they take on the coloration of their environment. ___ (B) Tropical fish are brightly colored because they inhabit warm waters. ___ (C) Tropical reefs are brightly colored environments. 11. Although sheepherding is an older and more beloved occupation, shepherds never caught the attention of American filmmakers the way cowboys did. ___ (A) There have been more American films about cowboys than about shepherds. ___ (B) Films about shepherds were popular before films about cowboys. ___ (C) Cowboys are generally younger than shepherds. 12. The Okefenokee Swamp is a fascinating realm that both confirms and contradicts popular notions of a swamp, because along with huge cypresses, dangerous quagmires, and dim waterways, the Okefenokee has sandy pine islands, sunlit prairies, and clear lakes. ___ (A) People generally feel that swamps are fascinating places. ___ (B) The Okefenokee has features that most people do not associate with swamps. ___ (C) Most swamps do not have huge cypresses, dangerous quagmires, and dim waterways. 13. As an architect, Thomas Jefferson preferred the Roman style, as seen in the University of Virginia, to the English style favored by Charles Bullfinch. ___ (A) The University of Virginia was influenced by the Roman style. ___ (B) Bullfinch was an English architect. ___ (C) Jefferson preferred to build in the English style of architecture. 14. In all cultures, gestures are used as a form of communication, but the same gestures may have very different meanings in different cultures. ___ (A) No two cultures use the same gestures. ___ (B) One gesture will never have the same meaning in two cultures. ___ (C) A person from one culture may misunderstand the gestures used by a person from another culture. 316 Section 3 • Guide to Reading Comprehension 15. Even spiders that do not build webs from silk use it for a variety of purposes, such as constructing egg sacs and nursery tents. _ _ (A) All spiders build webs. ___ (B) Spiders that build webs don't build egg sacs or nursery tents. _ _ (C) Silk is used by all spiders. Exercise 46.2 Focus: Recognizing valid inferences based on longer passages. Oi rections: Read the passages. If the statements following the passages are valid inferences based on those passages, mark the items I. If the statements qlOnot be inferred from the passage, mark those items X. The first one is done as an example. Questions 1-7 (line) (';) (10) ~ The term "neon light" was originally applied to a particular type of vapor lamp using the inert gas neon. A long tube was filled with neon, which then became luminous at low pressure when an electric current was passed through it. The lamp then emitted the characteristic reddish-orange light of neon. Today, the term "neon light" is given to lamps of this general type which may be filled with a variety of gases, depending on the color that is desired. Argon, for example, is used to produce blue light. Colors can also be altered by changing the color of the glass tube. The tubes must be quite long in all these lamps to produce light efficiently. As a result, high voltages are required. Neon tube lamps are not practical for indoor illumination, but they have found widespread outdoor use in glowing, colorful advertising signs. 1. The inert gas neon is reddish-orange in color. 2. The meaning of the term "neon light" has changed over time. 3. Today's "neon lights" never actually contain neon. ___ 4. All types of "neon lights" work on the same general principles. S. When stimulated by electricity, different types of gas produce different colors. 6. Modern "neon lights" are more efficient than those used in the past. ___ 7. The primary market for neon lights is businesses rather than private households. Section 3 • Guide to Reading Comprehension 317 Questions 8-15 (line) (';) (10) (l ';) (20) Natural flavorings and fragrances are often costly and limited in supply. For example, the vital ingredient in a rose fragrance is extracted from natural rose oil at a cost of thousands of dollars a pound; an identical synthetic substance can be made for 1% of this cost. Since the early twentieth century, success in reproducing these substances has created a new industry that today produces hundreds of artificial flavors and fragrances. Some natural fragrances arc easily synthesized; these include vanillin, the aromatic ingredient in vanilla, and benzaldehyde, the aromatic ingredient in wild cherries. Other fragrances, however, have dozens, even hundreds of components, Only recently has it been possible to separate and identify these ingredients by the use of gas chromatography and spectroscopy. Once the chemical identity is known, it is often possible to synthesize them, Nevertheless, some complex substances, such as the aroma of fresh coffee, have still not been duplicated satisfactorily. '\1any of the chemical compounds making up these synthetics arc identical to those found in nature, and are as harmless or harmful as the natural substances. New products must be tested for safety. and when used in food, must be approved by the ITS. Food and Drug Administration. The availability of synthetic flavors and fragrances has made possible a large variety of products, from inexpensive beveragcs to perfumed soap to used cars with applied "new car odor," 8. Natural rose fnlgrance is 100 times more expensive to produce than artificial rose fragrance, l). Vanillin is easier to synthesize than benzaldehyde, 10. In general, the more components there are in a fragrance, the harder it is to synthesize. 11, Once a substance has been chemically analyzed, it can al\vays be easily synthesized. 12. Only recently has it been possible to satisfactorily synthesize the aroma of fresh coffee. 13 ' Not all synthetic flavors are harmless. 14. Synthesized substances mllst be tested for safety only if the\" are used in food. 15. Synthetic fragrances can be used to make a used car smell like a new one. 318 Section 3 • Guide to Reading Comprehension Questions 16-20 (line) (';) (10) A legend is a popular type of folk tale. In some ways, legends resemble myths, another type of folk tale. But myths describe events from antiquity and usually deal with religious subjects, such as the birth of a god. Legends tell of recognizable people, places, and events and often take place in comparatively recent times. Some legends are based on real persons or events, but many are entirely fictional. The legends of the superhuman accomplishments of Paul Bunyan and Pecos Bill are imaginary, while the legends about Washington and Lincoln are mostly exaggerations of real qualities those two presidents had. All societies have legends. Most legends began as stories about the heroes of a particular region, occupation, or ethnic group. For example, John Henry was a legendary hero of black Americans, and Casey Jones of railroad workers. Over time, however, these figures have become national heroes. 16. Both legends and myths can be classified as folk tales. 17. Myths generally take place in comparatively recent times. 18. The stories of Paul Bunyan and Pecos Bill are not true, but they are based on actual people. 19. Legends about Washington and Lincoln are not entirely fictional. 20. John Henry and Casey Jones are today well-known only by certain groups of people. Exercise 46.3 Focus: Answering inference and purpose questions. Directions: Read the following passages and the questions about them. Decide which of the choices-(A), (B), (C), or (D)-best answers the question, and mark the answer. The first one is dom: as an example. Questions 1-4 (line) (5) (10) Pigeons have been taught to recognize human facial expressions, upsetting long-held beliefs that only humans had evolved the sophisticated nervous systems to perform such a feat. In recent experiments at the University of Iowa, eight trained pigeons were shown photographs of people displaying emotions of happiness, anger, surprise, and disgust. The birds learned to distinguish between these expressions. Not only that, but they were also able to correctly identify the same expressions on photographs of unfamiliar faces. Their achievement does not suggest, of course, that the pigeons had any idea what the human expressions meant. Some psychologists have theorized that because of the importance of facial expression to human communication, humans developed special nervous systems capable of recognizing subtle expressions. The pigeons cast doubt on that idea, however. Section 3 • Guide to Reading Comprehension (15) (20) 319 In fact, the ability to recognize facial expressions of emotion is not necessarily innate even in human babies, but may have to be learned in much the same way pigeons learn. In experiments conducted several years ago at the University of Iowa, it was found that pigeons organize images of things into the same logical categories that humans do. None of this work would come as any surprise to Charles Darwin, who long ago wrote about the continuity of mental development from animals to humans. 1. From the passage, which of the following can be inferred about pigeons' ___ (A) They can show the same emotions humans can. ___ (B) They can understand human emotions. ___ (C) They can only identify the expressions of people they are familiar with. ~ (D) They have more sophisticated nervous systems than was once thought. 2. The passage implies that, at birth, human babies ___ (A) have nervous systems capable of recognizing subtle expressions ___ (B) can learn from pigeons _ _ _ (C) are not able to recognize familiar faces ___ (D) may not be able to identify basic emotions through facial expressions 3. Why does the author mention the experiments conducted several years ago at the University of Iowa? ___ (A) They proved that pigeons were not the only kind of animal with the ability to recognize facial expressions. ___ (B) They were contradicted by more recent experiments. _ _ _ (C) They proved that the ability to recognize human expressions was not innate in human babies. ___ (D) They showed the similarities between the mental organization of pigeons and that of humans. 4. If Charles Darwin could have seen the results of this experiment, his most probable response would have been one of ___ (A) rejection ___ (B) surprise _ _ _ (C) agreement ___ (D) amusement Questions 5-7 (line) (5) (10) The spectacular eruptions of Old Faithful geyser in Yellowstone National Park do not occur like clockwork. Before the earthquake of 1959, eruptions came every 60 to 65 minutes; today they are as little as 30 minutes or as much as 90 minutes apart. The geyser usually gives a warning: a short burst of steam. Then a graceful column rises up to 150 feet in the air. The water unfurls in the sunlight with the colors of the rainbow playing across it. This eruption is only the visible part of the spectacle. The geyser is linked by an intricate plumbing network to some extremely hot rocks. As water seeps into the underground system, it is heated at the bottom like water in a tea kettle. But while water in a kettle rises because of convection, the narrow tubes of the geyser system prevent free circulation of the water. Thus, 320 Section 3 • Guide to Reading Comprehension the water in the upper tubes is far cooler than the water at the bottom. The weight of the water puts pressure on the column, and this raises the boiling point of the water near the bottom. Finally. the water in the upper part of the column warms and expands. some of it welling out of the mouth of the geyser. This decreases the pressure on the superheated water, which abruptly turns to steam. This in turn forces all the water and vapor out of the geyser. ( I "i) (20) '). It can be inferred from the passage that the earthquake of 1959 made Old Faithful geyser erupt ___ ___ ___ ___ (A) (ll) (C) (D) more frequently less regularly more suddenly less spectacularly 6. Why does the author mention a rainbow in line 7? _. ._ (A) The column of water forms an arc in the shape of a rainbow. ___.. (B) In the sunlight, the column of water may produce the colors of the rainbow. _ _ _ (C) Rainbows can be seen quite frequently in Yellowstone National Park. ____ . (D) The rainbow. like the geyser. is an example of the beauty of nature. - The passage implies that Old Faithful would probably not erupt at all if ___ (A) the tuhes of the geyser system were very wide _. _ _ (ll) the climate suddenly changed ___ (C) there had not been an earthquake in 1959 ___ (D) the underground tubes were longer Questions 8-12 (line) ("i) (10) (1'1) (20) In 1881, a new type of weed began spreading across the northern Great Plains. Unlike other weeds, the tumbleweed did not spend its life rooted to the soil; instead it tumbled and rolled across fields in the wind. The weed had sharp, spiny leaves that could lacerate the f1esh of ranchers and horses alike. It exploited the vast area of the plains, thriving in regions too barren to support other plants. With its ability to generate and disseminate numerous seeds qUickly, it soon became the scourge of the prairies. To present-day Americans, the tumbleweed symbolizes the Old West. They read the Zane Grey novels in which tumbleweeds drift across stark western landscapes and see classic western movies in which tumbleweeds share scenes with cowboys and covered wagons. Yet just over a century ago, the tumbleweed was a newcomer. The first sign of the invasion occurred in North and South Dakota in the late 1870's. Farmers had noticed the sudden appearance of the new. unusual weed. One group of immigrants, however, did not find the weed at all unfamiliar. The tumbleweed. it turns out, was a native of southern Russia. where it was known as Tartar thistle. It was imported to the l'nited States by unknown means. Section 3 • Guide to Reading Comprehension (25) 321 Frontier settlers gave the plants various names: saltwort, Russian cactus, and wind witch. But botanists at the Department of Agriculture preferred the designation Russian thistle as the plant's common name. However, these botanists had a much harder time agreeing on the plant's scientific name. Generally, botanists compare a plant to published accounts of similar plants, or to samples kept as specimens. Unfortunately, no book described the weed, and no samples existed in herbaria in the United States. 8. Which of the following can be inferred about tumbleweeds? ___ (A) They have strong, deep roots. ___ (B) They require a lot of care. _ _ _ (C) They reproduce efficiently. ___ (D) They provided food for ranchers and animals. 9. The passage suggests that most present-day Americans ___ (A) consider the tumbleweed beneficial ___ (B) don't know when tumbleweeds came to North America _ _ _ (C) have never heard of tumbleweeds ___ (D) believe tumbleweeds are newcomers to the United States lO. The author mentions the novels of Zane Grey and classic western movies (lines 11-12) because they ___ (A) tell the story of the invasion of tumbleweeds ___ (B) are sources of popular information about tumbleweeds _ _ _ (C) present very inaccurate pictures of tumbleweeds ___ (D) were written long before tumbleweeds were present in the United States 11. It is probable that the "group of immigrants" mentioned in line 18 ___ (A) was from southern Russia ___ (B) had lived in North and South Dakota for many years _ _ _ (C) imported tumbleweeds into the United States ___ (D) wrote a number of accounts about tumbleweeds 12. From the passage it can be inferred that the botanists at the Department of Agriculture ___ (A) could not find any tumbleweeds on the plains ___ (B) gave the names saltwort, Russian cactus, and wind witch to the tumbleweed _ _ _ (C) could not decide on a common designation for the tumbleweed ___ (D) found it difficult to classify the plant scientifically Questions 13-17 (line) (5) (lO) For most modern airports, the major design problem is scalehow to allow adequate space on the ground for maneuvering widebody jets while permitting convenient and rapid movement of passengers departing, arriving, or transferring from one flight to another. Most designs for airport terminals take one of four approaches. In the linear plan, the building may be straight or curved. The passengers board aircraft parked next to the terminal. This plan works well for small airports that need to provide boarding areas for only a few aircraft at a time. In the pier plan, narrow corridors or piers extend from a central building. This plan allows many aircraft to park next to the building. However, it creates long walking distances for passengers. 322 Section 3 • Guide to Reading Comprehension (15) (20) In the satellite plan, passengers board aircraft from small terminals that are separated from the main terminals. Passengers reach the satellites by way of shuttle trains or underground passageways that have shuttle trains or moving sidewalks. The transporter plan employs some system of transport to move passengers from the terminal building to the aircraft. If buses are used, the passengers must climb a flight of stairs to board the aircraft. If mobile lounges are used, they can link up directly with the aircraft and protect passengers from the weather. 13. It can be inferred that scale would not pose a major design problem at airports if ___ (A) airports were larger ___ (B) aircraft did not need so much space to maneuver on the ground ___ (C) other forms of transportation were more efficient ___ (D) airplanes could fly faster 14. The linear plan would probably be best at ___ (A) a busy airport ___ (B) an airport used by many small aircraft ___ (C) an airport with only a few arrivals or departures ___ (D) an airport that serves a large city 15. The passage implies that the term "satellite plan" is used because ___ (A) satellites are launched and tracked from these sites ___ (B) small terminals encircle the main terminal like satellites around a planet ___ (C) the plan makes use of the most modern, high-technology equipment ___ (D) airports that make use of this plan utilize data from weather satellites 16. The passage suggests that shuttle trains transfer passengers to satellite terminals from ___ (A) the main terminal ___ (B) airplanes ___ (C) downtown ___ (D) other satellite terminals 17. It can be inferred that mobile lounges would be more desirable than buses when ___ (A) passengers are in a hurry ___ (B) flights have been delayed _ _ _ (C) the weather is bad ___ (D) passengers need to save money Section 3 • Guide to Reading Comprehension Questions 18-20 (line) (5) (10) (15) The sea has been rising relative to the land for at least 100 years, geologists say. During that same period, the Atlantic Coast has eroded an average of 2 to 3 feet per year, the Gulf Coast even faster. Many engineers maintain that seawalls and replenished beaches are necessary to protect the nation's shoreline. Too many people live or vacation in Miami Beach, Atlantic City, or Martha's Vineyard to allow their roads and buildings to simply fall into the sea. The problem with seawalls is that they simply don't work. One study has shown that, in fact, seawalls accelerate the erosion of beaches. Faced with the loss of their beaches, other communities have tried a simple but expensive solution: replace the lost sand. These replenishment programs, however, are costly and of dubious value. Another study has shown that only 10% of replenished beaches lasted more than 5 years. 18. It can be inferred from the passage that the author ___ (A) opposes the use of both seawalls and beach replenishment ___ (B) believes beach replenishment would be more effective than seawalls ___ (C) opposes any actions to protect the shoreline ___ (D) denies that beach erosion is a problem 19. Why does the author mention Miami Beach, Atlantic City, and Martha's Vinyard? ___ (A) These are communities with seawalls. ___ (B) These are communities that have implemented replenishment programs. ___ (C) These are communities in danger of beach erosion. ___ CD) These are communities which have lost roads and buildings to erosion. 20. The author quotes the two studies in the passage in order to ___ (A) suggest that the sea is not rising as fast as was originally believed ___ (ll) strengthen the engineers' contention that seawalls and replenished beaches are necessary ___ (C) propose two new solutions to beach erosion ___ (D) support his own position 323 324 Section 3 • Guide to Reading Comprehension LESSON 47 VOCABULARY-IN-CONTEXT QUESTIONS In vocabulary-in-context questions, you must determine which of four words or phrases can best substitute for a word or words in the passage. Most of the questions ask about single words (usually nouns, verbs, adjectives, and adverbs). Some ask about two- or three-word phrases. Sometimes two or more of the answer choices for these items might be "correct" definitions of the word that is asked about. In those cases, you must decide which is correct in the context of the passage. There are a number of clues that can help you determine the meaning of an unknown word: • Synonyms The first state to institute compulsory education was Massachusetts, which made it mandatory for students to attend school 12 weeks a year. The word mandatory is a synonym for the word compulsory. • Examples Many gardeners use some kind of mulch, such as chopped leaves, peat moss, grass clippings, pine needles, or wood chips, in order to stop the growth of weeds and to hold in moisture. From the examples given, it is clear that mulch is plant matter. • Contrast In the 1820's the Southern states supported improvements in the national transportation system, but the Northern states balked. Since the Southern states supported improvements, and since a word signaling contrast (but) is used, it is clear that the Northern states disagreed with this idea, and that the word balked must mean objected or refused. • General Context In a desert, vegetation is so ~ as to be incapable of supporting any large human population. As is generally known, deserts contain little vegetation, so clearly the word scanty must mean scarce or barely sufficient. When answering vocabulary-in-context questions, you mllst often depend on the general context of the sentence to help you choose the correct answer. You should follow these steps to answer vocabulary-in-context items: 1. Look at the word being asked about and the four answer choices. If you are familiar with the word, guess which answer is correct. Do not mark your answer sheet yet. 2. Read the sentence in which the word appears. If you were familiar with the word and guessed at the answer, make sure that the word that you chose fits with the word as it is used in the sentence. If you were unfamiliar with the word, see if context clues in the sentence or in the sentences before or after help you guess the meaning. Section 3 • Guide to Reading Comprehension 325 3. If you are not sure which answer is correct, read the sentence with each ofthe four answer choices in place. Does one seem more logical, given the context of the sentence, than the other three? If not, do any seem illogical? (Those you can eliminate.) 4. If you're still not sure, make the best guess you can and go on. Sample Items (line) (5) (0) In Britain's NorthAmerican colonies, university trained physicians were at a premium. At the time of the Revolution, there were probably only around 400 physicians and some 3,000 practitioners who had on-the-job training as barber-surgeons or physicians' apprentices. Whether university trained or not, none had much knowledge of the causes of disease, and the "cures" they often recommended-bleeding, blistering, and the use of violent purgatives-were at best ineffective and at worst lethal. l. The phrase"at a premium" in line 2 is closest in meaning to (A) well-paid (B) not very numerous (C) very experienced (D) not well-respected The phrase "only around 400" indicates that there was a shortage of university-trained physicians. 2. Which of the following words could best be substituted for the word "lethal" in line 1O? (A) Impractical (B) Brutal (C) Impossible (D) Deadly The phrase "at best ineffectual and at worst lethal" indicates that the correct answer must describe a situation much worse than ineffectual. Choices (A) and (C) don't create logical sentences when substituted for lethal. Choice (B), brutal (which means savage or violent), is more logical, but only choice (D) is synonymous with the word that is asked about. ... Exercise 47.1 Focus: Using context clues to answer vocabulary-in-context questions involving words with multiple definitions. Directions: Read each item. Then mark the answer choice that could best be used in place of the underlined expression as it appears in the sentence. 1. The planet Mercury is visible to the naked eye but is not the easiest planet to spot. _ _ _ (A) unclothed ~ (B) unaided ___ (C) unarmed 326 Section 3 • Guide to Reading Comprehension 2. Every atlas has its own legend. _ _ (A) mythical story ___ (B) famous person ___ (C) explanation of symbols 3. Above the snowline, any mountain hollow is permanently occupied with snow. _ _ (A) filled _ _ (B) busy 4. The glass factories of Toledo, Ohio, boomed after Michael Owens invented a process that turned out bottles by the thousands. _ _ (A) exploded ___ (B) resounded _ _ (C) prospered 5. Dr. Rene Dubos, a French physician who came to the United States in 1924, searched for substances that would check the growth of bacteria. ___ (A) restrict ___ (B) investigate 6. The root of the horseradish plant has a biting taste. _ _ (A) chewing _ _ (B) sharp ___ (C) sarcastic 7. The double-bass is shaped like a viola and has a deep, rich tone. ___ (A) valuable ___ (B) resonant ___ (C) abundant 8. A public library is a resource the entire community can draw on. ___ (A) illustrate _ _ (B) approach _ _ (C) use 9. A business concern with two or more owners is referred to as a partnership. _ _ (A) firm ___ (B) worry 10. Table salt is finer than rock salt. ___ (A) made up of smaller particles ___ (B) of better quality ___ (C) freer of impurities 11. Shirley Jackson's sometimes chilling, sometimes hilarious stories were largely ignored by critics at the time they were published. ___ (A) freezing _ _ (B) frightening 12. As a child, the sharpshooter Annie Oakley hunted game with such success that, by selling it, she was able to payoff the mortgage on her family's farm. _ _ (A) animals _ _ _ (B) athletic competition 13. All chimpanzees are extremely curious about their surroundings. ___ (A) strange _ _ _ (B) inquisitive Section 3 • Guide to Reading Comprehension 327 14. Furniture design and manufacture were originally the work of individuals, but by the eighteenth century, many furniture makers had teams of craftsmen to help them carry out their plans. ___ (A) transport _ _ (B) obey ___ (C) implement 15. Samuel Latham Mitchell helped found Rutgers Medical College in New Jersey in 1826, and he produced several important works in chemistry and geology. _ _ (A) books ___ (B) accomplishments ___ (C) factories Exercise 47.2 Focus: Answering vocabulary-in-context questions about words or phrases in reading passages. Directions: Answer the questions about the vocabulary in the passages, and mark the words or phrases that are closest in meaning to the words or phrases that are asked about. The first one is done as an example. Questions 1-11 (line) (5) (10) (15) (20) The Civil War created feverish manufacturing activity to supply critical material, especially in the North. When the fighting stopped, the stage was set for dramatic economic growth. Wartime taxes on production vanished, and the few taxes that remained leaned heavily on real estate, not on business. The population flow from farm to city increased, and the labor force it provided was buttressed by millions of newly arrived immigrants willing to work for low wages in the mills of the North and on the railroad crews of the Midwest and West. Government was nothing if not accommodating. It established tariff barriers, provided loans and grants to build a transcontinental railroad, and assumed a studied posture of nonintervention in private enterprise. The social Darwinism of British philosopher Herbert Spencer and American economist William Graham Summer prevailed. The theory was that business, if left to its own devices, would eliminate the weak and nurture the strong. But as business expanded, rivalry heated up. In the 1880's, five railroads operating between New York and Chicago were vying for traffic, and two more were under construction. As a result of the battle, the fare between the cities decreased to $1. The petroleum industry suffered from similar savage competition, and in the 1870's, many oil industries failed. 1. The word "feverish" in line 1 is closest in meaning to ~ (A) extremely rapid ___ (B) sickly and slow _ _ _ (C) very dangerous ___ (D) understandable 2. Which of the following is closest in meaning to the word "critical" in line 2? ___ (A) Industrial _ _ _ (B) Serious ___ (C) Crucial _ _ (D) InSUlting 328 Section 3 • Guide to Reading Comprehension 3. The phrase "the stage was seC in line 3 is closest in meaning to which of the following? ___ (A) The play was over. ___ (B) The progress continued. ___ (C) The foundation was laid. ___ (D) The direction was clear. 4. The phrase "real estate" in line 5 refers to ___ (A) tools and machines ___ (B) actual income ___ (C) new enterprises ___ (D) land and buildings 5. The word "buttressed ,. in line 7 is closest in meaning to ___ (A) concerned _ _ (B) supplemented ___ (C) restructured ___ (D) enriched 6. The word "accommodating" in line lO is closest in meaning to ___ (A) persistent ___ (B) indifferent _ _ (C) balanced _ _ (D) helpful 7. W11ich of the following could best be substituted for the word "posture" in line 12? ___ (A) Stature ___ (B) Predicament ___ (C) Position ___ (D) Situation 8. The word "prevailed" in line 15 is closest in meaning to ___ (A) influenced ___ (B) triumphed _ _ _ (C) premiered ___ CD) evolved 9. The phrase"left to its own devices" in lines 15-16 means _ _ _ CA) forced to do additional work ___ (B) allowed to do as it pleased ___ (C) made to change its plans ___ CD) encouraged to produce more goods 10. The word "vying" in line 18 is closest in meaning to _ _ (A) competing _ _ (B) hoping ___ (C) arranging ___ (D) caring 11. The word "savage" in line 21 is closest in meaning to ___ (A) fierce _ _ (B) growing ___ (C) surprising ___ (D) genuine Section 3 • Guide to Reading Comprehension Questions 12-19 (line) (5) (10) (15) All birds have feathers, and all animals with feathers are birds. No other major group of animals is so easy to categorize. All birds have wings, too, but wings are not peculiar to birds. Many adaptations are found in both feathers and wings. Feathers form the soft down of geese and ducks, the long decorative plumes of ostriches, and the strong flight feathers of eagles and hawks. Wings vary from the short, broad ones of chickens, which seldom fly, to the long, slim ones of albatrosses, which spend almost all their lives soaring on air currents. In penguins, wings have been modified into flippers and feathers into a waterproof covering. In kiwis, the wings are almost impossible to detect. Yet diversity among birds is not so striking as it is among mammals. The difference between a hummingbird and a penguin is immense, but hardly as startling as that between a bat and a whale. It is variations in details rather than in fundamental patterns that have been important in the adaptation of birds to many kinds of ecosystems. 12. The word "categorize" in line 2 is closest in meaning to ___ (A) appreciate ___ (B) comprehend _ _ (C) classify ___ (D) visualize 13. Which of the following is closest in meaning to the phrase "peculiar to" in line 3? ___ (A) Unusual for ___ (B) Common to ___ (C) Necessary for ___ (D) Unique to 14. The word "slim" in line 8 is closest in meaning to ___ (A) slender _ _ (B) powerful _ _ (C) graceful _ _ (D) soft 15. The word "detect" in line 11 is closest in meaning to ___ (A) utilize ___ (B) extend ___ (C) observe ___ (D) describe 16. Which of the following is closest in meaning to the word "diversity" in line 12? ___ (A) Function _ _ (B) Heredity _ _ (C) SpeCialty _ _ (D) Variety 17. The word "hardly" in line 14 is closest in meaning to _ _ (A) definitely ___ (B) not nearly _ _ (C) possibly ___ (D) not always 329 330 Section 3 • Guide to Reading Comprehension 18. The word "startling" in line 14 is closest in meaning to ___ (A) initial _ _ _ (B) exciting ___ (C) tremendous ___ (D) surprising 19. The word "fundamental" in line 15 is closest in meaning to _ _ (A) basic _ _ (B) shifting _ _ (C) predictable ___ (D) complicated Questions 20-27 (line) (5) (10) (15) Manufactured in the tranquil New England town of Concord, New Hampshire, the famous Concord Coach came to symbolize the Wild West. Its rugged body and a suspension system of leather straps could handle the hard jolts from rough roads. A journalist in 1868, describing a railroad shipment of 30 coaches bound for Wells, Fargo and Company, wrote, "They are splendidly decorated ... the bodies red and the running parts yellow. Each door has a handsome picture, mostly landscapes, and no two coaches are exactly alike." Wells, Fargo and Company was founded in 1852 to provide mail and banking services for the gold camps of California and later won a monopoly on express services west of the Mississippi. A Wells, Fargo Concord Coach carried nine to fourteen passengers plus baggage and mail. The accommodations were by no means plush. However, the stagecoach was the swiftest method of travel through much of the Far West. 20. The word "tranquil" in line 1 is closest in meaning to _ _ (A) peaceful _ _ (B) bustling _ _ _ (C) industrial _ _ (D) tiny 21. The word "symbolize" in line 2 is closest in meaning to ___ (A) recollect _ _ (B) fulfill _ _ (C) deny ___ (D) represent 22. Which of the following could best substitute for the word "rugged" in line 3? _ _ (A) Streamlined _ _ (B) Roomy _ _ (C) Sturdy ___ (D) Primitive 23. Which of the following is closest in meaning to the word "jolts" in line 4? _ _ (A) Signs _ _ (B) Shocks _ _ (C) Sights _ _ (D) Shots Section 3 • Guide to Reading Comprehension 24. The phrase "bound for" in line 5 is closest in meaning to _ _ (A) belonged to ___ (B) destined for _ _ (C) built by _ _ (D) paid for 25. The word "splendidly" in line 6 is closest in meaning to _ _ (A) superbly _ _ (B) deliberately _ _ (C) specifically _ _ (D) slightly 26. The word "plush" in line 13 is closest in meaning to ___ (A) normal ___ (B) luxurious ___ (C) memorable ___ (D) unpleasant 27. Which of the following is closest in meaning to the word "swiftest" in line 14? ___ (A) Most comfortable _ _ (B) Cheapest _ _ (C) Most direct _ _ (D) Fastest Questions 28-35 (line) (5) (10) The Hopi people of Arizona stress the institutions of family and religion in a harmonious existence which makes the self-sacrificing individual the ideal. The Hopi individual is trained to feel his or her responsibility to and for the Peaceful People-the Hopi's own term for themselves. Fighting, bullying, or attempting to surpass others bring automatic rebuke from the community. Implicit in the Hopi view is an original and integrated theory of the universe. With this they organize their society in such a way to obtain a measure of security from a harsh and hazardous environment made up of human foes, famine, and plagues. They conceive of the universe-humans, animals, plants, and supernatural spirits-as an ordered system functioning under a set of rules known to them alone. These rules govern their behavior, emotions, and thoughts in a prescribed way. 28. The word "stress" in line 1 is closest in meaning to ___ (A) emphasize _ _ (B) define ___ (C) describe ___ (D) persuade 29. Which of the following could best substitute for the word "harmonious" in line 2? ___ (A) Cooperative ___ (B) Dangerous ___ (C) Philosophical ___ (D) Exclusive 331 332 Section 3 • Guide to Reading Comprehension 30. The word "term" in line 5 is closest in meaning to _ _ (A) era _ _ (B) name _ _ (C) area _ _ (D)law 31. The word "bullying" in line 5 is closest in meaning to _ _ (A) lying ___ (B) organizing ___ (C) entertaining ___ (D) tormenting 32. Which of the following can replace the word "rebuke" in line 6 with the least change in meaning? ___ (A) Prestige ___ (B) Criticism ___ (C) Reaction ___ (D) Acknowledgment 33. Which of the following could best be substituted for the word "hazardous" in line 9? _ _ (A) Changing _ _ (B) Random _ _ _ (C) Familiar ___ (D) Dangerous 34. The word "foes" in line 10 is closest in meaning to ___ (A) fears _ _ (B) needs ___ (C) enemies ___ (D) failures 35. Which of the following is closest in meaning to the word "prescribed" in line 14? _ _ (A) Set _ _ (B) Disorderly _ _ (C) Legal _ _ (D) Compatible Questions 36-42 (line) (5) (10) Canadian researchers have discovered a set of genes that determine the lifespan of the common nematode, a type of worm. This finding sheds new light on the aging process that may eventually allow scientists to delay the inexorable process of aging and death. By manipulating the newly discovered genes, the team at McGill University in Montreal was able to increase the lifespan of the nematode fivefold. Altering the genes apparently slowed the metabolism of the worms to a more leisurely pace. This in turn may slow the accumulation of the DNA defects thought to cause aging. Although the causes of aging in humans are undoubtedly more involved, researchers are confident that the discoveries will provide invaluable clues about this heretofore mysterious process. 36. The word "determine" in line 2 is closest in meaning to ___ (A) control _ _ (B) modify ___ (C) maintain ___ (D) shorten Section 3 • Guide to Reading Comprehension 37. Which of the following is closest in meaning to the phrase" sheds new light on" in line 3? ___ (A) Contradicts what is known about ___ (B) Gives new meaning to ___ (C) Provides more information about ___ (D) Calls more attention to 38. The word "inexorable" in line 4 is closest in meaning to ___ (A) cruel ___ (B) unstoppable ___ (C) essential ___ (D) incomprehensible 39. Which of the following could best be used in place of the phrase "more leisurely" in line 9? ___ (A) More relaxed ___ (B) Livelier ___ (C) Easier ___ (D) More irregular 40. The word "involved" in line 12 is closest in meaning to ___ (A) committed ___ (B) serious ___ (C) apparent ___ (D) complicated 4l. Which of the following is closest in meaning to the word "clues" in line 13? ___ (A) Plans ___ (B) Secrets _ _ (C) Signals _ _ (D) Hints 42. The word "heretofore" in line 13 is closest in meaning to ___ (A) universally ___ (B) almost ___ (C) previously ___ (D) somewhat 333 334 Section 3 • Guide to Reading Comprehension lESSON 48 REFERENCE QUESTIONS Reference questions ask what noun (called the referent) a pronoun or some other expression refers to. The correct answer is not always the noun that is closest to the pronoun in the passage. Incorrect choices are usually other nouns that appear in the passage. If you are unable to decide immediately which answer is correct, substitute the four choices for the word that is being asked about. Which one is the most logical substitute? In general, reference questions tend to be the easiest type of reading question. Sample Item (line) (5) There is a poisonous, plant-like animal called the anemone that lives among coral reefs. When small fish venture too close to the tentacles of these "living flowers;' they are stung and eaten. For unknown reasons, the anemone makes an exception of the clownfish, which swims through its deadly tentacles in safety. When in danger, the clownfish dashes among the anemone's tentacles where other fish are afraid to follow. The clownfish even builds its nest where the anemone can protect it. 1. The word "they" in line 4 refers to (A) coral reefs (B) small fish (C) tentacles (D) flowers Of the four choices, only "small fish" is a logical answer. 2. The word "it" in line 10 is a reference to the (A) clownfish (B) nest (C) anemone (D) exception Only the word "nest" is a logical substitute for "it." ... Exercise 48.1 Focus: Identifying the referents for pronouns and other expressions in sentences and very short passages. Directions: Read the items. Decide which choice is the correct referent for the underlined word, and mark the answer. The first one is done as an example. 1. Detergents clean clothes by first removing particles of dirt from the fabric, then suspending the particles until they can be washed away. _ _ (A) clothes ~ (B) particles of dirt ___ (C) detergents Section 3 • Guide to Reading Comprehension 335 2. Wooly mammoths were hunted by big cats called sabertooth tigers, which also became extinct by the end of the last Ice Age. They were also hunted by early men armed with spears and clubs. ___ (A) sabertooth tigers _ _ _ (B) early men ___ (C) wooly mammoths 3. X-rays allow art historians to examine paintings internally without damaging them. _ _ (A) x-rays _ _ _ (B) art historians _ _ (C) paintings 4. There is a New England influence in southern Ohio, manifesting itself in white churches and village greens. ___ (A) a New England influence _ _ _ (B) southern Ohio 5. Florists often refrigerate cut flowers to protect their fresh appearance. _ _ (A) florists' _ _ _ (B) flowers' 6. A flat kite needs a tail to supply drag and to keep the kite pointed toward the sky. A simple one consists of cloth strips tied end to end. _ _ (A) kite _ _ (B) tail _ _ (C) sky 7. A number of sculptors have rejected the abstractions of minimalist artists. These sculptors have developed a style of extreme realism involving ordinary subjects. ___ (A) extreme realists _ _ (B) minimalists 8. Water is an exception to many of nature's rules because of its unusual properties. ___ (A) nature's _ _ _ (B) water's 9. Compound bows are popular with bow hunters, but they are not permitted in international archery competitions. ___ (A) bow hunters _ _ _ (B) compound bows 10. Ropes are cords at least .15 inches in diameter and are made of three or more strands which are themselves formed of twisted yarns. _ _ (A) yarns _ _ (B) ropes _ _ (C) strands _ _ (D) cords 11. Grocers slice sides, quarters, and what are called primal cuts of beef into smaller pieces. These pieces are called retail cuts. ___ (A) smaller pieces _ _ _ (B) sides, quarters, and primal cuts 12. Leaves are found on all deciduous trees, but they differ greatly in size and shape. _ _ (A) trees _ _ _ (B) leaves 336 Section 3 • Guide to Reading Comprehension 13. Yasuo Kuniyashi was born in Japan in 1883 and studied art at the Los Angeles School of Art and Design. He also studied art in New York City, where he gave his fIrst one-man show. In 1925 he moved from there to Paris where he was influenced by the works of Chagall and other artists. _ _ (A) Japan _ _ (B) Paris _ _ (C) Los Angeles _ _ (D) New York City 14. In the past, biologists considered mushrooms and other fungi as a type of non-green plant. Today, however, they are most commonly regarded as a separate kingdom of living things. ___ (A) mushrooms and other fungi _ _ _ (B) biologists _ _ (C) plants 15. William Dean Howells, a contemporary and friend of Mark Twain, wrote a number of books that realistically portrayed life on farms in Midwestern America. One of his followers, Hamlin Garland, was even more bitter in his criticism of rural America than his mentor. _ _ (A) Hamlin Garland _ _ (B) Mark Twain ___ (C) William Dean Howells 16. The Wisconsin Dells is a region where the Wisconsin River cuts through soft sandstone. The strange formations that have been carved out of the rocks there are a delight to tourists. They have names such as Devil's Elbow, Grand Piano, and Fat Man's Misery. ___ (A) strange formations _ _ _ (B) tourists _ _ (C) rocks 17. The lives of beetles are divided into four stages, as are those of wasps, ants, and butterflies. _ _ (A) lives _ _ _ (B) stages ___ (C) insects 18. After electron microscopes were invented, scientists found many new viruses. Some of them were round, some oval, and some corkscrew-shaped. ___ (A) electron microscopes _ _ _ (B) viruses ___ (C) scientists 19. The detailed information in maps is now produced almost entirely from satellite photography rather than by ground surveying because this method is faster, cheaper, and more accurate. _ _ (A) satellite photography _ _ _ (B) ground surveying 20. An elephant is bigger than a mouse because it has trillions more cells, not because its cells are any bigger. ___ (A) a mouse's _ _ (B) an elephant's Section 3 • Guide to Reading Comprehension Exercise 48.2 Focus: Answering reference questions based on longer passages. Directions: Read the following passages and the questions about them. Decide which of the choices-(A), (B), (C), or (D)-best answers the question, and mark the answer. The first one is done as an example. Questions 1-6 (line) (5) (10) (15) (20) In addition to these various types of deep mining, several types of surface mining may be used when minerals lie relatively close to the surface of the Earth. One type is open-pit mining. The first step is to remove the overburden, the layers of rock and earth lying above the ore, with giant scrapers. The ore is broken up in a series of blasting operations. Power shovels pick up the pieces and load them into trucks or, in some cases, ore trains. These carry it up ramps to ground level. Soft ores are removed by drilling screws, called augers. Another type is called "placer" mining. Sometimes heavy metals such as gold are found in soil deposited by streams and rivers. The soil is picked up by a power shovel and transferred to a long trough. Water is run through the soil in the trough. This carries soil particles away with it. The metal particles are heavier than the soil and sink to the bottom, where they can be recovered. The finishing-off process of mining is called mineral concentration. In this process, the desired substances are removed from the waste in various ways. One technique is to bubble air through a liquid in which ore particles are suspended. Chemicals are added that make the minerals cling to the air bubbles. The bubbles rise to the surface with the mineral particles attached, and they can be skimmed off and saved. 1. The word "them" in line 7 refers to ___ (A) power shovels _ _ _ (B) layers of rock and earth ___ (C) giant scrapers ~ (D) pieces of ore 2. To which of the following does the word "These" in line 8 refer? _ _ (A) Ramps _ _ _ (B) Trucks or ore trains _ _ (C) Augers ___ (D) Blasting operations 3. The phrase "Another type" in line 10 is a reference to another type of _ _ (A) deep mining _ _ (B) ore _ _ (C) metal ___ (D) surface mining 4. The word "This" in line 13 refers to ___ (A) a power shovel _ _ (B) gold ___ (C) running water _ _ (D) a long trough 337 338 Section 3 • Guide to Reading Comprehension 5. In line 17, the phrase "this process"refers to _ _ (A) surface mining _ _ _ (B) the depositing of soil ___ (C) mineral concentration _ _ (D) placer mining 6. The word "they" in line 22 refers to ___ (A) the processes _ _ (B) the air bubbles _ _ (C) the chemicals _ _ (D) the minerals Questions 7-10 (line) (5) (10) (15) Mount Rainier, the heart of Mt. Rainier National Park, is the highest mountain in the state of Washington and in the Cascade Range. The mountain's summit is broad and rounded. It is 14,410 feet above sea level and has an area of about one square mile. Numerous steam and gas jets occur around the crater, but the volcano has been sleeping for many centuries. Mount Rainier has a permanent ice cap and extensive snow fields, which give rise to over forty glaciers. These feed swift streams and tumbling waterfalls that race through the glacial valleys. Forests extend to 4,500 feet. There are alpine meadows between the glaciers and the forests which contain beautiful wild flowers. The Nisqually Glacier is probably the ice region that is most often explored by visitors. Paradise Valley, where hotel accommodations are available, perches on the mountain's slope at 5,400 feet. The Wonderland Trail encircles the mountain. Its 90mile length can be covered in about a week's time. 7. To which ofthe following does the word "It" in line 3 refer? _ _ _ (A) Mt. Rainier _ _ (B) The summit ___ (C) The Cascade range _ _ (D) The national park 8. The word "These" in line 8 refers to which of the following? _ _ (A) Snow fields _ _ _ (B) Steam and gas jets ___ (C) Glaciers ___ (D) Streams and waterfalls 9. The word "which" in line 11 refers to _ _ (A) forests _ _ _ (B) wild flowers ___ (C) alpine meadows ___ (D) glacial valleys 10. What does the word "Its" in line 15 refer to? _ _ (A) The trail's _ _ _ (B) An ice region's ___ (C) The mountain's _ _ (D) A week's Section 3 • Guide to Reading Comprehension Questions 11-15 (line) (5) (10) (15) Some people associate migration mainly with birds. Birds do travel vast distances, but mammals also migrate. An example is the caribou, reindeer that graze on the grassy slopes of northern Canada. When the weather turns cold, they travel south until spring. Their tracks are so well-worn that they are clearly visible from the air. Another migrating mammal is the Alaska fur seal. These seals breed only in the Pribilof Islands in the Bering Sea. The young are born in June and by September are strong enough to go with their mothers on a journey of over 3,000 miles. Together they swim down the Pacific Coast of North America. The females and young travel as far as southern California. The males do not journey so far. They swim only to the Gulf of Alaska. In the spring, males and females all return to the islands, and there the cycle begins again. Whales are among the greatest migrators of all. The humpback, fin, and blue whales migrate thousands of miles each year from the polar seas to the tropics. Whales eat huge quantities of tiny plants and animals (called plankton). These are most abundant in cold polar waters. In winter, the whales move to warm waters to breed and give birth to their young. 11. The phrase "An example" in line 2 refers to an example of a ___ (A) migratory mammal ___ (B) place where animals migrate _ _ (C) bird ___ (D) person who associates migration with birds 12. In line 5, the word "Their" is a reference to the ___ (A) caribou's _ _ _ (B) grassy slopes' _ _ (C) birds' _ _ (D) seals' 13. To what does the word "They" in line 12 refer? _ _ (A) female seals _ _ _ (B) young seals ___ (C) the islands ___ (D) male seals 14. In line 14, the word "there" refers to _ _ (A) the Gulf of Alaska _ _ (B) the Pribilof Islands ___ (C) southern California ___ (D) the Pacific Coast of North America 15. The word "These" in line 18 refers to _ _ (A) three types of whales _ _ _ (B) tiny plants and animals _ _ (C) polar seas ___ (D) warm waters 339 340 Section 3 • Guide to Reading Comprehension Questions 16-19 (line) (5) (10) (15) (20) Design is the arrangement of materials to produce certain effects. Design plays a role in visual arts and in the creation of commercial products as well. Designers are concerned with the direction of lines, the size of shapes, and the shading of colors. They arrange these patterns in ways that are satisfying to viewers. There are various elements involved in creating a pleasing design. Harmony, or balance, can be obtained in a number of ways. It may be either symmetrical (in balance) or asymmetrical (out of balance, but still pleasing to the eye). Or a small area may balance a large area if it has an importance to the eye (because of color or treatment) which equals that of the larger area. Contrast is the opposite of harmony. The colors red and orange harmonize, since orange contains red. A circle and oval harmonize, as they are both made up of curved lines. But a short line does not harmonize with a long line. It is in contrast. Unity occurs when all the elements in a design combine to form a consistent whole. Unity resembles balance. A design has balance if its masses are balanced, or if its tones and colors harmonize. But unity differs from balance because it implies that balanced elements work together to form harmony in the design as a whole. 16. The word "They" in line 5 refers to ___ (A) designers _ _ _ (B) lines, shapes, and colors ___ (C) directions, size, and shape ___ (D) visual arts 17. The word "that" in line 12 is used as a reference to ___ (A) a color _ _ _ (B) an area _ _ (C) importance _ _ (D) balance 18. The word "It" in line 16 is used as a reference to _ _ (A) a circle _ _ _ (B) the color red _ _ (C) a long line ___ (D) a short line 19. In line 20, the word "it" refers to _ _ (A) unity _ _ _ (B) balance _ _ (C) a design ___ (D) a consistent whole Section 3 • Guide to Reading Comprehension Questions 20-22 (line) (5) (10) Although they had been used to haul freight and passengers between the Eastern seaboard and the Ohio Yalley since 1812, wagon trains were first used extensively in the 1820's on the Santa Fe Trail. Long trains of covered wagons drawn by oxen or mules carried manufactured goods to trade for fur, gold, and silver in Santa Fe. The independent traders who pooled their resources to form these trains elected a captain and several lieutenants who commanded the parallel columns in which the wagons usually moved. They enforced the rules, selected the routes, and designated stopping places. 20. To what does the word "they" in line 1 refer? ___ (A) the Eastern Seaboard and the Ohio Yalley _ _ _ (B) wagon trains ___ (C) freights and passengers ___ (D) oxen and mules 21. The word "their" in line 6 refers to ___ (A) the covered wagons' _ _ _ (B) the oxen and mules' ___ (C) the independent traders' ___ (D) the captain and lieutenants' 22. To what does the word "They" in line 9 refer? ___ (A) the leaders _ _ _ (B) the traders _ _ (C) the parallel columns _ _ (D) the stopping places Questions 23-26 (line) (5) (10) (15) In most of the earliest books for children, illustrations were an afterthought. But in the Caldecott "toy books," pictures were as important as the few lines of copy, and they occupied far more space. One can almost read the nursery rhymes from the dramatic action in the pictures. Since then, thousands of successful picture books have been published in the United States and in many countries around the world. In the best, the text and illustrations seem to complement each other perfectly. Often one person is the author and illustrator-for example, Robert McCloskey (Make Way for Ducklings) and Arnold Loebel (Frog and Toad Together). Many others have been produced by an author-artist team, as in The Happy Lion, written by Louise Fatio and illustrated by Roger Duvoisin. Wordless picture books have also become popular. With a little help, three- or four-year-olds can follow the sequence of events and they can understand the stories suggested in them. One of the most delightful examples of a wordless book is Jan Ormerod's Sunshine. 341 342 Section 3 • Guide to Reading Comprehension (20) American publishers have also drawn on artists from other countries whose original, imaginative works have brought their different visions to American children's book illustration. Among them are Leo Lionni from Italy, Feoddor Rojankowski from Russia, and Taro Yashima from Japan. 23. The word "they" in line 3 refers to _ _ _ (A) the earliest books for children _ _ (B) lines of copy _ _ _ (C) the Caldecott "toy books" ___ (D) pictures 24. The phrase "the best" in line 8 refers to the best ___ (A) picture books _ _ _ (B) illustrations _ _ _ (C) authors ___ (D) nursery rhymes 25. The word "they" in line 17 refers to _ _ (A) delightful examples ___ (B) events _ _ (C) 3- and 4-year-olds ___ (D) wordless picture books 26. The word "their" in line 22 refers to ___ (A) American publishers ___ (B) original, imaginative works ___ (C) artists from other countries _ _ _ (D) American children's Section 3 • Guide to Reading Comprehension 2. CA:YCDCDc[) <l.,,·CD@1/; 3. •. 1~,\CDCD~CiD ·.··:lO,QD CD.RQC[) 343 ~2.CA:YCDCDc[) CA:Y CD CDC[) CD~:;j"::" 24. QDCDCDc[) 15. ~~\J;,.; .. 25. QDCDCDc[) 1.~ . . . 26 .. CA:YCDCDc[) ,17; CA:Y c[>CDc[) 27. QDCDCDCQ) CDC]:>.,,: •. . 28. QDCDCDc[) ··t:,.: 29. QDCDCDc[) ""'30. CD cD CDC[) 23; ~~'~";'" ,-, .-',.---,' ·6.CD®©CQ5 7. CA:Y CD CD C[) .$;.CD~ >', ' ,",,'~ lO. ®jCDcn®jj ,i(,;, MINI-TEST 8: READING COMPREHENSION Directions: In this mini-test, there are several passages, each followed by a number of questions. Read the passages and, for each question, choose the one best answer. (A), (B), (C), or (D). You can mark the answer you have chosen either on the answer sheet above or on the blanks by the answer choices. All of your answers should be based on what is stated or implied in the passages. Time: 55 minutes Questions 1-12 (line) (5) Humans have struggled against weeds since the beginnings of agriculture. Marring our gardens is one of the milder effects of weeds-any plants that thrive where they are unwanted. They clog waterways, destroy wildlife habitats, and impede farming. Their spread eliminates grazing areas and accounts for one-third of all crop loss. They compete for sunlight, nutrients, and water with useful plants. 344 Section 3 • Guide to Reading Comprehension (10) (15) (20) (25) The global need for weed control has been answered mainly by the chemical industry. Its herbicides are effective and sometimes necessary, but some pose serious problems, particularly if misused. Toxic compounds threaten animal and public health when they accumulate in food plants, ground water, and drinking water. They also harm workers who apply them. In recent years, the chemical industry has introduced several herbicides that are more ecologically sound. Yet new chemicals alone cannot solve the world's weed problems. Hence, scientists are exploring the innate weed-killing powers of living organisms, primarily insects and microorganisms. The biological agents now in use are harmless to humans and are environmentally benign. They can be chosen for their ability to attack selected targets and leave crops and other plants untouched. In contrast, some of the most effective chemicals kill virtually all the plants they come in contact with, sparing only those that are naturally resistant or have been genetically modified for resistance. Furthermore, a number of biological agents can be administered only once, after which no added applications are needed. Chemicals typically must be used several times per growing season. 1. With what topic does this passage primarily deal? ___ (A) The importance of the chemical industry _ _ _ (B) The dangers of toxic chemicals ___ (C) Advantages of biological agents over chemical ones ___ (D) A proposal to ban the use of all herbicides 2. The word "Marring" in line 2 is closest in meaning to _ _ (A) spoiling _ _ (B) dividing _ _ (C) replacing _ _ (D) planting 3. The word "clog" in line 3 is closest in meaning to _ _ (A) drain _ _ _ (B) float on _ _ (C) obstruct _ _ (D) grow along 4. Which of the following terms does the author deftne in the ftrst paragraph? ___ (A) Grazing areas _ _ (B) Weeds _ _ (C) Wildlife habitats _ _ (D) Nutrients 5. Which of the following statements about the use of chemical agents as herbicides would the author most likely agree? _ _ (A) It should be increased. _ _ _ (B) It has become more dangerous recently. ___ (C) It is safe but inefftcient. _ _ _ (D) It is occasionally required. Section 3 • Guide to Reading Comprehension 6. Which of the following is NOT given as an advantage of biological agents over chemical herbicides? ___ (A) They are less likely to destroy desirable plants. _ _ _ (B) They are safer for workers. ___ (C) They are more easily available. _ _ _ (D) They do not have to be used as often. 7. Which of the following is closest in meaning to the word "Hence" in line 16? ___ (A) In addition _ _ _ (B) Consequently _ _ (C) Subsequently _ _ _ (D) In contrast 8. The word "innate" in line 17 is closest in meaning to _ _ (A) natural _ _ _ (B) effective _ _ (C) organic _ _ _ (D) active 9. According to the passage, biological agents consist mainly of ___ (A) insects and microorganisms _ _ _ (B) useful plants _ _ (C) weeds ___ (D) herbicides 10. The word "those" in line 23 refers to ___ (A) chemicals _ _ (B) targets _ _ (C) plants _ _ (D) agents 11. The word "applications" in line 25 could best be replaced by which of the following? _ _ (A) Requests _ _ (B) Special purposes ___ (C) Treatments ___ (D) Qualifications 12. Which of the following best describes the organization of the passage? ___ (A) A general idea is introduced, and several specific examples are given. _ _ _ (B) A recommendation is analyzed and rejected. ___ (C) A problem is described, and possible solutions are discussed. _ _ _ (D) Two possible causes for a phenomenon are compared. Questions 13-21 Oine) (5) (10) West Side Story is a musical tragedy based on William Shakespeare's play Romeo and Juliet. It is set in the early 1950's, when gang warfare in big cities led to injuries and even death. West Side Story transforms the Montagues and Capulets of Shakespeare's play into feuding street gangs, the Jets and the Sharks, one consisting of newly arrived Puerto Ricans and the other of native-born New Yorkers. The plot, tightly choreographed by Jerome Robbins, tells the story of the love of Maria, a Puerto Rican, forTony, who, while attempting to stop a street fight, kills Maria's brother and is ultimately killed himself. Leonard 345 346 Section 3 • Guide to Reading Comprehension (IS) Bernstein's musical score is brilliant, and Stephen Sondheim, making his Broadway debut, reveals a remarkable talent for writing lyrics. Among the hit songs of the play are "Tonight," "Maria;' "America," and "I Feel Pretty." The play opened on September 26, 1957. It ran for 734 performances, toured for 10 months, and then returned to New York for an additional 246 performances. A fllm version was released in 1961, and a successful New York revival opened in 1980. 13. The author's attitude toward the play is generally _ _ (A) regretful ___ (B) critical ___ (C) emotional _ _ (D) favorable 14. According to the passage, when does the action of the play West Side Story take place? ___ (A) In Shakespeare's time. _ _ (B) In the early 1950's. _ _ (C) In 1957. _ _ (D) In 1980. 15. It can be inferred from the passage that the Capulets and Montagues ___ (A) were rival groups in Shakespeare's play ___ (B) were 1950's street gangs _ _ (C) fought against the Jets and Sharks ___ (D) were groups of actors, dancers, and singers 16. The word "feuding" in line 5 is closest in meaning to _ _ (A) growing _ _ (B) hostile ___ (C) organized _ _ (0) criminal 17. Which of the following is closest in meaning to the word "ultimately" in line 1O? _ _ (A) Evidently _ _ (B) Immediately _ _ (C) Eventually _ _ (D) Savagely 18. According to the article, the words to the songs of West Side Story were written by _ _ _ (A) Jerome Robbins _ _ _ (B) Leonard Bernstein _-_ (C) William Shakespeare _ _ (0) Stephen Sondheim 19. The word "score" in line 11 could best be replaced by which of the following? _ _ (A) Talent _ _ (B) Music _ _ _ (C) Performance _ _ (D) Dialogue 20. During its initial appearance in New York, how many times was West Side Story performed? _ _ (A) 10 - - ( B ) 26 _ _ (C) 246 _ _ (0)734 Section 3 • Guide to Reading Comprehension 347 21. Where in the passage is the basic story of West Side Story summarized? _ _ (A) Lines 1-4 _ _ (B) Lines 7-10 _ _ (C) Lines 13-14 _ _ (D) Lines 15-17 Questions 22-33 (line) (5) (10) (15) (20) About 8,000 people looked over the horseless carriages on November 3, 1900, the opening day ofthe New York Auto Show, and the first opportunity for the automobile industry to show off its wares to a sizable audience. By happenstance, the number of people at the show equalled the entire car population at that time. At that time, 10 million bicycles and an unknown number of horse-and-buggies provided the prime means of transportation. Only about 4,000 cars were assembled in the United States in 1900, and only a quarter of those were gasoline powered. (The rest ran on steam or electricity.) After viewing the cars made by 32 car makers, the show's audience favored electric cars because they were quiet. The risk of a boiler explosion turned people away from steamers, and the gasoline powered cars produced smelly fumes. The Duryea Motor Wagon Company, which launched the American auto industry in 1892, offered an additive designed to mask the smell of the naphtha that it burned. The prices were not that different from what they are today. Most cost between $800 and $1,500, or roughly $11,000 to $18,500 in today's prices. Many of the 1900 models were cumbersome-the Gasmobile, the Franklin, and the Orient, for example, steered with tillers like boats rather than with steering wheels. The black-tie audience at the show viewed the display more as a social outing than as the extravaganzas that auto shows were about to become. 22. Approximately how many cars were there in the United States in 1900? _ _ (A) 4,000 _ _ (B) 8,000 _ _ (C) 10 million ___ (D) An unknown number 23. Which of the following is closest in meaning to the phrase "By happenstance" as used in line 5? _ _ (A) Generally _ _ _ (B) For example _ _ (C) Coincidentally _ _ (D) By design 24. Approximately how many of the cars assembled in the year 1900 were gasoline powered? _ _ (A) 32 _ _ (B) 1,000 _ _ (C) 2,000 _ _ (D) 4,000 348 Section 3 • Guide to Reading Comprehension 25. According to the passage, people at the 1900 New York Auto Show favored cars powered by ___ (A) electricity _ _ (B) naphtha _ _ (C) gasoline _ _ _ (D) steam 26. The word "fumes" in line 14 is closest in meaning to _ _ (A) fuels _ _ (B) grains _ _ (C) fires _ _ (D) gases 27. Which of the following is closest in meaning to the word "launched" in line 15? _ _ (A) Joined _ _ (B) Designed _ _ (C) Initiated _ _ _ (D) Anticipated 28. The purpose of the additive mentioned in line 16 was to ___ (A) increase the speed of cars _ _ _ (B) make engines run more efficiently _ _ _ (C) hide offensive smells _ _ _ (D) make cars look better 29. What was the highest price asked for a car at the 1900 New York Auto Show in the dollars of that time? _ _ (A) $800 _ _ (B) $1,500 _ _ (C) $11,300 _ _ (D) $18,500 30. The word "cumbersome" in line 20 is closest in meaning to _ _ (A) clumsy _ _ (B) unshapely _ _ (C) fragile ___ (D) inconvenient 31. Which of the following is NOT mentioned in the passage as steering with a tiller rather than with a steering wheel? _ _ (A) A Franklin _ _ (B) A Duryea _ _ (C) An Orient _ _ (D) A Gasmobile 32. The passage implies that the audience viewed the 1900 New York Auto Show primarily as ___ (A) a formal social affair _ _ _ (B) a chance to buy automobiles at low prices ___ (C) an opportunity to learn how to drive _ _ _ (D) a chance to invest in one of thirty-two automobile manufacturers 33. It can be inferred from the passage that auto shows held after 1900 ___ (A) were more spectacular _ _ _ (B) involved fewer manufacturers ___ (C) were more formal _ _ _ (D) involved less expensive cars Section 3 • Guide to Reading Comprehension 349 Questions 34-41 (line) (5) (10) When drawing human figures, children often make the head too large for the rest of the body. A recent study offers some insights into this common disproportion in children's illustrations. As part of the study, researchers asked children between four and seven years old to make several drawings of men. When they drew front views of male figures, the size of the heads was markedly enlarged. However, when the children drew rear views of men, the size of the heads was not so exaggerated. The researchers suggest that children draw bigger heads when they know they must leave room for facial details. Therefore, the odd head size in children's illustrations is a form of planning, and not an indication of a poor sense of scale. 34. The main subject of the passage is _ _ _ (A) what the results of an experiment revealed _ _ _ (B) how children learn to draw ___ (C) how researchers can gather data from works of art _ _ _ (D) what can be done to correct a poor sense of scale 35. It can be inferred that, during the research project, the children drew _ _ _ (A) pictures of men from different angles _ _ _ (B) figures without facial expression ___ (C) sketches of both men and women ___ (D) only the front view of men 36. The word "they" in line 6 refers to ___ (A) researchers _ _ (B) men _ _ (C) illustrations _ _ (D) children 37. The word "markedly" in line 7 is closest in meaning to _ _ (A) modestly _ _ _ (B) noticeably _ _ (C) merely _ _ (D) newly 38. The fmdings of the experiment described in the passage would probably be of LEAST interest to which of the following groups? _ _ _ (A) Teachers of art to children _ _ _ (B) Commercial artists _ _ (C) Experts in child development _ _ (D) Parents of young children 39. The word "odd" in line 10 is closest in meaning to ___ (A) unusual _ _ (B) huge ___ (C) average _ _ (D) expected 40. The word "scale" in line 12 is closest in meaning to ___ (A) measurement _ _ _ (B) proportion _ _ (C) balance _ _ (D) property 350 Section 3 • Guide to Reading Comprehension 41. The passage provides information to support which of the following conclusions? ___ (A) Children under the age of seven do not generally have a good sense of scale. ___ (B) With training, young children can be taught to avoid disproportion in their art. ___ (C) Children enlarge the size of the head because they sense that it is the most important part of the body. ___ (D) Children plan ahead when they are drawing pictures. Questions 42-50 (line) (5) (10) (15) (20) (25) Georgia O'Keeffe was born in Sun Prairie, Wisconsin, in 1887. She studied art in Chicago and New York from 1904 to 1908. Beginning as an advertising illustrator, she supported herself until 1918 by teaching in various schools and colleges in Texas. After that date, she devoted herself entirely to painting. Her paintings were first exhibited in 1916 at "291 ," an experimental art gallery in New York City owned by Alfred Steiglitz which was frequented by some of the most influential artists of the time. O'Keeffe married Steiglitz in 1924. O'Keeffe's early paintings were mostly abstract designs. In the 1920's she produced depictions of flowers and precise cityscapes of New York City. Whether painting flowers or buildings, she captured their beauty by intuitively magnifying their shapes and simplifying their details. O'Keeffe's style changed dramatically in 1929 during a visit to New Mexico. She was enchanted by the stark but beautiful landscapes under the bright Southwestern sun, and she then adopted her characteristic style. Thereafter, she most often painted desert landscapes, often with the blanched skull of a longhorn in the foreground. O'Keeffe's paintings were exhibited annually at several New York galleries until 1946, and she is represented in the permanent collections of most major American museums. In her later years, she settled in Taos, New Mexico, becoming the dean of Southwestern painters and one of the best known of American artists. 42. The author's main purpose in writing this passage was to ___ (A) criticize Georgia O'Keeffe's style of painting _ _ _ (B) discuss the early successes of an inlportant American artist ___ (C) compare abstract art and landscape art ___ (D) give the highlights of Georgia O'Keeffe's artistic career 43. According to the article, where did Georgia O'Keeffe receive her formal art training? ___ (A) Sun Prairie, Wisconsin _ _ _ (B) Chicago and New York _ _ (C) Texas ___ (D) Taos, New Mexico 44. The expression "that date" in line 4 refers to _ _ (A) 1887 _ _ (B) 1908 _ _ (C) 1916 _ _ (D) 1918 Section 3 • Guide to Reading Comprehension 4'5. The word "frequented" in line 7 is closest in meaning to ___ (A) visited ~~ (8) supported ~_ (C) founded ___ (D) favored 46. The word "intuitively" in line 13 is closest in meaning to ___ (A) deliherately ___ (B) defiantly ___ (C) instinctively ___ (D) instructively 47. Which of the following had the greatest influence on changing O'Keeffe's style of painting? ___ (A) A trip to the Southwest ___ (B) Alfred Steiglitz's photographs _ _ _ (C) Her job as an advertising illustrator ___ (D) Meeting influential artists 48. The word "blanched" in line 19 is closest in meaning to _ _ _ (A) shattered ___ (B) prominent ___ (C) whitened ___ (D) inexplicahle 49. Which of the following became the most common subject of O'Keeffe's paintings after 1929? _ _ _ (A) City scenes ___ (B) Desert scenes ___ (C) Flowers ___ (D) Abstract patterns '50. It can be inferred from the passage that, in her later years, O'Keeffe _ _ _ (A) continued to be successful ___ (B) returned to New York City ___ (C) could not match the successes of her early career _ _ (D) took up photography 351 352 Section 3 • Guide to Reading Comprehension MINI-LESSONS FOR SECTION 3 Vocabulary Building These mini-lessons consist of lists of over 500 words and their synonyms as well as practice exercises. Although vocabulary is no longer tested in discrete items in Part 3, there are vocabulary-in-context questions in the Reading Comprehension section. These exercises will improve your ability to use context to choose the word that best fits into a sentence. Mini-Lesson 3.1 abandon v. desert, leave behind able ad). capable, qualified, fit abolish v. end, eliminate abrupt a{{;' sudden, hasty, unexpected acclaim v. applaud, praise, honor n. praise, applause, honor acrid ad). bitter, sharp, biting adapt v. adjust, modify adept ad). skillful, expert adhere v. stick, cling admonish v. warn, caution, advise adorn v. decorate advent 11. coming, arrival adverse ad). hostile, negative, contrary affluent ad). rich, wealthy, prosperous, well-to-do aggravate v. (1) annoy, irritate (2) intensify, worsen aggregate ad). entire, total, combined agile ad). graceful, nimble, lively ailment 11. sickness, illness allot v. divide, distribute amazing ad). astonishing, astounding, surprising, startling amiable adj. agreeable, congenial, pleasant anticipate v. foresee, expect, predict anxious ad). (1) worried, nervous, apprehensive (2) eager, avid appraise 1.'. evaluate, estimate, assess apt ad). (1) appropriate, suitable, correct, relevant, proper (2) likely, prone arduous ad). difficult, exhausting arid ad). dry, barren aroma n. fragrance, smell, odor, scent artificial adj. synthetic, imitation, manmade astonishing ad). surprising, amazing, astounding astute ad). intelligent, clever, perceptive attain v. accomplish, achieve augment v. supplement, increase, strengthen, expand austere ad). strict, harsh, severe, stern authentic ad). genuine, true aversion 11. dislike, hostility, fear awkward ad). clumsy Section 3 • Guide to Reading Comprehension 353 Exercise: Complete the following sentences by filling in the blanks with vocabulary item (A), (B), or (C) according to the context of the sentences. The first one is done as an example. 1. Penicillin can have an _-LJA~_ (A) adverse 2. Burning rubber produces an (A) austere effect on a person who is allergic to it. (B) anxious (C) awkward smoke. (B) arid 3. Rationing is a system for _ _ _ _ scarce resources. (A) allotting (B) adapting 4. Anthrax is generally an humans. (A) ailment (C) acrid (C) appraising of sheep and cattle, but may also (B) aroma be transmitted to (C) aversion 5. The head of an academic department at a university should be not only a distinguished scholar but also an administrator. (C) abrupt (A) agile (B) able 6. Mountain climbing is an ____ sport. (A) austere (B) arduous (C) anxious 7. Turtles ____ their eggs after they lay them and never see their young. (A) abandon (B) appraise (C) adorn 8. Scholarships allow some students from less _ _ _ _ families to attend college. (C) amiable (A) artificial (B) affluent 9. Jewelers are sometimes asked to jewelry for insurance purposes. (A) attain (B) abandon (C) appraise 10. Acrobats must be extremely _ _ __ (A) awkward (B) affluent (C) agile 11. In a domed stadium such as the Georgia Dome, natural grass cannot be grown. _ _ _ _ turfis used on the playing field. (C) Austere (A) Artificial (B) Arid 12. Southern Arizona has an _ _ _ _ climate. (A) arid (B) astute (C) acrid 13. A person suffering from claustrophobia has an _ _ _ _ to confined spaces. (A) ailment (B) aversion (C) acclaim 14. I didn't care for the play because it ended so _ _ __ (A) amiably (B) abruptly (C) anxiously 15. The ballerina was _ _ _ _ for her wonderful performances. (A) augmented eB) anticipated (C) acclaimed 354 Section 3 • Guide to Reading Comprehension Mini-lesson 3.2 baffle v. confuse, puzzle, mystify balmy ad}. mild, warm ban v. prohibit. t()rbid bar v. prevent. obstruct. block barren ad}. sterile, unproductive, bleak, lifeless barter v. trade, exchange beckon v. summon, call, signal belligerent ad}. hostile, aggressive beneficial adj. helpful, useful, advantageous benevolent adj. benign, kind, compassionate bias n. prejudice, leaning bland ad}. mild, tasteless, dull blatant adj. flagrant. obvious, overt blend v. mix, mingle. combine n. mixture, combination bloom v. blossom, flower, flourish blunder 1'. make a mistake n. blunt aelj. error, mistake (1) unsharpened, dull (2) rude, abrupt, curt blurry aelj. unfocllsed, unclear, indistinct bold adj. brave, courageous bolster v. support, sustain bond v. join, connect n. tie, link, connection boomv. expand, prosper n. expansion, prosperity, growth brace v. support, reinforce brilliant adj. (1) bright, shiny, radiant, dazzling brisk adj. (2) talented, gifted, intelligent (I) lively, quick, vigorous (2) cool. chilly, invigorating brittle adj. bulkyadj. fragile, breakable, '''leak huge, large, clumsy Section 3 • Guide to Reading Comprehension 355 Exercise: Complete the following sentences by filling in the blanks with vocabulary item (A), (B), or (C) according to the context of the sentences. 1. Most flowers _ _ _ _ in the spring. (A) blend (B) brace (C) bloom 2. The Virgin Islands, located in the Caribbean, have a ____ climate. (B) brittle (C) balmy (A) blurry 3. Before currency came into use, people used the ____ system, exchanging goods directly for goods. (A) barter (B) blunder (C) bias 4. The airline _ _ _ _ . It sent me to Atlanta but my luggage to Montreal. (A) beckoned (B) bartered (C) blundered 5. People with ulcers must eat _ _ _ _ foods. (A) bold (ll) bland (C) bulky 6. Steel is not as _ _ _ _ as cast iron; it doesn't break as easily. (B) brittle (A) brisk (C) brilliant 7. Some people feel that violent sports such as boxing should be ____ because they are too dangerous. (A) banned (B) bloomed (C) braced 8. Many people think of deserts as regions, but many species of plants and animals have adapted to life there. (A) bland (B) barren (C) balmy 9. An autocratic ruler who serves his people well is sometimes called a dictator. (B) belligerent (C) benevolent (A) blatant lO. Robert Goddard was a ____ pioneer in the field of rocketry. (A) brilliant (B) balmy (C) brisk 11. I enjoy taking walks on ____ autumn mornings. (A) barren (B) brisk (C) blurry 12. The victim was apparently struck by a club or some other ____ object. (A) bland (B) brittle (C) blunt 356 Section 3 • Guide to Reading Comprehension Mini-lesson 3.3 calamity n. disaster, catastrophe capable adj. competent, able, efficient, skillful carve v. cut, sculpt, slice casual adj. (1) informal, relaxed (2) accidental, chance caustic adj. biting, harsh, sarcastic cautious adj. careful, alert, prudent celebrated adj. distinguished, famous, prominent charming adj. delightful, lovely, attractive cherish v. appreciate, esteem, treasure choice n. selection, option adj. exceptional, superior cite v. quote, mention, refer to, list clash v. argue, dispute, quarrel n. argument, conflict, dispute classify lJ. categorize clever ad}. smart, sharp, witty, bright cling v. stick, adhere, hold clumsy adj. awkward, inept coax v. persuade, urge colossal ad}. huge, enormous, gigantic commence v. begin, initiate, start commerce n. trade, business commodity n. good, product, merchandise compel v. force, require, coerce competent ad}. adept, skillful, capable, able Section 3 • Guide to Reading Comprehension 357 Exercise: Complete the following sentences by filling in the blanks with vocabulary item (A), (B), or (C) according to the context of the sentences. 1. The Red Cross provides relief in case of ____ such as floods, earthquakes, and hurricanes. (A) challenges (B) commodities (C) calamities 2. Spoken language is generally more ____ than written language. (C) cautious (A) casual (B) capable 3. When writing research papers, writers must ____ the sources they use. (A) coax (B) cite (C) clash 4. Monkeys are ____ as primates. (A) compelled (B) classified 5. (A) Casual remarks can offend people. (B) Caustic (C) coaxed (C) Clever 6. Sculptors use hammers and chisels to ____ statues out of stone. (A) clash (B) compel (C) carve 7. The Space Age in October 1957 when Sputnik, the first artificial satellite, was launched by the Soviet Union. (A) commenced (B) coaxed (C) cited 8. Workers must be very ____ when dealing with toxic substances. (A) caustic (B) casual (C) cautious 9. In seaside communities, building sites that have a view of the ocean are considered _ _ __ (A) choice (B) clever (C) competent 10. With the growth of international the economies of the world have become more interdependent. (A) commodity (B) commerce (C) choice 11. The Lincoln Memorial features a ____ statue of the sixteenth president. (A) colossal (B) caustic (C) casual 12. Corn, cotton, sugar, and many other goods are bought and sold in ____ markets. (A) clash (B) commerce (C) commodity 358 Section 3 • Guide to Reading Comprehension Mini-lesson 3.4 complement v. supplement, complete n. supplement, addition compliment v. praise, flatter, commend n. praise, flattery, commendation comprehensive adj. complete, thorough, exhaustive compulsory adj. necessary, obligatory, mandatory concede v. admit, acknowledge, recognize concise adj. brief, short, abbreviated concrete adj. tangible, specific, real, perceptible congregate v. assemble, gather conspicuous adj. noticeable, obvious, prominent contemplate v. think about, ponder, speculate controversial adj. disputable, debatable convenient adj. accessible, available, handy cope with v. deal with, manage, handle copious adj. abundant, ample, plentiful cordial adj. congenial, warm, friendly courteous adj. polite, refined, gracious covert adj. secret, hidden cozy adj. (1) comfortable, warm (2) friendly, intimate, close crave v. desire, long for, hope for crooked adj. (1) curved, twisted, zigzag (2) dishonest, corrupt crucial adj. critical, decisive, key crude adj. (1) rude, impolite, vulgar (2) unprocessed, raw, unrefined cruel adj. brutal, vicious, ruthless cryptic adj. secret, hidden, mysterious curb v. restrict, limit, control curious adj. (1) inquisitive (2) odd, strange, unusual curt adj. abrupt, blunt, impolite Section 3 • Guide to Reading Comprehension 359 Exercise: Complete the following sentences by filling in the blanks with vocabulary item (A), (B), or (C) according to the context of the sentences. 1. The use of seat belts is ____ in many states; failure to wear them may result in fines. (A) covert (B) cruel (C) compulsory 2. Every summer, bears from all over southern Alaska ____ along the McNeil River. (A) crave (B) curb (C) congregate 3. An abstract is a form of an academic article. Many journals publish abstracts so readers can decide if it is worthwhile to read the full version of the article. (A) concise (B) comprehensive (C) concrete 4. Before 1754, Britain and the North American colonies had a ____ relationship, but after that, their relationship became strained. (C) curt (A) conspicuous (B) cozy 5. Automatic teller machines provide a ____ means of banking twenty-four hours a day. (A) cordial (B) crooked (C) convenient 6. Lombard Street in San Francisco, which zigzags its way up Nob Hill, is known as the most ____ street in the world. (A) controversial (B) crooked (C) cryptic 7. A good writer supports his or her generalizations with ____ examples. (A) concrete (B) curious (C) crude 8. Many hunters wear orange and other bright colors in order to be as as pOSSible, and therefore avoid being shot by other hunters by mistake. (A) covert (B) cruel (C) conspicuous 9. Movie directors use music to ____ the action on the screen. (A) contemplate (B) complement (C) compliment 10. Workers in the service sector should be trained to act as ____ as possible. (A) crudely (B) courteously (C) curtly 360 Section 3 • Guide to Reading Comprehension Mini-Lesson 3.5 damp ad}. moist, wet, humid daring adj. bold, courageous, brave dazzling ad}. shining, sparkling, blinding, bright declare v. announce, proclaim deem v. believe, consider, regard, judge defective ad}. flawed, faulty, broken, malfunctioning defiant ad}. rebellious, insubordinate delicate ad}. exquisite, fragile delightful ad}. charming, attractive, enchanting delusion n. illusion, dream, fantasy demolish v. tear down, destroy, wreck, raze dense ad}. thick, solid desist v. stop, cease, discontinue device n. instrument, tool, mechanism devise v. invent, plan, figure out dim ad}. unclear, faint, indistinct din n. noise, clamor, commotion dire ad}. desperate, grievous, serious dismal ad}. gloomy, depressing, dreary disperse v. scatter, distribute, spread dispute n. argument, quarrel, debate, clash, feud distinct ad}. discrete, separate, different distinguished ad}. celebrated, notable, famous, well-known divulge v. reveal, admit, disclose dogged adj. stubborn, determined, persistent dominate v. rule, control, govern dot v. are located in, are scattered around n. spot, point downfall n. collapse, ruin, destruction doze v. sleep, nap drawback n. disadvantage, weakness, flaw dreary ad}. dismal, gloomy, bleak drench v. wet, soak drowsy ad}. sleepy, tired dubious ad}. doubtful, skeptical, uncertain durable ad}. lasting, enduring, resistant dwell v. live, reside, inhabit dwelling n. house, home, residence dwindle v. decrease, diminish dynamic ad}. energetic, forceful, active, vibrant Section 3 • Guide to Reading Comprehension 361 Exercise: Complete the following sentences by filling in the blanks with vocabulary item (A), (B), or (C) according to the context of the sentences. 1. The snow on the mountaintop was ____ in the bright morning sun. (A) dazzling (B) dogged (C) dim 2. A person who has been accused of a crime cannot be forced to ____ any information that is self-incriminating. (C) disperse (A) divulge (B) desist 3. Roses have a ____ beauty. (A) dense (B) delicate (C) dire 4. An odometer is a ____ for measuring distance. (A) device (B) delusion (C) dwelling 5. The amount of open space has ____ as more and more land is developed. (A) dominated (B) dwindled (C) dispersed 6. A battery can cause an electrical device to malfunction. (A) dogged (B) durable (C) defective 7. Richard Bird and his pilot Floyd Bennet undertook a ____ flight to the North Pole in May of 1926. (A) daring (B) defiant 8. Steep, round hills called knobs (A) demolish (B) dot (C) distinct southern Indiana. (C) dwell 9. Artists Nathaniel Currier and]. Merritt Ives produced some prints of nineteenth century New England scenes which collectors prize for their charm. (A) dreary (B) dim (C) delightful 10. Economists define ____ goods as ones intended to last more than four months. (A) durable (B) dense (C) delicate II, One cause of the American Revolution was a ____ over taxation. (A) drawback (B) delusion (C) dispute 12. Florida has a humid climate. Summers there are particularly hot and _ _ __ (A) dynamic (B) damp (C) dogged 362 Section 3 • Guide to Reading Comprehension Mini-Lesson 3.6 eerie ad}. strange, odd, unusual, frightening elderly ad}. old, aged elegant adj. sophisticated, polished eligible ad}. suitable, qualified, acceptable eminent ad}. celebrated, distinguished, famous emit v. send out, discharge enchanting ad}. delightful, charming, captivating encounter v. meet, find, come across n. meeting, confrontation endeavor n. attempt, venture endorse v. authorize, approve, support enhance v. intenSify, amplify, strengthen ensue v. follow, result entice v. lure, attract, tempt era n. period, age essential ad}. critical, vital, crucial, key esteem v. cherish, honor, admire evade v. escape, avoid, elude exhaustive ad}. thorough, complete, comprehensive exhilarating ad}. exciting, thrilling, stimulating extravagant ad}. excessive, lavish fable n. story, tale fabled ad}. legendary, mythical, famous facet n. aspect, point, feature faint ad}. dim, pale, faded, indistinct falter v. hesitate, waver fancyadj. decorative, ornate, elaborate fasten v. attach, secure fatal adj. mortal, lethal, deadly fatigue v. tire, exhaust n. exhaustion, weariness faulty adj. flawed, inferior feasible adj. possible feeble ad}. weak, fragile, frail ferocious adj. fierce, savage, violent fiery adj. (1) blazing, burning (2) passionate, fervent fitting adj. suitable, proper, apt, appropriate flagrant adj. blatant, obvious flaw n. defect, imperfection, fault flee v. escape, go away, elude flimsy ad}. fragile, frail, weak, feeble forego v. abandon, give up foremost ad}. chief, principal, leading fragment n. particle, piece, bit fragrant ad}. aromatic, scented fraudulent adj. false, deceptive, deceitful fundamental adj. basic, integral, elemental fusion n. blend, merger, union futile adj. useless, pOintless, vain Section 3 • Guide to Reading Comprehension 363 Exercise: Complete the following sentences by filling in the blanks with vocabulary items (A), (8), or (C) according to the context of the sentences. 1. In 1906, much of San Francisco was destroyed by an earthquake and the fires that _ _ __ (A) evaded (B) ensued (C) encountered 2. The writer H. P. Lovecraft wrote many ____ stories about the supernatural. (A) essential (B) eerie (C) exhilarating 3. A new of aviation began in 1947 when Chuck Yeager became the first pilot to fly faster than the speed of sound. (A) fable (8) endeavor (C) era 4. Vance Packard's book The Hidden Persuaders deals with the tactics advertisers use to _ _ __ consumers. (A) endorse (C) enhance (8) entice 5. Riding a roller coaster is an ____ experience. (A) exhilarating (8) elegant (C) exhaustive 6. Before the plane takes off, passengers must ____ their seat belts. (A) flee (8) emit (C) fasten 7. In the United States, citizens are to vote at the age of eighteen. (A) essential (8) elderly (C) eligible 8. Barracudas are ____ predators, sometimes called the "tigers" of tropical waters. (A) elegant (8) futile (C) ferocious 9. Certain gases such as neon ____ light when exposed to an electrical current. (A) emit (8) evade (C) esteem 10. People make more mistakes when they are ____ than when they are fresh. (A) exhaustive (8) eminent (C) fatigued 11. A in a jewel makes it less valuable. (A) fragment (B) facet (C) flaw 12. Honeysuckle is a shrub that has white or yellowish blossoms. (A) flagrant (8) fragrant (C) exhilarating 364 Section 3 • Guide to Reading Comprehension Mini-Lesson 3.7 gala ad}. festive, happy, joyous gap n. break, breach, opening garrulous ad}. talkative gaudy adj. showy, flashy, ostentatious genial ad}. pleasant, cordial, agreeable gentle ad}. mild, kind, considerate genuine ad}. authentic, real, valid glitter v. sparkle, shine, glisten glory n. grandeur, majesty, fame gorgeous ad}. attractive, beautiful graphic ad}. clear, explicit, vivid grasp v. (1) grab, seize, grip (2) understand grave ad}. serious, grievous, solemn, somber gregarious ad}. sociable, friendly grim ad}. severe, dreary, bleak, somber grip v. hold, grasp, seize grueling adj. exhausting, difficult gullible ad}. innocent, naive, trusting, credulous hamper v. delay, obstruct, hinder, block haphazard ad}. random, chance, aimless, unplanned hardship n. difficulty, trouble harm v. injure, damage harmony n. accord, agreement, peace harness v. control, use harsh ad}. severe, rough, strict hasty ad}. quick, rushed, hurried hazardous adj. dangerous, risky heed v. obey, listen to, mind, follow hinder v. block, obstruct, hamper hoist v. lift, raise, pick up hue n. color, tint, shade huge ad}. enormous, giant, colossal, immense hurl v. pitch, throw, fling Section 3 • Guide to Reading Comprehension 365 Exercise: Complete the following sentences by ftlling in the blanks with vocabulary item (A), (B), or (C) according to the context of the sentences. 1. During the construction of skyscrapers, cranes are used to ____ building materials to the upper floors. (C) hoist (A) hurl (B) harness 2. The twenty-six mile-long Boston Marathon is a ____ foot race. (C) hasty (A) gorgeous (B) grueling 3. Dams can ____ the power of rivers, but they may also destroy their beauty. (A) heed (B) harness (C) hurl 4. The more facets a diamond has, the more it _ _ __ (B) harms (A) glitters (C) hinders 5. Many people celebrate the New Year with ____ parties. (A) gala (B) grueling (C) haphazard 6. Think it over for a while; don't make a ____ decision. (B) gullible (A) genuine (C) hasty 7. Bad weather ____ the rescue crews trying to locate the life rafts. (A) hampered (B) grasped (C) harnessed 8. Gorillas look ferocious but are actually quite ____ creatures. (B) gentle (A) gaudy (C) gorgeous 9. Con artists are criminals who take advantage of ____ people by tricking them and taking their money. (A) garrulous (B) grim (C) gullible 10. Working with toxic materials is a occupation. (A) hazardous (B) genial (C) haphazard 11. Most minnows are tiny fish, but squaw fish, which can weigh as much as 30 pounds, are actually ____ minnows. (C) huge (A) gregarious (B) gaudy 12. At first, the results of the experiment seemed ____ , but finally a pattern emerged. (A) haphazard (B) grave (C) genuine 366 Section 3 • Guide to Reading Comprehension Mini-lesson 3.8 idea n. concept, notion, thought ideal n. model, standard adj. perfect, model, standard idle adj. (1) inactive, unused, inert (2) lazy illusion n. fantasy, delusion inlaginary adj. unreal, fantastic, fictitious inlaginative adj. creative, original, clever inunense ad}. huge, enormous, massive, colossal inlpair v. damage, injure, spoil inlpartial adj. fair, unbiased, neutral inlplement l1. realize, achieve, put into practice, execute n. tool, utensil, instrument incessant ad}. constant, ceaseless, continuous increment n. increase, amount indifferent ad}. uncaring, apathetic, unconcerned indigenous adj. native indispensable adj. necessary, essential, vital, critical indistinct ad}. unclear, blurry, hazy induce v. persuade, convince, coax inept adj. incompetent, awkward, clumsy inexorable adj. unstoppable infamous ad}. notorious, shocking infinite ad}. limitless, endless, boundless infinitesinlal ad}. tiny, minute, minuscule ingenious ad}. brilliant, imaginative, clever, inventive ingenuous ad}. naive, trusting inhabit v. live, dwell, reside, populate inhibit v. control, limit, restrain initial adj. original, first, beginning, introductory innate ad}. natural, inborn intense adj. powerful, heightened, concentrated intricate ad}. complicated, complex, involved irate adj. angry, furious, upset jagged ad}. rough, rugged, uneven, irregular jeopardy n. danger, hazard, risk, threat jolly ad}. joyful, happy, cheerful, jovial jolt v. shock, jar, shake up, surprise n. blow, surprise, shock keen ad}. (1) sharp (2) shrewd, clever, bright (3) eager, enthusiastic keyadj. principal, crucial, important knack n. skill, ability, aptitude, talent Section 3 • Guide to Reading Comprehension 367 Exercise: Complete the following sentences by filling in the blanks with vocabulary item (A), (B), or (C) according to the context of the sentences. 1. Many people feel that Hawaii has an almost ____ climate. (A) idle (B) impartial (C) ideal 2. A plow is a farm ____ used to break up soil and prepare the land for planting. (B) knack (C) implement (A) increment 3. A laser uses a synthetic ruby to concentrate light into an extremely ____ high-energy beam. (A) intense (B) indistinct (C) imaginary 4. Jesse James was an _ _ _ _ outlaw, well-known as a bank robber and gunfighter. (A) inept (B) ingenuous (C) infamous 5. Antibiotics (A) inhabit the growth of bacteria. (B) jolt 6. Optical ____ deceive the eye with tricks of perception. (A) illusions (B) ideals (C) inhibit (C) increments 7. Stockholders may be too ____ to vote in corporate elections, so they let management vote for them by proxy. (B) indifferent (C) ingenious (A) infamous 8. The heavily populated states of Ohio, Pennsylvania, Illinois, and Michigan are ____ states for any candidate in a presidential election. (A) initial (B) impartial (C) key 9. A virus is so ____ that it can be seen only with an electron microscope. (A) infinite (B) imaginary (C) infinitesimal 10. The (A) keen character Falstaff is one of Shakespeare's frnest comic creations. (B) jolly (C) irate 11. Anyone can learn basic cooking skills; you don't need a special _ _ __ (A) knack (B) idea (C) implement 12. Alcohol (A) jolts one's ability to drive. (B) impair (C) induce 13. The people of Australia were called Aborigines by the English settlers. (B) ingenuous (C) innate (A) indigenous 14. The rhinoceros has a poor sense of sight but ____ sense of smell. (A) an impartial (B) an inept (C) a keen 15. The equator is ____ line running around the center of the Earth. (A) an imaginative (B) a jagged (C) an imaginary 16. A glacier's progress is slow but _ _ __ (A) inexorable (B) impartial (C) infInite 368 Section 3 • Guide to Reading Comprehension Mini-Lesson 3.9 lack v. need, require, not have n. shortage, absence, scarcity lag v. fall behind, go slowly lavish ad}. luxurious, plentiful, abundant lax ad}. careless, negligent, loose legendary ad}. mythical, fabled, famous legitimate ad}. proper, authentic, valid lethargic adj. slow, listless, sluggish, lazy likely ad}. probable, plausible linger v. remain, stay link v. join, connect, fasten, bind n. connection, tie long v. desire, wish for lucid ad}. clear, plain, understandable lull v. soothe, calm, quiet n. pause, break lure v. attract, tempt, entice lurid ad}. shocking, sensational, graphic lurk v. prowl, sneak, hide luster n. shine, radiance, brightness luxurious ad}. lavish, elegant, plush magnificent adj. majestic, impressive, splendid magnitude n. size, extent, amount mandatory ad}. necessary, obligatory, compulsory mar v. damage, ruin, deface, spoil memorable ad}. unforgettable, impressive, striking mend v. ftx, repair mild adj. gentle, moderate, calm mingle v. blend, combine, mix minute ad}. tiny, minuscule, inftnitesimal monitor v. observe, watch moral ad}. honorable, ethical morale n. spirit, conftdence, attitude murky ad}. unclear, cloudy mysterious ad}. puzzling, strange mythical ad}. legendary, imaginary, ftctional Section 3 • Guide to Reading Comprehension Exercise: Complete the following sentences by filling in the blanks with vocabulary item (A), (B), or (C) according to the context of the sentences. 1. It's difficult for scuba divers to see when the water is _ _ __ (A) murky (B) lucid (C) magnificent 2. In most cultures, it is traditional to prepare ____ meals to celebrate holidays. (A) lurid (B) lethargic (C) lavish 3. Parents often sing to children to them to sleep. (A) lurk (B) mingle (C) lull 4. Julius Caesar is known not only for his military and political skills but also for his ____ , informative writing. (B) lurid (C) lethargic (A) lucid 5. A cobbler ____ damaged shoes. (B) mends (A) mars (C) lacks 6. One of the ____ exhibits ofImpressionist art is found at the Art Institute of Chicago. (B) most memorable (C) most lucid (A) mildest 7. The USO is a service organization which entertains U.S. troops and improves their _ _ __ (A) morale (B) luster (C) lack 8. Quarks are ____ particles that are believed to be the fundamental unit of matter. (A) massive (B) minute (C) mythical 9. Some people like to (A) lag after dinner over coffee and dessert. (B) long (C) linger 10. Paperback novels in the 1940's and 1950's often had ____ covers to attract readers' attention. (A) lurid (B) murky (C) legitimate 11. One problem caused by a rising crime rate is a ____ of space in prisons. (A) lag (B) lack (C) link 12. The lumberjack Paul Bunyan and his giant blue ox Babe are two of the most famous characters in American folklore. (A) legendary (B) moral (C) minute 369 370 Section 3 • Guide to Reading Comprehension Mini-Lesson 3.10 negligible adj. unimportant, trivial nimble adj. graceful, agile notable adj. remarkable, conspicuous, striking notify v. inform notion n. idea, concept, thought notorious adj. infamous, disreputable novel adj. new, innovative objective n. goal, purpose, aim adj. fair, impartial, unbiased, neutral oblong adj. oval obscure adj. unfamiliar, ambiguous, little-known obsolete adj. antiquated, out-of-date, outmoded odd adj. strange, unusual, peculiar, curious offspring n. young, children, descendants ominous adj. threatening, menacing, dangerous opulent adj. luxurious, plush, affluent ornamental adj. ornate, decorative, elaborate outgoing adj. (1) open, friendly (2) departing, leaving outlook n. (1) opinion, view (2) prospect, forecast outstanding ad}. excellent, exceptional, notable, well-known overall ad}. general, comprehensive overcast ad}. cloudy, gloomy overcome v. subdue, defeat, overwhelm overlook v. ignore, disregard, neglect oversee v. supervise, manage, direct oversight n. error, mistake, omission overt ad}. open, obvious overtake v. catch up with, reach overwhelm v. (1) astonish, astound (2) inundate, overcome, engulf (3) conquer, defeat, overpower Section 3 • Guide to Reading Comprehension 371 Exercise: Complete the following sentences by filling in the blanks with vocabulary item (A), (B), or (C) according to the context of the sentences. 1. Who should be ____ in case you are involved in an accident? (B) notified (A) overlooked (C) overtaken 2. Many of the world's most ____ restaurants are located in lUXury hotels. (A) outgoing (B) opulent (C) overt 3. The black clouds of a gathering thunderstorm look quite _ _ __ (A) ominous (B) negligible (C) overcast 4. Pulitzer Prizes are awarded to ____ journalists, novelists, poets, and other writers. (A) objective (B) outstanding (C) notorious 5. An plant is cultivated chiefly for its beauty. (A) opulent (B) obscure (C) ornamental 6. Franklin D. Roosevelt was able to ____ his physical handicaps; he didn't permit them to interfere with his living a vigorous life. (A) oversee (B) overcome (C) overtake 7. The poetry of Ezra Pound is sometimes difficult to understand because it contains so many ____ references. (A) notable (B) obscure (C) objective 8. The Bessemer Process was once the most common method of making steel, but today this process is considered _ _ __ (B) novel (C) obsolete (A) odd 9. Gregarious people are friendly and _ _ __ (A) overcast (B) nimble 10. The town planning commission said that their financial optimistic; they expect increased tax revenues. (A) outlook (B) oversight (C) outgoing for the next fiscal year was (C) notion 11. The new play was so successful that the demand for tickets was _ _ __ (A) odd (B) overwhelming (C) negligible 12. A book's table of contents provides readers with an ____ idea of what the book is about. (A) outgoing (B) overt (C) overall 13. Because ultraviolet light from the Sun can penetrate clouds, it is possible to get a sunburn on an ____ day. (A) obscure (B) overcast (C) opulent 14. Although the accident appeared serious, only a ____ amount of damage was done. (A) novel (B) notable (C) negligible 372 Section 3 • Guide to Reading Comprehension Mini-Lesson 3.11 pace n. rate, speed painstaking adj. careful, conscientious, thorough pale ad}. white, colorless, faded, blanched paltry ad}. unimportant, minor, trivial particle n. piece, bit, fragment path n. trail, track, way, route peculiar ad}. (1) strange, odd, puzzling (2) distinctive, characteristic, unique, special penetrate v. enter, go through, pierce, puncture perceive v. observe, sense, notice peril n. danger, hazard, risk, threat perpetual ad}. constant, endless, eternal perplexing ad}. puzzling, mystifying, confusing pierce v. penetrate, puncture, stab plausible ad}. likely, credible, believable plead v. appeal, beg plush ad}. opulent, luxurious, elegant ponder v. consider, think about, reflect on portion n. share, part, section, segment postpone v. delay, put off, defer potent adj. strong, powerful, effective pounce v. jump, leap, spring precious ad}. expensive, costly, rare precise adj. accurate, exact, definite premier ad}. (1) first, opening, earliest, initial (2) chief, leading, foremost pressing ad}. urgent, crucial, compelling pretext n. excuse, pretense, justification prevail v. succeed, win, triumph prevalent ad}. common, widespread, popular prior ad}. earlier, preceding, former probe v. investigate, inquire into procure v. obtain, acquire, secure profound ad}. important, significant, deep profuse ad}. plentiful, abundant prompt ad}. punctual, timely prosper v. flourish, thrive, succeed provoke v. (1) irritate, anger, annoy (2) cause, trigger prudent ad}. careful, sensible, cautious pulverize v. crush, grind, powder pungent ad}. (1) bitter, harsh, biting, sharp (2) spicy, sour, tart pursue v. chase, follow, seek puzzling adj. mystifying, confusing, baffling Section 3 • Guide to Reading Comprehension 373 Exercise: Complete the following sentences by filling in the blanks with vocabulary item (A), (B), or (C) according to the context of the sentences. 1. Turquoise is not valuable enough to be classified as a ____ stone. (B) pale (C) precious (A) perpetual 2. Employers often require job applicants to have (A) premier (B) prior experience in the field. (C) plush 3. Hospitals define urgent care as medical care given to somewhat less medical problems than emergency care. (A) perplexing (B) pressing (C) prudent 4. Tool makers must have the ability to work very ____ in order to meet exact specifications. (C) plausibly (A) precisely (B) profoundly 5. NASA ____ the launch of space vehicles on account of bad weather or technical problems. (A) ponders (B) postpones (C) probes 6. of dust in the air may trigger allergies in some people. (A) Portions (8) Pretexts (C) Particles 7. The Appalachian Trail, extending from Maine to Georgia, is the longest continuous hiking ____ in the world. (A) pace (B) peril (C) path 8. When a tiger spots its prey, it crouches down and then _ _ __ (B) ponders (A) pleads (C) pounces 9. X-rays cannot ____ lead. (A) provoke (C) pursue (B) penetrate 10. Morphine, a form of synthetic heroin, is a ____ painkiller. (A) potent (B) pungent (C) paltry 11. Sherlock Holmes, a fictional detective, solved many ____ crimes. (A) puzzling (B) prevalent (C) prompt 12. Mallows are a type of wildflower that grows ____ in prairies, woods, and marshes. (A) profusely (B) profoundly (C) preciously 13. Certain spices give foods a ____ taste. (B) pungent (A) prudent (C) pale 14. Trade with Britain and the West Indies allowed colonial seaports such as Boston to _ _ __ (A) postpone (B) provoke (C) prosper 15. A investor never takes unnecessary financial risks. (A) perplexing (8) prudent (C) premier 374 Section 3 • Guide to Reading Comprehension Mini-Lesson 3.12 quaint at(j. charming, picturesque, curious, old-fashioned quake [1. shiyer, shake, tremble quandary n. problem, dilemma, predicament quarrel n. argument, dispute, disagreement quest n. search, journey, Yenture radiant ad). bright, shiny, glowing ragged ad). torn, tattered, worn range 11. scope, extent, spectrum 0. (1) extend, vary, fluctuate (2) roam, wander rash ad';' thoughtless, careless, reckless raw ad). (1) uncooked (2) unprocessed, unrefined, crude raze v. demolish, level, knock down recede v. retreat, go back, subside, withdraw reckless ad). careless, rash recollect v. recall, remember recount II. narrate, tell refine u. improve, process, purify refuge n. shelter, haYen, retreat rehearse lI. practice, train, go over reliable ad). dependable, trustworthy relish u. enjoy, savor, like remedy n. treatment, cure remnant n. remainder, balance, fragment remote at(j. isolated, distant renowned at(j. famous, celebrated, notable resent t'. dislike, take offense at retract 1'. withdraw, pull back riddle n. puzzle, mystery rigid ad';' (1) stiff, unbending (2) harsh, severe, strict rip l( tear, cut, slash ripe ad). mature, developed, grown, ready risky ad). dangerous, hazardous, treacherous roam v. trayel, wander, range rough a{{j. (1) uneven, jagged, rugged (2) djmcult (3) impolite route n. way, course, path, road rudimentary ad). elementary, fundamental, primative rugged ad). (1) jagged, rough, uneven (2) strong, sturdy rumor n. gossip, hearsay, story rural (lc(j. agricultural ruthless ad). cruel, brutal, vicious Section 3 • Guide to Reading Comprehension 375 Exercise: Complete the following sentences by ftlling in the blanks with vocabulary item (A), (B), or (C) according to the context of the sentences. 1. Motorists can be fined for driving _ _ __ (B) reliably (A) recklessly (C) ruthlessly 2. Millions of bison once ____ the plains of North America. (A) recollected (B) ripped (C) roamed 3. Musicians have to (A) rehearse (C) recount before performing. (B) resent 4. At the end of the Ice Age, glaciers began to _ _ __ (B) raze (A) quake (C) recede 5. Big Sur, a wild section of California's coastline, is known for its ____ beauty. (A) ragged (B) rash (C) rugged 6. Wetlands provide ____ for many species of birds, reptiles, mammals, and amphibians. (A) riddles (B) refuge (C) rumors 7. The pirate Blackbeard had a reputation for being a harsh, ____ man. (B) quaint (C) reliable (A) ruthless 8. Wrecking balls are used to _ _ _ _ bUildings. (B) quake (A) rip (C) raze 9. The northernmost section of the Rocky Mountains, the Brooks Range, is located in a _ _ __ section of Alaska. (A) remote (B) reliable (C) radiant 10. Dogs can hear a greater ____ of sounds than humans. (B) quandary (A) remnant (C) range 11. Patent medicine salesmen have claimed to have ____ for all types of ailments, from cancer to baldness to the common cold. (A) remedies (C) riddles (B) quandaries 12. Visitors to Vermont delight in the beautiful scenery and picturesque villages and enjoy staying in some of the country inns there. (A) rudimentary (B) ragged (C) quaint 13. ____ materials have less economic value than processed materials. (A) Raw (B) Rash (C) Renowned 14. Many medieval stories dealt with , such as the story of the search for the Grail. (A) quarrels (B) quandaries (C) quests 15. The Tennessee Valley Authority helped bring cheap electricity to farmers in the ____ South. (A) reliable (B) rural (C) rugged 376 Section 3 • Guide to Reading Comprehension Mini-Lesson 3.13 salvage v. save, rescue, recover, retrieve scale v. climb n. (1) range, spectrum (2) proportion scarce adj. rare, sparse, unusual scatter v. disperse, spread scent n. aroma, fragrance, odor, smell scrap v. abandon, get rid of 11. piece, fragment seasoned adj. experienced, veteran secluded adj. hidden, isolated, secret sensational adj. thrilling, exciting, shocking serene adj. quiet, peaceful, calm, tranquil sever v. cut, slice off, break severe adj. (1) harsh, strict, austere (2) undecorated, plain shatter v. break, smash, fragment sheer adj. (1) steep, sharp, abrupt (2) transparent, thin, filmy shinuner u. shine, glow, glisten, gleam shred v. rip up, tear up shrewd adj. clever, sly shrill adj. piercing, high-pitched shun v. avoid, stay away from shyadj. timid, reserved significant adj. important, vital, major signify v. symbolize, stand for, indicate sketch v. draw 11. drawing, picture, diagram slender adj. thin, slim, slight sluggish adj. slow, listless, lazy, lethargic sly adj. cunning, clever, shrewd soak v. wet, drench, saturate solace 11. comfort, consolation, relief somber adj. serious, grave, solemn sort u. classify, categorize n. type, kind, variety sound 11. noise adj. safe, solid, secure Section 3 • Guide to Reading Comprehension Exercise: Complete the following sentences by filling in the blanks with vocabulary item (A), (B), or (C) according to the context of the sentences. 1. Skunks use a pungent ____ as their first line of defense. (B) scrap (A) scent (C) sound 2. One of the most popular peaks for mountain climbers to ____ is El Capitan in Yosemite National Park. (A) scale (B) soak (C) shun 3. Foxes are not particularly rare, but they are not often seen because they are so _ _ __ (A) sluggish (B) somber (C) shy 4. The Civil Rights Act of 1964 was a particularly ____ piece of legislation. (A) serene (B) significant (C) slender 5. (A) Shrill workers are more valuable to employers than beginners. (B) Seasoned (C) Sluggish 6. The Shakers were a strict religious group that ____ all types of pleasure. (A) scrapped (B) shunned (C) sketched 7. Denver's plan to build a subway system was ____ in the 1970's. (A) scattered (B) sorted (C) scrapped 8. Even after a ship has sunk, its cargo can often be _ _ __ (A) severed (B) shattered (C) salvaged 9. Some economists believe that the best way to get a ____ economy moving again is to cut taxes. (A) sensational (B) sluggish (C) shrewd 10. Government bonds and blue-chip stocks are ____ investments. (A) sound (B) shy (C) scarce 11. If a person's spinal cord is ____ , paralysis results. (A) soaked (B) severed (C) salvaged 12. Silk is a ____ fabric. (A) sheer (C) slender (B) shrewd 377 378 Section 3 • Guide to Reading Comprehension Mini-Lesson 3.14 span v. extend, bridge, connect n. length, extent, range spawn v. generate, create, produce n. offspring, descendants specific adj. definite, particular, exact specimen n. example, sample spectacular adj. dramatic, sensational, impressive spell n. interval, period, time spirited adj. lively, energetic, vigorous splendid adj. excellent, superb, wonderful spoil v. (1) ruin, mar (2) decay, deteriorate, decompose, rot spot v. locate, find, see n. (1) location, site (2) mark, stain, speck spur v. stimulate, impel, encourage, provoke n. inducement, stimulus stable adj. steady, secure, stationary, fixed stage v. present, put on n. grade, step, level, phase stain v. color, tint, discolor, dye n. spot, mark, blemish stale adj. (1) old, dry (2) dull, trite, uninteresting stall v. halt, delay, put off stately adj. dignified, grand, magnificent, elegant steep adj. sheer, perpendicular stern adj. firm, severe, strict, harsh strife n. conflict, dispute, struggle strive v. attempt, try struggle v. fight, argue, dispute n. conflict, strife, battle, effort stubborn adj. rigid, uncompromising, obstinate sturdy adj. strong, rugged, well-built subsequent adj. later, succeeding, following, ensuing subtle adj. indirect, suggestive, implied suitable adj. appropriate, correct, apt summit n. peak, apex, zenith sundry adj. miscellaneous, diverse, various superb adj. excellent, splendid supplant v. replace, substitute for supple adj. pliable, flexible, bendable sway v. (1) wave, rock, swing, bend (2) persuade, influence sweeping adj. complete, exhaustive, general, comprehensive swift adj. fast, quick, rapid swivel v. rotate, spin, turn Section 3 • Guide to Reading Comprehension 379 Exercise: Complete the following sentences by mling in the blanks with vocabulary item (A), (B), or (C) according to the context of the sentences. 1. High pressure cells may bring brief warm ____ even in the middle of winter. (A) struggles (B) spells (C) spans 2. The ____ cliffs of the Na Pali Coast of Kauai Island, Hawaii, rise some 4,000 feet from the sea. (A) still (B) steep (C) subtle 3. The process of refining oil involves a number of _ _ __ (B) spots (A) specimens (C) stages slightly. 4. In high winds, skyscrapers will (A) swivel (B) sway (C) stall 5. Severe thunderstorms may ____ tornadoes. (A) spot (B) spawn (C) span 6. The snow-covered ____ of Mount Hood is the highest point in the state of Oregon. (A) spur (B) summit (C) span 7. D. W Griffith was the first director of ____ mms. These were movies made on a colossal scale. (A) specific (C) spectacular (B) suitable 8. Cheetahs are the ____ of all land mammals, with top speeds of up to 70 miles per hour. (A) stalest (B) subtlest (C) swiftest 9. Salt can be used to keep meat from _ _ __ (A) struggling (B) spoiling (C) stalling 10. Because they must be able to break a path through icebound waters, icebreakers have to be very ____ boats. (A) stern (B) sturdy (C) supple 11. Most people who divorce ____ remarry. (B) subsequently (A) specifically (C) stubbornly 12. A roadbed supplies a (A) stable (C) sweeping base for a highway. (B) sundry 13. The geographical center of the North American continent is a ____ near Balta, North Dakota. (A) spot (B) stage (C) summit 14. Many medical tests require a blood _ _ __ (A) spell (B) specimen (C) stain 15. Because of their protective coloration, ghost crabs are hard to _ _ __ (B) spawn (C) spot (A) spur 380 Section 3 • Guide to Reading Comprehension Mini-Lesson 3.15 tact n. diplomacy, discretion, poise tale n. story tame v. domesticate, master ad}. docile, domesticated, gentle tamper (with) v. interfere (with) tangle v. knot, twist tart ad}. sour, tangy, piquant taunt v. insult, mock, torment tedious ad}. boring, dull, tiresome telling ad}. effective, convincing, forceful temperate ad}. mild, moderate tempting ad}. alluring, attractive, enticing tender adj. (1) delicate, soft (2) gentle, loving (3) sore, painful thaw v. melt thorough ad}. complete, comprehensive thoroughfare n. street, boulevard thrifty adj. economical, inexpensive thrilling ad}. exciting, stimulating, stirring thrive v. prosper, flourish thwart v. prevent, impede, obstruct tidings n. news, message tilt v. incline, slope timid ad}. fearful, shy, retiring tint v. color, hue, shade, tone n. color, stain, dye tiresome adj. tedious, dull, boring toil v. labor, work n. exertion, labor, work tolerant ad}. patient, impartial, open-minded torment v. taunt, abuse, bully torrent n. flood, deluge tow v. haul, draw, pull, drag toxic adj. poisonous, noxious trait n. characteristic, feature, quality treacherous ad}. dangerous, hazardous trickle n. drip, leak triumph n. victory, success, achievement trivial ad}. unimportant, minor trying adj. demanding, difficult, troublesome tug v. pull, draw Section 3 • Guide to Reading Comprehension 381 Exercise: Complete the following sentences by filling in the blanks with vocabulary item (A), (B), or (C) according to the context of the sentences. 1. Citric acid gives citrus fruit their (A) temperate taste. (B) toxic (C) tart 2. The use of robots and automated machinery has eliminated certain ____ factory jobs. (A) tedious (B) thrilling (C) timid 3. One should never buy a food or medicine if the packaging has obviously been _ _ __ (A) tangled (B) thwarted (C) tampered with 4. Alfred Hitchcock directed a number of ____ psychological dramas; among the most exciting are Psycho and North by Northwest. (C) thrilling (A) timid (B) trivial 5. Tides are caused by the ____ of the Moon's gravity. (A) tangle (B) torrent (C) tug 6. Many people find chocolate _ _ __ (A) tempting (B) tender (C) temperate 7. Peachtree Street is the main ____ in Atlanta. (A) triumph (B) thoroughfare (C) tilt 8. In her book Silent Spring, Rachel Carson wrote about insecticides and their ____ effects on animal life. (A) tiresome (B) tender (C) toxic 9. In the desert, dry creek beds may turn into raging ____ after heavy rainstorms. (A) trickles (B) torrents (C) toils 10. Barley can be grown almost anywhere in the temperate zone. Unlike most other grains, it even ____ at high altitudes. (A) tampers (B) thrives (C) thaws 11. (A) Traits such as hair color and eye color are inherited genetically from one's parents. (B) Triumphs (C) Tints 12. Washington Irving collected and interpreted many famous old ____ , including the legends of Rip Van Winkle and the Headless Horseman. (C) traits (A) tales (B) tidings 382 Section 3 • Guide to Reading Comprehension Mini-Lesson 3.16 ultimate ad}. (1) conclusive, definite, final (2) maximum, highest, best unbearable ad}. intolerable, agonizing uncouth ad}. impolite, rude, vulgar undertake v. try, attempt ungainly ad}. awkward, unskillful uniform adj. consistent, regular unique adj. singular, one of a kind, special unruly ad}. unmanageable, disorganized, disorderly unsound ad}. defective, faulty, unsafe uphold v. support, sustain upkeep n. maintenance uproar n. disorder, disturbance, commotion urge v. encourage, advise, implore urgent ad}. pressing, compelling utensil n. tool, implement, device utter v. say, speak ad}. total, absolute, complete vacant ad}. empty, unoccupied vague ad}. unclear, uncertain, ambiguous vain ad}. (1) useless, pointless (2) conceited, proud valid adj. genuine, authentic, legitimate vanish v. disappear variable ad}. changeable, shifting vast adj. huge, enormous, extensive, immense venomous ad}. poisonous verbose adj. talkative, wordy verge n. brink, edge, threshold vessel n. (1) container, bottle (2) ship vex v. irritate, anger, annoy viable ad}. (1) alive, living (2) feasible, practical, possible vicinity n. area, proXImIty, zone vigorous ad}. dynamic, energetic, spirited vital ad}. critical, crucial, key, essential vivid ad}. clear, distinct, graphic vow v. promise, pledge, swear n. oath, promise, pledge Section 3 • Guide to Reading Comprehension 383 Exercise: Complete the following sentences by filling in the blanks with vocabulary item (A), (B), or (C) according to the context of the sentences. 1. To be fair, laws must be ____ applied to all persons. (A) urgently (B) vaguely (C) uniformly 2. The rattlesnake is the most common snake in the United States. (A) ungainly (B) venomous (C) variable 3. The League of Women Voters ____ all citizens to vote. (B) vexes (A) urges (C) upholds 4. In his novel The Red Badge oj Courage, Stephen Crane ____ describes a Civil War battle. (A) vividly (8) uniformly (C) vitally 5. An Erlenmeyer flask is a glass ____ used in chemistry labs. (A) vessel (8) vow (C) verge 6. Aerobics is ____ form of exercise. (8) an uncouth (A) a viable (C) a vigorous 7. A metropolitan area consists of a central city and any suburban areas in its _ _ __ (A) vicinity (B) vessel (C) upkeep 8. Special police tactics are required to deal with riots or ____ crowds. (A) ungainly (B) unruly (C) unsound 9. The kidneys playa ____ role in maintaining health by removing impurities from the bloodstream. (A) vivid (B) viable (C) vital young man who stares at his 10. The myth of Narcissus tells the story of a handsome but reflection in a pool of water for so long that he turns into a flower. (A) vain (8) ungainly (C) verbose 11. The fork has been used as an eating ____ since at least the twelfth century. (A) vessel (B) utensil (C) verge 12. The Great Plains cover ____ area. (A) a vast (B) a viable (C) an ultimate 384 Section 3 • Guide to Reading Comprehension Mini-lesson 3.17 wage n. salary, pay, earnings wander v. roam, travel, range wane v. shrink, decrease, decline ware n. good, merchandise warp v. deform, bend, twist wary ad}. careful, cautious, alert weary ad}. tired, exhausted, fatigued well-to-do ad}. rich, wealthy, affluent wholesome ad}. healthy, nutritious, beneficial wicked ad}. evil, corrupt, immoral widespread ad}. extensive, prevalent, sweeping wily ad}. crafty, cunning, shrewd wise ad}. astute, prudent, intelligent withdraw v. retreat, pull out, remove wither v. dry, shrivel, wilt withhold v. reserve, retain, hold back witty ad}. comic, clever, amusing woe n. trouble, distress, sorrow wonder v. think about, speculate, ponder n. marvel, miracle wound lJ. injure, hurt n. injury yearn lJ. desire, crave, want yield v. (1) give up, surrender (2) produce, supply n. production, output, crop zealous ad}. enthusiastic, eager zenith n. peak, tip, apex zone n. area, vicinity, region Section 3 • Guide to Reading Comprehension Exercise: Complete the following sentences by filling in the blanks with vocabulary item (A), (B), or (C) according to the context of the sentences. 1. If boards become wet, they may _ _ __ (A) wither (B) yield (C) warp 2. Whole grains and fresh fruit and vegetables are _ _ _ _ foods. (A) wicked (B) wholesome (C) well-to-do 3. You must be _ _ _ _ when buying a used car; be sure the engine is in good condition. (A) weary (B) zealous (C) wary 4. In the past, many salesmen tried to sell their _ _ _ _ door- to-door. (B) woes (C) yields (A) wares 5. Humorist Will Rogers wrote many (A) wily (B) weary newspaper columns. (C) witty 6. Congress sets the minimum _ _ _ _ • which is the lowest amount of money workers may be paid per hour. (A) wage (B) yield (C) zone 7. Intelligent policies are needed so that public funds are used _ _ __ (A) wholesomely (B) zealously (C) wisely 8. Fertilizers can increase farmers' _ _ __ (A) wonders (B) yields (C) woes 9. Some superstitions are familiar to many cultures. For example, there is a _ _ _ _ belief that black cats bring bad luck. (A) widespread (B) wily (C) wicked 10. A green belt is a parklike _ _ _ _ around a city in which no development is permitted. (A) zenith (B) wound (C) zone 385 Guide to the Test of Written English (TWE) 387 388 Guide to the Test of Written English (TWE) About the Test of Written English Introd uction The TWE (Test of Written English) is designed to measure your ability to write an academic essay. The TWE is NOT given with Institutional TOEFL tests. However, it is ALWAYS given with paperbased tests that are administered by ETS. It is administered before the other three sections of TOEFL. There is no additional fee required to take the TWE. TWE differs from the rest of TOEFL in that it is productive. Instead of choosing one of four answer choices, you must write your own short essay. TWE consists of a single essay topic called a prompt; there is no choice as to what to write about. You have thirty minutes in which to write an essay based on the prompt. A typical TWE answer is about 200 to 300 words long and is divided into four or five paragraphs. The most common type ofTWE prompt asks you to write a contrast/opinion essay. In this type of essay, you must contrast two points of view, then defend one of those positions. Another type of essay asks you to select some development, invention, or phenomenon and explain its importance. Prompts are carefully chosen to avoid anything controversial, upsetting, or unfair to a particular group. At ETS, your essay is read by at least two trained readers who score it "holistically." In other words, the essay is not judged according to individual errors you might make but by the overall organization, development, clarity, and effectiveness of your writing. The score is based on a scale of 1 to 6; halfpoint scores (5.5, 4.5, and so on) are also given. The scoring system ETS uses is similar to the one that follows. (You can use this chart to estimate yourTWE score when you take theTWE Practice Tests.) Score Explanation of Score 6 Strongly indicates the ability to write a well-organized, well-developed, and logical essay. Specific examples and details support the main ideas. All the elements of the essay are unified and cohesive. A variety of sentence structures are used successfully, and sophisticated vocabulary is employed. Grammatical errors are infrequent, but a few minor mistakes may occur. 5 Indicates the ability to write an organized, developed, and logical essay. The main ideas are adequately supported by examples and details. Sentence structure may be less varied than that of a level 6 essay, and vocabulary less sophisticated. Some grammatical errors will appear. 4 Indicates a moderate ability to write an acceptable essay. Although main ideas may be adequately supported, weaknesses in organization and development will be apparent. Sentence structure and vocabulary may lack sophistication or be used inappropriately. Grammatical errors may be frequent. 3 Indicates some minimal ability in writing an acceptable essay, but involves serious weaknesses in organization and development. Significant sentence-structure and vocabulary problems occur, and there are frequent grammatical errors that sometimes make the writer's ideas difficult to comprehend. 2 Indicates the inability to write an acceptable essay. Organization and development are very weak or nonexistent. Lacks unity and cohesion. Few if any specific details may be given in support of the writer's ideas. If details are given, they may seem inappropriate. Significant and frequent errors in grammar occur throughout the essay. Writer may not have fully understood the essay topic. 1 Strongly indicates the inability to write an acceptable essay. No apparent development or organization. Sentences may be brief and fragmentary and unrelated to one another. Significant grammatical errors occur throughout the essay and make it difficult to Guide to the Test of Written English (TWE) 389 understand any of the author's ideas. Writer may have completely misunderstood the essay topic. OFF Did not write on the topic assigned. INR Did not write the essay. The average scoreTWE score is between 3.5 and 4.0. TWE is scored separately from the rest of the test and has no effect on your overall TOEFL score. Following are two sample TWE prompts and essays that respond to those prompts. (Note: These essays are based on composite student essays. They have been edited to correct errors in grammar and mechanics.) Sample TWE Prompt A Some people believe that money spent on space research benefits all of humanity. Others take the opposite view and say that money for this type of research is wasted. Discuss these two positions, using examples. Tell which view you agree with and explain why. Notes Intra: Space research + 50 years: expensive-$ well spent or wasted? -ideas costs billions; also human resources; no real benefits e.g., trip to Moon only brought back rocks many important uses for this $ on Earth: e.g., education environment housing + ideas consumer products; e.g., PC's, freeze-dried foods, pacemakers weather & commuI1ication satellites scientific knowledge about planets, Moon, even Earth Conclu: As shown, many benefits-also, human race needs challenge just as individuals do-therefore, space research is worth all the money spent 390 Guide to the Test of Written English (TWE) For over fifty years, a numl7er of nations have l7een involved in the exploration of outer space. This research has l7een very costly, of course. Has this money l7een well-spent or wasted? Some people l7elieve that all or most space research should l7e eliminated because of its incredible expense, not only in terms of money, but also in terms of scientific and human resources. These people point out the fact that it cost billions of dollars to send astronauts to the Moon, but all they brought back were some worthless rocks. These people say that the money and effort now being wasted in outer space could be spent on more important projects right here on Earth, such as providing housing for homeless people, improving the education system, saving the environment, and finding cures for diseases. However, other people believe that space research has provided many benefits to humanity. They point out that hundreds of useful products, from personal computers to heart pacemakers to freeze-dried foods, are the direct or indirect results of space research. They say that weather and communication satellites, which are also products of space programs, have benefited people all over the globe. In addition to these practical benefits, supporters of the space program point to the scientific knowledge that has been acquired about the Sun, the Moon, the planets, and even our own Earth as a result of space research. I agree with those people who support space research and want it to continue. Space research, as shown, has already brought many benefits to humanity. Perhaps it will bring even more benefits in the future, ones that we can't even imagine now. Moreover, just as individual people need challenges to make their lives more interesting, I believe the human race itself needs a challenge, and I think that the peaceful exploration of outer space provides just such a challenge. Guide to the Test of Written English (TWE) Notice that in the fIrst sampieTWE, the writer organized the essay in the following way: , ,, , ,, , ,, , ,, 1. Introduction ,:' 2. Negative points ---------------------~ / / / / / / / / / / / ~/ 4. Conclusion 3. Positive points 391 392 Guide to the Test of Written English (TWE) Sample TWE Prompt B Developments in transportation such as the invention of the automobile have had an enormous impact on modern society. Choose another development in transportation that you think is of great importance. Give reasons for your selection. Notes Intra: One of most important develop is internat'l jet transport-since '50'sbecause of speed & reI. low costs, has changed way people think Speed: 100 yrs ago, took weeks to cross ocean: today, few hrs-this has changed people's concept of space-world smaller Low costs: In past, only wealthy could travel comfortably; poor people had to save for years-today, more and more people can travel: businessmen students tourists Conclusion: countries no longer so isolated; people think of world as they used to think of their own hometowns Guide to the Test of Written English (TWE) 393 I believe that one of the m05t important development5 in tran5portation ha5 been the development of international jet tran5port. Since thi5 5tyle of tran5portation appeared in the 1950'5, it ha5 had 50me revolutionary effect5. BecaU5e of the high 5peed5 and the relatively low C05t of thi5 type of travel, it ha5 changed the way people look at the world. The m05t obvioU51y important characteri5tic of jet travel i5 the high 5peed involved. A hundred year5 ago, it took week5 to cr055 the Atlantic or Pacific ocean5 by 5hip. Today, tho5e 5ame trip5 can be completed in a matter of hour5. One can attend a meeting in Pari5 and have dinner in New York the 5ame day. The5e amazing 5peed5 have changed people'5 concept5 of 5pace. Today the world i5 much 5maller than it wa5 in the pa5t. Another important a5pect of jet travel i5 it5 relatively low C05t. An international journey one hundred year5 ago wa5 extremely expen5ive. Only wealthy people could afford to travel comfortably, in fir5t cla55. Poor people had to 5ave for year5 to purcha5e a ticket, and the condition5 in which they traveled were not very good. Today it i5 P055ible for more and more people in every country to travel in comfort. Thu5 it i5 P055ible for bU5ine55peopie to do bU5ine55 all over the world, for 5tudent5 to attend univer5itie5 in other countrie5, and for touri5t5 to take vacation5 anywhere in the world. In conclu5ion, the 5peed and low C05t of international jet travel have changed the world. Individual nation5 are not a5 i50lated a5 they were in the pa5t, and people now think of the whole planet a5 they once thought of their own hometown5. 394 Guide to the Test of Written English (TWE) Notice that the writer organized the second sample essay as follows: 1. Introduction I I I I I I , 2. 1st benefit (speed) I I I I I I , 3. 2nd benefit (cost) I I I I I I , 4. Conclusion Guide to the Test of Written English (TWE) 395 Ten Keys to Writing the TWE Essay You have only a half-hour in which to complete your work. You should use your time more or less as shown below: Reading and thinking about the prompt Planning and taking notes Writing the essay Checking the essay 2 3 22 3 minutes minutes minutes minutes As with all parts of TOEFL, be familiar with the directions forTWE so that you don't have to waste time reading them. You must write on the prompt exactly as it is given, so be sure you understand it. If you write about another topic, you won't receive a score at all. If you don't completely address the topic, you will receive a lower score. Before you begin to write, spend a minute or two just "brainstorming." Just think about the prompt and the best way to approach it. Remember: there is no "correct" answer forTWE. You can choose any position as long as you can adequately support it. While you're brainstorming, jot down your ideas in the section marked "NOTES". You don't have to write out a formal outline with Roman numerals, capital letters, and so on. However, you should make some notes. By following your notes, you can organize your essay before you write, leaving you free to concentrate on the task of writing. Wben making notes, don't worry about writing complete, grammatical sentences; use abbreviations if possible. The point of taking notes is to simply get your ideas down on paper as quickly as possible. Handwriting that is hard to read may unconsciously prejudice the readers who are grading your essay. Be sure your handwriting is not too small or too large. 396 Guide to the Test of Written English (TWE) AlllWE essays should consist of three basic parts: an introductory paragraph, a body that consists of two or three paragraphs, and a concluding paragraph. You need to include all of these elements in your essay. The introduction states the main idea of the essay in one sentence called the thesis statement and may provide some background about that idea. The body develops the main idea brought up in the introduction. Specific examples are given to make the thesis statement seem stronger and more believable to the reader. The conclusion evaluates and summarizes the material that is in the body. It provides the reader with a sense of closure-the feeling that the essay is really finished, not that the writer simply ran out of time. The exact plan of organization you use depends on the type of prompt you are given. The following patterns could be used for the two main types of prompt commonly given. Of course, these are not the only patterns that could be used in writing 1WE essays, but they are effective plans for organizing your ideas. Prompt Type A: Contrast/Opinion Introduction: Paragraph 1: Present the two sides of the issue; give a brief amount of background information. Body: Paragraph 2: Discuss the negative side of the issue; give examples. Paragraph 3: Discuss the positive side of the issue; give examples. Paragraph 4: Express you own opinion about the issue; give specific reasons for your decision. Conclusion: Prompt Type B: Explain the Importance of a Development, Invention, or Phenomenon Introduction: Paragraph 1: Explain what development you have chosen to write about, and why. Body: Paragraph 2: Discuss one aspect of why this development is important; give examples. Paragraph 3: Discuss another aspect of why this development is important; give examples. Paragraph 4: Summarize the points made in paragraphs 2 and 3. Conclusion: Now look back at the two 1WE sample essays to see if they follow these patterns of organization. Whenever you make a general statement, you should support it with specific examples. Don't just say, "Computers are important to modern business." Give specific examples of how computers can benefit businesses. If you state an opinion, give reasons. Don't just say, "I believe television is harmful to children." Explain exactly why you think television hurts children. Guide to the Test of Written English (TWE) 397 Signal words can be used to join paragraph to paragraph and sentence to sentence. These words make your essay clearer and easier to follow. Some of these expressions and their meanings are given below. Expressions Used to List Points, Examples, or Reasons First example or reason First, ... The first example is ... The first reason for this is that ... Additional examples or reasons Second ... (Third, Fourth) A second (third, fourth) example is that ... Another example is .. . Another reason is that .. . In addition, .. . Furthermore, .. . Moreover, ... Final examples or reasons Finally, ... To give individual examples For example, .. . For instance, .. . To give a specific example, ... X is an example of Y. To show contrast However, ... On the other hand, ... Nevertheless, ... To show a conclusion Therefore, ... Consequently, ... To show similarity Likewise, ... Similarly, . . . To begin a concluding paragraph In conclusion, .. . In summary, .. . 398 Guide to the Test of Written English (TWE) Examples of the Use of Signal Words I agree with the idea of stricter gun control for a number of reasons. First, statistics show that guns are not very effective in preventing crime. Second, accidents involving guns frequently occur. Finall); guns can be stolen and later used in crimes. I believe that a good salary is an important consideration when looking for a career. However, the nature of the work is more important to me. Thus, I would not accept a job that I did not find rewarding. For me, the reasons for living in an urban area are stronger than the reasons for living in a rural community. Therefore, I agree with those people who believe it is an advantage to live in a big city. Look back at the sample essays again. Did the authors use signal words to show transitions? Good writing in English consists of a more or less equal balance between short, simple sentences consisting of only one clause and longer sentences containing two or more clauses. Therefore, make an effort to use sentences of various lengths. You should also vary sentence structures. Begin some sentences with prepositional phrases or subordinate clauses. Examples of various sentence types Instead of ... I agree with this idea for several reasons. Try ... For several reasons, I agree with this idea. Instead of ... I support Idea A even though Idea B has some positive attributes. Try ... Even though Idea H has some positive attributes, I support Plan A. Allow a few minutes to proofread the essay. However, don't make any major changes at this time. Don't cross out long sections or try to add a lot of new material. Look for obvious errors in punctuation, spelling, and capitalization, as well as common grammatical mistakes: subject-verb agreement, wrong tense use, incorrect use of plurals, incorrect word forms, and so on. If you have ever taken a writing class in English, look at the corrections the teacher made on your papers to see what types of mistakes you commonly make, and look for these. Guide to the Test of Written English (TWE) 399 Three Practice TWE Tests The following exams are very similar to actual TWE tests. Time yourself carefully while taking these practice tests. You can use the scoring chart on page 388 to estimate your score. If you are taking an English course, you may want to ask your English teacher to score your test and to make recommendations for improving your essay. Practice TWE Directions 30 Minutes 1. When you are ready, turn the page and carefully read the essay prompt. 2. Before you begin writing, think about the prompt. You will probably want to make some notes to organize your thoughts. Use only the space marked NOTES to write notes or an outline. 3. Write on only one topic. If you do not write on the topic given, you will not receive a score. 4. Your essay should be clear and precise. Support your ideas with facts. The quality of your writing is of more importance than the quantity, but you will probably want to write more than one paragraph. S. Begin your essay on the first line of the essay page. Use the next page if you need to. Write as neatly as possible. Don't write in large letters. Don't skip lines or leave large margins. 6. Check your essay after you have finished. Give yourself enough time to read over your essay and make minor revisions before the end of the exam. 7. After 30 minutes, stop writing and put your pencil down. 400 Guide to the Test of Written English (TWE) TWE Essay Prompt 1 Some people believe that advertising on television is generally beneficial to viewers. Others take the position that television advertising has primarily negative effects. Which position do you agree with? Explain your decision, using specific examples. NOTES Use this space for essay notes only. Write the final version of your essay on the next two pages. Guide to the Test of Written English (TWE) 401 Name: __________________________________________________________________________ Write your essay here. 402 Guide to the Test of Written English (TWE) Guide to the Test of Written English (TWE) 403 TWE Essay Prompt 2 Some people say that university students should concentrate on their own field of study, and that all the classes they take should be closely related to that subject. Others believe that university students should get a general education, taking classes in many fields before concentrating on a single field. Discuss both points of view, using concrete examples. Which view do you support? Give reasons for your choice. NOTES Use this space for essay notes only. Write the final version of your essay on the next two pages. 404 Guide to the Test of Written English (TWE) Name: _______________________________________________________________________ Write your essay here. Guide to the Test of Written English (TWE) 405 406 Guide to the Test of Written English (TWE) TWE Essay Prompt 3 Good, affordable housing is one of the factors that make a community a desirable place to live. Choose one other factor that you feel is important. Give specific reasons for your choice. NOTES Use this space for essay notes only. Write the fmal version of your essay on the next two pages. Guide to the Test of Written English (TWE) 407 Name: __________________________________________________________________________ Write your essay here. 408 Guide to the Test of Written English (TWE) Three Complete Practice Tests 409 410 Three Complete Practice Tests About Taking the Practice Tests One of the best ways to ensure success on TOEFL is to take realistic practice exams. The three tests in this book are accurate and up to date. They duplicate actual exams in terms of format, content, and level of difficulty. They cover all the types of items that commonly appear on actual exams. All items have been pre-tested. These tests can be used in the classroom or by self-study learners. In order to get the most from these practice tests, suggestions for both types of users are given below. Using the Tests in the Classroom • The test should be given under actual testing conditions-for example, desks should be arranged as they would be during a test, and no talking should be allowed. If possible, each test should be given in its entirety rather than section by section. • Sections 2 and 3 should be carefully timed, and students should not work ahead. • Students should mark answers on the answer sheets provided at the end of the book. • An analysis and a discussion of all three parts of the exam is an important follow-up for the practice tests. These activities can be done as a class or in small groups. Test-takers should understand why a choice is correct. Using the Tests for Self-Study • Take each test all at one time rather than section by section. • Use a watch to time yourself carefully during Sections 2 and 3. Do not go ahead to the next section even if you finish early. Do not give yourself extra tinle even if you haven't finished the section. • Sit at a desk or table, not in an easy chair or on a sofa, and work away from distractions such as a television or a stereo. • Mark your answers on the answer sheets rather than in the book. • After completing the test, mark incorrect answers but do not write in the corrections. Instead, go back and answer these questions a second time. • Read the explanations in the Written Expression section and the Reading Comprehension section for all items that you answer incorrectly. • If you have time, take the entire test over again on another answer sheet. (You may want to make photocopies of the answer sheets in the back of the book before you begin.) • Use the scoring charts in the back of this book to calculate your scores for each practice test. • Keep track of your scores in the Personal Score Record in the Scoring section of this book. If you have consistently lower scores on one section of the test, you might want to review the corresponding section in the Guide. • If you have enough time, take the entire test over on a separate answer sheet. Three Complete Practice Tests 411 Scoring the Practice Tests The level of difficulty varies slightly from one TOEFL test to another. ETS uses a statistical process called "test equating" to adjust each set of scores. The chart given here can only be used to determine a range of scores. ETS, of course, reports your score as a single number, not as a range. After completing each test, obtain a raw score for each of the three sections by counting the number of correct answers in the three sections. Then look at the conversion chart to determine the range of scaled scores for each section. Add the three low scores from the range of scores for each section, then the three high scores. Multiply both totals by 10 and divide by 3. Your "actual" TOEFL score will lie somewhere in that range of numbers. For example, suppose that you had 32 correct answers in Listening Comprehension, 29 in Structure and Written Expression, and 37 in Reading Comprehension: 49 + 50 + 53 = 152 50 + 52 + 54 = 156 152 X 10 = 1,520 156 X 10:;;:: 1,560 -7-7- 3 = 507 3 = 520 Your score on the practice test would be between 507 and 520. 48-50 45-47 42-44 39-41 36-38 33-35 30-32 27-29 24-26 21-23 18-20 15-17 12-14 9-11 6-8 3-5 0-2 65-68 57-64 55-57 54-55 52-54 50-52 49-50 47-48 45-47 44-45 42-44 39-41 36-38 33-36 29-32 25-28 23-24 39-40 36-38 34-35 31-33 29-30 27-28 24-26 21-23 18-20 15-17 12-14 9-11 6-8 3-5 0-2 64-68 60-64 57-59 53-56 50-52 49-50 48-49 46-48 43-45 39-42 36-38 32-35 28-32 24-27 20-23 48-50 45-47 42-44 39-41 36-38 33-35 30-32 27-29 24-26 21-23 18-20 15-17 12-14 9-11 6-8 3-5 0-2 65-67 57-64 56-57 55-56 53-54 51-52 50-51 48-49 46-47 44-45 42-44 39-41 36-38 33-36 29-32 25-28 21-24 412 Three Complete Practice Tests Personal Score Record Practice Test 1 Practice Test 2 Practice Test 3 Practice Test 1 413 414 Practice Test 1 [IJ Listening Comprehension This section tests your ability to comprehend spoken English. It is divided into three parts, each with its own directions. You are not permitted to turn the page during the reading of the directions or to take notes at any time. Part A l1li))) Directions: Each item in this part consists of a brief conversation involving two speakers. Following each conversation, a third voice will ask a question. You will hear the conversations and questions only once, and they will not be written out. When you have heard each conversation and question, read the four answer choices and select the one-(A), (B), (C), or (D)-that best answers the question based on what is directly stated or on what can be inferred. Then ftll in the space on your answer sheet that matches the letter of the answer that you have selected. Here is an example. You will hear: Sample Answer You will read: (A) (B) (C) (D) Open the window. Move the chair. Leave the room. Take a seat. From the conversation you find out that the woman thinks the man should put the chair over by the window. The best answer to the question, "What does the woman think the man should do?" is (B), "Move the chair." You should fill in (B) on your answer sheet. 8 Section 1 • Listening Comprehension 415 [I] [I] [I] [I] [I] [I] [I] [I] [I] [I] [I] [I] 1. (A) She doesn't have an appointment. (B) Her problem is complicated. (C) She must live somewhere else. (D) Her apartment isn't far away. 2. (A) She can use his phone if she wants. (B) There's no charge for phone calls. (C) His phone is out of order too. (D) She can call him later if she likes. 3. (A) (B) (C) (D) He couldn't find it. It was too hard to solve. It was simpler than he'd thought. He solved it even though it was hard. 8. (A) He was too busy to take it. (B) He did well on it. (C) He left some questions unanswered on it. (D) He took it two times. 9. (A) Breaking the glass. (B) Warming the lid. (C) Hitting the lid. (D) Filling the jar. 10. (A) It was too expensive. (B) She bought it at the shop next door. (C) It was given to her as a gift. (D) She paid very little for it. 4. (A) He cleaned up after cooking. (B) He forgot to put the pots and pans away. (C) He was out in a terrible storm. (D) He put some plants in the kitchen. 11. (A) She doesn't want to discuss the traffic. (B) She didn't have to go downtown today. (C) She was in the traffic herself. (D) She thinks the traffic was better today. 5. (A) He studied forestry in school. (B) He worked in a forest. (C) He read a lot of books about trees. (D) His father taught him. 12. (A) (B) (C) (D) 6. (A) How many pages he must write. (B) What Professor Barclay discussed. (C) How long the class lasted. (D) When the paper is due. 13. (A) Gary doesn't need a audio player. (B) She wants her audio player back. (C) She's glad Gary is finally here. (D) Gary can keep her audio player. 7. (A) She doesn't like any music except classical. (B) There is some classical music she doesn't like. (C) She likes classical music, but she can't play it. (D) Classical music doesn't interest her at all. 14. (A) Stay inside and read it. (B) Look in it for advertisements for umbrellas. (C) Cover her head with it. (D) Throw it away. 15. (A) (B) (C) (D) The classes aren't interesting. Classes have been canceled. The weather is pleasant. It isn't very sunny today. She originally supported Margaret. She can no longer support Ed. Ed has dropped out of the race. She's not interested in the election. @.i.",t·"UJUiii¥t-. 416 Practice Test 1 [IJ[IJ[IJ[IJ[IJ[IJ[IJ[IJ[IJ[IJ[IJ[IJ 16. (A) She and her brother painted the apartment. (B) Her brother owes her some money. (C) Her brother painted the apartment by himself. (D) She painted her brother's apartment. 17. (A) Give him a map. (B) Cut his hair for him. (C) Drive him to the lake. (D) Show him another route. 18. (A) (B) (C) (D) Hanging it. Buying it. Painting it. Framing it. 19. (A) (B) (C) (D) Borrow Stephanie's computer. Buy her own computer. Save some money. Stay home and complete her assignment. 20. (A) (B) (C) (D) He doesn't need to practice anymore. His team has won a lot of games. He doesn't want to play volleyball. His.team needs to improve. 21. (A) She seems to be feeling better. (B) She has quite an imagination. (C) She takes beautiful pictures. (D) She's too sick to go out. 22. (A) (B) (C) (D) Lou has been here once before. They'll start when Lou arrives. Lou has already started. Everyone is getting hungry. 24. (A) (B) (C) (D) To improve his game quickly. To take more lessons. To train with a professional. To teach people to play tennis. 25. (A) Wrap the present. (B) Playa game. (C) Point out a problem. (D) End the discussion. 26. (A) (B) (C) (D) He wants to buy some books. Two of the books are the same. He needs some matches. The couple is a good match. 27. (A) Neither street goes downtown. (B) California Street is better than Oak Street. (C) There's not enough time to go downtown. (D) He can take either street. 28. (A) It was hard to hear. (B) It wasn't true. (C) It was surprising. (D) It wasn't very interesting. 29. (A) (B) (C) (D) The handle on the suitcase is broken. His hands are already full. The luggage is too heavy for him. He'll be happy to help. 30. (A) (B) (C) (D) She had to prepare for an exam. She'd passed the physics test. She was going camping. She'd dropped the physics class. 23. (A) She thinks they're reasonably priced. (B) She doesn't like them at all. (C) She'd buy them if she had enough money. (D) She doesn't need them, but she still likes them. @.i.",t.ilUjU43 i UW t- - ' Section 1 • Listening Comprehension 417 [IJ[IJ[IJ[IJ[IJ[IJ[IJ[IJ[IJ[IJ[IJ[IJ Part B Directions: This part of the test consists of extended conversations between two speakers. After each of these conversations, there are a number of questions. You will hear each conversation and question only once, and the questions are not written out. When you have heard the questions, read the four answer choices and select the one-(A), (B), (C), or (D)-that best answers the question based on what is directly stated or on what can be inferred. Then fIll in the space on your answer sheet that matches the letter of the answer that you have selected. Don't forget: During actual exams, taking notes or writing in your test book is not permitted. 8 418 Practice Test 1 [I][I][I][IJ[IJ[IJ[IJ[IJ[IJ[IJ[IJ[IJ 31. (A) They are both studying social anthropology. (B) Both of them are going to the museum on Saturday. (C) They both have the same teacher. (D) Both of them have studied anthropology before. 32. (A) In the morning. (B) In the afternoon. (C) In the evening. (D) Only on Saturdays. 33. (A) Relationships between parents and children. (B) The tools used by ancient people. (C) Leadership in contemporary society. (D) Marriage customs. 34. (A) He found (B) He found (C) He found (D) He found 35. (A) At a university. (B) At a television station. (C) At a newspaper office. (D) At a hospital. 36. (A) He needs a well-paying position. (B) He was told to by a professor. (C) He wants the experience. (D) He recently lost another job. 37. (A) Drama. (B) Journalism. (C) Telecommunications. (D) History. 38. (A) Talk to Ms. Wagner. (B) Drop a class. (C) Change his major. (D) Complete a form. it uninteresting. it useful. it fascinating. it difficult. @.i.,;,t·""iUW'@[--' Section 1 • Listening Comprehension 419 ITJITJITJITJITJITJITJITJITJITJITJITJ Part C Directions: This part of the test consists of several talks, each given by a single speaker. After each of these talks, there are a number of questions. You will hear each talk and question only once, and the questions are not written out. When you have heard each question, read the four answer choices and select the one-(A), (B), (C), or (D)-that best answers the question based on what is directly stated or on what can be inferred. Then fill in the space on your answer sheet that matches the letter of the answer that you have selected. Here is an example. You will hear: Now here is a sample question. You will hear: Sample Answer You will read: (A) (B) (C) (D) Philosophy. Meteorology. Astronomy. Photography. The lecture concerns a lunar eclipse, a topic that would typically be discussed in an astronomy class. The choice that best answers the question, "In what course is this lecture probably being given?" is (C), "Astronomy." You should fill in (C) on your answer sheet. Here is an example. You will hear: Sample Answer You will read: (A) The Earth's shadow moves across the .\1oon (B) Clouds block the view of the Moon. (C) The Moon moves between the Earth and the Sun. (D) The Sun can be observed without special equipment. 420 Practice Test 1 []][]][]][IJ[IJ[IJ[IJ[IJ[IJ[IJ[IJ[IJ From the lecture you learn that a lunar eclipse occurs when the Earth moves between the Sun and the Moon and the shadow of the Earth passes across the Moon. The choice that best answers the question, "According to the speaker, which of the following occurs during a lunar eclipse?" is (A), "The Earth's shadow moves across the Moon." Don't forget: During actual exams, taking notes or writing in your test book is not permitted. 8 Section 1 • Listening Comprehension 421 ITJITJITJITJITJITJITJITJITJITJITJITJ 39. (A) A football game. (B) Jet transportation. (C) The Hindenburg disaster. (D) Lighter-than-air craft. 45. (A) As confusing. (B) As innovative. (C) As amusing. (D) As wasteful. 40. (A) Early twentieth century airships. (B) Blimps. (C) Jet aircraft. (D) Modern airships. 46. (A) To standardize daylight saving time. (B) To establish year-round daylight saving time. (C) To abolish daylight saving time. (D) To shorten daylight saving time. 41. (A) The age of zeppelins ended in disaster there. (B) It was there that the first blimp was designed. (C) Helium was first substituted for hydrogen there. (D) It was there that the last zeppelin was built. 42. (A) They would be safer. (B) They would use less fuel. (C) They would be faster. (D) They could fly higher. 43. (A) The Uniform Time Act. (B) The role of daylight saving time in wartime. (C) Ways to save energy. (D) The history of daylight saving time. 44. (A) In the (B) In the (C) In the (D) In the 47. (A) A program the city is starting. (B) The uses of recycled materials. (C) A proposed schedule. (D) A recent newspaper article. 48. (A) Newspapers. (B) Aluminum cans. (C) Plastic bottles. (D) Glass containers. 49. (A) The north. (B) The east. (C) The south. (D) The central. 50. (A) Look in the local newspaper. (B) Keep listening to radio. (C) Stop by the recycling center. (D) Call the radio station. spring. summer. fall. winter. TInS IS THE END OF SECTION 1, USTENING COMPREHENSION. STOP WORK ON SECTION 1. ~TO' ~TO' ~TO' ~TO' ~TO' ~TO' 422 Practice Test 1 [I] Structure and Written Expression Time: 25 minutes This section tests your ability to recognize grdmmar and usage suitable for standard written English. This section is divided into two parts, each with its own directions. Structure Directions: Items in this part are incomplete sentences. Following each of these sentences, there are four words or phrases. You should select the one word or phrase-CA), (B), CC), or (D)-that best completes the sentence. Then fill in the space on your answer sheet that matches the letter of the answer that you have selected. Example! Sample Answer Pepsin ____ an enzyme used in digestion. (A) that (B) is (C) of (D) being This sentence should properly read "Pepsin is an enzyme used in digestion." You should fill in (B) on your answer sheet. Example II Sample Answer c.FJCDCD . . ____ large natural lakes are found in the state of South Carolina. (A) There are no (B) Not the (C) It is not CD) No This sentence should properly read "No large natural lakes are found in the state of South Carolina." You should ftll in CD) on your answer sheet. As soon as you understand the directions, begin work on this part. @.i.",t.',Uj U43 i ¥t--' Section 2 • Structure and Written Expression 423 •••••••••••• 1. team sports require cooperation. (A) Of all (B) They are all (C) All (D) Why are all 2. Anyone who has ever pulled weeds from a garden roots firmly anchor plants to the soil. (A) is well aware of (B) is well aware that (C) well aware (D) well aware that 3. Centuries of erosion have exposed ____ rock surfaces in the Painted Desert of northern Arizona. (A) in colors of the rainbow (B) colored like a rainbow (C) rainbow-colored (D) a rainbow's coloring 4. The higher the temperature of a molecule, 7. A person of ____ age may suffer from defects of vision. (A) every (B) some (C) certain (D) any 8. ____ have settled, one of their first concerns has been to locate an adequate water supply. (A) Wherever people (B) There are people who (C) Whether people (D) People 9. If a bar magnet is , the two pieces form two complete magnets, each with a north and south pole. (A) broken (B) broke (C) breaking (D) break 10. The type of plant and animal life living in (A) the more energy it has (B) than it has more energy (C) more energy has it (D) it has more energy 5. Frontier surgeon Ephraim MacDonald had to perform operations anesthesia. (A) no (B) not having (C) without (D) there wasn't 6. ____ young, chimpanzees are easily trained. (A) When are (B) When (C) They are (D) When they and around a pond depends on the soil of the pond, , and the pond's location. (A) what the quality of the water is (B) how is the water quality (C) the quality of the water (D) what is the water quality 11. Clifford Holland, civil engineer, was in charge of the construction of the first tunnel under the Hudson River. (A) he was a (B) a (C) being a (D) who, as a @,i""t."UjU§3i@t.-' 424 Practice Test 1 eeeeeeeeeeee 12. parrots are native to tropical regions is untrue. (A) That all (B) All (C) Why all (D) Since all 13. A major concern among archaeologists today is the preservation of archaeological sites, are threatened by development. (A) of which many (B) many of them (C) which many (D) many of which 14. In 1775 Daniel Boone opened the Wilderness Trail and made ftrst settlements in Kentucky. (A) possibly it was (B) as possible (C) possible CD) it possible the 15. Rarely seen far from water. (A) spotted turtles (B) spotted turtles are (C) have spotted turtles (D) are spotted turtles IU.i.';'i.',Uj U43 i @t.-' Section 2 • Structure and Written Expression 42S •••••••••••• Written Expression Directions: The items in this part have four underlined words or phrases, (A), (B), (C), and (D). You must identify the one underlined expression that must be changed for the sentence to be correct. Then find the number of the question on your answer sheet and fill in the space corresponding to the letter. Example I Sample Answer Lenses may to have either concave or convex shapes. ABC D This sentence should read "Lenses may have either concave or convex shapes." You should therefore select answer (A). /<'xample II Sample Answer When painting a fresco, an artist is applied paint directly to the damp plaster of a wall. A B C D This sentence should read "When painting a fresco, an artist applies paint directly to the damp plaster of a wall." You should therefore select answer (B). As soon as you understand the directions, begin work on this section. 426 Practice Test 1 16. Ceramics can be harder, !_ight, and more resistant to heat than metals. A B C D 17. Not everyone realizes that the most largest organ of the human body is the skin. A -B- C D 18. Mold is extremely destruction to books in a library. A B --c--n 19. Sidney Lanier achieved fame both as a poet or as a symphony musician. A 13 C 0 20. The horses used play polo are not of any special breed or of any definite size. A B C 0 21. A tapestry consists of a foundation weave, called the warp, which across are passed different A B colored threads, called the weft, forming decorative patterns. C D 22. The works of early American woodcarvers had many artistic qualities, but these craftsmen --A- -B-- probably did not think of them as artists. C -0- 23. Perhaps mankind's first important musical influence were the songs of birds. i\ -c-.- B --0- 24. The techniques of science and magic are quite different, but their basic aims-to understand --A- B C and control nature-they are very similar. o 25. It was in a cave near Magdalena, New Mexico, when the oldest known ears of cultivated corn -----A B C were discovered. D 26. The fossil remains of much extinct mammals have been found in the tar pits at Rancho La Brea -A- B -c- in Los Angeles. D 27. Sharks can detect minute electrical discharges coming from its prey. A --B-C D @,i""t,ilUjUY:j.@t-. Section 2 • Structure and Written Expression 427 28. A dark nebula consists of a cloud of interstellar dust enough dense to obscure the stars A B C beyond it. D 29. Commercially honey is heated and filtered in order to stabilize and clarify !!:. A B C D 30. The various parts of the body require so different surgical skills that many surgical specialties A B C -D- have developed. 31. One reason birds have been so successful is because of their able to escape from danger A B C quickly. D 32. The wood of the rosewood tree is used to do fine musical instruments. -B C D 33. Chemical engineering is based on the principles of physics, chemists, and mathematics. A B C I) 34. Ballet performers must be believable actors and actresses as well as experts dancers. A -B- D --C- 35. Venus, the second planet from the Sun, is exactly almost the same size as the Earth. D A B C 36. P T. Barnum opened his own circus in 1871 and become the most famous showman of his A B C time. -1)- 37. The way a child plays with other children reveals a lots about the child's emotional A B C development. D 38. Sheep have been domesticated for over 5,000 years ago. A B -C D @.i.",t."UjU§3'UW t. • 428 Practice Test 1 eeeeeeeeeeee 39. Chemical compounds with barium, cobalt, and strontium are responsible to many of the vivid -AB C colors in fireworks. -D- 40. Duke University in North Carolina has an outstanding collecting of documents concerning ~ B C Southern history. D THIS IS THE END OF SECTION 2. IF YOU FINISH BEFORE TIME IS CALLED, CHECK YOUR WORK ON SECTION 2 ONLY. DO NOT READ OR WORK ON ANY OTHER SECTION OF THE TEST. ~TO' ~TO' ~TO' ~TO' ~TO' ~TO' Section 3 • Reading Comprehension 00 429 Reading Comprehension Time: 55 minutes This section of the test measures your ability to comprehend written materials. Directions: This section contains several passages, each followed by a number of questions. Read the passages and, for each question, choose the one best answer-(A), (B), (C), or (D)-based on what is stated in the passage or on what can be inferred from the passage. Then fill in the space on your answer sheet that matches the letter of the answer that you have selected. Read the following passage: Like mammals, birds claim their own territories. A bird's territory may be small or large. Some birds claim only their nest and the area right around it, while others claim far larger territories that include their feeding areas. Gulls, penguins, and other waterfowl nest in huge colonies, but even in the biggest colonies, each male and his mate have small territories of their own immediately around their nests. Male birds defend their territory chiefly against other males of the same species. In some cases, a warning call or threatening pose may be all the defense needed, but in other cases, intruders may refuse to leave peacefully. (line) (5) (10) Example! Sample Answer What is the main topic of this passage? (A) (B) (C) (D) Birds that live in colonies Birds'mating habits The behavior of birds Territoriality in birds The passage mainly concerns the territories of birds. You should fill in (D) on your answer sheet. Example!! Sample Answer According to the passage, male birds defend their territory primarily against (A) female birds (B) birds of other species (C) males of their own species (D) mammals The passage states that "Male birds defend their territory chiefly against other males of the same species." You should fill in (C) on your answer sheet. As soon as you understand the directions, begin work on this section. 430 Practice Test 1 Questions 1-11 (line) (5) (10) (15) (20) The Sun today is a yellow dwarf star. It is fueled by thermonuclear reactions near its center that convert hydrogen to helium. The Sun has existed in its present state for about 4 billion, 600 million years and is thousands of times larger than the Earth. By studying other stars, astronomers can predict what the rest of the Sun's life will be like. About 5 billion years from now, the core of the Sun will shrink and become hotter. The surface temperature will fall. The higher temperature of the center will increase the rate of thermonuclear reactions. The outer regions of the Sun will expand approximately 35 million miles, about the distance to Mercury, which is the closest planet to the Sun. The Sun will then be a red giant star. Temperatures on the Earth will become too hot for life to exist. Once the Sun has used up its thermonuclear energy as a red giant, it will begin to shrink. After it shrinks to the size of the Earth, it will become a white dwarf star. The Sun may throw off huge amounts of gases in violent eruptions called nova explosions as it changes from a red giant to a white dwarf. After billions of years as a white dwarf, the Sun will have used up all its fuel and will have lost its heat. Such a star is called a black dwarf. After the Sun has become a black dwarf, the Earth will be dark and cold. If any atmosphere remains there, it will have frozen onto the Earth's surface. 1. What is the primary purpose of this passage? (A) To alert people to the dangers posed by the Sun (B) To discuss conditions on Earth in the far future (C) To present a theory about red giant stars (D) To describe changes that the Sun will go through 2. The word "fueled" in line 1 is closest in meaning to (A) powered (B) bombarded (C) created (D) propelled 3. The word "state" in line 3 is closest in meaning to (A) shape (B) condition (C) location (D) size lfi.i.];. &11 hjh§!; i@(. • Section 3 • Reading Comprehension 431 4. It can be inferred from the passage that the Sun (A) is approximately halfway through its life as a yellow dwarf (B) has been in existence for 10 billion years (C) is rapidly changing in size and brightness (D) will continue as a yellow dwarf for another 10 billion years 5. What will probably be the first stage of change as the Sun becomes a red giant? (A) Its core will cool off and use less fuel. (B) Its surface will become hotter and shrink. (C) It will throw off huge amounts of gases. (D) Its center will grow smaller and hotter. 6. When the Sun becomes a red giant, what will conditions be like on Earth? (A) Its atmosphere will freeze and become solid. (B) It will be enveloped in the expanding surface of the Sun. (C) It will become too hot for life to exist. (D) It will be nearly destroyed by nova explosions. 7. As a white dwarf, the Sun will be (A) the same size as the planet Mercury (B) thousands of times smaller than it is today (C) around 35 million miles in diameter (D) cold and dark 8. According to the passage, which of the following best describes the sequence of stages that the Sun will probably pass through? (A) Yellow dwarf, white dwarf, red giant, black giant (B) Red giant, white dwarf, red dwarf, nova explosion (C) Yellow dwarf, red giant, white dwarf, black dwarf (D) White dwarf, red giant, black dwarf, yellow dwarf 9. The phrase "throw off" in line 17 is closest in meaning to (A) eject (B) burn up (C) convert (D) let in 10. The word "there" in line 23 refers to (A) our own planet (B) the outer surface of the Sun (C) the core of a black dwarf (D) the planet Mercury 11. Which of the following best describes the tone of the passage? (A) Alarmed (B) Pessimistic (C) Comic (D) Objective @.i.,;,t",Uj;t=ti .¥(-. 432 Practice Test 1 ------------------------------------ ------------------------------------------------------- Questions 12-23 (line) (5) (10) (15) (20) (25) (30) (35) It is said that George Washington was one of the first to realize how important the building of canals would be to the nation's development. In fact, before he became President, he headed the first company in the United States to build a canal which was to connect the Ohio and Potomac rivers. It was never completed, but it showed the nation the feasibility of canals. As the country expanded westward, settlers in western New York, Pennsylvania, and Ohio needed a means to ship goods. Canals linking natural waterways seemed to offer an effective solution. In 1791 engineers commissioned by the state of New York investigated the possibility of a canal between Albany on the Hudson River and Buffalo on Lake Erie, which would link the Great Lakes area with the Atlantic seacoast. It would avoid the mountains that served as a barrier to canals from the Delaware and Potomac rivers. The first attempt to dig the canal, to be called the Erie Canal, was made by private companies, but only a comparatively small portion was built before the project was halted for lack of funds. The cost of the project was an estimated five million dollars, an enormous amount for those days. There was some on-again-offagain Federal funding, but the War of 1812 put an end to this. In 1817 DeWitt Clinton was elected Governor of New York and persuaded the state to finance and build the canal. It was completed in 1825, costing two million dollars more than expected. The canal rapidly lived up to its sponsors' faith, quickly paying for itself through tolls. It was far more economical than any other form of transportation at the time. It permitted trade between the Great Lake region and East Coast, robbing the Mississippi River of much of its traffic. It allowed New York to supplant Boston, Philadelphia, and other Eastern cities as the chief center of both domestic and foreign commerce. Cities sprang up along the canal. It also contributed in a number of ways to the North's victory over the South in the Civil War. An expansion of the canal was planned in 1849. Increased traffic would undoubtedly have warranted its construction had it not been for the development of the railroads. 12. Why does the author most likely mention George Washington in the first paragraph? (A) He was President at the time the Erie Canal was built. (B) He was involved in pioneering efforts to build canals. (C) He successfully opened the first canal in the United States. (D) He commissioned engineers to study the possibility of building the Erie Canal. lij.i·';'GlIUJUg:i.@t-. Section 3 • Reading Comprehension 433 13. The word "feasibility" in line 6 is closest in meaning to (A) profitability (B) difficulty (C) possibility (D) capability 14. According to the passage, the Erie Canal connected the (A) Potomac and Ohio rivers (B) Hudson River and Lake Erie (C) Delaware and Potomac rivers (D) Atlantic Ocean and the Hudson River 15. Which of the following is closest in meaning to the word "comparatively" in line 17? (A) Relatively (B) Contrarily (C) Incredibly (D) Considerably 16. The phrase "on-again-off-again" in line 20 could be replaced by which of the following with the least change in meaning? (A) Intermittent (B) Unsolicited (C) Ineffectual (D) Gradual 17. The completion of the Erie Canal was financed by (A) the state of New York (B) private companies (C) the federal government (D) DeWitt Clinton 18. The actual cost of building the Erie Canal was (A) five million dollars (B) less than had been estimated (C) seven million dollars (D) more than could be repaid 19. The word "tolls" in line 26 is closest in meaning to which of the following? (A) Jobs (B) Grants (C) Links (D) Fees lij.i.",t."UjU§!i.¥t-. 434 Practice Test 1 20. Which of the following is NOT given in the fourth paragraph as an effect of the building of the Erie Canal? (A) It allowed the East Coast to trade with the Great Lakes area. (B) It took water traffic away from the Mississippi River. (C) It helped determine the outcome of the Civil War. (D) It established Boston and Philadelphia as the most important centers of trade. 21. What can be inferred about railroads in 1849 from the information in the last paragraph? (A) They were being planned but had not yet been built. (B) They were seriously underdeveloped. (C) They had begun to compete with the Erie Canal for traffic. (D) They were weakened by the expansion of the canal. 22. The word "warranted" in line 35 is closest in meaning to (A) guaranteed (B) justified (C) hastened (D) prevented 23. At what point in the passage does the author focus on the beginning of construction of the Erie Canal? (A) Lines 3-5 (B) Lines 10-13 (C) Lines 16-18 (D) Lines 25-26 Questions 24-33 (line) (5) (10) (15) It's a sound you will probably never hear, a sickened tree sending out a distress signal. But a group of scientists has heard the cries, and they think some insects also hear the trees and are drawn to them like vultures to a dying animal. Researchers with the u.s. Department of Agriculture's Forest Service fastened sensors to the bark of parched trees and clearly heard distress calls. According to one of the scientists, most drought-stricken trees transmit their plight in the 50- to 500kilohertz range. (The unaided human ear can detect no more than 20 kilohertz.) Red oak, maple, white pine, and birch all make slightly different sounds in the form of vibrations at the surface of the wood. The scientists think that the vibrations are created when the water columns inside tubes that run the length of the tree break, a result of too little water flowing through them. These fractured columns send out distinctive vibration patterns. Because some lij.i.,;,t.ilUj U4:J i ¥t-. Section 3 • Reading Comprehension (20) 435 insects communicate at ultrasonic frequencies, they may pick up the trees'vibrations and attack the weakened trees. Researchers are now running tests with potted trees that have been deprived of water to see if the sound is what attracts the insects. "Waterstressed trees also smell differently from other trees, and they experience thermal changes, so insects could be responding to something other than sound," one scientist said. 24. Which of the following is the main topic of the passage? (A) The vibrations produced by insects (B) The mission of the U.S. Forest Service (C) The effect of insects on trees (D) The sounds made by distressed trees 25. The word "them" in line 4 refers to (A) trees (B) scientists (C) insects (D) cries 26. The word "parched" in line 6 is closest in meaning to which of the following? CA) Burned (B) Dehydrated CC) Recovered (D) Fallen 27, The word "plight" in line 8 is closest in meaning to (A) signal (ll) condition (C) need (D) agony 28. It can be inferred from the passage that the sounds produced by the trees CA) serve as a form of communication with other trees (B) are the same no matter what type of tree produces them (C) cannot be heard by the unaided human ear CD) fall into the 1-20 kilohertz range 29, The word "fractured" in line 15 is closest in meaning to (A) long (ll) blocked (C) hollow CD) broken 30. Which of the following could be considered a cause of the trees' distress signals? (A) Torn roots (B) Attacks by insects (C) Experiments by scientists (D) Lack of water @.i·'I,t.iIIUi U43'@(-. 436 Practice Test 1 31. In line 17, the phrase "pick up" could best be replaced by which of the following? (A) Perceive (B) Lift (C) Transmit (D) Attack 32. All of the following are mentioned as possible factors in drawing insects to weakened trees EXCEPT (A) thermal changes (B) smells (C) sounds (D) changes in color 33. It can be inferred that, at the time the passage was written, research concerning the distress signals of trees (A) had been conducted many years earlier (B) had been unproductive up to then (C) was continuing (D) was no longer sponsored by the government Questions 34--41 (line) (5) (10) The concepts of analogy and homology are probably easier to exemplify than to define. When different species are structurally compared, certain features can be described as either analogous or homologous. For example, flight requires certain rigid aeronautical principles of design, yet birds, bats, and insects have all conquered the air. The wings of all three types of animals derive from different embryological structures, but they perform the same functions. In this case, the flight organs of these creatures can be said to be analogous. In contrast, features that arise from the same structures in the embryo but are used in different functions are said to be homologous. The pectoral fins of a fish, the wings of a bird, and the forelimbs of a mammal are all homologous structures. They are genetically related in the sense that both the forelimb and the wing evolved from the fin. 34. Which of the following best describes the organization of the passage? (A) A contrast is drawn between two concepts by means of examples. (B) A general concept is introduced, examples are given, and a conclusion is offered. (C) Two definitions of the same concept are compared. (D) Two proposals are suggested and support for both is offered. 35. According to the passage, the concepts of analogy and homology are (A) difficult to understand (B) easier to understand through examples than through definitions (C) impossible to explain (D) simple to define but hard to apply @.i·,;.t:jlUjU§ii.@t-. Section 3 • Reading Comprehension 437 36. The word "rigid" in line 4 is closest in meaning to (A) inflexible (B) ideal (C) unnatural (D) steep 37. According to the information provided in the passage, which of the following would most probably be considered analogous? (A) A shark's fin and a tiger's claws (8) A man's arms and a bird's wings (C) A monkey's tail and an elephant's tail (D) A spider's legs and a horse's legs 3R. According to the passage, one way in which homologous organs differ from analogous organs is that they (A) are genetically related (B) are only found in highly developed animals (C) perform the same general functions (D) come from different embryological structures 39. As used throughout the passage, the term "structures" most nearly means (A) buildings (B) features of an animal's anatomy (C) organizational principles (D) units of grammar 40. The word "sense" in line 13 is closest in meaning to (A) feeling (B) logic (C) meaning (D) perception 41. Where in (A) Lines (B) Lines (C) Lines (D) Lines the passage does the author first focus his discussion on the concept of homology? 2-4 6-8 9-1 1 13-14 Questions 42-50 (line) (5) Probably the most famous film commenting on twentieth century technology is Modern Times, made in 1936. Charlie Chaplin was motivated to make the film by a reporter who, while interviewing him, happened to describe working conditions in industrial Detroit. Chaplin was told that healthy young farm boys were lured Id·i .,,'i.ilUj U43.@t-. 438 Practice Test 1 (10) (15) (20) (25) (30) to the city to work on automotive assembly lines. Within four or five years, these young men's health was destroyed by the stress of work in the factories. The film opens with a shot of a mass of sheep making their way down a crowded ramp. Abruptly the scene shifts to a scene of factory workers jostling one another on their way to a factory. However, the rather bitter note of criticism in the implied comparison is not sustained. It is replaced by a gentler note of satire. Chaplin prefers to entertain rather than lecture. Scenes of factory interiors account for only about one-third of the footage of Modern Times, but they contain some of the most pointed social commentary as well as the most comic situations. No one who has seen the film can ever forget Chaplin vainly trying to keep pace with the fast-moving conveyor belt, almost losing his mind in the process. Another popular scene involves an automatic feeding machine brought to the assembly line so that workers need not interrupt their labor to eat. The feeding machine malfunctions, hurling food at Chaplin, who is strapped into his position on the assembly line and cannot escape. This serves to illustrate people's utter helplessness in the face of machines that are meant to serve their basic needs. Clearly, Modern Times has its faults, but it remains the best film treating technology within a social context. It does not offer a radical social message, but it does accurately reflect the sentiments of many who feel they are victims of an over-mechanized world. 42. The author's main purpose in writing this passage is to (A) criticize the factory system of the 1930's (B) analyze an important film (C) explain Chaplin's style of acting (D) discuss how film reveals the benefits of technology 43. According to the passage, Chaplin got the idea for the film Modern Times from (A) a newspaper article (B) a scene in a movie (C) a job he had once held (D) a conversation with a reporter 44. The word "abruptly" in line 10 is closest in meaning to (A) suddenly (B) mysteriously (C) finally (D) predictably lij.i·';'&11 Uj U43 i @t--' Section 3 • Reading Comprehension 45. It can be inferred from the passage that two-thirds of the film Modern Times (A) is completely unforgettable (B) takes place outside a factory (C) is more critical than the other third (D) entertains the audience more than the other third 46. Which of the following could best replace the phrase "losing his mind" in lines 19-20? (A) Getting fired (B) Doing his job (C) Going insane (D) Falling behind 47. The word "This" in line 24 refers to which of the following? (A) The machine (B) The food (C) The assembly line (D) The scene 48. According to the passage, the purpose of the scene involving the feeding machine is to show people's (A) ingenuity (B) adaptability (C) helplessness (D) independence 49. The word "utter" in line 25 is closest in meaning to which of the following? (A) Notable (B) Complete (C) Regrettable (D) Necessary 50. The author would probably be LEAST likely to use which of the following words to describe the film Modern Times? (A) Revolutionary (B) Entertaining (C) Memorable (D) Satirical THIS IS THE END OF THE SAMPLE READING COMPREHENSION SECTION. IF YOU FINISH BEFORE TIME IS CALLED, GO BACK AND CHECK YOUR WORK IN THIS SECTION ONLY. ~TO~ ~TO~ ~TO~ ~TO' ~TO' ~TO~ 439 THERE IS NO TEST MATERIAL ONTHIS PAGE. 442 Practice Test 2 I . [}] Listening Comprehension This section tests your ability to comprehend spoken English. It is divided into three parts, each with its own directions. You are not permitted to turn the page during the reading of the directions or to take notes at any time. Part A liliiii))) Directions: Each item in this part consists of a brief conversation involving two speakers. Following each conversation, a third voice will ask a question. You will hear the conversations and questions only once, and they will not be written out. When you have heard each conversation and question, read the four answer choices and select the one-(A), (B), (C), or (D)-that best answers the question based on what is directly stated or on what can be inferred. Then fill in the space on your answer sheet that matches the letter of the answer that you have selected. Here is an example. You will hear: Sample Answer You will read: (A) (B) (C) (D) Open the window. Move the chair. Leave the room. Take a seat. From the conversation you find out that the woman thinks the man should put the chair over by the window. The best answer to the question, "What does the woman think the man should do?" is (B), "Move the chair." You should fill in (B) on your answer sheet. 8 Practice Test 2 441 442 Practice Test 2 m listening Comprehension This section tests your ability to comprehend spoken English. It is divided into three parts, each with its own directions. You are not permitted to turn the page during the reading of the directions or to take notes at any time. Part A l1li»)) Directions: Each item in this part consists of a brief conversation involving two speakers. Following each conversation, a third voice will ask a question. You will hear the conversations and questions only once, and they will not be written out. When you have heard each conversation and question, read the four answer choices and select the one-(A), (B), (C), or (D)-that best answers the question based on what is directly stated or on what can be inferred. Then fill in the space on your answer sheet that matches the letter of the answer that you have selected. Here is an example. You will hear: Sample Answer You will read: (A) Open the window. (B) Move the chair. (C) Leave the room. (D) Take a seat. From the conversation you find out that the woman thinks the man should put the chair over by the window. The best answer to the question, "What does the woman think the man should do?" is (B), "Move the chair." You should fill in (B) on your answer sheet. 8 Section 1 • Listening Comprehension 443 [IJ[IJ[IJ[IJ[IJ[IJ[IJ[IJ[IJ[IJ[IJ[IJ 1. (A) (B) (C) (D) The plane hasn't taken off yet. The cost of flying has increased. More flights will soon be scheduled. He hasn't changed his vacation plans. 10. (A) (B) (C) (D) 2. (A) (B) (C) (D) All of the books are cheap. None of the books is required. Half of the books should be returned. Only four of the books are expensive. 11. (A) The bucket has been broken. (B) The water was spilled. (C) They still need more money. (D) They have run out of time. 3. (A) (B) (C) (D) It will probably rain today. She likes to watch the rain come down. She'll play even if it rains. It has rained a lot lately. 12. (A) (B) (C) (D) 4. (A) Nearly all of the students can meet. (B) The meeting time must be changed. (C) Only Lisa will be at the library. (D) Lisa dropped the class on Friday. 5. (A) He walked so far that he was exhausted. (B) He was wearing shorts on his hike. (C) He encountered some wild animals in the woods. (D) He should get some comfortable hiking shoes. 6. (A) He intends to see Michelle. (B) Michelle may visit him anytime. (C) He has to return some money to Michelle. (D) Michelle owes him some money. 7. (A) (B) (C) (D) Buying some red chairs. Renting a bigger auditorium. Moving chairs from the auditorium. Getting more chairs. 8. (A) (B) (C) (D) He He He He can't read the sign. didn't make the sign. didn't sign his name. doesn't like the sign. 9. (A) Housing near campus is getting cheaper and cheaper. (B) She doesn't need to live close to campus. (C) It's not easy to find inexpensive housing near campus. (D) The man could tind housing if he looked carefully. To To To To a game. buy tickets. get some groceries. a party. He is kind. He is impolite. He is somewhat busy. She doesn't want to say. 13. (A) It's not surprising that Tony went fishing. (B) He already knew Tony had caught only one fish. (C) He doesn't think Tony is a good fisherman. (D) Tony usually catches a lot of fish. 14. (A) Don't complete the form. (B) Don't waste time. (C) Take a form. (D) There's no hurry. 15. (A) (B) (C) (D) He is telling her the truth. He's never been to Seattle. He has visited Seattle once. She's only spoken to him once. 16. (A) (B) (C) (D) Disapproved of her plan. Watered Lily's plants. Traveled overseas. Caught colds. 17. (A) (B) (C) (D) It should have been turned up. The people across town enjoyed it. It's extremely popular. It was much too loud. 18. (A) (B) (C) (D) It was quite relaxing. The weather wasn't good. It was unexpectedly busy. It was perfectly planned. lij.i.",t·iIIUiU4:J i ¥t--' 444 Practice Test 2 [IJ[IJ[IJ[I]ITJ[I][I][I][I][I][I][I] 19. (A) The lab is generally locked on Saturdays. (B) The man doesn't have a key to the lab. (C) Something strange happened in the lab on Saturday. (D) The lab should never be locked. 20. (A) He needs the insurance no matter how much it costs. (B) There are other types of insurance he should buy. (C) The man doesn't have enough money to buy insurance. (D) The cost of insurance is becoming more reasonable. 21. (A) (B) (C) (D) She's an art student. She's afraid of flying. She did well on the test. She got her pilot's license. 22. (A) (B) (C) (D) An elevator. A television. An automobile. A telephone. 23. (A) (B) (C) (D) Meg's sister took it to the cleaner's. Meg cleaned it. Meg wore it to class. Meg's sister borrowed it. 24. (A) (B) (C) (D) They must go to an orientation session. They are not new students. They won't be allowed to register. They were given the wrong schedule. 25. (A) (B) (C) (D) He doesn't mind moving. His brother won't move for two weeks. He'd rather not help his brother move. His brother decided not to move. 26. (A) (B) (C) (D) She has a coin like his. She knows a lot about coins. She thinks the coin is worthless. She's never seen this type of coin. 27. (A) Its lyrics are hard to understand. (B) It needs a stronger melody. (C) It has become very popular. (D) Its melody is hard to forget. 28. (A) He hadn't been smiling. (B) His picture hadn't been taken. (C) It wasn't a good picture. (D) The woman wouldn't show him the picture. 29. (A) They're always expensive. (B) They haven't been cleaned. (C) They're inexpensive now. (D) There aren't any available. 30. (A) (B) (C) (D) From college. Through her roommate. From the reception. Through her sister. @,i""al UjU§3 i @t. • Section 1 • Listening Comprehension 445 [IJ[IJ[IJ[IJ[IJ[IJ[IJ[IJ[IJ[IJ[IJ[IJ Part B Directions: This part of the test consists of extended conversations between two speakers. After each of these conversations, there are a number of questions. You will hear each conversation and question only once, and the questions are not written out. When you have heard the questions, read the four answer choices and select the one-(A), (B), (C), or (D)-that best answers the question based on what is directly stated or on what can be inferred. Then fill in the space on your answer sheet that matches the letter of the answer that you have selected. Don't forget: During actual exams, taking notes or writing in your test book is not permitted. 446 Practice Test 2 [IJ[IJ[IJ[IJ[IJ [IJ[IJ [IJ[IJ[IJ[IJ[IJ 31. (A) At a newspaper. (B) At an advertising agency. (C) At a furniture store. (D) At a real estate office. 32. (A) A two-bedroom apartment. (B) A sofa. (C) A chair. (D) A roommate. 33. (A) (B) (C) (D) Her phone number. The location of the apartment. The best time to call her. Her first name. 34. (A) (B) (C) (D) $5. $15. $30. $250. 35. (A) (B) (C) (D) From a newspaper advertisement. From a magazine article. From a television program. From an automobile dealer. 36. (A) To warn of dangers. (B) To explain traffic regulations. (C) To wake up drivers who are falling asleep. (D) To give directions. 37. (A) (B) (C) (D) He has a good sense of direction. He owns a "smart" car. He doesn't know how to drive. He doesn't know the way to the woman's house. 38. (A) (B) (C) (D) He got lost. He ran out of gas. He was in an accident. His car broke down. (ij.i.,;,t.ilUj U43.¥t. • Section 1 • Listening Comprehension 447 ITJITJITJITJITJITJITJITJITJITJITJITJ Part C Directions: This part of the test consists of several talks, each given by a single speaker. After each of these talks, there are a number of questions. You will hear each talk and question only once, and the questions are not written out. When you have heard each question, read the four answer choices and select the one-(A), (B), (C), or (D)-that best answers the question based on what is directly stated or on what can be inferred. Then fill in the space on your answer sheet that matches the letter of the answer that you have selected. Here is an example. You will hear: Now here is a sample question. You will hear: Sample Answer You will read: (A) (B) (C) (D) Philosophy. Meteorology. Astronomy. Photography. The lecture concerns a lunar eclipse, a topic that would typically be discussed in an astronomy class. The choice that best answers the question, "In what course is this lecture probably being given?" is (C), "Astronomy." You should fill in (C) on your answer sheet. Here is another sample question. You will hear: Sample Answer You will read: (A) The Earth's shadow moves across the Moon. CB) Clouds block the view of the Moon. CC) The Moon moves between the Earth and the Sun. CD) The Sun can be observed without special equipment. lij.i.,;,t·"nj U43.@t--' 448 Practice Test 2 IITJ ITJ ITJ ITJ ITJ ITJ ITJ ITJ ITJ ITJ ITJ ITJ From tbe lecture you learn that a lunar eclip~e occurs when the Earth moves between the Sun and the \1oon and the shadow of the Earth pas'ies acro~s the Moon. The choice that best answers the question, 'According to the speaker, which of the following occurs during a lunar eclipse?" is (A), "The Earth's shadow moves acros~ the Moon," Don't forget: During actual exams, taking notes or writing in your test book is Hot permitted, 8 Section 1 • Listening Comprehension 449 ITJITJITJITJITJITJITJITJITJITJITJITJ 46. (A) (ll) (C) (D) Take part in the race. Travel to the country. Avoid the downtown area. Ride a bicycle to work. A faculty member. A chancellor of the college. A graduate student. 47. (A) (ll) (C) (D) In In In In 4l. (A) (ll) (C) (D) Computer science. History. Economics. Physics. 48. (A) (ll) (C) (D) A famous person. A beautiful landscape. An empty phone booth. Geometric shapes. 42. (A) (ll) (C) CD) Two. Four. Six. Eight. 49. (A) They are very valuable. (ll) They are quite large. (C) They are highly ahstract. (D) They are extremely lifelike. 43. (A) (ll) (C) (D) A bicycle racer. A radio announcer. A coach. A television reporter. 50. (A) (ll) (C) (D) 44. (A) (ll) (C) (D) The benefits of bicycle commuting. Local traffic problems. A series of bicycle races. The American university system. 45. (A) (ll) (C) (D) On the college campus. On downtown streets. In Woodland Park. In the nearby countryside. 39. CA) (ll) (C) (D) To To To To present an award. say goodbye to Professor Callaghan. explain computer models. welcome a new college president. 40. (A) (ll) (C) (D) An administrator. an art history class. a painter's studio. a photography class. an art museum. Paint pictures. Write papers. View some slides. Discuss their reactions. THIS IS THE END OF SECTION 1, liSTENING COMPREHENSION. STOP WORK ON SECTION 1. 450 Practice Test 2 ~ Structure and Written Expression Time: 25 minutes This section tests your ability to recognize grammar and usage suitable for standard written English. This section is divided into two parts, each with its own directions. Structure Directions: Items in this part are incomplete sentences. Following each of these sentences, there are four words or phrases. You should select the one word or phrase-(A), (B), (C), or (D)-that best completes the sentence. Then fill in the space on your answer sheet that matches the letter of the answer that you have selected. Example! Sample Answer Pepsin ____ an enzyme used in digestion. (A) that (B) is (C) of (D) being This sentence should properly read "Pepsin is an enzyme used in digestion." You should fill in (B) on your answer sheet. Example II Sample Answer ____ large natural lakes are found in the state of South Carolina. (A) There are no (B) Not the (C) It is not (D) No This sentence should properly read "No large natural lakes are found in the state of South Carolina." You should fill in (D) on your answer sheet. As soon as you understand the directions, begin work on this part. lij.i.,;,t"Uj;tM.¥t a - ' Section 2 • Structure and Written Expression 451 1. Sharp knives are actually safer to use _ _ _ _ __ (A) as dull ones (B) as ones that are dull (C) than dull ones (D) that are dull ones 2. Daniel Webster, Thaddeus Stevens, and many others _ _ _ _ _ _ prominent in public life began their careers by teaching school. (A) they became (B) once they became (C) became (D) who became 3. As coal mines became deeper, the problems of draining water, bringing in fresh air, and _ _ _ _ _ _ to the surface increased. (A) transporting ore (B) to transport ore (C) how ore is transported (D) ore is transporting 4. because of the complexity of his writing, Henry James never became a popular writer, but his works are admired by critics and other writers. (A) It may be (B) Perhaps (C) Besides (D) Why is it 5. Piedmont glaciers are formed _ _ _ _ _ _ several valley glaciers join and spread out over a plain. (A) by (B) when (C) from (D) that 6. As late as 1890, Key West, with a population of 18,000, _ _ _ _ _ _ Florida's largest city. (A) that was (B) to be (C) was (D) it was 7. A mastery of calculus depends on _ _ _ _ _ _ of algebra. (A) an understanding (B) is understood (C) to understand (D) understand @.i."'i.',UjUMi!$t-. 452 Practice Test 2 eeeeeeeeeeee 8. he was not a musician himself, Lawrence Hammond developed an electronic keyboard instrument called the Hammond organ. (A) Although (B) That (C) Despite (D) For 9. Agnes De Mille's landmark musical play Oklahoma! was _ _ _ _ _ _ of story, music, and dance. (A) successfully combined (B) a successful combination (C) to combine successfully (D) successful combining single dialect of American English has ever become dominant. 10. (A) No (B) Not only a Not (D) Nor a (C) 11. In 1837 the University of Michigan became the first state university of regents elected by the voters of the state. (A) under the control (B) it was controlled (C) being controlled (D) to be controlled _ _ _ _ _ by a board 12. Indoor heating systems have made _ _ _ _ _ _ for people to live and work comfortably in temperate climates. (A) it is possible (B) possible (C) it possible (D) possibly 13. Certain fish eggs contain droplets of oil, _ _ _ _ _ _ to float on the surface of the water. (A) allowing them (B) allows them (C) they are allowed (D) this allows them 14. Considered America's first great architect, (A) many of the buildings at Harvard University were designed by Henry Hobson Richardson (B) Henry Hobson Richardson designed many of the buildings at Harvard University (C) Harvard University has many buildings that were designed by Henry Hobson Richardson (D) it was Henry Hobson Richardson who designed many of the buildings at Harvard University 15. is caused by a virus was not known until 1911. (A) That measles (B) As measles (C) Measles ~ (D) What if measles @.i.hi(.ihti;rtiiWn-~ Section 2 • Structure and Written Expression 453 eeeeeeeeeeee Written Expression Directions: The items in this part have four underlined words or phrases, CA), CB), (C), and CD). You must identify the one underlined expression that must be changed for the sentence to be correct. Then find the number of the question on your answer sheet and fill in the space corresponding to the letter. Sample Answer Example I Lenses may to have either concave or ABC convex shapes. --D- This sentence should read "Lenses may have either concave or convex shapes." You should therefore select answer (A). Example II Sample Answer When painting a fresco, an artist is applied paint A B directly to the damp plaster of a wall. ---c-·-- D This sentence should read "When painting a fresco, an artist applies paint directly to the damp plaster of a wall." You should therefore select answer CB). As soon as you understand the directions, begin work on this section. lij.i.",(.i,UJU§:ii@t-. 454 Practice Test 2 eeeeeeeeeeee 16. Dreams are commonly made up of either visual and verbal images. A B --C-- D 17. The Yale Daily News is oldest than any other college newspaper still in operation in the A ---c- -B- D United States. 18. Mary Rinehart was a pioneer in the field of journalist in the early twentieth century. A --B- C D 19. The Dave Brubek Quartet, one of the most popular jazz bands of the 1950's, had a particularly A B loyal following on campuses college. C D 20. In the architecture, a capital is the top portion of a column. --A-- --B- C D 21. Today, successful farmers are experts not only in agriculture, but also in market, finance, and ABC accounting. D 22. In the early days of jet development, jet engines used great numbers of fuel. --A--BC -023. Georgia has too many types of soil that virtually any temperate-zone crop can be grown there. ABC -D24. The sum of all chemical reactions in an organism's living cells are called its metabolism. A B C 0 25. River transportation in the United States consists primarily of barges pull by towboats. A B C ~ 26. Most modern barns are both insulated, ventilated, and equipped with electricity. A B C D 27. Many bridges in New England were covered with wooden roofs to protect it from -AB C rain and snow. D @.i.",t·"Uj U43 i @t--' Section 2 • Structure and Written Expression 455 eeeeeeeeeeeel 28. It is their nearly perfect crystal structure that gives diamonds their hardness, brilliance, and A B C transparent. D 29. Needles are simple-looking tools, but they are very relatively diff1cult to make. A B C D 30. Ducks are less susceptible to infection than another types of poultry. ~ B -cD 31. Unlike competitive running, race walkers must always keep some portion of their feet A ---c: B in contact with the ground. D 32. One of the most beautiful botanical gardens in the United States is the wildly and lovely A B C Magnolia Gardens near Charleston, South Carolina. D 33. Composer John Cage used many unusual objects as instrument in his music, including A B C cowbells, flower pots, tin cans, and saw blades. D 34. Woody Guthrie wrote thousands of songs during the lifetime, many of which became classic A B C folk songs. D 35. RunnerWilma Rudolph win three gold medals at the 1960 Olympics, and she set the A B C world record for the laO-meter dash in 1961. D 36. Some critics have called Theodore Dreiser's book Sister Carrie a first modern novel because -BA c it broke so many traditions. D 37. Abigail Adams' letters to her husband present a graphic picture of the age which she lived. --A- -'-I C D lij.i·'I'&1!IUi U§:J i ¥t-. 456 Practice Test 2 •••••••••••• 38. Viscosity is a measurement describing the relative difficulty or easy with which liquids flow. A --B-- (~- 0 39. More than 10,000 years ago, glaciers moved across the Minnesota region four time, leveling A -B- -C- most of the land. [) 40. The discover of gold and silver in the rugged mountains of Nevada in 1858 attracted many A B C fortune-seekers to that area. o THIS IS THE END OF SECTION 2. IF YOU FINISH BEFORE TIME IS CALLED, CHECK YOUR WORK ON SECTION 2 ONLY. DO NOT RFAD OR WORK ON ANY OTHER SECTION OF THE TEST. ~TO' ~T07 ~TO' ~TO' ~T07 ~TO' Section 3 • Reading Comprehension 457 @] Reading Comprehension Time: 55 minutes This section of the test measures your ability to comprehend written materials. Directions: This section contains several passages, each followed by a number of questions. Read the passages and, for each question, choose the one best answer-(A), (B), (C), or (D)-based on what is stated in the passage or on what can be inferred from the passage. Then fill in the space on your answer sheet that matches the letter of the answer that you have selected. Read the following passage: Like mammals, birds claim their own territories. A bird's territory may be small or large. Some birds claim only their nest and the area right around it, while others claim far larger territories that include their feeding areas. Gulls, penguins, and other waterfowl nest in huge colonies, but even in the biggest colonies, each male and his mate have small territories of their own immediately around their nests. Male birds defend their territory chiefly against other males of the same species. In some cases, a warning call or threatening pose may be all the defense needed, but in other cases, intruders may refuse to leave peacefully. (line) (5) (10) Example! Sample Answer What is the main topic of this passage l (A) (B) (C) (D) Birds that live in colonies Birds' mating habits The behavior of birds Territoriality in birds The passage mainly concerns the territories of birds. You should fill in (D) on your answer sheet. 458 Practice Test 2 Example 1/ Sample Answer According to the passage, male birds defend their territory primarily against (A) (B) (C) (D) female birds birds of other species males of their own species mammals The passage states that "Male birds defend their territory chiefly against other males of the same species." You should fill in (C) on your answer sheet. As soon as you understand the directions, begin work on this section. lij.i.';'i.ilUj;t=tJi ¥(-. Section 3 • Reading Comprehension 459 Questions 1-11 (line) (5) (10) (15) (20) (25) The time when humans crossed the Arctic land bridge from Siberia to Alaska seems remote to us today, but actually represents a late stage in the prehistory of humans, an era when polished stone implements and bows and arrows were already being used, and dogs had already been domesticated. When these early migrants arrived in North America, they found the woods and plains dominated by three types of American mammoths. These elephants were distinguished from today's elephants mainly by their thick, shaggy coats and their huge, upward-curving tusks. They had arrived on the continent hundreds of thousands of years before their human followers. The wooly mammoth in the North, the Columbian mammoth in middle North America. and the imperial mammoth of the South, together with their distant cousins the mastodons, dominated the land. Here, as in the Old World, there is evidence that humans hunted these elephants, as shown by the numerous spear points found with mammoth remains. Then, at the end of the Ice Age, when the last glaciers had retreated, there was a relatively sudden and widespread extinction of elephants. In the New World, both mammoths and mastodons disappeared. In the Old World, only Indian and African elephants survived. Why did the huge, seemingly successful mammoths disappear? Were humans connected with their extinction? Perhaps, but at that time, although they were cunning hunters, humans were still widely scattered and not very numerous. It is difficult to see how they could have prevailed over the mammoth to such an extent. 1. With which of the following is the passage primarily concerned' (A) Migration from Siberia to Alaska (B) Techniques used to hunt mammoths (C) The prehistory of humans (D) The relationship between man and mammoth in the New World 2. The word "implements" in line 4 is closest in meaning to (A) tools (B) ornaments (C) houses (D) carvings 3. The phrase "these early migrants" in line 6 refers to (A) mammoths (B) humans (C) dogs (D) mastodons @.i·';'t:jl;tj U4!iiijWt-. 460 Practice Test 2 4. Where were the imperial mammoths the dominant type of mammoth? (A) In Alaska (B) In the central portion of North America (C) In the southern part of North America (D) In South America 5. It can be inferred that when humans crossed into the New World, they (A) had previously hunted mammoths in Siberia (B) had never seen mammoths before (C) brought mammoths with them from the Old World (D) soon learned to use dogs to hunt mammoths 6. Which of the following could best substitute for the word "remains" in line 17? (A) Bones (B) Drawings (C) Footprints (D) Spear points 7. The word "seemingly" in line 23 is closest in meaning to (A) tremendously (B) apparently (C) formerly (D) obviously 8. The passage supports which of the following conclusions about mammoths? (A) Humans hunted them to extinction. (B) The freezing temperatures of the Ice Age destroyed their food supply. CC) The cause of their extinction is not definitely known. CD) Competition with mastodons caused them to become extinct. 9. The word "cunning" in line 25 is closest in meaning to (A) clever (B) determined (C) efficient (D) cautious 10. Which of the following is NOT true about prehistoric humans at the time of the mammoths' extinction? (A) They were relatively few in number. (8) They knew how to use bows and arrows. CC) They were concentrated in a small area. (D) They were skilled hunters. 11. Which of the following types of elephants does the author discuss in the most detail in the passage? (A) The mastodon (B) The mammoth CC) The Indian elephant (D) The African elephant @,i""t,',Uj;tWii!Wt-. Section 3 • Reading Comprehension 461 Questions 12-23 -------------------- (line) (5) (10) (15) (20) (25) Just before and during World War I, a number of white musicians came to Chicago from New Orleans playing in an idiom they had learned from blacks in that city. Five of them formed what eventually became known as the Original Dixieland Band. They moved to New York in 1917 and won fame there. That year they recorded the first phonograph record identified as jazz. The first important recording by black musicians was made in Chicago in 1923 by King Oliver's Creole Jazz Band, a group that featured some of the foremost jazz musicians of the time, including trumpet player Louis Armstrong. Armstrong's dynamic trumpet style became famous worldwide. Other band members had played in Fate Marable's band, which traveled up and down the Mississippi River entertaining passengers on riverboats. The characteristics of this early type of jazz, known as Dixieland jazz, included a complex interweaving of melodic lines among the cornet or trumpet, clarinet, and trombone, and a steady chomp-chomp beat provided by the rhythm section, which included the piano, bass, and drums. Most bands used no written notations, preferring arrangements agreed on verbally. Improvisation was an indispensable element. Even bandleaders such as Duke Ellington, who provided his musicians with written arrangements, permitted them plenty to freedom to improvise when playing solos. In the late 1920's, the most influential jazz artists in Chicago were members of small bands such as the Wolverines. In New York, the trend was toward larger groups. These groups played in revues, large dance halls, and theaters. Bands would become larger still during the next age of jazz, the Swing era. 12. What is the main topic of this passage(A) The early history of jazz (B) The music of World War I (C) The relationship of melody and rhythm in jazz (D) The New York recording industry in the 1920's 13. The word "idiom" in line 2 is closest in meaning to (A) slang (B) tempo (C) tune (D) style 14. The musicians who made the earliest jazz recordings were originally from (A) New Orleans (B) Chicago (C) New York (D) Mississippi lij.i·';'GlIUjUg:i.@t-. 462 Practice Test 2 15. When was the first important recording by black jazz musicians made? (A) In 1917 (B) In 1923 (C) In the late 1920's (D) In the early 1930's 16. According to the passage, Louis Armstrong was a member of which of the following' (A) The Original Dixieland Band (B) Fate Marable's riverboat band (C) King Oliver's Creole Jazz Band (D) The Wolverines 17. The word "steady" in line 17 is closest in meaning to (A) constant (13) basic (C) urgent (I)) happy IR. According to the passage, which of the following instruments helped provide the beat for Dixieland jazz? (A) The cornet (B) The piano (C) The trombone (D) The clarinet 19. Duke Ellington is given as an example of a bandleader who (A) could not read music (B) did not value improvisation (C) discouraged solo performances (D) used written arrangements 20. Which of the following phrases would be LEAST likely to be applied to Dixieland jazz' (A) Relatively complex (B) Highly improvisational (C) Rhythmic and melodic (D) Carefully planned 21. According to the passage, who were the Wolverines? (A) A band that played in large dance halls (B) A New York group (C) A Swing band (D) A small group 22. The author provides the most detailed description of early jazz music in the (A) first paragraph (B) second paragraph (C) third paragraph (D) fourth paragraph @,i,';'(,ilUjU4:J i ¥t-. Section 3 • Reading Comprehension 463 23. The paragraph following this one most likely deals with (A) the music of small bands (B) the Swing era (C) music that influenced Dixieland Jazz (D) other forms of music popular in the 1920's Questions 24-32 (line) (5) (10) (15) (20) (25) (30) A pioneering study by Donald Appleyard made the astounding discovery that a sudden increase in the volume of traffic through an area affects people in the way that a sudden increase in crime does. Appleyard observed this by finding three blocks of houses in San Francisco that looked much alike and had the same kind of middle-class and working-class residents, with approximately the same ethnic mix. The difference was that only 2,000 cars a day ran down Octavia Street (LIGHT street, in Appleyard's terminology) while Gough Street (MEDIUM street) was used by 8,000 cars daily, and Franklin Street (HEAVY street) had around 16,000 cars a day. Franklin Street often had as many cars in an hour as Octavia Street had in a day. Heavy traffic brought with it danger, noise, fumes, and soot, directly, and trash secondarily. That is, the cars didn't bring in much trash, but when trash accumulated, residents seldom picked it up. The cars, Appleyard determined, reduced the amount of territory residents felt responsible for. Noise was a constant intrusion into their homes. Many Franklin Street residents covered their doors and windows and spent most of their time in the rear of their houses. Most families with children had already left. Conditions on Octavia Street were much different. Residents picked up trash. They sat on their front steps and chatted with neighbors. They had three times as many friends and twice as many acquaintances as the people on Franklin. On Gough Street, residents said that the old feeling of community was disappearing as traffic increased. People were becoming more and more preoccupied with their own lives. A number of families had recently moved, and more were conSidering it. Those who were staying expressed deep regret at the destruction of their community. 24. The word "astounding" in line 1 is closest in meaning to (A) startling (B) disappointing (C) dubious (D) alternative @.i.U'i.'jUj,JM'W$t-. 464 Practice Test 2 25. The three streets mentioned in this passage are different in that (A) they are in different cities (B) the residents are of different ethnic backgrounds (C) they have varying amounts of traffic (D) the income levels of the residents vary considerably 26. Approximately how many cars use Fntnklin Street daily? (A) 2,000 (13) 8,000 (C) 16,000 (D) 20,000 27. All of the following are direct results of heavy traffic EXCEPT (A) increased amounts of trash (B) greater danger to residents (C) more pollution (D) more vibrations 28. The author's main purpose in the second paragraph is to (A) discuss the problems of trash disposal (B) point out the disadvantages of heavy traffic (C) propose an alternate system of transportation (D) suggest ways to cope with trafflc problems 29. On which street is there the most social interaction? (A) Octavia Street (B) Gough Street (C) Franklin Street (D) There is no significant social interaction on any of the three streets. 30. The word "chatted" in line 23 is closest in meaning to (A) joked (13) talked (C) argued (D) walked 31. Which of the following is NOT a statement you would expect from a resident of Gough Street? (A) People on this street are unhappy because the neighborhood is deteriorating. (13) People on this street think mostly of themselves. (C) People on this street have more and more space for which they feel responsible. (D) A number of people are preparing to leave this street. 32. In what order does the author present detailed discussions of the three streets? (A) LIGHT, MEDIUM, HEAVY (B) HEAVY, MEDIUM, LIGHT (C) HEAVY, LIGHT, MEDIUM (D) LIGHT, HEAVY, MEDIUM @.i.",t.',UjU43 i ¥t-. Section 3 • Reading Comprehension 465 Questions 33-42 (line) (5) (10) (15) (20) Rachel Carson was born in 1907 in Springsdale, Pennsylvania. She studied biology at college, and zoology at Johns Hopkins University, where she received her master's degree in 1933. In 1936, she was hired by the U.S. Fish and Wildlife Service, where she worked most of her life. Carson's first book, Under the Sea Wind, was published in 1941. It received excellent reviews, but sales were poor until it was reissued in 1952. In that year she published The Sea Around Us, which provided a fascinating look beneath the ocean's surface, emphasizing human history as well as geology and marine biology. Her imagery and language had a poetic quality. Carson consulted no fewer than 1,000 printed sources. She had voluminous correspondence and frequent discussions with experts in the field. However, she always realized the limitations of her non-technical readers. In 1962 Carson published Silent Spring, a book that sparked considerable controversy. It proved how much harm was done by the uncontrolled, reckless use of insecticides. She detailed how they poison the food supply of animals, kill birds and fish, and contaminate human food. At the time, spokesmen for the chemical industry mounted personal attacks against Carson and issued propaganda to indicate that her findings were flawed. However, her work was vindicated by a 1963 report of the President's Science Advisory Committee. 33. The passage mainly discusses Rachel Carson's work (A) as a researcher (B) at college (C) at the U.S. Fish and Wildlife Service (D) as a writer 34. According to the passage, what did Carson primarily study at Johns Hopkins University? (A) Oceanography (ll) History (C) Literature (D) Zoology 35. When she published her first book, Carson was closest to the age of (A) 26 (B) 29 (C) 34 (D) 45 @"""t.'Uti.t§!i'WU-. 466 Practice Test 2 36. It can be inferred from the passage that in 1952, Carson's book Under the Sea Wind (A) was outdated (B) became more popular than her other books (C) was praised by critics (D) sold many copies 37. Which of the following was NOT mentioned in the passage as a source of information for The Sea Around Us? (A) Printed matter (B) Talks with experts (C) A research expedition (D) Letters from scientists 38. Which of the following words or phrases is LEAST accurate in describing The Sea Around Us? (A) Highly technical (B) Poetic (C) Fascinating (D) Well-researched 39. The word "reckless" in line 18 is closest in meaning to (A) unnecessary (B) limited (C) continuous (D) irresponsible 40. According to the passage, Silent Spring is primarily (A) an attack on the use of chemical preservatives in food (B) a discussion of the hazards insects pose to the food supply (C) a warning about the dangers of misusing insecticides (D) an illustration of the benefits of the chemical industry 41. The word "flawed" in line 22 is closest in meaning to (A) faulty (B) deceptive (C) logical (D) offensive 42. Why does the author of the passage mention the report of the President's Science Advisory Committee (lines 23-24)? (A) To provide an example of government propaganda (B) To support Carson's ideas (C) To indicate a growing government concern with the environment (D) To validate the chemical industry's claims @.i.,;,t.,,;@U43 i ¥(. • Section 3 • Reading Comprehension 467 Questions 43-50 (line) (5) (10) (15) (20) What is meant by the term economic resources? In general, these are all the natural, man-made, and human resources that go into the production of goods and services. This obviously covers a lot of ground: factories and farms, tools and machines, transportation and communication facilities, all types of natural resources, and labor. Economic resources can be broken down into two general categories: property resources-land and capitaland human resources-labor and entrepreneurial skills. What do economists mean by land? Much more than the noneconomist. Land refers to all natural resources that are usable in the production process: arable land, forests, mineral and oil deposits, and so on. What about capital? Capital goods are all the man-made aids to producing, storing, transporting, and distributing goods and services. Capital goods differ from consumer goods in that the latter satisfy wants directly, while the former do so indirectly by facilitating the production of consumer goods. It should be noted that capital as defined here does not refer to money. Money, as such, produces nothing. The term labor refers to the physical and mental talents of humans used to produce goods or services (with the exception of a certain set of human talents, entrepreneurial skiIIs, which will be considered separately because of their special significance). Thus the services of a factory worker or an office worker, a ballet dancer or an astronaut all fall under the general heading of labor. 43. What is the author's main purpose in writing this passage? (A) To explain the concept of labor (B) To criticize certain uses of capital (C) To contrast capital goods and consumer goods (D) To define economic resources 44. In lines 3-4, the author uses the expression "This obviously covers a lot of ground ..." to indicate that (A) the factories and farms discussed in the passage are very large (B) economic resources will be discussed in great depth (C) the topic of economic resources is a broad one (D) land is an important concept in economics 45. When non-economists use the term "land," its definition (A) is much more general than when economists use it (B) is much more restrictive than when economists use it (C) changes from place to place (D) includes all types of natural resources 'St""I' t."1M' ,t*I'¥'. 468 Practice Test 2 46. The word "arable" in line 11 is closest in meaning to (A) dry (B) fertile (C) developed (D) open 47. The phrase "the latter" in line 15 refers to (A) economists (B) non-economists (C) capital goods (D) consumer goods 48. Which of the following could be considered a capital good as defined in the passage? (A) A railroad (B) Money (C) A coal deposit (D) Human skills 49. The word "heading" in line 24 is closest in meaning to (A) direction (B) practice (C) category CD) utility 50. The skills of all the following could be considered examples of labor, as defined in the passage, EXCEPT (A) artists and scientists (B) workers who produce services, not goods (C) office workers (D) entrepreneurs THIS IS THE END OF THE SAMPLE READING COMPREHENSION SECTION. IF YOU FINISH BEFORE TIME IS CAll.ED, GO BACK AND CHECK YOUR WORK IN THIS SECTION ONLY. ~TO' ~TO' ~TO' ~TO' ~TO' ~TO' Practice Test 3 469 Practice Test 3 469 470 Practice Test 3 Listening Comprehension This section tests your ability to comprehend spoken English. It is divided into three parts, each with its own directions. You are not permitted to turn the page during the reading of the directions or to take notes at any time. Part A Directions: Each item in this part consists of a brief conversation involving two speakers. Following each conversation, a third voice will ask a question. You will hear the conversations and questions only once, and they will not be written out. When you have heard each conversation and question, read the four answer choices and select the onc-(A), (B), (C), or (D)-that best answers the question based on what is directly stated or on what can be inferred. Then fill in the space on your answer sheet that matches the letter of the answer that you have selected. Here is an example. You will hear: Sample Answer You will read: (A) Open the window. (B) Move the chair. (C) Leave the room. (0) Take a seat. From the conversation you find out that the woman thinks the man should put the chair over by the window. The best answer to the question, "What does the woman think the man should do?" is (B), "Move the chair." You should fill in (n) on your answer sheet. 8 Section 1 • Listening Comprehension 471 mmmmmmmmmmmm 1. (A) She broke the window herself. (B) She repaired the broken window. (C) She was able to get the widow open. (D) She hurt herself on the broken glass. 2. (A) The golf tournament made it famous. (B) It's grown a lot lately. (C) It can't be found without a map. (D) Very few people there play golf. 3. (A) Where she got her information. (B) How the copy was made. (C) Who painted the picture. (D) Why the copy was made. 4. (A) Groceries. (B) A used car. (C) Gasoline. (D) Medicine. 5. (A) She read it again and again. (B) She covered it up. (C) She read every page of it. (D) She ripped its cover off. 6. (A) He can no longer play. (B) He's played every day for years. (C) His playing has improved. (D) He played quite well. 7. (A) He needs to get more camping equipment. (B) He is an experienced camper. (C) He is taking too much equipment. (D) He shouldn't go camping for such a long time. 8. (A) Try to get elected mayor. (B) Attend a class. (C) Interview the mayor. (D) Apply for a job. 9. (A) She was mistaken about Professor Leguin. (B) She just returned from San Francisco. (C) She doesn't know what the professor looks like. (D) She's an admirer of Professor Leguin. 10. (A) He's speaking a language they don't know. (B) He doesn't have a microphone. (C) He's speaking much too quickly. (0) He's using a defective microphone. 11. (A) It's near the elevator. (B) He doesn't know where it is. (C) It's on another floor. (D) The directory doesn't list it. 12. (A) She's heard it only a few times. (B) She doesn't get to listen to it very often. (C) She once liked it, but she's heard enough. (D) She enjoys it very much. 13. (A) It's no wonder that she had to work. (B) It wasn't busy because of the weather. (C) She was very busy at work. (0) The snow made her late for work. 14. (A) He doesn't have any money left either. (B) The club is looking for some new members. (C) He can lend the woman some money. (0) It doesn't cost much to join the club. 15. (A) He doesn't enjoy receptions. (B) He uses his computer a lot. (C) His computer isn't working. (0) He will defmitely attend. 16. (A) Only one person in the group is older than he. (B) His group is almost the oldest. (C) He's the youngest person in the group. (D) He appears only in the second photograph. 17. (A) Pancakes are not his favorite dish. (B) His pancakes don't taste very good. (C) He never makes enough pancakes. (D) He can't cook many dishes. l§.i.';'i.ijUjU§!iiijWt-. 472 Practice Test 3 18. (A) She thinks Professor Bryant is unfair. (B) She doesn't know Professor Bryant. (C) She agrees with the man. (D) She doesn't understand the man's remark. 19. (A) In a few days. (B) Before they eat. (C) During lunch. (D) When lunch is over. 20. (A) The woman would enjoy the mountain scenery. (B) The weather has been hot this month. (C) The weather in the mountains is unusual. (D) The woman probably doesn't like cool weather. 25. (A) When she will answer the questions. (B) Where she drove. (C) What kind of car she has. (D) Why she asked so many questions. 26. (A) He thinks it will be better than the old one. (B) He's anxious for it to be completed. (C) He's worried that it's not long enough. (D) He feels that it shouldn't have been built. 27. (A) Where he's studying. (B) What subject he's studying. (C) How long he's been in Europe. (D) When he's returning. 21. (A) It has exceptionally good service. (B) It has excellent food. (C) The service there is disappointing. (D) Everything there is great. 28. (A) Charlotte wouldn't be attending graduate school. (B) Charlotte had gotten a scholarship. (C) Graduate school wouldn't start until September. (D) Scholarships were easy to get. 22. (A) If his name sounds familiar. (B) If she's spoken to him on the phone. (C) If he's a musician. (D) If she likes his name. 29. (A) He paid it today for the flrst time. (B) He pays it on the last day of the month. (C) He pays it after it's due. (D) He's planning to pay it tomorrow. 23. (A) He thought other science courses would be harder. (B) It's a required class for all students. (C) He's studied geology before. (D) It was the only science course open to him. 30. (A) The debate involved only a few issues. (B) Many people changed their plans. (C) A lot of people attended. (D) The debate lasted longer than expected. 24. (A) He's never been to the zoo. (B) He's seen only one bear. (C) He's never seen a bear in the wild. (D) There weren't any bears at that zoo. @.i.,,'i·"Uj h43 i @t-. Section 1 • Listening Comprehension 473 ITJITJITJITJITJITJITJITJITJITJITJITJ Part B Directions: This part of the test consists of extended conversations between two speakers. After each of these conversations, there are a number of questions. You will hear each conversation and question only once, and the questions are not written out. When you have heard the questions, read the four answer choices and select the one-(A), (B), (C), or (D)-that best answers the question based on what is directly stated or on what can be inferred. Then flll in the space on your answer sheet that matches the letter of the answer that you have selected. Don't forget: During actual exams, taking notes or writing in your test book is not permitted. 8 474 Practice Test 3 [I] [I] [IJ[I][I][I] [I] [I] [I] [I] [I] [I] 31. (A) Because it was cheap. (B) Because it is in such good condition. (C) Because it is a collector's item. (D) Because he can re-sell it at a high price. 35. (A) To attend a conference. (B) To see the planetarium. (C) To change planes. (D) To go sightseeing. 32. (A) In the (B) In the (C) In the (D) In the 36. (A) (B) (C) (D) 1930's. 1940's. 1950's. 1960's. 33. (A) Replace its engine. (B) Enter it in some shows. (C) Take it on a long drive. (D) Re-seU it for more money. She recently went there. It's not a very good one. There's one in her hometown. It will be closed when she's free. 37. (A) On foot and by boat. (B) By car and on foot. (C) By air and by car. (D) By car and by bus. 34. (A) At a conference hall. (B) At an art gallery. (C) At an airport. (D) At a hotel. lij.i.';it.',;Uj U43 i @t. • Section 1 • Listening Comprehension 475 ITJITJITJITJITJITJITJITJITJITJITJITJ Part C Directions: This part of the test consists of several talks, each given by a single speaker. After each of these talks, there are a number of questions. You will hear each talk and question only once, and the questions are not written out. When you have heard each question, read the four answer choices and select the one-(A), (B), (C), or (D)-that best answers the question based on what is directly stated or on what can be inferred. Then fill in the space on your answer sheet that matches the letter of the answer that you have selected. Here is an example. You will hear: Now here is a sample question. Sample Answer You will read: (A) Philosophy. (B) Meteorology. (C) Astronomy. (D) Photography. The lecture concerns a lunar eclipse, a topic that would typically be discussed in an astronomy class. The choice that best answers the question, "In what course is this lecture probably being given?" is (C), "Astronomy." You should fill in (C) on your answer sheet. Here is an example. You will hear: Sample Answer You will read: (A) The Earth's shadow moves across the Moon. (B) Clouds block the view of the Moon. (C) The moon moves between the Earth and the Sun. (D) The Sun can be observed without special equipment. From the lecture you learn that a lunar eclipse occurs when the Earth moves between the Sun and the Moon and the shadow of the Earth passes across the Moon. The choice that best answers the question, "According to the speaker, which of the following occurs during a lunar eclipse?" is (A), "The Earth's shadow moves across the Moon." Don't forget: During actual exams, taking notes or writing in your test book is not permitted. 8 476 Practice Test 3 45. (A) In the spring. (B) In the summer. (C) In the fall. (D) In the winter. 38. (A) Tourists. (B) Professional dancers. (C) Students. (D) Traditional musicians. 39. (A) It will be different from the ones performed in Hawaii today. (B) It will involve women wearing grass skirts. (C) It will involve only male dancers. (0) It was once performed for great Hawaiian leaders. 40. (A) They prohibited it. (B) They sponsored it. (C) They proposed some small changes in it. (D) They exported it to other islands. 41. (A) Attend a live performance. (B) Go on a tour. (C) Perform a dance. (D) Watch a DVD. 42. (A) To discuss a weather phenomenon. (B) To explain how to drive during storms. (C) To describe supercooled water. (D) To warn gardeners of the danger of hail. 43. (A) Because of its size. (B) Because of its color. (C) Because of its layers. (D) Because of its weight. 44. (A) As (B) As (C) As (D) As 46. (A) New theories about the origin of language. (B) How to teach grammar to children. (C) Mistakes children sometimes make. (D) The stages of children's language learning. 47. (A) "Koo, Koo." (B) "Da-da." (C) "More milk!" (D) "Na-na." 48. (A) Between four and eight months. (B) Between one year and eighteen months. (C) Between two and three years. (0) Between three and four years. 49. (A) They are the same in all languages. (B) They are often misinterpreted. (C) They are learned by imitation. (D) They are quite logical. 50. (A) They are too complicated. (B) She doesn't have time to talk about them today. (C) The class didn't have a chance to read about them. (D) She doesn't agree with them. a drop of supercooled water. a snowflake. a particle of dust. a ball of ice. TIllS IS THE END OF SECTION 1, LISTENING COMPREHENSION. STOP WORK ON SECTION 1. ~TO' ~TO' Section 2 • Structure and Written Expression 477 ~ Structure and Written Expression Time: 25 minutes This section tests your ability to recognize grammar and usage suitable for standard written English. This section is divided into two parts, each with its own directions. Structure Directions: Items in this part are incomplete sentences. Following each of these sentences, there are four words or phrases. You should select the one word or phrase-(A), (B), (C), or (D)-that best completes the sentence. Then fill in the space on your answer sheet that matches the letter of the answer that you have selected. Example I Sample Answer CD"COC[) Pepsin ____ an enzyme used in digestion. (A) that (B) is (C) of (D) being This sentence should properly read "Pepsin is an enzyme used in digestion." You should fill in (B) on your answer sheet. Example II Sample Answer CD CD CO . . ____ large natural lakes are found in the state of South Carolina. (A) There are no (B) Not the (C) It is not (D) No This sentence should properly read "No large natural lakes are found in the state of South Carolina." You should fill in (D) on your answer sheet. As soon as you understand the directions, begin work on this part. @.i·"'(·"UjUM.@t-. 478 Practice Test 3 eeeeeeeeeeee 1. Ellen Swallow Richards became the flrst woman to enter, graduate from, and _ _ _ _ __ at the Massachusetts Institute of Technology. (A) teach (B) a teacher (C) who taught (D) to teach 2. Coins last approximately twenty times _ _ _ _ _ _ paper bills. (A) longer (B) as long (C) long (D) longer than 3. It has been estimated that _ _ _ _ _ _ species of animals. (A) more than a million (B) it is a million or more (C) there are over a million (D) are over a million of 4. Dr. Seuss, was Theodore Seuss Geisel, wrote and illustrated delightfully humorous books for children. (A) his real name (B) who had as his real name (C) with his real name (D) whose real name 5. American landscape architects was Hideo Sasaki. (A) The most famous one of (B) One of the most famous (C) Of the one most famous (D) The one most famous of 6. Most young geese leave their nests at an early age, and young snow geese are _ _ _ _ __ exception. (A) not (B) no (C) none (D) never 7. Vancouver, British Columbia, has a temperate climate for a city situated _ _ _ _ _ _ far north. (A) as (B) so (C) very (D) by lij.i.",t.ilUJUtii!Wt. • Section 2 • Structure and Written Expression 479 •••••••••••• 8. in 1849, Manuel A. Alonso recorded the customs, language, and songs of the people of Puerto Rico in his poetry and prose. (A) Beginning (B) He began (C) Having begun (D) The beginning was 9. the sails of a distant ship are visible before the body of the ship. (A) The curve of the Earth makes (B) The Earth, in that it curves, makes (C) Because the curve of the Earth, CD) Because of the curve of the Earth, 10. Printing ink is made _ _ _ _ _ _ of a paste, which is applied to the printing surface with rollers. (A) to form (B) the form (C) in the form (D) so that it forms cold climates, they can thrive in hot, dry climates as well. 11. Although (A) sheep adapted well (B) well-adapted sheep (C) sheep, well adapted to (D) sheep are well adapted to 12. Not only as a shade tree, but it also produces wood used for boxes and furniture. (A) the Linden tree is of value (B) the value of the Linden tree is (C) is the Linden tree valuable (D) the valuable Linden tree is used 13. one of Laura Ingalls Wilder's many books about the American frontier are based on her own childhood experiences. (A) Except (B) All but (C) Without (D) Not any 14. One of the ftrst industries to be affected by the Industrial Revolution _ _ _ _ __ (A) was the textile industry (B) the textile industry (C) in the textile industry (D) the textile industry was 15. the outer rings of a gyroscope are turned or twisted, the gyroscope itself continues to spin in exactly the same position. (A) However (B) Somehow • (C) Otherwise (D) No matter @.'.U,t.ihtjU=tii@t- 480 Practice Test 3 Written Expression Directions: The items in this part have four underlined words or phrases, (A), (B), (C), and (D). You must identify the one underlined expression that must be changed for the sentence to be correct. Then fmd the number of the question on your answer sheet and fill in the space corresponding to the letter. Example! Sample Answer Lenses may to have either concave or ABC convex shapes. --D- This sentence should read "Lenses may have either concave or convex shapes." You should therefore select answer (A). Example II Sample Answer CD-'CDCD When painting a fresco, an artist is applied paint --A-- B directly to the damp plaster of a wall. e D This sentence should read "When painting a fresco, an artist applies paint directly to the damp plaster of a wall." You should therefore select answer (B). As soon as you understand the directions, begin work on this section. @.i.".t.',UjlI§3 i @t--' Section 2 • Structure and Written Expression 481 16. The rock formations in the Valley of Fire in Nevada has been worn into many strange shapes A -BC by the action of wind and water. o 17. The author Susan Glaspell won a Pulitzer Prize in 1931 for hers play, Alison's House. Cl) -B- --A- 18. Haywood Broun was a read widely newspaper columnist who wrote during the 1920's and A B C --0- 1930's. 19. Researches in economics, psychology, and marketing can help businesses. I) A B C 20. Because of their color and shape, seahorses blend so well with the seaweed in which they live A B C that it is almost impossible to see themselves. o 21. Although the social sciences different a great deal from one another, they share a common A B C interest in human relationships. o 22. Herman Melville's novel Moby-Dick describes the dangers, difficult, and often violent life -A-- B C aboard a whaling ship. --0-- 23. Near equator, the slant of the sun's rays is never great enough to cause temperatures to fall A --c-- B below the freezing point. o 24. Stephen Hopkins was a cultural and political leadership in colonial Rhode Island. ~ B C D 25. A mousebird's tail is double as long as its body. ABC 0 lij.i·",t."UjUii .@. 482 Practice Test 3 eeeeeeeeeeee 26. The Uinta Mountains of northeastern Utah are the only range of mountains in North America A that runs from east and west for its entire length. c- B --0- 27. The tools used most often by floral designers are the knives, scissors, and glue gun. A B -C- -0- 28. Most types of dolphins live at less 25 years, and some species may reach 50 years of age. -A- --B- -C- -0- 29. Isle Royale National Park in Lake Superior can only be reached by the boat. A 13 C 0 30. The main divisions of geologic time, called eras, are subdivided to periods. A 0 -B- --C- 31. All root vegetables grow underground, and not all vegetables that grow underground are ~ -Bc--0 roots. 32. The process of fermentation takes place only in the absent of oxygen. A B C --0- 33. In about 1920, experimental psychologists have devoted more research to learning than to A B C any other topic. o 34. Transfer taxes are imposed on the sell or exchange of stocks and bonds. A 13" C ----n35. One of the greatest of mountains climbers, Carl Blaurock was the ftrst to climb all of the A B C mountains higher than 14,000 feet in the United States. ----r> 36. Biochemists have solved many of the mysteries about photosynthesis, the process which A B C plants make food. o @.i.,;,t·"UjUiii@t-. Section 2 • Structure and Written Expression 483 37. Oceanic islands have been separated from the mainland for too long that they have evolved --A1B C distinctive animal populations. D 38. Certain species of penicillin mold are used to ripe cheese. A B -----c: - D - 39. Many of the important products obtained from trees, one of the most important is wood pulp, A B C which is used in paper-making. D 40. Not longer are contributions to the advancement of industry made primarily by individuals. A B C -D- THIS IS THE END OF SECTION 2. IF YOU FINISH BEFORE TIME IS CALLED, CHECK YOUR WORK ON SECTION 2 ONLY. DO NOT READ OR WORK ON ANY OTHER SECTION OF THE TEST. ~TO' ~TO' ~TO' ~TO' ~TO' ~TO' 484 Practice Test 3 00 Reading Comprehension Time: 55 minutes This section of the test measures your ability to comprehend written materials. Directions: This section contains several passages, each followed by a number of questions. Read the passages and, for each question, choose the one best answer-(A), (B), (C), or (D)-based on what is stated in the passage or on what can be inferred from the passage. Then fill in the space on your answer sheet that matches the letter of the answer that you have selected. Read the following passage: like mammals, birds claim their own territories. A bird's territory may be small or large. Some birds claim only their nest and the area right around it, while others claim far larger territories that include their feeding areas. Gulls, penguins, and other waterfowl nest in huge colonies, but even in the biggest colonies, each male and his mate have small territories of their own immediately around their nests. Male birds defend their territory chiefly against other males of the same species. In some cases, a warning call or threatening pose may be all the defense needed, but in other cases, intruders may refuse to leave peacefully. (line) (5) (10) Example! Sample Answer CDCDCD'What is the main topic of this passage? (A) Birds that live in colonies (B) Birds' mating habits (C) The behavior of birds (D) Territoriality in birds The passage mainly concerns the territories of birds. You should fill in CD) on your answer sheet. Example II Sample Answer According to the passage, male birds defend their territory primarily against (A) (B) (C) (D) female birds birds of other species males of their own species mammals The passage states that "Male birds defend their territory chiefly against other males of the same species." You should fill in (C) on your answer sheet. As soon as you understand the directions, begin work on this section. Section 3 • Reading Comprehension 485 Questions 1-11 (line) (5) (10) (15) (20) To date, Canada has produced only one classic children's tale to rank with Alice's Adventures in Wonderland and the works of Mark Twain; this was Lucy Maud Montgomery's Anne of Green Gables. Lucy Maud Montgomery was born in Clinton, Prince Edward Island. Her mother died soon after her birth, and when her father went to Saskatchewan to assume a business position, she moved in with her grandparents in Cavendish, Prince Edward Island. There she went to school, and later qualified to be a teacher. Montgomery wrote the Anne books while living in Cavendish and helping her grandmother at the post office. The fIrst of the books, Anne of Green Gables, was published in 1908, and in the next three years she wrote two sequels. Like Montgomery, the heroine of the book is taken in by an elderly couple who live in the fictional town of Avonlea, and Montgomery incorporated many events from her life in Cavendish into the Anne books. In 1911, Montgomery married Ewan MacDonald, and the couple soon moved to Ontario, where she wrote many other books. However, it was her first efforts that secured her prominence, and the Anne books are still read all around the world. Her novels have helped create a warm picture of Prince Edward Island's special character. Several movies, a television series, and a musical play have been based on her tales, and today visitors scour the island for locations described in the book. 1. The main purpose of this passage is to (A) introduce Montgomery and her Anne books (B) contrast Canadian children's literature with that of other countries (C) provide a brief introduction to Prince Edward Island (0) show the similarities between Montgomery's life and that of her fictional character Anne. 2. The word "this" in line 3 refers to (A) Canada (B) the work of Mark Twain (C) Alice's Adventures in Wonderland (0) a Canadian children's classic 3. According to the passage, Montgomery was raised primarily (A) in an orphanage (B) by her grandparents (C) by her mother (D) by her father @.i·';'(·"ltjlIiii@t-. 486 Practice Test 3 4. Approximately when did Lucy Maud Montgomery write the two sequels to her book Anne of Green Gables? (A) From 1874 to 1908 (B) From 1908 to 1911 (C) From 1911 to 1913 (D) From 1913 to 1918 5. The word "elderly" in line 13 is closest in meaning to (A) kindly (B) old (C) friendly (D) sly 6. In the Anne books, the main character lives in (A) the town of Cavendish (B) Saskatchewan (C) the town of Avonlea (D) Ontario 7. Which of the following can be concluded from the passage about the Anne books? (A) They were at least partially autobiographical. (B) They were influenced by the works of Mark Twain. (C) They were not as successful as Montgomery's later works. (D) They were not popular until after Montgomery had died. 8. The word "prominence" in line 18 is closest in meaning to (A) reputation (B) excellence (C) effort (D) permanence 9. Which of the following is closest in meaning to the word "character" in line 21? (A) A person in a novel (B) Nature (C) A written symbol (D) Location 10. All of the following have been based on the Anne books EXCEPT (A) a television series (B) movies (C) a play (D) a ballet 11. In line 22, the word "scour" could be replaced by which of the following without changing the meaning of the sentence? (A) Cleanse (B) Admire (C) Search (D) Request lij.i.,;,t,',UjUWi@t-. Section 3 • Reading Comprehension 487 Questions 12-23 (line) (5) (10) (15) (20) (25) (30) Certain animals have an intuitive awareness of quantities. They know without analysis the difference between a number of objects and a smaller number. In his book The Natural History of Selbourne (1786), the naturalist Gilbert White tells how he surreptitiously removed one egg a day from a plover's nest, and how the mother laid another egg each day to make up for the missing one. He noted that other species of birds ignore the absence of a single egg but abandon their nests if more than one egg has been removed. It has also been noted by naturalists that a certain type of wasp always provides five-never four, never sixcaterpillars for each of their eggs so that their young have something to eat when the eggs hatch. Research has also shown that both mice and pigeons can be taught to distinguish between odd and even numbers of food pieces. These and similar accounts have led some people to infer that creatures other than humans can actually count. They also point to dogs that have been taught to respond to numerical questions with the correct number of barks, or to horses that seem to solve arithmetic problems by stomping their hooves the proper number of times. Animals respond to quantities only when they are connected to survival as a species-as in the case of the eggs-or survival as individuals-as in the case of food. There is no transfer to other situations or from concrete reality to the abstract notion of numbers. Animals can "count" only when the objects are present and only when the numbers involved are small-no more than seven or eight. In lab experiments, animals trained to "count" one kind of object were unable to count any other type. The objects, not the numbers, are what interest them. Animals' admittedly remarkable achievements simply do not amount to evidence of counting, nor do they reveal more than innate instincts, refmed by the genes of successive generations, or the results of clever, careful conditioning by trainers. 12. What is the main idea of this passage? (A) Careful training is required to teach animals to perform tricks involving numbers. (B) Animals cannot "count" more than one kind of object. (C) Of all animals, dogs and horses can count best. (D) Although some animals may be aware of quantities, they cannot actually count. @.i.",t·",t:i U43 i @t-. 488 Practice Test 3 13. Why does the author refer to Gilbert White's book in line 3? (A) To show how attitudes have changed since 1786 (B) To contradict the idea that animals can count (C) To provide evidence that some birds are aware of quantities (D) To indicate that more research is needed in this field 14. The word "surreptitiously" in line 4 is closest in meaning to (A) quickly (B) secretly (C) occasionally (D) stubbornly 15. The word "abandon" in line 8 is closest in meaning to (A) vacate (B) rebuild (C) move (D) guard 16. The word "odd;' as used in line 14, refers to which of the following? (A) Unusual numbers (B) Numbers such as 1, 3, 5, and so on (C) Lucky numbers (0) Numbers such as 2, 4, 6, and so on 17. The author mentions that all of the following are aware of quantities in some way EXCEPT (A) plovers (B) mice (C) caterpillars (D) wasps 18. The word "accounts" in line 15 is closest in meaning to (A) invoices (B) reasons (C) reports (D) deceptions 19. According to information in the passage, which of the following is LEAST likely to occur as a result of animals' intuitive awareness of quantities? (A) A pigeon is more attracted by a box containing two pieces of food than by a box containing one piece. (B) When asked by its trainer how old it is, a monkey holds up five fingers. (C) When one of its four kittens crawls away, a mother cat misses it and searches for the miSSing kitten. (D) A lion follows one antelope instead of a herd of antelopes because it is easier to hunt a single prey. @.i.';'i.',UjUtii@t. • Section 3 • Reading Comprehension 489 20. How would the author probably characterize the people who are mentioned in line 15? (A) As mistaken (B) As demanding (C) As clever (D) As foolish 21. The word "admittedly" in line 29 is closest in meaning to (A) improbably (B) arguably (C) apparently CD) undeniably 22. In line 31, the word "they" refers to (A) numbers (B) animals (C) achievements (D) genes 23. Where in the passage does the author mention research that supports his own view of animals' inability to count? (A) Lines 3-6 (B) Lines 12-14 (C) Lines 15-16 (D) Lines 27-28 Questions 24-33 (line) (5) (10) (15) It would be hard to cite a development that has had more impact on American industry than the Bessemer process of making steel. It made possible the production of low-cost steel and established the foundation of the modern steel industry. In many ways it was responsible for the rapid industrialization of the United States that took place in the formative period of the late 1800's. The first Bessemer plant in the United States was built in Wyandotte, Michigan, in 1864, near the end of the Civil War. It was capable of producing only 2 tons of steel ingots at a time. The ingots were rolled into rails-the first steel rails made in the United States. Acceptance of the process was initially slow. By 1870, the annual output of Bessemer steel was a mere 42 thousand tons. Production grew rapidly after about 1875, rising to 1.2 million tons in 1880, when it exceeded that of wrought iron for the first time. Id.i.';it.ilUj U4:J i UW t- . 490 Practice Test 3 (20) (25) The rise of the US. steel industry in the last quarter of the 19th century was brought about largely by the demand for Bessemer steel rails for the nation's burgeoning rail network. Steel rails were far more durable than those made of iron. Spurred by this demand, the us. steel industry became the largest in the world in 1886, when it surpassed that of Great Britain. The Bessemer Process was the chief method of making steel until 1907, when it was overtaken by the open-hearth process. By the 1950's, the Bessemer Process accounted for less than 3% of the total U.S. production. 24. With what topic is this passage mainly concerned? (A) The history of metal-working (B) A comparison of the US. and British steel industries in the nineteenth century CC) The technical details of the Bessemer process CD) The effects of one method of making steel 25. According to the passage, the Bessemer process contributed to all of the following EXCEPT (A) the establishment of the modern steel industry in the United States CB) the manufacture of weapons during the Civil War CC) lowered costs for steel CD) industrial development in the United States during an important period 26. What can be inferred from the passage about wrought iron? (A) At one time, more of it was produced than Bessemer steel. (B) It is a by-product of the Bessemer process. (C) It was once primarily imported from Great Britain. CD) It later became a more important product than Bessemer steel. 27. The word "burgeoning" in line 19 is closest in meaning to (A) overpowering (B) planned (C) expanding CD) vital 28. According to the passage, why were Bessemer steel rails used in place of iron rails? (A) They lasted longer. (B) They did not have to be imported. (C) They could be installed faster. (D) They provided a smoother ride for passengers. 29. The word "Spurred" in line 20 is closest in meaning to which of the following? (A) Driven CB) Challenged (C) Dominated (D) Broken lij.i.",t.',UjUY:'i@t-. Section 3 • Reading Comprehension 491 30. According to the passage, in what year did the steel industry of the United States begin to produce more steel than that of Great Britain did? (A) 1864 (B) 1875 (C) 1880 (D) 1886 31. What can be inferred about the steel industry in the United States during the 1950's? (A) It had begun producing many new types of products. (B) It was in a period of severe decline. (C) It primarily involved methods of production other than the Bessemer Process. (D) It was becoming more and more important. 32. The paragraph following this one probably concerns (A) innovations in the railroad industry (B) the open-hearth method of making steel (C) industrialization in the twentieth century (D) new methods of making wrought iron 33. The author first begins to discuss the growth of the Bessemer Process in (A) lines 3-4 (B) lines 8-9 (C) lines 14-16 (D) lines 23-24 Questions 34-41 (line) (5) (10) (15) Nearly 515 blocks of San Francisco, including almost all of Nob Hill, were destroyed by the 1906 earthquake and fires. Many of San Francisco's "painted ladies" -its gaudy, nineteenth century Victorian houses-were lost in the disaster. Today, some 14,000 surviving houses have been preserved, particularly in the Cow Hollow, Mission, Pacific Heights, and Alamo Square districts. Distinguished by their design characteristics, three styles of San Franciscan Victorians can be found today. The Italianate, which flourished in the 1870's, is characterized by a flat roof, slim pillars flanking the front door, and bays with windows that slant inward. The ornamentation of these narrow row houses was patterned after features of the Roman Classical styles. The Stick style, which peaked in popularity during the 1880s, added ornate woodwork outlines to the doors and windows. Other additions included the French cap, gables, and three-sided bays. Designs changed dramatically when the Queen Anne style became the rage in the 1890's. Turrets, towers, steep gabled roofs, and glass art windows distinguished Queen Anne houses from their predecessors. (ij.i.,;,(.ilUjU§iii@[-. 492 Practice Test 3 (20) (25) In the period after the earthquake, the Victorians came to be regarded as impossibly old-fashioned, but beginning around 1960, owners began peeling off stucco, tearing off false fronts, reapplying custom woodwork, and commissioning multi-hued paint jobs. Before long, many of these houses had been restored to their former splendor. 34. Which of the following is NOT one of the author's purposes in writing the passage? (A) To talk about the restoration of Victorian houses in San Francisco in the 1960's (B) To discuss housing problems in San Francisco today (C) To briefly trace the history of Victorian houses in San Francisco (D) To categorize the three types of Victorian houses found in San Francisco 35. The word "gaudy" in line 3 is closest in meaning to (A) showy (B) enormous (C) antiquated (D) simple 36. According to the passage, in what district of San Francisco are authentic Victorian houses LEAST likely to be found today? (A) Cow Hollow (B) Pacific Heights (C) The Mission (D) Nob Hill 37. According to the passage, which of the following styles of architecture was the last to become fashionable in San Francisco? (A) Roman Classical (B) Italianate (C) Stick (D) Queen Anne 38. As used in the second paragraph, the word "bays" refers to (A) bodies of water (B) colors (C) architectural features (D) trees 39. Which of the following is most likely to be seen only on a Queen Anne style house? (A) A flat roof (B) A tower (C) A French cap (D) Gables @.i.,;,t.',Uj,t§!i.@t. • Section 3 • Reading Comprehension 493 40. During which of the following periods were San Francisco's Victorian houses generally thought of as old-fashioned? (A) From 1870 to 1890 (B) During the 1890's (C) From 1907 to 1960 (D) During the 1960's 41. What can be inferred from the passage about Victorian houses after they had been restored? (A) They were painted in many colors. (B) They looked exactly like modern houses. (C) They were covered with new fronts made of stucco. (D) They were more attractive than the original houses. Questions 42-50 (5) (10) (15) (20) (25) (30) Sea otters dwell in the North Pacific. They are the largest of the mustelids, a group which also includes freshwater otters, weasels, and badgers. They are from four to five feet long, and most weigh from 60 to 85 pounds. Large males may weigh 100 pounds or more. Unlike most marine mammals, such as seals or dolphins, sea otters lack a layer of blubber, and therefore have to eat up to 30% of their body weight a day in clams, crabs, fish, octopus, squids, and other delicacies to maintain body heat. Their voracious appetites do not create food shortages, though, because they are picky eaters, each animal preferring only a few food types. Thus no single type of food source is exhausted. Sea otters play an important environmental role by protecting forests of seaweed called kelp, which provide shelter and nutrients for many species. Certain sea otters feast on invertebrates like sea urchins and abalones that destroy kelp. Sea otters eat and sleep while floating on their backs, often on masses of kelp. They seldom come on shore. Sea otters keep warm by means of their luxuriant double-layered fur, the densest among animals. The soft outer fur forms a protective cover that keeps the fme underfur dry. One square inch of underfur contains up to 1 million hairs. Unfortunately, this essential feature almost led to their extinction, as commercial fur hunters drastically reduced their numbers. Under government protection, the sea otter population has recovered. While elated by the otters return, scientists are concerned about the California sea otter population growth of 5% a year, lagging behind the 18% a year rate among Alaska otters. Sea otters are extremely sensitive to pollution. In 1989 up to 5,000 sea otters perished when the Exxon Valdez spilled oil in Prince William Sound, Alaska. 494 Practice Test 3 42. According to the passage, what are mustelids? (A) A family of marine mammals that have blubber (B) A type of sea otter (C) A group of mammals that contains sea otters (D) A kind of sea animal that includes clams, crabs, and many other creatures 43. It can be inferred from that passage that, if a large male sea otter weighs 100 pounds, it must eat approximately how many pounds of food a day to maintain its body heat? (A) 5 pounds (B) 15 pounds (C) 30 pounds (D) 60 pounds 44. The author refers to sea otters as "picky eaters" Oine 11) because (A) all sea otters eat many types of food (B) each sea otter eats only one type of food (C) all sea otters have voracious appetites (D) each sea otter eats only a few kinds offood 45. The word "exhausted" in line 12 is closest in meaning to (A) needed (B) used up (C) desired (D) tired out 46. According to the passage, which of the following best describes sea otters' relationship with kelp forests? (A) The kelp serves as food for the otters. (B) The otters protect the kelp by eating animals that destroy it. (C) The otters eliminate the kelp's source of nutrients. (D) The kelp is destroyed when the otters build shelters. 47. Which of the following could best replace the word "luxuriant" in line 19? (A) Expensive (B) Soft (C) Abundant (0) Attractive 48. According to the passage, the outer fur of sea otters (A) keeps the underfur from getting wet (B) seems finer than the underfur (C) is more desirable to hunters than the underfur (D) is not as soft as the underfur 49. The word "elated" in line 26 is closest in meaning to (A) disappointed (B) shocked (C) concerned (D) overjoyed li.i·'lit,"IUiUMiQWt-. Section 3 • Reading Comprehension 50. According to the passage, why are scientists concerned about the population of California sea otters? (A) It has been growing at too fast a rate. (B) Its growth rate has been steadily decreasing. (C) Its growth rate is not as fast as that of the Alaska sea otters. (D) It has been greatly reduced by oil spills and other forms of pollution. THIS IS THE END OF THE SAMPLE READING COMPREHENSION SECTION. IF YOU FINISH BEFORE TIME IS CAllED, GO BACK AND CHECK YOUR WORK IN TIllS SECTION ONLY. ~TO~ ~TO~ ~TO~ ~TO' ~TO~ ~TO' 495 About the Author 497 About the Author Bruce Rogers has taught English as a Second Language and test preparation courses since 1979. He was a senior instructor in the English Program at the Economics Institute, University of Colorado, for 20 years and later became chair of the program. He has also taught in Indonesia, Vietnam, Korea, and the Czech Republic. He is the author of many books including The Complete Guide to the TOEFL: iBJ'Edition and The Complete Gllide to the TOBe both published by Heinle. He is currently president of Colorado TESOL. He lives in Boulder, Colorado, USA. Answer Sheet SAMPLE LISTENING COMPREHENSION TEST ANSWER SHEET 1. 2. 3. 4. 5. 6. 7. 8. 9. 10. 11. 12. 13. 14. 15. 16. 17. 18. 19. 20. 21. 22. 23. 24. 25. CD CD U":) CQ:) CD CD U":) CQ:) CD CD u":) CQ:) CD CD CD CQ:) CDCDCDCQ:) CDCDCDCI> CDCDCDCQ) CD CD CDCQ) CDCDCDCQ) CDCDCDCQ:) CDCDCDCQ:) CDCDCDCQ:) CDCDCDCQ:) CDCDCDCQ:) CDCDCDCQ:) CDCDCDCQ:) CD CD u":) CQ) CDCDCDCQ:) CDCDCDCQ:) CDCDCDCQ) CDCDCDCQ:) CDCDCDCQ:) CDCDCDCQ:) c£)CDCDCQ:) CDCDCDCB 26. 27. 28. 29. 30. 31. 32. 33. 34. 35. 36. 37. 38. 39. 40. 41. 42. 43. 44. 45. 46. 47. 48. 49. 50. CD CD U":) CQ:) CD CD U":) CQ:) CD CD CD CQ:) CDCDCDCQ:) CDCDCDCQ) c£)CDCDCI> CDCDCDCQ) CDCDCDCQ) CDCDCDCQ) CDCDCDCQ) CDCDu":)CQ) CDCDCDCQ) CD CD CD CQ:) CDCDCDCQ:) CD CD u":) CQ:) CDCDCDCQ:) CDCDCDCQ) CDCDCDCQ:) CDCDCDCQ) CDCDCDCI> CDCDCDCQ) CDCDCDCQ) CDCDCDCQ) CDCDCDCQ) CDCDCDCQ:) 499 Answer Sheet SAMPLE STRUCTURE AND WRITTEN EXPRESSION TEST ANSWER SHEET 1. 2. 3. 4. 5. 6. 7. 8. 9. 10. 11. 12. 13. 14. 15. 16. 17. 18. 19. 20. CDCDCDc[) CDCDCDCQ) CDCDCDc[) CD CD CDC[) CDCDCDc[) CD CD CDC[) CD CD CD C[) CDCDCDc[) CD CD CD c[) CDCDCDc[) CDCDCDc[) CDCDCDc[) CDCDCDc[) CDCDCDc[) CDCDCDc[) CD CD CDc[) CD CD CDc[) CDCDCDc[) CDCDCDc[) CD CD CDc[) 21. 22. 23. 24. 25. 26. 27. 28. 29. 30. 31. 32. 33. 34. 35. 36. 37. 38. 39. 40. CDCDCDc[) CDCDCDc[) CDCDCDc[) CDCDCDc[) CDCDCDc[) CDCDCDc[) CDCDCDc[) CDCDCDc[) CD CD CDc[) CD CD CDc[) CDCDCDc[) CDCDCDc[) CDCDCDCJD CDCDCDc[) CDCDCDc[) CDCDCDc[) CD CD CDC[) CDCDCDc[) CD CD CDc[) CDCDCDc[) 501 Answer Sheet SAMPLE READING COMPREHENSION TEST ANSWER SHEET CE:> CD CD CD 2. CE:> CD CD CD 3. CE:> CD CD CD 26. c£)CDCDCD CE:> CD CD CD c£)CDCDCD c£) CD CD CD c£) CD CD CD CD CD CD CD c£) CD CD CD CDCDCDCQ) CDCDCDCQ) c£) CD CD CD c£) CD CD CD c£)CDCDCQ) c£) CD CD CD CE:>CDCDCQ) CE:> CD CD CD c£) CD CD CD c£) CD CD CD CDCDCDCQ) c£) CD CD CD CDCDCDCD c£)CDCDCD Q:) CD CD CD 29. 1. 4. 5. 6. 7. 8. 9. 10. 11. 12. 13. 14. 15. 16. 17. 18. 19. 20. 21. 22. 23. 24. 25. CE:> CD CD CD 27. CE:> CD CD CD 28. CE:> CD CD CD 30. 31. 32. 33. 34. 35. 36. 37. 38. 39. 40. 41. 42. 43. 44. 45. 46. 47. 48. 49. 50. c£) CD CD CD c£) CD CD CD c£)CDCDCD CD CD CD CD CDCDCDCD CDCDCDCQ) c£) CD CD CD CD CD CD CD CDCDCOCQ) CD CD CD CD c£) CD CD CD Cl0CDCDCQ) c£) CD CD CD CDCDCDCQ) c£) CD CD CD CDCDCDCQ) c£) CD CD CD CE:> CD CDCQ) c£)CDCDCD CD CD CD CD c£)CDCDCD CD CD CD CD 503 Answer Sheet 505 PRACTICE TEST 1 ANSWER SHEET Name: 1. 2. 3. 4. 5. 6. 7. S. 9. 10. 11. 12. 13. 14. 15. 16. 17. IS. 19. 20. 21. 22. 23. 24. 25. 26. 27. 2S. 29. 30. 31. 32. 33. 34. 35. 36. 37. 3S. 39. 40. 41. 42. 43. 44. 45. 46. 47. 4S. 49. 50. CD c[) CD CD CD c[) CD CD CD CD CD CD CD CD CD CD CDc[)CDCD CDc[)CDCD CDc[)CDCD CDc[)CDCD CD c[) CD CD CDCDCDc[) CDCDCDCD CD CD CDc[) CDCDCDCD CDCDCDCD CDCDCDc[) CDCDCDCD c£)CDCDCD CDCDCDc[) CDCDCDc[) CDCDCDc[) CD CD CD CD CD CD CD CD CD CD CDc[) CD CD CDc[) CDCDCDCD CDCDCDCQ:) CDCDCDCQ:) CDCDCDCQ:) CDCDCDc[) CDCDCDCD CDCDCDCD CDCDCDc[) CDCDCDCD CD CD CDc[) CDCDCDc[) CDCDCDc[) CDCDCDc[) CDCDCDc[) CDCDCDc[) CDCDCDCD CDCDCDc[) CDCDCDCD CD CD CDc[) CDCDCDc[) CDCDCDCD CDCDCDCQ) CDCDCDc[) c£)CDCDc[) CDCDCDc[) CDCDCDCQ:) 1. 2. 3. 4. 5. 6. 7. S. 9. 10. 11. 12. 13. 14. 15. 16. 17. IS. 19. 20. 21. 22. 23. 24. 25. 26. 27. 2S. 29. 30. 31. 32. 33. 34. 35. 36. 37. 3S. 39. 40. CDCDCDc[) CD CD CD CQ:) CD CD CD CQ:) CD CD CD CD CD CD CDC[) CD CD CD CD CDCDCDCD CD CD CDC[) CDCDCDc[) CDCDCDc[) CDCDCDc[) CDCDCDCQ) CDCDCDCQ) CDCDCDCQ) CDCDCDCQ) CDCDCDc[) CD CD CDC[) CDCDCDCQ:) CDCDCDCQ) CD CD CDC[) CD CD CDC[) CDCDCDCQ:) CDCDCDc[) CDCDCDc[) CDCDCDc[) CD CD CDC[) CDCDCDc[) CDCDCDc[) CDCDCDc[) CDCDCDc[) CDCDCDc[) CDCDCDc[) CDCDCDCQ:) CDCDCDc[) CDCDCDc[) CD CD CDC[) CDCDCDc[) CDCDCDc[) CDCDCDc[) CD CD CDC[) 1. 2. 3. 4. 5. 6. 7. S. 9. 10. 11. 12. 13. 14. 15. 16. 17. IS. 19. 20. 21. 22. 23. 24. 25. 26. 27. 2S. 29. 30. 31. 32. 33. 34. 35. 36. 37. 3S. 39. 40. 41. 42. 43. 44. 45. 46. 47. 4S. 49. 50. CDCDCDCQ) CDCDCDCD CDCDCDc[) CD CD CD CD CD CD CD CD CDCDCDCD CDCDCDCD CDCDCDc[) CDCDCDCD CD CD CDc[) CDCDCDc[) CDCDCDc[) CD CD CDC[) CD CD CDC[) CDCDCDc[) CDCDCDc[) CDCDCDCQ) CDCDCDCD CD CD CDC[) CDCDCDc[) CDCDCDc[) CDCDCDCD CDCDCDCQ) CDCDCDCQ) CDCDCDCQ:) CDCDCDCB CDCDCDCQ) CDCDCDCQ) CDCDCDc[) CDCDCDCQ) CDCDCDc[) CDCDCDc[) CDCDCDc[) CDCDCDc[) CDCDCDCD CD CD CDC[) CDCDCDc[) CD CD CDC[) CDCDCDCQ:) CDCDCDc[) CDCDCDc[) CDCDCDc[) CDCDCDCD CDCDCDCD CDCDCDCD CDCDCDCD CDCDCDCQ) CDCDCDCQ) CDCDCDCQ) CDCDCDCD Answer Sheet PRACTICE TEST 2 ANSWER SHEET Name: 1. 2. 3. 4. 5. 6. 7. 8. 9. 10. 11. 12. 13. 14. 15. 16. 17. 18. 19. 20. 21. 22. 23. 24. 25. 26. 27. 28. 29. 30. 31. 32. 33. 34. 35. 36. 37. 38. 39. 40. 41. 42. 43. 44. 45. 46. 47. 48. 49. 50. CDCDCDCD CDCDCDCD CDc[)CDCD CDc[)CDCD CDCDCDCD CD CD CD CQ:) CD CD CD CQ:) CD c[) CD CQ:) CD CD CD CQ:) CDc[)CDCQ:) CDc[)CDCQ:) CDc[)CDCQ:) CDc[)CDCD CDc[)CDCD CDCDCDCD CDc[)CDCD CDc[)CDCD CDc[)CDCD CDc[)CDCD CDCDCDCD CDc[)CDCQ:) CD CD CD CD CDc[)CDCQ:) CDc[)CDCQ:) CDc[)CDCQ:) CDc[)CDCQ) CDCDCDCQ:) CDCDCDCD CDCDCDCD CDc[)CDCD CDc[)CDCQ) CDc[)CDCQ) CDc[)CDCQ:) CDc[)CDCD CDCDCDCD CDCDCDCQ:) CDCDCDCQ:) CDCDCDCQ:) CDCDCDCQ:) CDCDCDCQ:) CDCDCDCQ:) CDc[)CDCQ) CDCDCDCB CD CD CD CD CDc[)CDCD CDCDCDCD CDc[)CDCD CDCDCDCD CDCDCDCD CDc[)CDCQ) 1. 2. 3. 4. 5. 6. 7. 8. 9. 10. 11. 12. 13. 14. 15. 16. 17. 18. 19. 20. 21. 22. 23. 24. 25. 26. 27. 28. 29. 30. 31. 32. 33. 34. 35. 36. 37. 38. 39. 40. CDc[)CDCQ) CDCDCDCQ) CDc[)CDCQ) CDCDCDCD CD c[) CD CQ:) CD CD CDCQ) CDCDCDCQ) CDCDCDCQ) CDc[)CDCQ) CDCDCDCQ:) CDc[)CDCQ) CDc[)CDCQ) CDCDCDCB CDCDCDCQ:) CDCDCDCB CDCDCDCQ) CDc[)CDCQ) CDCDCDCQ) CDc[)CDCD CDCDCDCQ:) CDCDCDCQ:) CDCDCDCD CDCDCDCQ) CDc[)CDCQ) CDc[)CDCQ) CDc[)CDCQ) CDCDCDCQ) CDc[)CDCD CDCDCDCQ) CDc[)CDCQ) CDCDCDCQ) CDc[)CDCQ:) CDCDCDCQ) CDCDCDCQ) CDCDCDCB CDc[)CDCQ:) CDCDCDCB CDCDCDCQ:) CDCDCDCQ:) CDCDCDCQ:) 1. 2. 3. 4. 5. 6. 7. 8. 9. 10. 11. 12. 13. 14. 15. 16. 17. 18. 19. 20. 21. 22. 23. 24. 25. 26. 27. 28. 29. 30. 31. 32. 33. 34. 35. 36. 37. 38. 39. 40. 41. 42. 43. 44. 45. 46. 47. 48. 49. 50. CDc[)CDCD CDCDCDCD CDc[)CDCD CDc[)CDCD CDc[)CDCD CD c[) CD CD CDc[)CDCD CD CD CD CD CDCDCDCD CD CD CD CD CDc[)CDCD CDCDCDCQ) CDCDCDCD CDCDCDCD CDCDCDCD CDCDCDCD ClD c[) CDCQ:) CDCDCDCQ:) CDCDCDCQ:) CDCDCDCD CDCDCDCD ClDCDCDCD CDCDCDCQ) CDc[)CDCQ) CDCDCDCQ) CD CD CDCQ) ClDCDCDCQ) CDCDCDCD CD CD CD CD CD CD CD CD CDCDCDCQ) ClDc[)CDCQ) ClDCDCDCQ) CDCDCDCQ) CDCDCDCD ClD CD CD CD CDCDCDCD CDCDCDCD ClDCDCDCQ:) CDCDCDCD CDCDCDCD CDc[)CDCQ) CD CD CD CD CDCDCDCD ClDCDCDCD CDCDCDCD ClDCDCDCD CDCDCDCD ClDc[)CDCD ClDCDCDCD 507 Answer Sheet 509 PRACTICE TEST 3 ANSWER SHEET Name: ~:~I.~;;~t·:·:~1ec:ti6tll 1. 2. 3. 4. 5. 6. 7. 8. 9. 10. 11. 12. 13. 14. 15. 16. 17. 18. 19. 20. 21. 22. 23. 24. 25. 26. 27. 28. 29. 30. 31. 32. 33. 34. 35. 36. 37. 38. 39. 40. 41. 42. 43. 44. 45. 46. 47. 48. 49. 50. CDCDCDeJD CDCDCDeJD CDCDCDeJD CDCDCDeJD CD CD CI) C[) CDCDCDc[) CDCDCDc[) CD CD CDC[) c£)CDCDc[) c£)CDCDc[) c£)c[)CDc[) c£)C[)CDc[) c£)CDCDeJD c£)CDCI)C[) c£)CDCDc[) c£)CDCDeJD c£)CDCDeJD c£)CDCDeJD c£)CDCI)C[) c£)CDCDc[) c£)CDCDc[) c£)CDCDc[) c£)CDCDc[) CD CD CDc[) CDCDCDc[) CD CD CDc[) CD CD CDc[) CD CD CD C[) CDCDCDCQ) CDCDCI)C[) CDCDCDCQ) CDCDCDCQ) CDCDCDCQ) CDCDCDCQ) CDCDCDc[) CD CD CDC[) c£)CDCDc[) c£)CDCDc[) c£)CDCDCQ) c£)CDCDc[) CDCDCDCQ) CDCDCDc[) CDCDCDc[) CDCDCDeJD CDCDCDeJD CDCDCDeJD c£)CDCDeJD CD CD CDeJD CDCDCDeJD CDCDCDCQ) ~.3 1. 2. 3. 4. 5. 6. 7. 8. 9. 10. 11. 12. 13. 14. 15. 16. 17. 18. 19. 20. 21. 22. 23. 24. 25. 26. 27. 28. 29. 30. 31. 32. 33. 34. 35. 36. 37. 38. 39. 40. c£)CDCDeJD CDCDCDc[) CDCDCDc[) CDCDCDCQ) C£) CD CD CQ) CDCDCDeJD CDCDCDeJD CDCDCDeJD CDCDCDc[) CDCDCDc[) CDCDCDc[) c£)CDCDc[) c£)CDCDc[) c£)CDCDc[) c£)CDCDc[) c£)CDCDc[) CD CD CDC[) c£)CDCDc[) CDCDCDc[) c£)CDCDc[) CDCDCDc[) CD CD CDC[) CDCDCDeJD CD CD CD C[) CDCDCDeJD CDCDCDeJD CDCDCDeJD CD CD CDC[) CD CD CDC[) CDCDCDc[) c£)CDCDc[) c£)CDCDc[) CDCDCDc[) c£)CDCDc[) CDCDCDc[) CD CD CDC[) CDCDCDCQ) CD CD CDC[) CDCDCDeJD CDCDCDCQ) 1. CDCDCDCQ) 3. 4. 5. 6. 7. CDCDCDCQ) c£)CDCDCQ) CDCDCDCQ) CDCDCDCQ) CDCDCDeJD CD CD CDC[) CDCDCDeJD CD CD CDC[) CDCDCDeJD CD CD CDc[) CDCDCI)C[) CDCDCDeJD CDCDCI)C[) CDCDCDeJD CDCDCDeJD CD CD CDc[) CD CD CDc[) CDCDCDeJD CDCDCDc[) CDCDCDc[) CDCDCDeJD CD CD CDC[) CDCDCDeJD CD CD CDC[) CDCDCDeJD CDCDCDCD CDCDCDeJD CDCDCDCD CDCDCDCD CDCDCDeJD CDCDCDeJD CDCDCDeJD CDCDCDc[) CDCDCDc[) CDCDCDc[) CDCDCDeJD CDCDCDCQ) CDCDCDc[) CDCDCDeJD CD CD CDC[) CDCDCDCD CD CD CD CD CDCDCDeJD CDCDCDeJD CDCDCDeJD CD CD CDc[) CDCDCDc[) CDCDCDeJD 2. C£) CD CD CQ) 8. 9. 10. 11. 12. 13. 14. 15. 16. 17. 18. 19. 20. 21. 22. 23. 24. 25. 26. 27. 28. 29. 30. 31. 32. 33. 34. 35. 36. 37. 38. 39. 40. 41. 42. 43. 44. 45. 46. 47. 48. 49. 50. - - - - - - - - ISBN-13: 978-1-111-22059-4 ISBN-10: 1-111-22059-X 90000 ~.. 1­ HEINLE (ENGAGE LearningHeinle, a part of (engage Learning, is a leading provider of materials for English language teaching and learning throughout the world. Visit elt.heinle.com 9 781111 220594 -= Answer Key The Complete Guide To The TOEFL® Test PBT Edition Answer Key and Audio Scripts Bruce Rogers TOEFL® is a registered trademark of Educational Testing Service (ETS). This publication is not endorsed or approved by ETS. Copyright © 2011 Heinle, a Part of Cengage Learning, Inc Answer Key CONTENTS ANSWER KEY SECTION 1: GUIDE TO LISTENING COMPREHENSION Sample Listening Comprehension Test Part A: Dialogs Part B: About Extended Conversations Part C: Mini-Talks Mini-Lessons for Section 1: Idiomatic Expressions 3 3 5 6 6 SECTION 2: GUIDE TO STRUCTURE AND WRITTEN EXPRESSION Sample Structure and Written Expression Test Part A: About Structure Part B: Written Expression Mini-Lessons for Section 2: Preposition Use 9 10 12 19 SECTION 3: GUIDE TO READING COMPREHENSION Sample Reading Comprehension Test Reading Comprehension Exercises and Mini-Test Mini-Lessons for Section 3: Vocabulary Building 20 21 24 THREE COMPLETE PRACTICE TESTS Practice Test 1 Practice Test 2 Practice Test 3 27 30 33 AUDIO SCRIPTS SECTION 1: GUIDE TO LISTENING COMPREHENSION Sample Listening Comprehension Test Part A: Dialogs Part B: Extended Conversations Part C: Mini-Talks 37 41 62 69 THREE COMPLETE PRACTICE TESTS Practice Test 1 Practice Test 2 Practice Test 3 76 80 84 Answer Key SECTION 1: LISTENING COMPREHENSION SAMPLE LISTENING COMPREHENSION TEST 1.C 2.B 3.B 4.A 5.A 6.C 7.B 8.D 9.B 10.C 11.C 12.D 13.A 14.B 15.A 16.D 17.C 18.A 19.D 20.B 21.A 22.D 23.A 24.D 25.D 26.A 27.C 28.B 29.C 30.B 31.A 32.C 33.C 34.A 35.D 36.B 37.D 38.D 39.D 40.A 41.D 42.C 43.A 44.B 45.B 46.A 47.A 48.A 49.C 50.D PART A: DIALOGS Exercise 1 1.b 2.f 3.a Exercise 2.1 1.B 2.B 3.A Set A 4.e 5.c Set B 9.c 10.b 6.d 7.a 8.f 4.A 5.A 6.A 7.B 8.B 9.B Set C 14.f 15.d 11.e 12.a 13.c 10.A 11.B 12.A 13.B 14.B 15.B Exercise 2.2 1. B center later 2. C appointment appointed 3. C plants cattle 4. A drain train 5. A sister missed her 6. C copy cough drops 7. B hear pain 8. C food boots 9. C weakened awakened 10. B texts collect 11. A van fine 12. B list police Exercise 3.1 1.B 2.B 3.B 7.B 8.B 9.B Exercise 3.2 1.A 2.B Exercise 3.3 1.A 2.B 3.B 4.A 5.B 6.A 3.A 4.B 5.A 6.A 4.A 5.B 6.A 10.A 11.B 12.B 7.A 8.B 7.A 8.A 9.B 9.B 10.B 10.A 11.B 12.A Answer Key Exercise 4.1 1.A was in trouble 2.A met … unexpectedly 3.B became friends 4.A simple 5.B 6.A 7.B 8.A immediately nervous a little sick looks like 9.A 10.B 11.B 12.B permanently Help close to didn’t like Exercise 4.2 1.A 2.B 3.A 4.A Set A 5.A 6.B 7.B 8.B Set B 13.B 14.B 15.A 16.B 9.A 10.B 11.A 12.B Set C 21.B 22.B 23.A 24.A 17.B 18.A 19.B 20.A Exercise 4.3 1.C 2.B 3.A 4.B 5.A 6.C 7.C 8.A 9.C 10.C 11.A 12.B 13.C 14.B 15.B 16.A 17.B 18.C 19.A 20.C Exercise 5 1.C 2.A 3.C 4.B 5.A 6.C 7.C 8.B 9.B 10.A 11.B 12.B 13.A 14.C 15.B 16.B 17.A 18.A 19.B 20.C Exercise 6.1 1.A 2.A 3.B 4.A 5.B 6.B 7.A 8.B 9.A 10.B 11.A 12.A Exercise 6.2 1.A 2.C 3.C 4.C 5.A 6.B 7.A 8.B 9.B 10.A 11.C 12.B Exercise 7.1 1.A 2.A 3.B 4.B 5.B 6.A 7.A 8.B 9.B 10.A 11.A 12.B 13.A 14.B 15.B 16.B Exercise 7.2 1.C 2.C 3.C 4.A 5.A 6.B 7.C 8.A 9.B 10.C 11.A 12.B 13.B 14.C 15.C 16.B Exercise 8 1.C 2.B 3.B 4.A 5.C 6.A 7.C 8.C 9.B 10.A 11.A 12.C 13.C 14.B 15.B 16.A 17.A 18.B 19.B 20.A Answer Key Exercise 9 1.A 2.A 3.C 4.A 5.B 6.B 7.C 8.C 9.B 10.B 11.C 12.B 13.A 14.C 15.A Exercise 10 1.B 2.B 3.B 4.A 5.A 6.B 7.B 8.A 9.A 10.A 11.B 12.A 13.B 14.B 15.B 13.A 14.D 15.A 16.C 17.C 18.A 19.B 20.D 21.C 22.B 23.C 24.A 25.C 26.A 27.D 28.B 29.D 30.D Mini-Test 1: Dialogs 1.D 7.C 2.A 8.D 3.A 9.B 4.C 10.D 5.A 11.C 6.B 12.B PART B: ABOUT EXTENDED CONVERSATIONS Exercise 11 Conversation 1 1.b 2.a 3.d 4.c Conversation 2 5.c 6.a 7.b 8.d B A Exercise 12 1.B 2.C 3.A 4.A Exercise 13.1 1.C 2.B 3.C Exercise 13.2 1.B 2.C 3.A Conversation 3 9.d 10.e 11.c 12.a 13.b D 5.C 6.A 7.B 8.C 4.C 5.A 6.A 9.A 10.C 11.C 12.B 7.A 8.B 9.A 4.C 5.B 6.A Mini-Test 2: Extended Conversations 1.A 3.C 2.C 4.D 10.A 11.C 12.C 13.C 14.A 15.C 16.B 13.C 14.C 15.B 16.C 17.B 7.B 8.B 9.C 10.C 11.B 12.B 5.D 6.A 7.B 8.C Answer Key PART C: MINI-TALKS Exercise 14 Talk 1 1.b 2.c 3.a 4.d 5.e Exercise 15 1.B 2.A 3.C 4.B Talk 2 6.d 7.c 8.b 5.B 6.A 7.C 8.A Exercise 16.1 Talk A 1.B 2.A 3.C 4.B 5.C 6.B Exercise 16.2 1.B 2.C 3.B 4.A 5.B Mini-Test 3: Mini-Talks 1.D 2.C 3.B 9.B 10.C 11.A 12.C Talk 3 9.d 10.b 11.e 12.a 13.C 14.A 15.B 16.B 17.A 18.A 19.A 20.C Talk B Talk C 7.B 8.C 9.B 10.B 11.C 12.A 13.A 14.C 15.B 16.A 17.B 18.A 19.C 20.C 21.B 6.A 7.C 8.A 9.C 10.B 11.C 12.C 4.D 5.D 6.C 7.A 8.C 9.C 10.A 11.A 12.C MINI-LESSONS FOR SECTION 1: IDIOMATIC EXPRESSIONS Mini-Lesson 1.1 1. about to 2. As a matter of fact 3. bank on 4. broke in on 5. better off 6. As a rule 7. broke down 8. all of a sudden 9. bound to 10. brought up 11. by and large 12. add up 13. be my guest 14. by heart 15. bring ...up 16. brought about 17. by no means 18. break the ice 19. brush up on 20. bit off more than ...could chew 21. at the drop of a hat 22. Beats me 23. a breeze 24. break up 25. at ease Answer Key Mini-Lesson 1.2 1. called off 2. calm down 3. came across 4. count on 5. come up with 6. checked into check... out 7. checked...out from 8. call it a day 9. chip in 10. cut off 11. clear up 12. cared for 13. cost an arm and a leg 14. care for 15. Cheer up 16. caught up with 17. cut out for 18. catch on 19. come around to 20. calls on 21. clear up Mini-Lesson 1.3 1. figure out 2. drop off 3. dreamed up 4. feel like 5. fallen behind 6. few and far between 7. do ... over 8. fed up with 9. fell through 10. a far cry from 11. drop out of 12. died down 13. drop in on 14. day in and day out 15. eyes ...bigger than ...stomach 16. feel free 17. drop ... a line Mini-Lesson 1.4 1. find out 2. fill out 3. fill in 4. gave ... the cold shoulder 5. for good for the time being 6. get rid of 7. give away 8. a fish out of water 9. get a kick out of 10. got on 11. get off the ground 12. get along with 13. fixed ...up 14. gotten in touch with 15. get under way 16. gets in...blood 17. get off 18. filled in for Mini-Lesson 1.5 1. grew up 2. handed out 3. handed in 4. heard of 5. hold on 6. go on with 7. Hold still 8. go easy on 9. go... with 10. getting the hang of 11. gave...hand 12. goes without saying 13. held up 14. heard from 15. give... a hand with 16. hold on to 17. hard to come by 18. hit the road 19. have a word with 20. went overboard Mini-Lesson 1.6 1. keep an eye on 2. keeping up with 3. lay off 4. looking forward to 5. jump to conclusions 6. keep an eye out for 7. in the long run 8. left out 9. in no time 10. kill ... time 11. let up 12. in the same boat 13. keep on 14. leave... alone 15. looked... for 16. looks after 17. in the dark 18. looking into 19. iron out 20. in hot water 21. learned the ropes 22. in favor of 23. know like the back of...hand 24. keep track of 25. in store Mini-Lesson 1.7 1. looked over 2. on second thought 3. make...up 4. out of order 5. looks up to 6. on hand 7. make sense of 8. make a point of over and over 9. on the tip of … tongue 10. out of the question 11. out of 12. music to ... ears 13. on end 14. odds and ends 15. on the go 16. make up...mind 17. look out for 18. on needles and pins 19. look...up 20. make way for 21. next to nothing 22. on the whole 23. No harm done 24. out of...mind 25. mean to 26. an old hand at 27. mixed up 28. on...own 29. Not at all 30. over...head Answer Key Mini-Lesson 1.8 1. put up with 2. picked up the tab for 3. point out 4. play it by ear 5. part with 6. picked...up 7. put. ..on 8. put together 9. a pretty penny 10. push.. .luck 11. pick up 12. picked up put. ..away 13. put off 14. picked out 15. pass ...up 16. Pay attention 17. pat. ..on the back 18. passed...with flying colors 19. put. .. aside 20. pulling.. .leg Mini-Lesson 1.9 1. saw … off 2. see to 3. ran out of 4. right away 5. shut down 6. sign up for 7. So far so good 8. ring a bell with 9. spell. .. out for 10. run of the mill 11. singing another tune 12. rough it 13. show up 14. run for office 15. sleep on 16. showed... around 17. Save... breath 18. rule ... out 19. ran into 20. slowly but surely 21. short for 22. snowed under 23. running a temperature 24. see eye to eye with...on Mini-Lesson 1.10 1. stamp out 2. stay up 3. stay out 4. take after 5. taking apart 6. take a break 7. stand for 8. stuck with 9. takes a lot of nerve 10. Stick with 11. spick and span 12. stood out 13. Take it easy 14. stand for 15. straighten up stop by 16. take advantage of 17. stock up on 18. a stone's throw from 19. took a lot out of 20. stack up against Mini-Lesson 1.11 1. try on 2. talked .. .into 3. tore up 4. tell ... apart 5. take up 6. took part in 7. taking off 8. tear... away from 9. throw the book at 10. Take off 11. tried out 12. talked ...out of 13. talk... over 14. throw away 15. think...over 16. took a short cut 17. try out for take the plunge 18. take ... time out from Mini-Lesson 1.12 1. turns into 2. Turn off 3. turn down 4. turn up 5. turn on 6. worn out 7. warm up 8. turned out 9. turn in 10. what the doctor ordered 11. without a hitch 12. work out 13. watch out for 14. Turn around 15. worked...out 16. wait on 17. turned ...down 18. warm up 19. turned out 20. under the weather 21. walking on air Answer Key SECTION 2: GUIDE TO STRUCTURE AND WRITTEN EXPRESSION SAMPLE STRUCTURE AND WRITTEN EXPRESSION TEST  1.C 2.A 3.C  16.B 17.A 18.C 19.B 20.C Structure 4.D 5.B 6.B Written Expression 21.C 22.B 23.C 24.D 25.C 7.D 8.A 9.C 10.C 11.A 12.B 13.D 14.C 15.C 26.A 27.D 28.C 29.D 30.A 31.D 32.A 33.A 34.C 35.B 36.B 37.C 38.C 39.A 40.B Explanation of Written Expression Items 16. The plural verb are indicates that a plural subject, thousands, must be used 17. The preposition in should replace the preposition since. (The preposition since is used only in sentences in which the verb is in the present perfect tense, not the simple past tense.) 18. The noun variety should be used in place of the adjective various. 19. The word alike is only used in the pattern A and B are alike. The correct pattern in this sentence is A, like B, .... 20. The subject of the sentence is use; it is an unnecessary repetition of the subject. 21. A plural pronoun (their) should be used to agree with the plural noun cowboys. 22. The relative pronoun who can only refer to a person, not to a thing. The relative pronoun that or which should be used instead. 23. An adverb (potentially) not an adjective (potential) is needed. 24. The subject of the clause (one species) is singular, so the singular verb is must be used. 25. Almost is the wrong word choice. The best word choice is most (or almost all). 26. Despite is used only before noun phrases. An adverb-clause marker (although) must be used with a clause. 27. A noun (injury), not a verb (injure), is required. 28. The preposition must precede the relative pronoun: in which. 29. After the verb permit, an infinitive (to exist) is used. 30. The correct pattern is between A and B. 31. In order to be parallel with the other words in the series (logic and probability), the name of the field (engineering) must be used. 32. The verb in this sentence should be passive; therefore, the past participle known (not the -ing form knowing) must be used. 33. Before a word beginning with a vowel sound (honor), the article an must be used. 34. The superlative form of a one-syllable adjective (old) is formed with the suffix -est: oldest. 35. The noun belief should be used in place of the verb believe. 36. The plural pronoun those should be used to refer to the plural noun phrase public buildings. 37. The correct pattern is neither... nor. 38. A past participle (held) is needed in place of the -ing form. 39. The correct pattern is so + adjective + that clause (too is used in the pattern too + adjective + infinitive). 40. A plural noun (teeth) is required. Answer Key PART A: ABOUT STRUCTURE Note: Items marked with an asterisk (*) do not focus on the structures that are presented in that lesson. Exercise 17 1.B 2.D 3.A 4.B 5.A 6.A 7.D 8.C 9.A* 10.B 11.A 12.B 13.A 14.D* 15.A 16.D 17.B 18.D 19.C 20.C 21.A 22.D 23.C 24.C 25.D Exercise 18 1.D 2.C* 3.B 4.A 5.A 6.B 7.C 8.A* 9.A 10.C 11.B 12.D 13.C 14.C 15.D 16.A Exercise 19 1.A 2.B 3.B 4.D 5.A 6.C 7.B* 8.D* 9.D 10.C 11.B 12.A Exercise 20 1.B 2.C 3.A 4.A* 5.B 6.D 7.C 8.C 9.A 10.D 11.B 12.D* Exercise 21 1.C 2.B 3.D 4.D 5.B 6.C* 7.C 8.C 9.D 10.A 11.C 12.B* 13.D 14.D 15.C* 16.A 17.B 18.C Exercise 22 1.D 2.B 3.A 4.D 5.B* 6.C 7.A 8.A 9.D 10.C 11.D 12.B* Exercise 23 1.C 2.B* 3.A 4.B 5.A 6.A 7.D 8.B 9.C* 10.A 11.D* 12.D Mini-Test 4: Structure 1.D 4.A 2.C 5.B 3.D 6.D 7.C 8.D 9.B 10.A 11.A 12.D 13.C 14.C 15.A Exercise 24 1.A 2.B 3.A 4.C 9.C 10.B 11.A 12.D 13.B 14.A 15.D 16.B 17.D 18.C 19.C 20.C 5.B 6.D 7.A 8.B Answer Key Exercise 25 1.B 2.D 3.C 4.A 5.C 6.A 7.B 8.B 9.C* 10.C 11.A 12.D Exercise 26 1.D 2.C 3.B* 4.C 5.A 6.B 7.D 8.A 9.B 10.B 11.C 12.C 13.A* 14.B 15.A Exercise 27 1.C 2.C 3.A 4.A 5.B 6.A 7.C 8.D 9.C 10.C 11.A 12.C* 13.C 14.B 15.D Exercise 28 1.A 2.D 3.B 4.D 5.C 6.C 7.A 8.B 9.A 10.C 11.B 12.D Exercise 29 1.A 2.B 3.B 4.A 5.B 6.C 7.C 8.D 9.C 10.B 11.D 12.A Exercise 30 1.B 2.D 3.B 4.C* 5.A 6.C 7.A 8.A 9.D* 10.C 11.B 12.C Exercise 31 1.D 2.A 3.B 4.A 5.A 6.C 7.B 8.C 9.D 10.D 11.A 12.B 13.A 14.B Mini-Test 5: Structure 1.C 4.D 2.A 5.C 3.D 6.B 7.C 8.B 9.C 10.A 11.A 12.B 13.A 14.A 15.C Answer Key PART B: WRITTEN EXPRESSION Note: Items marked with an asterisk (*) do not focus on the structures that are presented in that lesson. Exercise 32.1 1. 2. invent 3. 4. 5. deepen 6. decide 7. 8. 9. 10. emphasize 11. inconvenience 12. glorify (or glory) 13. 14. equalize (or equal) 15. generalize 16. simplify 17. familiarize 18. purify 19. 20. Exercise 32.2 1. 2. surgery 3. poetry 4. electricity 5. 6. finance 7. photography 8. 9. athletics 10. 11. philosophy 12. crime 13. politics 14. law 15. Exercise 32.3 1. greatly 2. annually 3. Regular 4. simple 5. beautiful difference invention competition (or competitiveness) fertility (or fertilization, fertilizer) depth different differently inventively competitively competitive fertile beauty (or beautification) prohibition origin ( or originality) emphasis inconvenience deep decisive beautiful decisively beautifully prohibitive original mystery (or mystification) equality generality (or generalization) simplicity (or simplification) familiarity (or familiarization) purity freedom restriction prohibitively originally emphatically inconveniently gloriously mysteriously glorious mysterious equal generally simple familiarly purely freely restrictively free restrictive musician musical surgical Poet electric administrative administrator financier theoretician (or theorist) athlete editor photographic theoretical editorial philosophical criminal politian legal humorous humorist 6. simultaneously 7. generally 8. painstakingly 9. permanently 10. widely 11. close 12. easy 13. incredible 14. automatically 15. formal 16. profound 17. commercially 18. persuasively 19. masterful deeply 20. distinct Answer Key Exercise 32.4 1. fictional (N/Adj) 2. industry (N/Adj) products (V/N) 3. fragrant (N/Adj) 4. mathematical (N/Adj) equal (Adj/N) 5. evidence (N/Adj) illegal (Adj/Adv) 6. severity (Adv/N) 7. development (G/N) 8. transport (N/V) 9. differ (V/Adj) originate (V/N) 10. magician (N/PN) 11. depth (Adj/N) 12. distinction (N/Adv) 13. collection (N/G) 14. present (Adj/N) open (Adj/Adv) 15. choices (V/N) approved (V/N) 16. scholarly (PN/Adj) immigration (PN/N) 17. food (V/N) rainy (N or V/Adj) 18. symbolize (V/N) occupation (N/Adj) 19. relieve (V/N) 20. respire (V/N) chemical (N/Adj) 21. member (PN/N) interpreter (V/PN) 22. strength (Adj/N) directly (Adj/Adv) 23. tropical (N/Adj) ability (Adj/N) 24. lose (N/V) rapidly (N/Adv) 25. ripen (Adj/V) Exercise 32.5 1.B intellectual 2.A destructive 3.C importance 4.C analyzes 5.A dances 16.C life 17.D health 18.D neighborhoods 19.D successful 20.C collection 6.A outer 7.D weight 8.D* purposes 9.D ranching 10.B well 11.B measurement 12.C literature 13.D* exposed to 14.B reaction 15.C sharpness 21.D* and 22.D luck 23.A absence 24.C politicians 25. D harden Exercise 33.1 1. made 2. done 3. made 4. do 5. did 6. make 7. made 8. make 9. do 10. make Exercise 33.2 1. alike 2. like 3. Like 4. as 5. like 6. like 7. as 8. like 9. as 10. alike Exercise 33.3 1. so 2. too 3. So 4. such a 5. too 6. as 7. such a 8. so 9. too 10. such Exercise 33.4 1. another 2. other 3. other 4. another 5. another 6. other 7. other 8. another 9. another 10. other Exercise 33.5 1. Although 2. Because 3. In spite of 4. Because of 5. When 6. Despite 7. Although 8. because 9. because of 10. Although 11. during 12. because Answer Key Exercise 33.6 1. Many 2. little 3. much 4. few Exercise 33.7 1. age 2. twice 3. afterwards 4. Most Exercise 33.8 1.C no longer 2.C as much 3.D nearly 4.C old (or of age) Exercise 34.1 1. is 2. was 3. are Exercise 34.2 1. X shipped 2.X was built 3.C 4.X worn 5.X has been growing (or has grown) 5. number … amount 6. little 7. amounts 8. many 5. before 6. earliest 7. round 8. somewhat 9. tell 10. near 11. doubled 12. live 5.C little 6.B never 7.B many 8.D* symbol 4. was 5. moves 6. are 9. Many 10. little 13. percent 14. old 15. after 16. No 17. Almost 18. tell 19. nearly 20. ever 9.C doing 10.D like 11.D other 12.A number 7. are 8. was 9. makes 13.C* is 14.A earliest 15.A during 16.C a live 10. are 11. is 12. varies 13. are 14. was 15. goes 6.X does 7.X are played 8.X was 9.X came 10.C 11.X contain 12.X ran 13.X were chosen 14.X have used 15.X were produced Exercise 34.3 1.A study 2.D lived 3.D do 4.B requires 5.C have 6.D spun 7.D eat 8.C is suited 9.A took 10.D was 11.B described 12.A shaken 13.D built 14.A* emphasis 15.B dipped Exercise 35.1 1.X grinding 2.X industrial 3.C 4.X warm 5.X stories 6.C 7.C 8.X religion 9.X heat 10.X a critic 11.X is inexpensive 12.X control floods Exercise 35.2 1.B adjust 2.D and on mountain 3.D* other 4.A historical 5.A designer 6.C more specialized 7.D economic 8.B package 9.C loyal 10.D a game 11.C mineral 12.D* have 13.C economics 14.A* front teeth 15.C hearing Answer Key Exercise 36.1 1.X its 2.X them 3.X it 4.C 5.X that (or which) 6.X their 7.X it 8.X who 9.C 10.X those 11.X their 12.X its 13.X which 14.X their 15.X that Exercise 36.2 1.X her 2.X them 3.C 4.X that (or which) 5.X our 6.X themselves 7.X they 8.X whose 9.X its 10.X themselves Exercise 36.3 1.X it 2.X which 3.X that 4.X he 5.C 6.X they 7.X them 8.C 9.X she 10.C 11.X which 12.X it Exercise 36.4 1.C are 2.D their 3.B them 4.B uses 5. A* unusual 6.D themselves 7.A resemble 8.D those 9.D their 10.B its 11.A who 12.D itself 13.A this 14.C themselves 15.A* illustrated Exercise 37 1.D mammals 2.C human 3.B automobile 4.A year 5.B sources 6.B percent 7.A All colleges 8.C thousands 9.C sixty-mile 10.C men 11.D editorial 12.C appliances 13.D feet 14.C* grown 15.A pioneer 16.C* be made 17.B* most 18.D women 19.B trillion 20.D sunlight Mini-Test 6: Written Expression 1.C 6.C 2.A 7.C 3.C 8.A 4.D 9.D 5.C 10.C 11.A 12.C 13.D 14.B 15.A 16.C 17.D 18.C 19.C 20.B 21.D 22.B 23.C 24.D 25.A Explanation 1. 2. 3. 4. 5. 6. 7. 8. 9. 10. 11. 12. The adjective different is required. Before a clause, an adverb-clause marker such as although is required. For parallelism, the noun color must be used. The noun beliefs must be used in place of the verb believes. The past participle played must be used after has (or have) to form the present perfect tense. The plural form designers is required. The verb made (meaning "to manufacture") is needed. The word like is required in this pattern. The correct pronoun is them. (The animals can't carry themselves!) For parallelism, the adjective safe is needed. The adjective pure is required. The word amounts is used to refer to uncountable nouns like food. Answer Key 13. 14. 15. 16. 17. 18. 19. 20. 21. 22. 23. 24. 25. Only the second noun of a compound noun is pluralized: railroad workers. Such a is used with an adjective + noun + that clause. The adjective light is required. The pronoun she is an unnecessary repetition of the subject. To be parallel with the two other adjectives (biological and psychological), the adjective medical must be used. A passive verb phrase is needed: were given. The object form of the pronoun them must be used. For parallelism, a noun phrase is needed: composer of The adjective spicy must be used. The plural pronoun themselves must be used to agree with its referent. A singular verb (is) is required to agree with the subject knowledge. Furniture is an uncountable noun and cannot be pluralized. Soonest is the wrong word choice; the correct word is earliest. Exercise 38.1 1. to control 2. to move 3. producing 4. to have 5. to grow Exercise 38.2 1. known 2. written 3. astonishing 6. to catch 7. bend 8. bringing 9. to communicate 10. miss 4. twisting 5. working 6. filled Exercise 38.3 1.C packaging 2.C to cut 3. B classifying 4.A mixed 5.D* or 7. named 8. appearing 9. deposited 6.D to catch 7.B obtained 8.A Bathing 9.D sparkle 10.B painting Exercise 39.1 1. For against within of 2. in on of 3. of to since into 4. At of of along between 5. of in of on in 6. at on to on 7. of in by with on 8. off of for of Exercise 39.2 1.X According polls 2.X thanks improved 3.C 4.X of 5.X on 6.X regardless the 7.C 8.X of 9.X attached bones 10.X to 11. to snap 12. to rupture 13. to describe 14. to be awarded 15. achieving 10. regarded 11. cracked 12. stimulating 11.D* 40 miles 12.A crushed 13.C for playing 14.D smoking 15.C leading 9. In for from to 10. In at to on 11. In on of of at 12. to of by in of in 13. In of by of in through 14. on of to on 15. in for of in since to to of to 11.X 12.X 13.C 14.X 15.X 16.X 17.X 18.X 19.X 20.C related the expert the by by away the of side the familiar people to on from of to Answer Key Exercise 39.3 1.A which 2.B deals with 3.A Many 4.C on its 5.D* life 6.A from 7.A in 8.C belongs to one 9.D native to 10.B in 11.A For 12.A thousands of eggs 13.A Nowadays 14.D* to reach 15.A Since Exercise 40.1 1. one the water fresh 2. The mineral the most fertilizers 3. The a electrical 4. Humor American the earliest the present 5. The ozone an most the Sun’s 6. the early a Cherokee the a North 7. The Goddard New the the United the seventeenth 8. Popcorn the corn 9. the most research a well-written 10. the American the the twentieth the 11. The nineteenth refracting his 12. The Hawaiian the most the world Exercise 40.2 1.A Most 2.C their 3.D attention 4.A an underwater 5.D the young 6.A the only 7.A a third 8.D their customers 9.B is an imaginary 10.B the most common 11.C her career 12.A the first 13.C a heated 14.A the highest 15.D* and 16.B an accurate 17.B history 18.A At the beginning 19.B* opened 20.D a height Exercise 41 1.X the easiest 2.X stronger and larger 3.C 4.X lower 5.C 6.X heavier 7.C 8.X most destructive 9.X least 10.X worse 11.C 12.X more bitter (or bitterer) 13.X best-known 14.X finest 15.C 16.X larger 17.C 18.X less dense Exercise 42.1 1. X chief source 2.X brightly colored 3.C 4.X long before 5.X at which 6.C 7.X miles longer 8.X natural habitats 9.X they are 10.X dense enough 11.X any other 12.X almost entirely 13.C 14.X formal training 15.X children’s books Answer Key Exercise 42.2 1.C major barrier 2.D large enough 3.D greatly reduced 4.C in which 5.D it is 6.B it possible 7.C each second 8.B* little resemblance 9.A slow-moving 10.A such as Exercise 43 1.X either 2.C 3.X but also 4.X nor 5.X or Mini-test 7: Written Expression 1.C 6.D 2.A 7.C 3.A 8.B 4.D 9.C 5.B 10.A 11.A the sky is 12.B highly original 13.B only one 14.D feet long 15.C is surrounded 16.B workers’ satisfaction 17.D trading center 18.C about how 19.D* loneliness 20.C light is 6.X and 7.X both 8.C 9.X but 10.X and 11.B 12.B 13.C 14.B 15.B 11.C 12.X not only 13.X frogs 14.X neither 15.C 16.A 17.A 18.C 19.D 20.C 21.B 22.B 23.A 24.C 25.B Explanation 1. 2. 3. 4. 5. 6. 7. 8. 9. 10. 11. 12. 13. 14. 15. 16. 17. 18. 19. 20. 21. 22. 23. 24. 25. The wrong preposition is used; the phrase should correctly read at one time. When a noun phrase includes an ordinal number (third), a definite article (the third) must be used. (However, an indefinite article is used in a fraction to mean one: a third of the Earth's surface, for example.) The correct pattern is neither... nor. After a preposition, a gerund form (photographing) is needed. An infinitive (to transport) is needed. The correct superlative form is highest. In a wh- clause that is not a direct question, the correct word order is subject-verb: They are. The conjunction but is used before the word rather to show contrast. The preposition in is used before months such as September. The correct form of the infinitive is to walk. The preposition of is used in the phrase instead of a. The article an must be used before words that begin with a vowel sound such as enormous. The correct word order is almost all. The past participle written is required. The verb result is used with the preposition in. (The noun result is followed by the preposition of) A comparative form (much rarer) must be used. An indefinite article is needed before the noun: a picture. The preposition for is used after the adjective suitable. The correct word order is human psychology. A full infinitive (to teach) is required in place of the simple form. Enough must follow adjectives: safe enough. After the verb allow, the infinitive to approach is needed. The definite article is used before most when it is part of a superlative adjective phrase: The most. The correct pattern is whether... or. An indefinite article is required: a banana. Answer Key MINI-LESSONS FOR SECTION 2: PREPOSITION USE Mini-Lesson 2.1 1. to 2. in (or with) 3. on 4. to 5. of 6. with 7. of 8. from 9. for 10. to11. of 12. to (or for) 13. of 14. of 15. for 16. with 17. to 18. on 19. for 20. to Mini-Lesson 2.2 1. to 2. to 3. of 4. for 5. for 6. to 7. at (or by) 8. of 9. to 10. for 11. to 12. of 13. to 14. with 15. to 16. for 17. to Mini-Lesson 2.3 1. in of in of 2. of for 3. in of 4. on 5. with 6. to 7. of 8. for 9. in 10. of 11. of 12. to 13. to 14. of 15. on of 16. to 17. for 18. of 19. on 20. of Mini-Lesson 2.4 1. to 2. with 3. with 4. on 5. on 6. of 7. on 8. in 9. with 10. on 11. in 12. with 13. with 14. on 15. to 16. on 17. to 18. for 19. in 20. for 21. in 22. to Mini-Lesson 2.5 1. of 2. In of 3. with 4. to 5. to 6. in 7. On of of 8. of 9. to 10. of 11. by of 12. to Mini-Lesson 2.6 1. in 2. in 3. in in 4. in at 5. in at in 6. on in 7. in 8. in 9. on 10. in on on 11. in in 12. at on 13. in on 14. in Mini-Lesson 2.7 1. on in 2. at 3. at in 4. in 5. at 6. on 7. at in 8. On 9. in 10. in 11. in in 12. at 13. in in 14. in 15. In 16. in 17. at 18. in 19.on 20. In on in Mini-Lesson 2.8 1. with 2. by 3. for 4. by 5. without 6. by 7. by 8. by 9. with 10. on of Answer Key SECTION 3: READING COMPREHENSION SAMPLE READING COMPREHENSION TEST 1.B 2.B 3.D 4.A 5.C 6.D 7.B 8.C 9.A 10.A 11.C 12.D 13.D 14.A 15.A 16.D 17.D 18.B 19.A 20.B 21.D 22.C 23.A 24.A 25.C 26.B 27.A 28.D 29.C 30.D 31.D 32.A 33.B 34.A 35.C 36.B 37.D 38.A 39.B 40.B 41.C 42.B 43.A 44.B 45.D 46.C 47.C 48.A 49.D 50.C Explanation 1. 2. 3. 4. 5. 6. 7. 8. 9. 10. 11. 12. 13. 14. 15. 16. 17. 18. 19. 20. 21. 22. 23. 24. The passage discusses the plentiful supply of wood in the colonies and the advantages and disadvantages this involved. Strikingly means "dramatically." Lines 5-6 state, "The first colonists did not, as many people imagine, find an entire continent covered by a climax forest." Abounded means "present in large numbers." Lines 10-11 state that "by the end of the colonial age, the price of wood had risen slightly in eastern cities." Lines 15-16 indicate that, in the colonies, "buildings were made of wood to a degree unknown in Britain." Therefore, many British houses must have been made of materials other than wood. According to lines 17-18, wood was the source of industrial compounds, and charcoal is given as an example. Charcoal, according to line 19, is a component of gunpowder. In the context of the passage, the word conferred means "provided." (However, in other contexts, conferred may mean "consulted.") The phrase follow suit means "do the same." Lines 31--33 state that "the former colonies lagged behind Britain...because their supply of wood led them to cling to charcoal iron." In the context of the passage, cling to means "continue to use." The author begins to discuss the disadvantages brought on by an abundance of wood in the colonies in lines 21-22. The passage deals with the entire Peale family; (A) and (C) are too narrow, and (B) is too general. The passage indicates that the portrait was "so realistic" that Washington mistook the painted figures for real ones. The word settings is closest in meaning to "environments." The author defines the term mastodon in line 15 as "a huge, extinct elephant." The other terms are undefined. There is no information about when the museum was founded. All of the other questions are answered in the second paragraph: Charles Willson Peale found and prepared the animal exhibits; the museum was located in Philadelphia; its most popular exhibit, a mastodon's skeleton, was found on a farm in New York. In the context of the passage, the word unearthed means "dug up," "removed from the ground." As used in this context, rage means the "current style or fashion." Charles Willson Peale painted over a dozen portraits of Washington (line 4); Rembrandt Peale also painted at least one (lines 24-25). Sarah Miriam Peale is the daughter of Charles Willson Peale's brother James Peale (line 28); Titian and Raphaelle are identified as Charles's sons in line 5, Rubens in line 25. The author praises the art and work of Charles Willson Peale and other members of the family; that, together with the absence of any critical comments, makes admiring the best choice. The main theme of this passage is the idea of transforming Mars; choice (A) best summarizes this idea. The word stark means "harsh," "severe." Answer Key 25. The word there refers to Mars. 26. The passage states that "daytime temperatures may reach above freezing," but there is no mention that temperatures ever become dangerously hot. The other characteristics are given in the first paragraph. 27. According to the passage, "The air there is 95% carbon dioxide" (line 5). 28. According to the passage, building up the atmosphere "could create a 'greenhouse effect' that would stop heat from radiating back into space." The author points out in lines 8-10 that it is because heat radiates back into space that Mars is so cold. 29. The word suitable is closest in meaning to "appropriate." 30. According to scientist Christopher McKay, the project could be started "in four or five decades"-forty or fifty years (lines 27-28). 31. Terra-forming refers to the process of "transforming Mars into a more Earth-like planet" (lines 16-17). 32. The phrase more profound means "deeper." 33. The passage indicates that the possibility of transforming Mars comes from an "understanding of how Earth's ecology supports life" (line 31). 34. The word staggering means "astonishing." 35. The first paragraph indicates that age is "another" factor in susceptibility to colds; therefore, it is logical that a previous paragraph must deal with some other factor. 36. Specific facts is closest in meaning to the word "particulars." 37. Lines 3-4 state that the study "revealed particulars that seem to hold true for the general population." 38. Line 4 indicates that "Infants are the most cold-ridden group" and that infant boys have more colds than infant girls (lines 5-6). 39. No matter what age they are, parents of young children show an increase in cold infections; it is reasonable to assume that these parents are infected by their children. 40. The reference is to people in their twenties. 41. This paragraph deals with the influence of economics on incidence of colds. 42. The word cramped means "small and crowded." 43. This is a neutral, objective scientific report. 44. The passage generally discusses an experiment in which plant roots are grown in isolation-in other words, without the tops of the plants. 45. The reference is to the roots of plants. 46. According to the passage (lines 9-10), ATP is a "high energy compound...which drives the biochemical reactions." 47. The word intact means "whole." 48. The phrase comes in handy means "is useful." 49. The fact that roots provide organic nitrogen compounds is useful for "the growth of buds in the early spring when leaves are not yet functioning" (lines 17-18) 50. The passage discusses an experiment involving plant roots and the significance of that experiment. READING COMPREHENSION EXERCISES AND MINI-TEST Exercise 44.1 1. (A) S (B) C (C) X 2. (A) G (B) S (C) C Exercise 44.2 1.A 2.B 3.D 4.A 3. (A) C (B) I (C) S 4. (A) X (B) C (C) I 5.C 6.C 7.B 8.D 5. (A) G (B) C (C) S 6. (A) X (B) I (C) C 7. (A) G (B) C (C) X 8. (A) C (B) X (C) I 9.A 10.D 11.D 12.A 9. (A) X (B) I (C) C 10. (A) X (B) S (C) C 13.C 14.B 15.A 16.D Answer Key Exercise 44.3 1.C 2.A 3.D 4.B 5.A 6.C 7.A 8.A 9.C Exercise 45.1 1. lines 2-3 2. lines 4-5 3. lines 8-10 4. lines 12-13 5. lines 17-18 6. lines 18-19 7. lines 20-21 10.A 11.B 12.A 8. line 1 9. line 3 10. lines 5-6 11. line 8 12. lines 9-10 13. lines 15-16 13.A 14.C 15.D 14. line 1 15. line 2 16. line 2 17. lines 3-4 18. lines 6-7 19. lines 10-11 20. lines 11-12 21. lines 16-17 22. lines 18-19 23. line 19 Exercise 45.2 1.B 2.B 3.A 4.A 5.C 6.A 7.B 8.C 9.B 10.C 11.A 12.B 13.A 14.C 15.B 16.C 17.A 18.B 19.D 20.D 21.A 22.D 23.B 24.B 25.A 26.D 27.D 28.A 29.B 30.C 31.C 32.D Exercise 46.1 1.B 2.C 3.C 4.B 5.B 6.A 7.A 8.A 9.C 10.C 11.A 12.B 13.A 14.C 15.C Exercise 46.2 1.X 2.I 3.X 4.I 5.I 6.X 7.I 8.I 9.X 10.I 11.X 12.X 13.I 14.X 15.I 16.I 17.X 18.X 19.I 20.X Exercise 46.3 1.D 2.D 3.D 4.C 5.B 6.B 7.A 8.C 9.B 10.B 11.A 12.D 13.B 14.C 15.B 16.A 17.C 18.A 19.C 20.D Exercise 47.1 1.B 2.C 3.A 4.C 5.A 6.B 7.B 8.C 9.A 10.A 11.B 12.A 13.B 14.C 15.A Exercise 47.2 1.A 2.C 3.C 4.D 8.B 9.B 10.A 11.A 15.C 16.D 17.B 18.D 22.C 23.B 24.B 25.A 29.A 30.B 31.D 32.B 36.A 37.C 38.B 39.A Answer Key 5.B 6.D 7.C 12.C 13.D 14.A 19.A 20.A 21.D 26.B 27.D 28.A 33.D 34.C 35.A 40.D 41.D 42.C Exercise 48.1 1.B 2.C 3.C 4.A 5.B 6.B 7.A 8.B 9.B 10.C 11.A 12.B 13.D 14.A 15.C 16.A 17.A 18.B 19.A 20.B Exercise 48.2 1.D 2.B 3.D 4.C 5.C 6.D 7.B 8.C 9.C 10.A 11.A 12.A 13.D 14.B 15.B 16.A 17.C 18.D 19.A 20.B 21.C 22.A 23.D 24.A 25.C 26.C Mini-Test 8: Reading 1.C 11.C 2.A 12.C 3.C 13.D 4..B 14.B 5.D 15.A 6.C 16.B 7.B 17.C 8.A 18.D 9.A 19.B 10.C 20.D 21.C 22.B 23.C 24.B 25.A 26.D 27.C 28.C 29.B 30.A 31.B 32.A 33.A 34.A 35.A 36.D 37.B 38.B 39.A 40.B 41.D 42.D 43.B 44.D 45.A 46.C 47.A 48.C 49.B 50.A Explanation 1. 2. 3. 4. 5. 6. 7. 8. 9. 10. 11. 12. 13. 14. The passage generally concerns the advantages of biological agents and the disadvantages of chemical agents. The word marring means "spoiling," "ruining." The word clog is closest in meaning to the word "obstruct." The author defines weeds as "any plants that thrive where they are unwanted" (line 3). No definitions are offered for the other terms. Lines 9-10 say "herbicides... are sometimes necessary." Choice (A) is given in lines 20-22, which says that biological agents "leave crops and other plants untouched," while chemical agents "kill virtually all the plants they come in contact with." Choice (B) is also given; chemical agents "harm workers who apply them" (lines 12-13), while biological agents "are harmless to humans" (line 19). Choice (D) is given in lines 25-28; "biological agents can be administered only once," while chemical agents "must be used several times per growing season." The word hence means "consequently" or "therefore." The word innate means "natural" or "in-born." According to the passage, the living organisms used to kill weeds are "primarily insects and microorganisms." The reference is to plants. In this context, applications means treatments with biological agents. The problem is the need to control weeds; the possible solutions are the use of chemical or biological agents. The author refers to the fact that the plot is "tightly choreographed"; that Bernstein's score is "brilliant," and that Stephen Sondheim revealed "a remarkable talent." All of these positive factors, and the absence of negative ones, add up to a favorable attitude. Lines 2-3 say the play "is set in the early 1950's." Answer Key 15. Since the Jets and Sharks were rival gangs, and were based on the Montagues and Capulets, it is reasonable to assume that the latter groups were also rivals. 16. The word feuding means "hostile," "antagonistic." 17. The word ultimately means "eventually." 18. Lines 11-12 state: "Stephen Sondheim...reveals a remarkable talent for writing lyrics." 19. A score is the written form of a piece of music. 20. Lines 15-16 indicate that, after it first opened, the play ran for 734 performances. 21. The summary sentence (beginning "The plot. ..") runs from lines 7 to 10. 22. There were 8,000 people at the 1900 New York Auto Show, according to line 1. By coincidence, this was the same number of cars as there were in the United States in 1900 (lines 5-6). 23. By happenstance means "by chance," "coincidentally." 24. According to the passage, only around 4,000 cars were assembled in the United States in 1900, and only a quarter of those were gasoline powered (lines 8-10). One quarter of 4,000 is 1,000. 25. Lines 11-12 state that "the show's audience favored electric cars." 26. The word fumes means "gases," "vapors." 27. The word launched means "initiated," "began." 28. According to the passage, "The Duryea Motor Works ...offered an additive designed to mask the smell of the naphtha that it burned." 29. Line 19 indicates that the highest-priced cars sold for $1,500 in 1900 dollars. 30. The word cumbersome means "clumsy, difficult to control" 31. Lines 21-23 indicate that the Gasmobile, Franklin, and Orient steered with tillers; the Duryea probably used a steering wheel 32. Lines 23-25 state that "the black tie (i.e., very formal) audience viewed the display ... as a social outing." 33. According to the passage, auto shows were about to become "extravaganzas." 34. The passage deals with an interpretation of an experiment involving children's art. 35. The passage says that the children drew both "front views" (line 6) and "rear views" (line 7). 36. The reference is to the children. 37. The word markedly means "noticeably." 38. There is nothing in the article particularly useful to commercial artists. 39. The word odd means "unusual" 40. In the context of this passage, scale means "proportion." 41. Lines 10-12 indicate that the head size "is a form of planning, and not an indication of a poor sense of scale." There is no information in the passage to support any of the other choices. 42. Choice (A) is not correct; the author is not critical of 0'Keeffe's style. Choice (B) is too specific. There is no comparison of abstract art and landscape art, so (C) is not correct. Choice (D) is the best statement of the author's purpose. 43. Line 2 states that 0'Keeffe "studied art in Chicago and New York." 44. The expression refers to 1918. 45. The word frequented is closest in meaning to "visited." 46. The word intuitively means "instinctively." 47. Lines 15-16 indicate that "her style changed dramatically...during a visit to New Mexico." The reference to the "Southwestern sun" (line 17) tells you that New Mexico is in the Southwest. 48. The word blanched means "whitened," "bleached." 49. According to the passage, she "most often painted desert landscapes" after a trip to New Mexico in 1929. 50. Lines 24-25 state that she became "the dean of Southwestern painters and one of the best known of American artists." There is no information to support the other choices. MINI-LESSONS FOR PART 3 Mini-Lesson 3.1 1.A 2.C 3.A 4.A 5.B 6.B 7.A 8.B 9.C 10.C 11.A 12.A 13.B 14.B 15.C Answer Key Mini-Lesson 3.2 1.C 3.A 2.C 4.C 5.B 6.B 7.A 8.B 9.C 10.A 11.B 12.C Mini-Lesson 3.3 1.C 3.B 2.A 4.B 5.B 6.C 7.A 8.C 9.A 10.B 11.A 12.C Mini-Lesson 3.4 1.C 2.C 3.A 4.B 5.C 6.B 7.A 8.C 9.B 10.B Mini-Lesson 3.5 1.A 2.A 3.B 4.A 5.B 6.C 7.A 8.B 9.C 10.A 11.C 12.B Mini-Lesson 3.6 1.B 2.B 3.C 4.B 5.A 6.C 7.C 8.C 9.A 10.C 11.C 12.B Mini-Lesson 3.7 1.C 2.B 3.B 4.A 5.A 6.C 7.A 8.B 9.C 10.A 11.C 12.A Mini-Lesson 3.8 1.C 2.C 3.A 4.C 5.C 6.A 7.B 8.C 9.C 10.B 11.A 12.B 13.A 14.C 15.C 16.A Mini-Lesson 3.9 1.A 2.C 3.C 4.A 5.B 6.B 7.A 8.B 9.C 10.A 11.B 12.A Mini-Lesson 3.10 1.B 2.B 3.A 4.B 5.C 6.B 7.B 8.C 9.C 10.A 11.B 12.C 13.B 14.C Mini-Lesson 3.11 1.C 2.B 3.B 4.A 5.B 6.C 7.C 8.C 9.B 10.A 11.A 12.A 13.B 14.C 15.B Mini-Lesson 3.12 1.A 2.C 3.A 4.C 5.C 6.B 7.A 8.C 9.A 10.C 11.A 12.C 13.A 14.C 15.B Mini-Lesson 3.13 1.A 4.B 7.C 10.A Answer Key 2.A 3.C 5.B 6.B 8.C 9.B 11.B 12.A Mini-Lesson 3.14 1.B 2.B 3.C 4.B 5.B 6.B 7.C 8.C 9.B 10.B 11.B 12.A 13.A 14.B 15.C Mini-Lesson 3.15 1.C 2.A 3.C 4.C 5.C 6.A 7.B 8.C 9.B 10.B 11.A 12.A Mini-Lesson 3.16 1.C 2.B 3.A 4.A 5.A 6.C 7.A 8.B 9.C 10.A 11.B 12.A Mini-Lesson 3.17 1.C 2.B 3.C 4.A 5.C 6.A 7.C 8.B 9.A 10.C Answer Key THREE COMPLETE PRACTICE TESTS PRACTICE TEST 1 SECTION 1: LISTENING COMPREHENSION PARTS A, B, AND C 1.C 11.C 2.A 12.C 3.B 13.D 4.A 14.C 5.D 15.A 6.D 16.A 7.B 17.D 8.C 18.A 9.B 19.B 10.D 20.D 21.A 22.B 23.B 24.A 25.D 26.C 27.D 28.C 29.B 30.A 31.C 32.B 33.B 34.A 35.B 36.C 37.B 38.D 39.D 40.B 41.A 42.B 43.D 44.C 45.C 46.A 47.A 48.C 49.D 50.A SECTION 2: STRUCTURE AND WRITTEN EXPRESSION  1.C 2.C 3.C  16.A 17.C 18.B 19.C 20.B Structure 4.A 5.C 6.B Written Expression 21.B 22.D 23.C 24.D 25.A 7.D 8.A 9.A 10.C 11.B 12.A 13.C 14.C 15.D 26.A 27.D 28.B 29.A 30.B 31.C 32.C 33.C 34.D 35.C 36.B 37.B 38.D 39.C 40.B Explanation of Written Expression Items 16. In order to be parallel with the other adjectives in the series (harder and more resistant), the comparative form lighter must be used. 17. The correct superlative form is largest. 18. The adjective form destructive is required in place of the noun form. 19. The correct pattern is both... and. 20. The correct form of the infinitive (to play) is needed. 21. The correct word order is preposition + relative pronoun: across which. 22. Both the noun phrase (these craftsmen) and the pronoun refer to the same person, so the reflexive pronoun themselves should be used. 23. The singular form of the verb, was, should be used to agree with the singular subject, influence. 24. The pronoun subject they is used unnecessarily and should be omitted. 25. The relative word where must be used to describe a place. (When is used to describe a time.) 26. With countable nouns such as mammals, the word many is used. 27. In order to agree with a plural noun (sharks), the possessive adjective their should be plural. 28. The correct word order is adjective + enough: dense enough. 29. The adjective form (Commercial) is required. 30. Such... that is used with an adjective + noun. (So... that is used when an adjective appears alone.) 31. The noun ability is needed in place of the adjective able. 32. The correct verb is make. 33. In order to be parallel with the other items in the series (physics and mathematics), the name of the field (chemistry) must be used. Answer Key 34. The adjective expert cannot be pluralized. 35. The correct word order is almost exactly. 36. The past tense form is required: became. 37. The word lot should not be pluralized. 38. The word ago is used unnecessarily. 39. The preposition for should be used with the adjective responsible. 40. The noun form collection should be used in place of the gerund. SECTION 3: READING COMPREHENSION 1.D 2.A 3.B 4.A 5.D 6.C 7.B 8.C 9.A 10.A 11.D 12.B 13.C 14.B 15.A 16.A 17.A 18.C 19.D 20.D 21.C 22.B 23.C 24.D 25.A 26.B 27.B 28.C 29.D 30.D 31.A 32.D 33.C 34.A 35.B 36.A 37.D 38.A 39.B 40.C 41.C 42.B 43.D 44.A 45.B 46.C 47.D 48.C 49.B 50.A Explanation 1. 2. 3. 4. 5. 6. 7. 8. 9. 10. 11. 12. 13. 14. 15. 16. 17. 18. 19. The primary purpose of this passage is to detail the stages of the Sun's life in the future. The word fueled is closest in meaning to "powered." As it is used in the passage, the word "condition" is closest in meaning to "state." The Sun has existed in its present state for about 4 billion, 600 million years (lines 34). It is expected to become a red giant in about 5 billion years. Therefore, it is about halfway through its life as a yellow dwarf. Line 8 states that "the core of the Sun will shrink and become hotter." The second paragraph describes the process by which the Sun becomes a red giant star. The last sentence of that paragraph states: "Temperatures on the Earth will become too hot for life to exist." Lines 16-17 indicate that the Sun will be a white dwarf "After it shrinks to about the size of the Earth." Lines 4-5 indicate that the Sun today is thousands of times Jarger than the Earth. Therefore, the Sun will be thousands of times smaller than it is today. According to the passage, the Sun is now a yellow dwarf star; it will then expand to a red giant star, shrink to a white dwarf star, and finally cool to a black dwarf. The word eject has the same meaning as "throw off." The reference is to the planet Earth. The tone is scientifically objective. Although the passage describes the end of the Earth, that event is so far in the future that the author's tone is dispassionate. Washington was one of the first persons to realize the importance of canals, and he headed the first company formed to build a canal. This indicates that he was a pioneer in canal construction. Choice (C) is incorrect because the canal was never finished. The word possibility is closest in meaning to "feasibility." According to lines 11-12, the canal linked Albany on the Hudson River with Buffalo on Lake Erie. The word relatively is closest in meaning to "comparatively." The word intermittent is closest in meaning to "on-again-off-again." According to the passage, the Governor of New York "persuaded the state to finance and build the canal" (lines 22-23). The cost had been estimated at $5 million (line 19) but actually cost $2 million more (line 24), a total of $7 million. The word fees is closest in meaning to the word "tolls." Answer Key 20. According to lines 29-31, the canal "allowed New York to supplant Boston, Philadelphia, and other eastern cities as the chief center of both domestic and foreign trade." The other effects are mentioned in the fourth paragraph. 21. Lines 34-36 indicate that the expansion of the Canal would have been warranted "had it not been for the development of the railroads." (This means, "if the railroads had not been developed.") The railroads must have taken so much traffic away from the canal that the expansion was no longer needed. 22. The word warranted most nearly means "justified." 23. The passage begins to discuss the actual construction of the Erie Canal in Line 16. 24. The passage mainly deals with the distress signals of trees. Choices (B) and (C) concern minor details. There is no information about (A). 25. The reference is to "trees" in line 4. 26. The word parched has the same meaning as the word "dehydrated." 27. The word plight means "condition." 28. The trees' signals are in the 50-500 kilohertz range; the unaided human ear can detect no more than 20 kilohertz (lines 9-10). 29. The word fractured is closest in meaning to "broken." 30. The signals are caused when the water columns inside tubes in trees break, "a result of too little water" (line 15). 31. In the context of the passage, pick up means "perceive." 32. Choices (A) and (B) are mentioned in lines 21-22; (C) is mentioned throughout the passage; there is no mention of (D). 33. Lines 18-19 say, "Researchers are now running tests," implying that research is continuing. 34. The passage explains the difference between two concepts, analogy and homology, and gives examples of both. 35. Lines 1-2 state, "The concepts ... are probably easier to exemplify than define." 36. The word rigid is closest in meaning to "inflexible." This means that there are certain principles that cannot be altered. 37. Analogous organs are those that perform the same functions but are not derived from the same embryological structures. The structures given in (D) most likely demonstrate this relationship in that they both provide the same functions-locomotion and support-but are not otherwise related. 38. Homologous organs "are genetically related," according to line 13. 39. In the context of the passage, the term structures refers to different physical parts of animals: wings, limbs, fins, and so on. 40. The word sense is closest in meaning to "meaning." 41. The author begins to discuss homology in the sentence beginning "In contrast … " in line 9. 42. The purpose of the passage is primarily to describe Charlie Chaplin's movie Modern Times. 43. Lines 2-4 state that Chaplin "was motivated to make the film by a reporter" during an interview. 44. The word abruptly is closest in meaning to the word "suddenly." 45. According to lines 15-16, "scenes of factory interiors account for only about one third of the footage." Therefore, about two thirds of the film must have been shot outside the factory. 46. The phrase losing his mind means "going insane" (from the pressure of work). 47. The reference is to the phrase "another popular scene" in line 20. 48. Lines 24-26 state: "This serves to illustrate people's utter helplessness in the face of machines that are meant to serve their basic needs." 49. The word utter is closest in meaning to that of "complete." 50. The film does not offer "a radical social message," and so would not be considered "revolutionary" (A). Line 14 states that "Chaplin prefers to entertain rather than lecture"; thus, it is "entertaining" (B). Lines 1719 mention that people who have seen the film cannot forget certain scenes, and so it is "memorable" (C). According to lines 12-14, the opening scene's "rather bitter note of criticism …is replaced by a gentler note of satire"; therefore, the author would consider the film "satirical" (D). Answer Key PRACTICE TEST 2 SECTION 1: LISTENING COMPREHENSION PARTS A, B AND C 1.B 11.C 2.D 12.B 3.A 13.C 4.A 14.D 5.B 15.C 6.A 16.A 7.D 17.D 8.A 18.C 9.C 19.A 10.B 20.A 21.C 22.A 23.A 24.B 25.C 26.B 27.D 28.B 29.C 30.D 31.A 32.D 33.A 34.C 35.B 36.D 37.A 38.B 39.A 40.B 41.C 42.C 43.B 44.C 45.D 46.D 47.A 48.C 49.D 50.C SECTION 2: STRUCTURE AND WRITTEN EXPRESSION  1.C 2.C 3.A  16.C 17.A 18.C 19.D 20.A Structure 4.B 5.B 6.C Written Expression 21.C 22.C 23.A 24.C 25.D 7.A 8.A 9.B 10.A 11.D 12.C 13.A 14.B 15.A 26.B 27.C 28.D 29.C 30.D 31.B 32.C 33.B 34.B 35.A 36.B 37.D 38.C 39.C 40.A Explanation of Written Expression Items 16. The correct pattern is both... and. 17. The comparative older is needed in place of the superlative oldest. 18. The noun that names a field (journalism) is needed in place of the noun naming a person (journalist). 19. The correct word order is college campuses. 20. The definite article the should not be used before the name of a field such as architecture. 21. To be parallel with the other words in the series (agriculture, finance, and accounting), a noun that names a field should be used (marketing). 22. Before an uncountable noun, the word amounts should be used. 23. The correct expression is so many... that. (Too is used in phrases with infinitives: too many to.... ) 24. The singular verb is must be used to agree with the singular subject sum. 25. The past participle pulled must be used to express a passive idea. 26. Before a series of three elements (insulated, ventilated, and equipped), the conjunction both cannot be used. 27. The plural pronoun them must be used to agree with the plural noun bridges. 28. To be parallel with the other nouns in the series (hardness and brilliance), another noun form (transparency) is needed. 29. The word very cannot be used to modify the word relatively. 30. The word other should be used in place of another before a plural noun (types). 31. This sentence incorrectly compares people and a sport (running and race walkers). For a logical comparison, the word running must be changed to runners. 32. The adjective wild should replace the adverb wildly because the phrase modifies a noun (Magnolia Gardens). The word lovely is used correctly in this sentence because it is an adjective ending in -ly, not an adverb. 33. The plural noun instruments is needed here. 34. The possessive form his should be used in place of the article the. Answer Key 35. The past tense form of the verb (won) is needed. 36. The definite article the is required before the ordinal number first. 37. The preposition in has been omitted: age in which. 38. The noun ease is needed to be parallel with the noun difficulty. 39. The noun should be pluralized: times. 40. The noun discovery is required in place of the verb discover. SECTION 3: READING COMPREHENSION 1.D 2.A 3.B 4.C 5.A 6.A 7.B 8.C 9.A 10.C 11.B 12.A 13.D 14.A 15.B 16.C 17.A 18.B 19.D 20.D 21.D 22.C 23.B 24.A 25.C 26.C 27.A 28.B 29.A 30.B 31.C 32.C 33.D 34.D 35.C 36.D 37.C 38.A 39.D 40.C 41.A 42.B 43.D 44.C 45.B 46.B 47.D 48.A 49.C 50.D Explanation 1. 2. 3. 4. 5. 6. 7. 8. 9. 10. 11. 12. 13. 14. 15. 16. 17. 18. 19. 20. The passage generally deals with the time humans and mammoths co-existed in the New World and the possible role humans played in the extinction of the mammoths. No specific details are offered about (A) or (B), and (C) is too general. The word implements is closest in meaning to "tools." The phrase refers to "humans." Line 13 mentions "the imperial mammoth of the South," meaning the southern section of North America. Lines 14-16 state that "Here, as in the Old World, there is evidence that humans hunted these elephants," implying that humans had also hunted mammoths in Siberia. The word remains can be defined as those parts of an animal's body that can be found after many years. In this case, they are mainly the bones of the mammoths. The meaning of the word "apparently" is closest to that of seemingly. The author argues that choice (A) is unlikely. Choice (B) is not possible because the extinction of the mammoths came at the end of the Ice Age. There is no information about (D). Only (C) is a possible conclusion. The word cunning means "clever." Choice (A) is true; line 26 states that humans were "not very numerous." Choice (B) is true; line 4 states that humans had bows and arrows at the time that they crossed from Siberia, and that crossing took place before the extinction of the mammoths. Choice (D) is also true; line 25 states that humans were "cunning hunters." Only (C) is not true; lines 25-26 say that humans were "still widely scattered." The author provides the most detailed information about the mammoth. The passage chiefly deals with the first decades of jazz, the Dixieland era. As used in this sentence, the word idiom means a style of playing music. According to the first paragraph, the earliest recordings were made by the Original Dixieland Band, who were among those white musicians who "came to Chicago from New Orleans." According to the second paragraph, the first important recording made by black musicians was recorded by King Oliver's Creole Jazz Band in 1923. Lines 8-10 state that King Oliver's Creole Jazz Band "featured some of the foremost jazz musicians of the time, including ...Louis Armstrong." The word steady is closest in meaning to "constant." Lines 16-18 indicate that the beat was provided by the rhythm section, which included the piano. According to lines 21-22, Duke Ellington "provided his musicians with written arrangements." Line 20 states that "improvisation was an indispensable element," indicating that Dixieland was not carefully planned. All of the other answer choices are referred to in the second paragraph. Answer Key 21. Line 25 refers to the Wolverines as an example of a small Chicago jazz band. 22. The author provides the most detailed description of early jazz in the third paragraph. 23. The last sentence of the passage indicates that the next era of jazz would be the Swing era, so it is logical that the next paragraph will continue with a discussion of this period. 24. The word astounding is closest in meaning to "startling." 25. All three streets are in San Francisco (lines 4-5); the residents have the same approximate levels of income. (They are all middle class or working class, according to lines 5-6.) They all have approximately the same ethnic mix as well (lines 6-7). The only difference is the amount of traffic. 26. Lines 10-11 say that Franklin Street "had almost 16,000 cars a day." 27. According to lines 13-16, trash is a secondary effect of heavy traffic. 28. The author's main purpose in this paragraph is to discuss the negative impact heavy traffic has on Franklin Street. 29. The third paragraph deals with how Octavia Street residents interact; they have more friends and acquaintances on their block than do Franklin Street residents, and by implication, than do Gough Street residents as well. 30. The word chatted means "talked" (informally). 31. According to the passage, increased traffic reduces the amount of territory for which residents feel responsible (lines 16-17). All the other statements would be consistent with information given about Gough Street residents in the fourth paragraph. 32. The author first presents a detailed discussion of Franklin Street (HEAVY), the Octavia Street (LIGHT), and finally Gough Street (MEDIUM). 33. The passage concentrates on the books written by Rachel Carson and on her career as a writer. 34. Lines 2-3 state that Carson studied zoology at Johns Hopkins University. 35. Carson was born in 1907 (line 1) and published Under the Sea Wind in 1941 (lines 67), so she must have been around 34 years of age at the time of publication. 36. According to lines 7-8, when Under the Sea Wind was first published "it received excellent reviews, but sales were poor until it was reissued in 1952." 37. There is no mention that Rachel Carson took part in a research expedition.,The other sources are given in lines 12-13. 38. Carson "realized the limitations of her non-technical readers" (lines 14-15), implying that the book was not highly technical. It did have a poetic quality (line 11), and it was fascinating (interesting), according to line 9, and well-researched (lines 12-14). 39. The word reckless is closest in meaning to "irresponsible." 40. Lines 17-18 state that the book Silent Spring "proved how much harm was done by the uncontrolled, reckless use of insecticides." 41. The word flawed is closest in meaning to the word "faulty." 42. Carson's work "was vindicated" by the report (line 23), implying that the report contradicted the chemical industry's claims and supported her ideas. 43. The passage deals with the two main divisions of economic resources: property resources and human resources. The other choices refer to minor details in the passage. 44. This expression is used figuratively in the passage to mean that economic resources is a broad topic. 45. According to lines 9-10, economists "mean much more than the non-economist" by the term "land." 46. The word arable means "able to be cultivated"-and therefore, "fertile. 47. The term the latter (which means the second concept mentioned before) refers to "consumer goods." 48. Capital goods include aids to transporting goods (line 13), such as a railroad. Choice (B) is specifically mentioned as not being a type of capital (lines 17-18). Choices (C) and (D) are examples of land, not of capital. 49. The word heading as used in this sentence means "category." 50. The third paragraph indicates that the term labor involves all types of human talents except entrepreneurial skills, which are considered a separate category. Answer Key PRACTICE TEST 3 SECTION 1: LISTENING COMPREHENSION PARTS A, B AND C 1.B 11.B 2.A 12.D 3.A 13.C 4.D 14.A 5.C 15.B 6.D 16.A 7.C 17.D 8.C 18.C 9.A 19.C 10.D 20.B 21.C 22.A 23.A 24.C 25.D 26.B 27.C 28.A 29.B 30.B 31.C 32.C 33.B 34.D 35.A 36.C 37.B 38.C 39.A 40.A 41.D 42.A 43.C 44.B 45.A 46.D 47.A 48.C 49.D 50.B SECTION 2: STRUCTURE AND WRITTEN EXPRESSION  1.A 2.D 3.C  16.A 17.D 18.A 19.A 20.D Structure 4.D 5.B 6.B Written Expression 21.B 22.B 23.A 24.C 25.B 7.B 8.A 9.D 10.C 11.D 12.C 13.B 14.A 15.A 26.C 27.D 28.B 29.D 30.D 31.C 32.D 33.A 34.B 35.A 36.C 37.B 38.C 39.A 40.A Explanation of Written Expression Items 16. The plural verb form have must be used to agree with the subject of the sentence, the plural noun formations. 17. The possessive adjective her should be used in place of the possessive pronoun hers. 18. The correct word order is adverb + participle: widely read. 19. Research is properly an uncountable noun and should not be pluralized. 20. The personal pronoun them should be used instead of the reflexive pronoun themselves. 21. The verb differ should be used in place of the adjective different. 22. In order to be parallel with the other items in the series (difficult and violent), the adjective dangerous is needed in place of the plural noun dangers. 23. Before the word equator, the definite article the must be used. 24. The noun leader (a person who leads) should be used in place of the noun leadership (the quality that leaders have). 25. The word twice should be used in place of double in this sentence. 26. The phrase should correctly read from east to west. 27. To be parallel with the other items (scissors and glue gun), a singular noun (knife) should be used in place of the plural noun knives. 28. The phrase should correctly read at least. 29. The definite article the should not be used in the phrase by boat. 30. The preposition into should be used after the verb subdivided. 31. There is a contrast between the two clauses of this sentence, so the conjunction but should be used in place of and. 32. The noun absence is needed in place of the adjective absent. 33. The preposition Since should be used in place of the preposition In. (This is clear because the verb-have devotedis in the present perfect, not the past tense.) Answer Key 34. The noun sale is needed in place of the verb sell. 35. Only the second noun (climbers) of the compound noun (mountain climbers) should be pluralized. 36. The preposition by has been incorrectly omitted; the phrase should read the process by which. 37. The word so should be used in place of too. 38. The verb ripen should be used in place of the adjective ripe. 39. The correct word order for the opening phrase of the sentence is Of the many. 40. This phrase should correctly read No longer. SECTION 3: READING COMPREHENSION 1.A 2.D 3.B 4.B 5.B 6.C 7.A 8.A 9.B 10.D 11.C 12.D 13.C 14.B 15.A 16.B 17.C 18.C 19.B 20.A 21.D 22.C 23.D 24.D 25.B 26.A 27.C 28.A 29.A 30.D 31.C 32.B 33.C 34.B 35.A 36.D 37.D 38.C 39.B 40.C 41.A 42.C 43.C 44.D 45.B 46.B 47.C 48.A 49.D 50.C Explanation 1. 2. 3. 4. 5. 6. 7. 8. 9. 10. 11. 12. 13. 14. 15. 16. 17. 18. 19. The passage provides an introduction to Lucy Maud Montgomery's life and works, especially her Anne books. The reference is to Anne of Green Gables, which is a classic children's tale written by a Canadian author. According to lines 5-7, Montgomery "moved in with her grandparents" after her mother died and her father moved to Saskatchewan. Montgomery's first book was published in 1908 (line 11), and she wrote the two sequels in the next three years." The word elderly is closest in meaning to "old." In lines 12-14, the author states that "the heroine of the book is taken in by an elderly couple who live in the fictional town of Avonlea." According to lines 14-15, "Montgomery incorporated many events from her life in Cavendish into the Anne books. The word prominence is closest in meaning to "reputation." In the context of the passage, the word character means "nature." There is no mention in the passage that a ballet was ever based on the Anne books. The other choices are mentioned in lines 21-22. In the context of the passage, scour means "search." Choices (A) and.(B) are details; choice (C) implies that animals can count, an idea which is contradicted in the third paragraph. The accounts from White's book indicate that certain animals are aware of quantities on an intuitional level. The word surreptitiously is closest in meaning to the word "secretly." As used in this passage, the word abandon means "vacate." The word odd in this context refers to numbers which are not divisible by 2 (1, 3, 5, 7, and so on). (Numbers that are divisible by 2 are called even numbers.) According to the first paragraph, caterpillars are used by wasps to supply food for their young. There is no mention that they are aware of quantities. The other choices are given in the paragraph as examples of creatures that are aware of quantities in some way. The word accounts means "reports" or "stories" in the context of this passage. According to the passage, "animals respond to quantities when they are connected to survival as a species ...or survival as individuals." Choices (A) and (D) are connected to the pigeon's and the lion's survival as individuals (since these incidents involve food), and choice (C) is an incident involving the survival of a Answer Key 20. 21. 22. 23. 24. 25. 26. 27. 28. 29. 30. 31. 32. 33. 34. 35. 36. 37. 38. 39. 40. 41. 42. 43. 44. 45. 46. 47. 48. 49. species (since it involves the cat's young). It can be inferred that choice (B) is the result of conditioning by a trainer rather than the result of the monkey's instinctive awareness of quantities. Since these people believe that "creatures other than humans can actually count" (line 16) and the author believes that none of the animal's achievements shows evidence of counting, the author must consider these people mistaken. The word admittedly is closest in meaning to "undeniably." The reference is to the remarkable achievements of animals. The reference comes in the sentence beginning, "In lab experiments ...." The research mentioned in the first paragraph shows animals' ability to recognize quantities, not their inability to count. The main topic of the passage is the impact of the Bessemer process. Choice (A) is too general; choices (B) and (C) are not directly discussed in the passage. There is no mention that Bessemer steel was used in Civil War weapons; the other effects are cited in the first paragraph. According to the passage, the production of Bessemer steel exceeded that of wrought iron in 1880 (lines 15-16), implying that wrought iron production was greater than Bessemer steel production before then. The word burgeoning is closest in meaning to the word "expanding." According to lines 19-20, "steel rails were far more durable (long-lasting) than those made of iron." In the context of this passage, the word driven is closest in meaning to that of "spurred." According to lines 21-22, "the U.S. steel industry became the largest in the world in 1886, when it surpassed that of Great Britain." Because steel made by the Bessemer Process accounted for only 3 percent of the total U.S. production in the 1950's, steel production must have involved methods other than the Bessemer process. Because the last paragraph deals with the declining importance of the Bessemer Process and mentions that the open-hearth method supplanted it, it is logical to assume that the next paragraph discusses the openhearth method. The author first mentions the expansion of the Bessemer steel method in lines 14 -16, in the sentence beginning "Production grew rapidly ...." The author does not discuss San Francisco's current housing problems. Restoration is discussed in the third paragraph; the three styles are explained in the second paragraph; the entire passage provides a brief history of San Francisco's Victorian houses. The word gaudy is closest in meaning to the word "showy." The first paragraph indicates that almost all of Nob Hill was destroyed in 1906, and goes on to say that surviving Victorians can be found in Cow Hollow, Pacific Heights, and the Mission district. Roman Classical is not one of the three styles of Victorian built in San Francisco; it is the style Italianate houses were influenced by, and so must have been an earlier style. Italianate styles were popular in the 1870's, Stick houses in the 1880's, and Queen Anne houses in the 1890's. Queen Anne houses were therefore the last to become fashionable. In the context of the sentence, bays are features of Victorian houses. (They are a type of window.) In lines 17-20, the author says that towers were among the features that "distinguished Queen Anne houses from their predecessors." The other choices are given as features of Italianate or Stick styles. Victorian houses were considered "impossibly old-fashioned" in the period after the earthquake (which occurred in 1906), but interest in them was renewed during the 1960's, when many were restored. According to lines 23-24, the owners of the houses commissioned "multi-hued paint jobs." According to the first paragraph, sea otters, along with freshwater otters and other animals, are members of the group known as mustelids. Lines 7-9 indicate that sea otters have to eat about 30 percent of their body weight a day. Thirty percent of 100 pounds is 30 pounds. The term picky eaters (meaning creatures with selective appetites) is applied to sea otters because each animal prefers only a few food types (lines 11-12). In the context of this passage, the word exhausted means "used up." Lines 15-16 indicate that some sea otters protect kelp forests because they "feast on invertebrates such as sea urchins and abalones that destroy kelp." The term luxuriant means "abundant." According to lines 20-21, "the soft outer fur forms a protective cover that keeps the fine under-fur dry." The word elated is closest in meaning to "overjoyed." Answer Key 50. According to lines 26-28, scientists are worried because the California otter population rate of growth is only 5 percent a year, “lagging behind the 18%-a-year rate among Alaska otters.” Answer Key AUDIO SCRIPTS SECTION 1: LISTENING COMPREHENSION SAMPLE LISTENING COMPREHENSION TEST Part A CD 1, TR 2 1. M1: I like your new bicycle, Helen. F1: Thanks, but it isn't new. I had my old one repainted. M2: What can be said about Helen's bicycle? 2. F2: Will that be cash, check, or charge? M1: I'm going to write a check, but I just realized I left my checkbook in my car. I'll be right back. M2: What will the man probably do next? 3. M1: I'll never be able to get through all these books on Professor Grey's reading list. F2: But Mark, not all of them are required. M2: What does the woman tell Mark? 4. M1: Was there anything you didn't like about the novel? F1: Only that it was too short! M2: What does the woman imply about the novel? 5. M1: Should I fill out this form with a pen or a pencil? F2: What difference does it make? M2: What does the woman mean? 6. F1: This new computer software is really easy to use. M1: Isn't it, though! M2: What does the man mean? 7. F2: You did bring the slide projector, didn't you? M1: I brought it, but I'm afraid it's out of order. M2: What is the problem with the slide projector? 8. F1: Your brother called. He wants you to meet him for dinner. M1: Oh, really? Did he say when? M2: What does the man want to know? 9. F2: Have you heard from Howard lately? M1: Funny you should ask. Yesterday, from out of the blue, I got a letter from him. M2: What does the man say about Howard? 10. F2: If you don't like this studio apartment, I can show you a one-bedroom unit up on the third floor. M1: All right. This one just doesn't have enough room for me. M2: How does the man feel about the studio apartment? CD 1, TR 3 Answer Key 11. M1: I think I deserved a higher grade in chemistry class. Does Professor Welch ever change the grades he gives? F1: Sure-about once a century! M2: What can be inferred about Professor Welch from this conversation? 12. F1: I wonder how I did on Professor Porter's test. M1: Oh, she's already posted the grades on her office door. M2: What does the man say about Professor Porter? 13. M1: I've invited some friends over to watch the game on television. I think I'll go out and get some drinks before they arrive here. F1: Shouldn't you get some snacks too? M2: What does the woman tell the man? 14. M1: I'm going to drop my political science class. It meets too early in the morning for me. F2: Allen, is that really a good reason to drop the class? M2: What does the woman imply? 15. F1: I can hardly hear anything from back here! Let's just go home. M1: Why don't we ask an usher if we can sit closer to the stage? M2: What does the man suggest? 16. F1: William comes up with some weak excuse or another for just about every mistake he makes, doesn't he? F2: Wait till you hear his latest. M2: What do the speakers imply about William? 17. M1: What a beautiful sunset! You should photograph it, Melissa. F1: If I had some film in my camera, I would. M2: What does Melissa mean? 18. F2: We should be arriving at the airport in another ten minutes. M1: Wait a second. This bus is going to the airport? M2: What can be inferred about the man? 19. M1: I wonder when the board of regents will pick a new dean of students. F1: Who knows? They're not even scheduled to meet until next month. M2: What does the woman imply? 20. M1: Are Randy and his friends still going to play cards this evening? F2: He's setting up the folding card table right now. M2: What does the woman say about Randy? CD1, TR4 21. M1: We should never have listened to Harvey. F1: If only we'd asked someone else for advice! M2: What do they mean? 22. M1: All the people in the audience certainly seemed to enjoy the performance. F1: Well, almost all of them did. M2: What does the woman mean? 23. M1: How was your room last night? M2: I slept like a baby. And the rates were quite reasonable. F2: What are these people probably discussing? Answer Key 24. F1: Professor White? A few of us in the back of the room didn't get a copy of your syllabus. M1: Hmm... there are 23 names on my class list, so I only brought 23 copies of the syllabus. M2: What can be inferred from Professor White's remark? 25. F2: We're not far from Mount Pleasant Street. There are some antique stores there that have some wonderful things, and they're fairly cheap. M1: They have some nice antiques, all right, but I sure wouldn't call them cheap. M2: What does the man mean? 26. F1: Daniel said that San Diego is a great place to go to a conference. M1: He should know. He's been there often enough. M2: What does the man say about Daniel? 27. F2: Have you ever gone for a ride with Charlie? F1: I sure have. He seems to think he's a race-car driver, doesn't he? M2: What does the woman imply about Charlie? 28. FI: Try a bowl of this soup and see how you like it. It's a new brand. M1: Um.. .I'd say it stacks up pretty well against the other kinds. M2: What does the man mean? 29. F2: Peter is favored to win the tennis match Saturday. M1: Oh, then that match wasn't canceled after all? M2: What had the man originally assumed? 30. F2: Adam, do you remember the tools I lent you when you were building those bookshelves last month? I'd like to have them back. M1: Uh, well, I hate to tell you this ...but I can't seem to lay my hands on them. M2: What does Adam imply? Part B CD1, TR 6 Questions 31-33: Listen to the following conversation. M1: Good morning, Diana. What did you want to talk to me about? F1: Good morning, Professor Lane. I wanted to talk to you about changing my major. You see, I've decided that when I graduate in three years, I'd like to work in an art museum. I think I should change my major to art history. M1: You know, Diana, I think you should give this decision some more thought. You've done well in your year as a business major. Besides, all organizations need good managers, whether they're private companies or nonprofit foundations like museums. F1: I suppose that's true, but wouldn't I still have to know a lot about art? M1: Why not take a few elective courses in art history? And try working a few hours a week as a volunteer at the local art museum. See if you really like working there. 31. What is the probable relationship between these two speakers? 32. When does Diana hope to begin working at a museum? 33. What does the man advise Diana to do? CD 1, TR 7 Questions 34-37: Listen to a conversation between two students. M1: Gloria, hello! You're not looking too happy. What's the matter? Have you been studying too much? F2: Oh, hi. No, that's not it. The problem is that I was planning to go home over spring break, but my travel agent just told me all the airlines are fully booked that week. Answer Key M1: Why not go by car? F2: It's too long a trip to take by myself, and gasoline is so expensive. M1: Have you checked the ride board? Maybe you can get a rider to go with you. F2: The ride board-what's that? M1: It's a bulletin board that has a map of the United States on it. The map is divided into different regions, and each region has a different number. Say you want to go to New England-that would be box number one. There are boxes for each number. You can put a white card or a blue card in one of the boxes. F2: What's the difference between a blue card and a white one? M1: Blue means you have a car and need riders to share the driving. White means that you're looking for a ride. F2: So I should go look at the white cards to see if anyone needs a ride to where I'm going, right? Ml: Yeah, and fill out a blue card too. F2: So where is this ride board? Ml: It's on the second floor of the Student Union building, right by the campus cinema. 34. Why does Gloria look unhappy? 35. According to the man, what do the numbers on the boxes at the ride board represent? 36. What does Gloria hope to obtain through the ride board? 37. Where is the ride board? Part C CD1, TR9 Questions 38-42: Listen to a student telling his friends about an experience he had. M1: I saw something rather unusual late last Sunday night. My roommate Ron and I were driving back here to the university from my parents' house. They live in a small town about seventy miles from here. We'd stopped so that I could driveRon was really tired-when all of a sudden, the whole sky was lit up. I thought I was seeing a UFO-a real flying saucer. Ron thought maybe it was an airliner going down, or a satellite coming out of orbit. A fireball shot across the sky. It looked almost like fireworks. Then the light disappeared behind the hills. It wasn't until the next morning, when I was listening to the morning news on the radio, that I found out what had happened. Apparently I'd seen a large meteorite. The light was caused by the heat of friction as the meteor traveled through the atmosphere. Most meteors burn up in the upper atmosphere, according to the announcer, but large ones can hit the ground. This one was unusual in that pieces of it were recovered. They landed in a parking lot about twenty miles from where we were. My mother told me that when she was young, she read a story in a magazine about a meteorite that crashed through the roof of a house and hit a woman sleeping in her bedroom and broke her leg. Anyway, it was an incredible sight, and I'm glad I was lucky enough to see it. 38. What did the man initially believe he had seen? 39. Why had the man stopped the car? 40. How did the man learn what he had actually seen? 41. What happened to the meteor that the man had seen? 42. How did the man feel about what he had seen? CD 1, TR 10 Questions 43-46: Listen to a talk given by a tour guide. F2: Ladies and gentlemen, the bus will stop next at the Washington Monument, which honors the first President of the United States. The first stone of the monument was laid in 1848, but because of a variety of problems, work was delayed. So it was not until 1884 that the monument was completed. Imagine that-it took 36 years! And then it was another four years before it opened to the public. Now, if you like, you can go to the top of the Monument. On the way up, you'll have to take the elevator. You can take the elevator down, too, or you can walk-if you don't mind climbing down 898 steps. The monument is made of marble, except for a tiny aluminum tip. Incidentally, before the tip was put on the monument, it was displayed around the country. Some young people jumped over the tip so that later, they could truthfully claim that they'd jumped over the top of the Washington Answer Key Monument. After visiting the Washington Monument, we'll be walking over to the Lincoln Memorial. Watch your step as you get off the bus, please. 43. Where is this talk probably being given? 44. How long did it take to build the Washington Monument? 45. According to the speaker, what must those people who go to the top of the Washington Monument do? 46. According to the speaker, what did some young people do when the tip of the monument was displayed? CD 1, TR 11 Questions 47-50: Listen to the fo